You are on page 1of 225

i

~l -
$"
~
~
..~
-
~f I
I
PREFACE
TABLE OF CONTENTS

This book is designed to give more emphasis in Description Page


solving problems in Power Plant Engineering subjects due to
the increase number of problem solving on recent board
examinations. Thermodynamics 1
Fuels & Combustion 72
The main purpose is to show the way by presenting Variable Load Problems 95
subject matter and material that have appeared in many board Steam cycles 97
examinations, together with worked-out solutions that are Boilers 123
Steam engine 135
acceptable to board examiners. By following a rather simple Steam Turbine 142
pattern of self-discipline and working out many problems can Geothermal Plant 149
help you pass the jist time. Diesel Plant 158
Gas Turbine Plant 188
Hydro-electric plant 199
The contents of each topic are also well designed for
Chimney 219
familiarization of expected questions that will come out in the Machine Foundation 226
actual board examinations. Heat Transfer 237
Air-compressor 253
The author can guarantee a good passing grade in Pumps 276
Fans & Blowers 304
Power Plant Engineering subjects if mastery of topics
Fluid Mechanics 314
including the principles behind in this reviewer were done. Past Board Examination Elements 331
Refrigeration 390
Air-conditioning 422
Conversion of Units- 453
,,"0_

Thermodynamics

THERl\10DYNAMICS

THERMOD YNAI~lICS PROPERTIES

Thermodynamics - 1 (Math-ME Ed Oct. 1998)

What is the pressure 8,000 ft (2000 m) below the surface of the ocean?
Neglect the compressibility factor, in Sl units.
Ao 21.4 Mpa C. 21.0 Mpa
B. 20 I Mpa D. 22.3 Mpa

SOLUTION:
Sea water

p = '>'1 h
U sing typical SG of sea water equal to 1.03 ~~o~=r:2:~m
p = (1.03 x 98 J )(2000)
p = 20,208 Kpa
P c 20.21 Mpa 1 P=wh

Thermodynamics - 2 (Math-ME Ed Oct. 1998)

What is the temperature at which water freezes using the Kelvin scale'!
A. 373 C. 278
B. 273 D. 406

SOLUTION:
Freezing temperature of water is o-c.
"K = °C + 273
"K = 0 + 273
"K = 273

Thermodynamics - 3 (Math-ME Ed Oct. 1998)

The SI unit of temperature is:


A. of C. BTU
B. oK D. oR
411.\', B
~

::' Thermodynamics Thermodynamics .J

St lllJTlON
Thermodynamics - 4 (Math-ME Bd Oct. 1998)
h = u + Pv
The pressure reading of 35 psi in kpa is: 9500 -r-u + 900(58)
A. 342.72 kpa C. 273.40 II ~c 4280/t-lh/lh
iii
~~ B. 724.00 D. 42730
fi']
i:~'
; SOL UIION' Thermodynamics - 8 (Math-ME Bd Oct. 1997)
~

Pg 35 (101325 14.7) The barometer reads 29.0 inches (737 mm) of mercury. What is the
Pg 241.25 kpag absolute pressure if a vacuum gage reads 9.5 psi (66 kpa) in 51?
P abs = Pg + Palm A. 3202 kpa C 31.36 kpa
Pabs 24l.2" + 10132" B. 3304 k p a . D. 31.86 kpa
Pabs 342.57 kpaa
SOLUTION:
101.325
Patrn (29)(---)
29.92
Thermodynamics - 5 (Math-ME Bd Apr. 1999) 98.2 kpa
Paun
r., Pgage + Pann
1 torr is equivalent to pressure _ -66 ~ 98.2
p abs
A. 1 atm C 14.7
PaLs 32.2 kpa
C 2 mm Hg D. 1/760 atm

An5. D
Thermodynamics - 9 (Math-ME Bd Oct. 1997)

A fluid with a vapor pressure of 0.2 Pa and a specific gravity of 12 is


used in a barometer. If the fluids column height is 1 rn, what is the
Thermodynamics - 6 (Math-ME Bd Apr. 1999)
atmospheric pressure?
A. 150.6 kpa C 144.4 Kpa
What is the standard temperature in the US? B. 115.5 Kpa D. 117.7 Kpa
A. Fahrenheit C Celsius
B. Rankine D. Kelvin
SOLUTION
Ans. A
Pressure = (specific weight)(height)
Pressure (12 x 9.81)(1 m)
rr;

Pressure c= 117.72 Kpa


Thermodynamics - 7 (Math-ME Bd Apr. 1999)

Given steam pressure of 900 Ib/fe, temperature of 300°F, specific Thermodynamics - 10 (Math-ME Bd Apr. 1997)
volume of 5.8 fellb. If the specific enthalpy is 9500 ft-lb/lb, what is the What is the atmospheric pressure on a planet if the pressure is 100 kpa
internal energy per Ib of the system? and the gage pressure is 10 kpa?
A. 4400 C. 3600
A. 10 kpa C. so kpa
B. 3900 D. 4280
B. 100 kpa D. 90 kpa
4 Thermodynamics Thermodynamics ';

SOLUTION'
Thermodynamics - 13
Pabs P atm + P gage
If the temperature inside the furnace is 700 oK, what is the
100 = P atm + 10 corresponding reading in OF?
A. 700.60 C. 860.60
PanTI = 90 kpa B. 750.60 D. 800.60

SOLUTION:
Thermodynamics - 11 (Math-ME Bd Apr. 1997)
OK = "C + 32
A column of water 200 cm high will give a pressure equivalent to: 700= "C + 273
A. 9810 dyne/em 2 C. O. JO bar "C ~ 427
2
B 0.1 atrn D. 19,620 N/m of = 9/5 De +- 32
of = 9/5 (427) + 32
SOLl:TTON:
°F= 800.6
h= 200 em
h~ 2 m
3
r
1
w = 9810 N/m r:" Thermodynamics - 14
P = wh 200cm
P = (9810)(2 m)
P == 19,620 Nlm 3
11' the of scale is twice the °C scale, what will be the corresponding
reading in each scale'?
A. zz-c and 44°F e. 40 0e
and 80°F
lher mudynamics - 12 B. 160 0e and 320°F D. 1oo-c and 200°F

0K? SOLUTION:
What is the equivalent "R of 400
OF 9/5 -c + 32
A n0600R C. 670.2rR
B. 851.15°R D 344.25°R
of z-c

SOLUTION 2°e = 9/5 <C + 32


°K- -c + 273 "C = 160
WI) "C + 273
OF = 2(160)
°e = 127
of = 320
of 9/5°e + 32
of 9/5 (127) + 32
OF 260.6
oR = of + 460
Thcrmodynamics - 15
oR 260.6 + 460
oR 720.6 Water enters the condenser at 25°C: and leaves at 40°C. What is the
temperature difference in of'?
6 Thermodynamics Thermodynamics 7

A. 25 en A 102.3 KN/m] C. 150.9 KN/rn J


B. 26 D. 28 B. 132.9KN/m J D. 82.2 KN/m J

SOLUTION: SOLUTION:

6°C = °C 2 - °C I Specific weight of mercury (Specific gravity) (Density of H 20)


Specific weight of mercury (1355)(9.81)
J
,:'.°C =40-25 Specific weight of mercury 132.9 KlV/m

6°C = 15 t, =40°C Thermodynamics - 18 (Math-ME Bd Apr. 1996)


6°F 9
---- An iron block weighs 5 N and has a volume of 200 em]. What is the
6°C 5 density of the block?
A. 988 kg/m ' C. 2550 kg/rrr'
6°F 9 B. 1255 kg/rn' D. 800 kg/rn '
-------
15 5
SOLUTION:
6°F 27°F
Mass = 5/9.81
Mass = 0.5097 kg
Thermodynamics - 16 Volume = 200 cm ' (1/100 3 )
Volume = 2 x 10.4 m'
Wafer enters the heater with 28°C and leaves at 75°C. What is the Mass
w
temperature change in OF? Volume
A. 7480 C. 84.80 0.5097
B 38.29 D. 57.36 w
2xl0- 4
w 2548.42 kg/m'
SOLUT10N:

6°C °C 2 - °C l

~
=
Thermodynamics - 19
6°C = 75 - 28
6°C ~c 47°C The suction pressure of a pump reads 540 mm Hg vaccum. What is the
absolute pressure in Kpa?
i\OF/6°C = 9/5 HEATER A. 40 C. 60
6°F/47 = 9/5 B. 3.3 D. 29.3
6°F = 84.6 of SOLUTION:

Thermodynamics - 17 (Math-ME Bd Apr. 1996)


# rr®] J)
Pa b , = Pg + Paull
Pabv -540 mrn Hg + 760 rnrn Hg
The specific gravity of mercury relative to water is 13.55. What is the 1\\" 220 mm Hg
specific weight of mercury? The specific weight of water is 62.4 lb per 1'.11" 220 X (101.325/760)
cubic foot. p.= -540 mm Hg
1',,1," 29.33 Kpa
8 Thermodynamics 9
Th crmodynamics

Thermodynamics - 20 SOltiTlON:

Weight of water ~ !S.'7481)(62A)


A boiler installed where the atmospheric pressure is 755 mm Hg. Has
a pressure 12 kg/ern", What is the absolute pressure in Mpa? Weight of water 4L7 lb
::200 + 350
A. 1.350 C. 1.200 Volume of sand and gravel
B. 1.277 D. 1,700 2.65(62.4)
Volume of sand and gravel cc 3.326 ft'
Patm=755 mm Hg 94
SOLUTION: Volume of cement
3.1 O(62..4)
Volume of cement -; 0.486 flO
P'b, p ~ -'- Parm
Pabs ~­ 12 x (101325/1.033) + 755 x (101.325/760)
Total weight 41.'7 -i- 200 350 ,. 9..+
r., 127769 Kpa
2 .r.

r., 1.277 Mpa Total weight - 685.7 lbs

BOILER Total volume 5!7.4S! - 33::26 ' CdS6


Total volurne- 4.48 jf
Thermodynamics - 21
Weight of concrete per CLl. it. concrete 68".'7,4.48
A storage tank contains oil with specific gravity of 0.88 and depth of 20 Weight of concrete/cu. n. concrete c-. 153.0S8 lb.ft \
m. What is the hydrostatic pressure at the bottom of the tank in
kg/ern"?
A. 1.76 C. 60
R. 2.0 D. 3.0 Thermodynamics - 23 (Math-ME Bd Oct. 19(5)

SOLUTION. A batch of concrete consisted of 200 Ibs fine aggregate, 350 lhs (031 'it
aggregate, 94 Ills, cement, and 5 gallons water. The specific gr:n ity cf

~G"0'd ,'om
P - w x h the sand and gnwel may be taken as 2.65 and that of 011' <:,~m~-m :b
J
P - (0.88 x 981 KN/m ) ( 20 m) 3.t O. Hew m uch by weight of cement is req uired to produce one cubic
P ~- 172.66 Kpa yard'.'
P J72.66 x (1.033/101.325) A. 765 C. 675
P - 1.76 kg/em' 13. 657 D. 567

SOLLTIC.JN:
Thermodynamics - 22 (Math-ME Bd Oct. 1997)
Volume of water ~ '7 .48l
Volume of water C~ 0668 ft
A batch of concrete consisted of 200 Ibs fine aggregate, 350 Ibs coarse
]no+3:<J
aggregate, 94 Ibs, cement, and 5 gallons water. The specific gravity of Volume of sand and gravtl
the sand and gravel may be taken as 2.65 and that of the cement as ':' t,5( 62.4)
3.~,:'6 n
J
3.10. What was the weight of concrete in place per cubic foot? Volume of sand ;mj zravel
A. 1721b C. 1621b 94
B. 236 Ib D. 153 Ib Volume of cement
.1. i ()~ 62 A)
10 Thermodynamics Thermodynamics II

Volume of cernent > 0.486 ft° Total volume 289.9 cm '


Total volurne > 0.668 + 3.326 -i- 0.486
Total volume ~ 448 ft3 Specific volume .~ Total volume
Weight of cement per ft° of concrete mixture Total mass
~ 94/4.48
3 3 289.9
2098 Ibs/ft 3 (27ft /yd ) Specific volume
566.5 lbs or 567lbs 250
Specific volume 1.1596 cm'rgr

Thermodynamics - 24

A cylindrical tank 2 m diameter, 3 m high is full of oil. If the specific Thermodynamics - 26 (Math-ME Bd. Oct. 1997)
gravity of oil is 0.9, what is the mass of oil in the tank?
A 8482 kg C. 1800 kg 100 g of water are mixed with 150 g of alcohol (w = 790 kg/m'). What
B. 4500 kg D. 7000 kg is the specific gravity of the resulting mixtures, assuming the fluids
mixed completely?

o .,
SOLUTION: A. 0.96 C. 0.82
B 0.63 D. 0.86
Volume of cylinder n/4 0' h
=
SOLUTION
Volume ofc~linder n/4 (2)2 (3)
=
Volume of cylinder = 9.425 m'
Total mass O. J 00 + 0.1 SO
Mass .~ Density x Volume
Total mass 0.250 kg
Mass = w x V _---- ~_'!1_ __
Mass ~ (09 x rooo kg/m )(9 .425)
3

Mass = 8482.3 kg
0.100 o.isu
Total volume = - - + -----
1000 790
Total volume 2.899 x io' m 3
Thermodynamics - 25 (Math-ME Bd Apr. 1998)
Total mass
3 Density of mixture
100 g of water arc mixed with 150 g of alcohol (w = 790 kg/m ) . What Total volume
is the specific volume of the resulting mixtures, assuming the fluids
0.250
mixed completely? - Density of mixture
3 4
A. 0.82 cm /kg C. 0.63 cmvkg 2899 x 10-
B. 0.88 cm]/kg D. 1.20 crrr'zkg Density Of mixture = 862 kg/rn:
Specific Gravity = 86211000
SOLUTION: Specific gravity = 0.862

Total mass = 100 -t- 150


Total mass = 250 grams Thermodynamics - 27

0100 0.150 A spherical tank is full of water that has a total mass of 10.000 kg. If
Total volume ----+---
1000 790 the out side diameter of the tank is 2722 mm, how th ick is the wall of
Total volume 2.899 x 10.4 m 3 the tank?
I nerrnouynumus 1,)
12 Thermodynamics

SOLUTION'
A. 50 rnrn C. 30 mm
B. 25 mm D. 35 mm Q= volume flow of water flowing
Q = A x vel
SO}"-,UTI()r~: 1ft
Q = n/4 D 2 x vel
i
Q ~
v "' t l i +- n/4(1)2(10)

~
rn/w
~
Q J
= 7.85jr/sec
'lS
EI%1§

O
V 10,000 kgll,OOO kg/rrr' i, . r, :r;
I

V = 10 m 3 ..-- ; -- t+
V = 4/3 it [3 I
I
10 = 4/3 it [3 IV
Thermodynamics - 30
r = 1336 m
r = 1336 mm
A certain fluid is flowing in a 0.5 m x 0.3 m channel at the rate of 3
1 2722/2-1336 m/sec and has a specific volume of 0.0012 m 3/kg. Dctermine the mass
1 = 24.49 mm of water flowing in kg/sec.
A 380 kg/sec C. 375 kg/sec
i3 390 kg/sec D.370kg/sec
Thermodynamics - 28
a.3m
A cylindrical t~WK is filled with water at the rate of 5.000 gal/min. The
height of water ill the tank after minutes is 20.42 ft. What is the
SOLUTION: ~
3m/s r = c _ P-O O.5m

diameter of the tank'?


A 30 ft C. 20 ft A x vel
Q =
B. 25 ft D.9m Q (05 X 0.3)(3)
Q OA5 m ' /sec
SOLUTION: Q = fiX v
OA5 = rn (0.0012)
m = 375 kg/sec
After J5 minutes,
V c, 5000(15) ~, 75,000 gal
V = 75,000x(l fe/7ASI gal)
V = 10,025.39 ft3 20.42
V c. n'4 DC h
2
10,025.39 = n:/4 D (20.42)
D = 25ft. ~-_Q --+

Thermodynamics - 29

Water is flowing through a 1 foot diameter pipe at the rate of W It/sec,


What is the volume flow of water flowing?
A. 7.50 ft3/sec C. 7.85 m 3/sec
B. 7.95 ft3/sec D. 0.22 mzsec
J. IICI ".uuy"u,"'U.. . ..., ............ .. _'_~,.a ...... v

UWS OF THERMODYNAMICS
Thermodynamics - 33

The flow energy of 124 IiImin of a fluid passing a boundary to a


Thermodynamics - 31 (Math-ME Bd. Apr. 1998) system is 2 KJ/sec. What is the pressure at this point.
A. 100 Kpa C. 1,000 Kpa
One useful equation used is the change of enthalpy of compressible B. 140.39 psi D. 871 Kpa
liquid with constant specific heat is:
hs ub2 - h,u b l = c(T, ub1 - T.u b l ) + V(P.ub2 - P.ub l ) SOLUTION:
where: T. ubu = temperature at state n
p. u bn = pressure at state n W = Pressure x Volume
v = specific volume of liquid
Water with enthalpy with C,ubp = 4.187 KJIkg_oK and v = 1.00 x 10 to 2 KJ/sec x 60sec/min = P(0.124 m 3/min)
rd
the _3 power cu.mIkg has the following states:
State I: T su b l = 19°C P. ub l = 1.013 x 10 to the 5th power Pa P = 967.74 Kpa x 14.7/101.325
State II: T, ub2 = 30°C P.u b2 = 0.113 Mpa
What is the change in enthalpy from state I to state II? P = 140.39 psi
A. 46.0 Kpa/kg C. 46.0 KJlkg
B. 56.0 KJlkg D. 46.0 KNlkg

SOLUTION: Thermodynamics - 34 (Math-ME Bd Apr. 1996)

h 2 - h. Cp(Tz-T 1) + v(P 2-P t ) Steam at 1000 Ib/fr pressure and 300 0 R has a specific volume of 6.5
h z - hi 4.187(30 - 19) + 0.001(113 - 101.3) fe/lb and a specific enthalpy of 9800 ft-Ib/lb. Find the internal energy
h z - h, 46KJ/kg per pound mass of steam.
A. 5400 C. 6400
B. 3300 D. 2500

Thermodynamics - 32 SOLUTION:

What is the potential energy of a 500 kg body if it is dropped to a h=u+Pv


height of 100 m?
A. 490.50 KJ C. 560.50 KJ 9800 = u + 1000(6.5)
B. 765.50 KJ D. 645.48 KJ
u = 3300 ft-lbfllb m
SOLUTION:

Potential Energy = mxz Thermodynamics - 35


Potential Energy = 500 x 100
Potential Energy = 50,000 kg.m x 0.00981 KNlkg Air and fuel enter a furnace used for home heating. The air has an
Potential Energy = 490.50 KJ enthalpy of 302 KJ/kg and the fuel has an enthalpy of 43,207 KJ/kg.
The gases leaving the furnace have an enthalpy of 616 KJ/kg. There
Thermodynamics
Thermodynamics 17

re 17 kg air/ kg fuel. The house requires 17.6 KW of heat, What is the


lei consumption per day?
A 85 kg C. 45 kg Thermodynamics - 3 7 (ME Rd. Apr. 1995)
B 41 kg D. 68 kg
The enthalpy of air is increased by 139.586 KJ/kg in a compressor,
SOLUTION: The ~ate of air flow is 16.42 kg/min. The power input is 48.2 KW.
Which of the following values most nearly equals the heat loss from the
gas compressor in KW?
By mass balance:
rn, + m, = m g ~
--. A. -10.0 C. -9.95
air
....... B. +10.2 D. +9.95
m/mr = 17
rna
m, = 17mr EY-RJ*.A~e
i 7mr + m, = mg ......
heat
17.kw
SOLUTION
mg 18mr U.42k

---
C.C

fuel Q
W 1 mh. = mh, -"- Q h.
mf
By heat balance:
rn, h, mrhr= m g h g Q .~ W + m(h i - h1 )
I..W=~8.2kW
.i..

(l7rnrJ(302) -t mt<43,207) = (l 8mf)(6 16) + \7.6 Q W - mth, - hi)


cc
/
m, = 4.7244 x \0,4 kg/sec x 3600 x 24 Q = 48.2 - (16.42/60)(139.586) COMPRESSOR
m. 40.819 kg/day
Co
Q= 10 KW
Q = -10 KJV ( heat is rejected)

t
h
,
Thermodynamics - 36
Thermodynamics - 38 (ME Bd, Oct. 1982)
The power plant furnace burns coal at the rate of 108,200 kglhr. Air at
100.8 Kpa, 28°C is supplied at the rate of 13.8 kg/kg coal. Determine A steam turbine receives 70 pounds of steam per minute with an
the volume flow rate of air flow in mJ/min. enthalpy of 1600 Btu per pound and a velocity of 100 It/sec. It 11:'3\'l:s
3/min 3
A. 21,327.64 rn C. 20,435.26 rn /m in the turbine at 900 It/sec and 1320 Btu/lb enthalpy. The radiation loss is
B. 19,41462 m3/mim D. 24,535.54 m3/min 84.000 Btu/hr. Find the horsepower output.
h.v, Q:::B4.000Btu!h
SOLUTION: SOLUTION'

A/F
AiF

rna
m,
=

C~
m/ml
13.8

13.8mt
13.8(\08,200)
100.8kPa

28°Cma_ . .. B
.
:;~t FUR~~fE.
By heat balance:

mh, t KE, co

W ~.. nuh, - h:) , m2


mh- + KE 2 +Q+ W

[VIC - v/] - Q
VJ

rn, = 1493160 kglhr


kg/h
rna = 24886 kg/min ) 7
PY ~. mRT 70( 1600 1320) 70/60 100 - - 900- 84,000
W .~ - . - - - - +- - - - - J ---------1 V;
(IOO.8)(Y) = 24886(0.287)(28 + 273) 42 ~ 2(32.2) 55!) 2~.t"
Y 21,327.64 m 3/min
-z-

W .c 4()J Hp
Thermodynamics Thermodynamics 19

(80)- (v ? i\]
ierrnodynamics 39 (M.E Bd. Oct 1986) 800 = 750 -+-

---.,
T

2(9.81)(427) 2(9.81)(427)
earn enters a turbine stage with a enthalpy of 3628 KJ/kg at 10 m/sec h,=800Kcal/kg
---<>----,
d leaves the same stage 'with an enthalpy of 2846 KJ/kg and a Y2 652.14 m/sec V,=80m/s
locity of 124 m/sec. Calculate the work done by steam.
A. 77676 KJ/kg C. 567.23 KJ/kg
B 873.45 KJ/kg D. 923.34 KJ/kg TURBiNE ~ W

SOLUTION:
0=0 V, Ih, =750kcal/kg
For ITl ~c 1 kg (basis)
By heat balance: ~

mh.! KE[ = mh 2+KE 2 + Q -rW


w
Il
~,
Thermodynamics - 41 (Power-MEEd Apr. 1998)

W m (11:
.
- h 2 ) + -- (v: - v 2 )
ill
2g
. 2 2
-
""----1'
Q------
h,=2846KJlkg t,
w
f,
A volume of 450 cc of air is measured at a pressure of 740 rnm Hg
absolute and a temperature of 20°C. What is the volume in cc at 760
mm Hg absolute and O°C?
I A. 516.12 C. 620.76
w~ 1(3628 - 2846, -t _ 1 _ [(70)2 -(24)2)(0.00981) \
£,
2(9.81) t;, B. 408.25 D. 375.85
v,
"~.
VI 776.762 Kllkg SOLUTION:
1~
CD II @
I, V,= 450c:c: V,
~ P,=760
! PlY' P 2Y] P, = 470
er ruodynamics - 40 (Math-ME Bd Apr. 1998)
I
~
-----
t, =20°C =
t, O°C
Ii TI T2

am with all enthalpy of 800 Kcal/kg enters a nozzle at a velocity of


r
740( 450) (760)(Y 2 )
rn/sec, Find the velocity of the steam at the exit of the nozzle if its
halpy is reduced to 750 Kcal/kg, assuming the nozzle is hortzontal (20+273) (0 + 273)
! disregarding heat losses. Take g(9,81) m/sec and J = 427 kg-
(cal Y 2 = 408.25 cc
A. 56124 m/s C 52.41 m/s
B 142.5 m/s D. 652.14 mls
Thermodynamics - 42 (Power-ME Bd \pr. 1998)
SOLUTION:
Assuming compression is according to the law PV = constant.
V 2 V 2 Calculate the initial volume of gas at a pressure of 2 bar which will
I
hi-+---- = h 2 +
2gJ
-=--
2

LgJ
occupy a volume of 6 cubic meters when it is compressed to a pressure
of 42 bar.
A. 126 m ' C. 130 m'
B. 120 m' D. 136 mY
20 Thermodynamics Th ermadynumics .!l

SOLUTION
Thermodynamics - 45 (Math-ME Bd Apr. 1998)
PI V; P2 V 2 CD Q)
The mass of air in the room 3 m x 5 m x 20 m is known to be 350 kg.
2 (VI) = 42 (6) P, = ·2bar P,=42 bar Find its density.
V,=6m' V,=? . A 1617kg/m J C. 1.167kg/m J
3
VI = 126 m B.1.716kg/m' D I.176kg/m '

SOLUTION

Thermodynamics - 43 (Power-ME Bd Apr. 1998) V ~ 3(5)(20)

How much heat , KJ must be transferred to 20 kg of air to increase the


temperature from 20 degrees C to 280 degrees C if the pressure is
maintained constant.
A 2500 C. 5200
V - 300m'

Density
m
v
,meso" [J 20
5m

B. 2050 D. 5500
t
350
r Density -
300
SOLUTION: 1,=20°C 280°C=t, \
HEATER -------.
m=20kg f!
Q = m cp (t 2 - t.) Density 1.167 kg/m'
Q = 20 (1.0) (280 - 20) r
Q = 5200 KJ Q ~
I Thermodynamics - 46 (Math-ME Bd Apr. 1998)

Thermodynamics - 44 (Math-ME Bd Apr. 1998)


I A transportation company specializes in the shipment of pressurized

If air is at pressure, p, of 3200 Iblft\ and at a temperature, T, of 800


what is the specific volume, v?
oR,
I
f.
gaseous materials. An order is received for 100 liters of a particular
gas at STP (32°F and 1 atm). What minimum volume tank is
necessary to transport the gas at 80°F and a maximum pressure of 8
a trn?
A. I 4.2 fe/lb C. 11.2 ftJlIb
A 16 liters C. 10 liters
B. 13.3 ft 3 /1b D. 9.8 fellb
B 14 liters D. 12 liters
SOLUTION
SOLUTION
PV = mRT CD @
PI VI T\ V 2
V P, = 1atm P,=8atm
v 1'[ 1', t, = 32°F t, = 80°F
m V 2 = ?•
V,= 1001i
RT
v 1(100) (8)(V 2 )
P -~--- - - -

(53.3)(800) (32 t 4(0) (80 + 460)


v c~

3200
v = 13.375/f/lb V.' /4 liters
22 Thermodynamics Thermodynamics 23

Thermodynamics - 47 (Math-ME Ed Apr. 1997) Thermodynamics - 49 (Math-ME Ed Apr. 1998)

A bicycle has a volu me of 600 em:'. It is inflated with carbon dioxide to Air compressed in a diesel engine from an initial pressure of 13 psia
pressure of 80 psi at 20"e. How many grams of carbon dioxide are and a temperature of 120°F to one-twelfth of its original volume.
contained in the tire? Calculate the final temperature assuming compression to be adiabatic.
A 5.98 g C 4.63 g A. 987 C. 981
B 6.4:-1 g D 3.83 g B. 980 D. 1010

SOLUTION SOLUTION: 2
p

M
M
M
=
C.C

=
molecular weight of CO 2
12 +2(16)
44
1'2
r,
=l~j
(
V2
y-I
\ '~=c
R = 8.314/44 ( 4-1

L
R
V
=
=
0.189 KJ/kg- oK
600 em' 1 (100)' --l-
1'2
120+460
.
V I / 12
VJ
V,-
~' 120"F >

V = 0.0006 m 3
1'2 = 1567°R ._-~~

P = 80 psi x (101.325/14.7) v,
P = 551.43 kpa
t 2 co 1567 - 460 ....,.-.v
t2 = 1101'F
T = 20 I 273
T = 293 uK
PV=mR1'
(551.43 )(0.0006) = m(0.189)(293)
Thermodynamics - 50 (Math-ME Bd. Oct. 1997)
m ~ 0.00598 kg
m ~ 5.98 grams
The compression ratio of an Otto cycle is 6:1, P sub l is 14.7 psia, T su b 1 is
68°F. Find the pressure and temperature at state 2.
~fe. A. 180.6 psia, 1081 OF C. 180.6 psia, 139°F
Thermodynamics - 48 (Math-ME Bd Apr. 1997) ~'
B. 180.6psia,139°F D. 180.6psig, 1081°R
f'
I
I
An Ideal gas at 0.60 atmospheres and 87"C occupies 0.450 liter. How r, p
~
many moles are in the sample? (R = 0.0821 atm/mole K)
SOLUTION:
A. 0.0002 mole C. 00198 mole ~'

I P,
B 0.0378 mole D. 0.0091 mole ;:

~t'
i~
(~
Compression ratio VIN]
~,
Compression ratio 6 2 P. ,=14.7 PSia
SOLUTION: S=c t,=68 0F
(P2/P\) = (V/V 2) k
(P 2fl4.7) =_ (6)14 • 1
Pv=nR1'
P 2 = 180.6 psia
liter - atm

--
I
(060 atm)(0.450 li) = n (0.0821 )(87 + 273)K ~

!v
n =
mole- K
0.009135 mole
1'2fT I =. (V /V 2)k.\
1'2/(68+460) = (6)14.1 + v,
1'2 1081.11'R

l
4 Thermodvnamics Thermodvnamics 25

'hermodynamics - 51 Thermodynamics - 53

,IIautomobile tire is inflated to 30 psig pressure at 50°F. After being If 8 Ihs of a substance receives 240 Btu of heat at constant volume and
riven, the temperature rise to i5°F. Determine the final gage pressure undergo a temperature change of 150°F. What is the average specific
ssuming the volume remains constant. heat of the substance during the process?
A. 32. J9 psig c. 0 psig
B 55 psig D. 38.9 psig A 0.30 Btu/lb-oF C G50 Btu/lb-oF
B. 025 Btu/Ib-oF D 0.20 Btu/lb· OF
SOLUTION:
SOLUTION
P2 PI T2 / T 1
Q ~.' m c, VH) m = Bibs
P2 (75 + 460) aF= 150
--------
(30+147) (50+460) 240 = 8(c,)( 150)

P= = 46 89 psia c, = 0.2 Btu/lb- OF 0=240 Btu

Peo 46.89 - 14.7


P2 32.19 psig
cc

Thermodynamics - 54 (ME Bd, Oct. 1995)


_ _ _ _ _ _ _ _ _ _ _ _ _ _ _ _ _ _ _ _ _ _T
.crmodynamics - 52 A certain gas at 101.325 Kpa and 16 cC whose volume is 2.83 m J are
compressed into a storage vessel of 0.3] m' capacity, Before admission,
flO 111} of atmospheric air at zero degree centigrade temperature are the storage vessel contained the gas at a pressure and temperature of
.ompressed to a volume of 1 m) at 100°C, what will be the pressure of 137.,8 Kpa and 24°C; after admission the pressure hasmcreased to
ur in Kpa? l1il.S Kpa. What should be the final temperature of the gas in the
A. J 500 C. 2,000 vessel in Kelvin'!
B. J,384 D. JOOO A. 298.0 C. [800
B. 319.8 [) 4200
SOLUTION:
SOLUTION
PlY]2Y .
2 P
----_
Solving for the mass of gas which is to be compressed:
T1 T2 PV -r- mRT
JOJ 32~(283) = m ,R( J6 + 2 7J )
P, = 101.325 Kpa (atmospheric air) m , ~ o 9922/R
Solving fur the mass of gas Initially contained in the
(101.325)(\0) (p} )(\) vessel:
(0 + 273) (\ 00 + 272\ PV mRT
137.8(031) = m2R(24 + 273)
P2 = 1384.4 Kpa m=- 01433/R
Solving for the final temperature:
Thermodynamics Thermodynamics 27

p]v, ~ rnJ RT 3
m, = rnlT rn2
Thermodynamics - 57
m, ~. 0.9922/R+01438/R
m, cc 1.13 6/R
What is the specific volume of an air at 30°C and 150 Kpa?
1171.8(0.31) = (l.136/R)RT
A. I ml/kg C. 12 m 3 /kg
1'3 = 319.8°[( B. 0.5 m 3/kg
D. 0.579 mJ/kg

SOLUTION:
ermodynamics - 55
PV = rnRT
e temperature of an ideal gas remains constant while the absolute v = Vim
.ssure changes from 103.4 Kpa to 827.2 Kpa. If the initial volume is v = RTfP
liters, what is the final volume?
A. 100 Ii c. 8 Ii 0.287(30 + 273)
B. 10 Ii D. 1000 Ii v
ISO
SOLUTION
3
v = 0.579 m / kg
PI VI ~c P2 V 2

1034(80) .~ 827.2(VJ)
Thermodynamics - 58
V, •• 10 liters
The compression ratio of an Otto cycle is 5. If initial pressure is 100
Kpa, determine the final pressure.
-rmodynarnics - 56 A. 1000 Kpa C. 300 Kpa

at is the density of air under standard condition: If SOLUTION


B. 952 Kpa P 3 D. 100 Kpa

A. j ] kg/m ' C. 1.2 kg/rn 3 ~ M


~. P1V 1 = P"V,/'
B. I kg/rn ' D. I KN/m' ~.

PI V/ ~ P 2 V 2 k

SOLUTION P 2 / PI - (V I/V 2 )k
P •.• 14.7 psi P2 / P I ~ (rK/ P,=100kPa

P = 101.325 Kpa P2 / 100 = (5)14


t c. 70°F
P 2 .~ 951.82 Kpa
_v,- v, v
t ~ 21.11 °C
PV rnRT
w - m/R Thermodynamics - 59
w P/RT
How much work is done when 20 fe of an air initially at a pressure of
101J25
V\,';::-- --.-- 15 psia and a temperature of 40°F experience an increase of pressure
0.287(2 LlI + 273) to 80 psi while the volume remains constant.
w .c 1.2I,g/m 3 A 1000 Btu C zero
B 3000 Btu 0 2000 Btu
28 Thermodynamics Thermodynamics 29

~
SOLUTION:
Thermodynamics - 62
For constant volume process, W = 0
L=J During the polytropic process of an ideal gas, the state changes from
20 psia and 40°F to 120 psia and 340°F. Determine the value of n.
A. 1.233 C. 1.355
Thermodynamics - 6C B. L255 D. 1.400

A perfect gas has a value of R = 58.8 ft-lb/lb-oR and k = 1.26. If 20 Btu SOLUTION:
are added to Sibs of this gas at constant volume when initial
temperature is 90°F, find the final temperature. T 2/T 1 = (PZ/Pit-i/n
n-I
A. 100°F C. 154°F
B. 104°F D. 185°F (340 -+- 460) j .!2.~J ~­
(40 + 460) -l20
SOLUTION:
1.6 6 n- l i n
=

Q m c, (t 2 - t 1) n - I In 1.6

Y,='{ n In6
c, =, R / k-I n- 1 0.2623n
=
m = 51bs
58.8 n = 1.355
c y
t
(1.26-1)778
c, = 0.29086 Btu/lb-vf 1
0=20 Btu
Thermodynamics - 63 (ME Bd, Apr. 1984)
20 = 5(0.29086)(t2 - 90)
t = 103. 76°F A group of 50 persons attend a secret meeting in a room which is 12
meters wide by 10 meters long and a ceiling height of 3 m. The room is
completely sealed off and insulated. Each person gives off 150 Kcal per
Thermodynamics - 61 hour of heat and occupies a volume of 0.2 m', The room has an initial
pressure of 101.3 Kpa and temperature of 16°C. Calculate the room
While the pressure remains constant and 689.5 Kpa, the volume of air temperature after 10 minutes.
changes from 0.567 m 3 to 0.283 m'. What is the work done? A. 36.35 C. 23.24
A. -196 KJ C. 206 KJ B. 33.10 D. 76.32 12m
B. 204 KJ D. -406 KJ
SOLUTION:
SOLUTION:
Volume of room = 12 x 10 x 3
W = P(V 2 - VI) Volume of room = 360 m'
Volume of air = 360 - (02 x 50)
of air = 350 m 3
W

W
689.5(0.283 - 0.567)

-195.82 KJ
Volume
Q
Q
50 x 150
7,500 Kcal/hr
i 0 =7500K
A ca
I/h

m PV/RT
m = (101.3)(350)/(0.287)(16~273)
30 Thermodynamics
Thermodynamics 31

m 427.46 kg
SOLUTION.
c, 0 171 Kcal/kg-OC
After 10 minutes:
7500(10/60) ~- 427.46(0.171)(tz- 16) Since the molecular weight of ammonia is 17, then
R 8.3143/M
t2 = 33.1 cr R = 8.3143/17
R 0.489 KJlkg-OK

T1 38-1-273
Thermodynamics - 64 (ME Bd. Oct. 1994)
T! 311°K
The compression ratio of an ideal Otto cycle is 6:1. Initial conditions
T2 = 100 + 273
are 101.3 Kpa and 20 c C. Find the pressure and temperature at the end
T2 = 373°K
of adiabatic compression.
A 1244.5 Kpa. 599.96°K
PIV 1 '.~ mRT I
B 12445 Kpa, gage, 60°C
413(V I) = 22(0.489)(311)
C 1244.5 Kpa, 60°C
VI = 8.101 m'
D. 1244.5 Kpa, 599.96°C
P2 V 2 = mRT 2
SOLUTION: (413)(V 2 ) = 22(0.489)(373)
V 2 c 9.716 rrr' .
Compression ratio .C V IIV
Compression ratio = 6 W = P(V,-V 2)
(P/P j ) = (V/V,;' W == 413(9.716 - 8.1(1)
(P/I013) = (6)14 W = 667 KJ
P2 = /244.5 Kpa

T/T = (V IV ) k. 1
J I 2
T 2/(20 + 273)= (60)14. Thermodynamics - 66 (Power-ME Bd Oct. 1997)
T 2 = 599.96 oK
Determine the average C, value in KJ/kg-K of a gas if 522 KJ/kg of
heat is necessary to raise the temperature from 300 OK to 800 0 K
making the pressure constant:
Thermodynamics - 65 (ME Bd. Apr. 1996) A. 1.440 C. 1038
B. 1.044 D. 1026
Ammonia weighing 22 kgs is confirmed inside a cylinder equipped
with a piston has an initial pressure of 413 Kpa at 38°C. If 2900 KJ of SOLUTION
heat is added to the ammonia until its final pressure and temperature
are 413 Kpa and 100°C, respectively, what is the amount of work done For constant pressure process,
by the fluid in KJ?
A. 630 C 420 Q .• m c p (t 2 - td
B 304 D. :'102 522 = I (c p ) (800 - 3(0)
cpc UJ44 IU/kg-" J(
32 Thermodynamics Thermodynamics 33

Thermodynamics - 67 (ME Bd. Oct. 1993) Thermodynamics - 69 (Power-ME Bd Oct. 1(97)


3
A tank contains 80 fe of air at a pressure of 350 psi; if the air is cooled A large mining company was provided with a 3 m of compressed air
until its pressure and temperature decreases to 200 psi and 70°F tank. Air pressure in the tank drops from 700 kpa to 180 kpa while
respectively, what is the decrease in internal energy? the temperature remains constant at 28°C. What percentage has the
A. +4575 C. 5552 mass or air in the tank been reduced?
B. -5552 D. 0 A. 74 C. 76
8.72 D. 78
SOLUTION:
SOLUTION:
G) Q)
m PV/RT
P,=180K;~
=
Solving for m., PI =700Kpa
m
m
=
=
(200 + 14.7)(144)(80)/(53.3)(70 + 460)
87.55 lbs PI VI = m, R T I V, =3m' V, =3m' I
700(3) = ill] (0.287)(28 + 273)
1,= 28°1£ _ t.,;:: 2S°t: I'

For constant volume process: ill I = 24.31 kg


ill, I m, .

PI/T] = P2/T2 Before use Afterl1se


T 2 = 70 + 460 Solving for rn-;
T 2 = 530 0 R P2 V 2 = m R T 2
(350 + 14.7) (200 + 14.7) 180(3) = m2 (0.287)(28 + 273)
rn, = 6.25 kg
T1 530
0
TI = 900 R Percent of mass reduced:
6U = mc v (T 2 - T I ) 24.3 i -- 6.25
- - - - -

6U = 87.55(0.171)(530-900)
24.31
6U = -5544 Btu 74.29%

Thermodynamics - 70
Thermodynamics - 68 (ME Bd. Oct. 1993)
In a diesel cycle, the air is compressed to one-tenth of its uriginal
If 10 Ibs of water are evaporated at atmospheric pressure until a volume. If the initial temperature of the air is 27"C, what is the final
volume of 288.5 fe is computed, how much work is done? tern peratu re?
A. 1680 Btu C. -610,000 ft-lb A. 420°C C. 460~C
B. no work D. 550.000 ft-lb
B. 440°C D. 480"C
SOLUTION:
SOLUTION:
VI = 10/62.4
V2 '=1/10 V I
V I = 0 . 1 6 f t3
Vi/Vz=lO
W = P(V 2 - VI) T: / T I = (VI / V2) k. l
W cc (14.7 x 144)(288.5 - 016)
W -r- 610,358 ft-lb
36 Thermodynamics Thermodynamics 37
I"
f,
IY
I
A 14.33 Kl/kg-OK C. 13.23 Kl/kg-OK
B. 2.34 Btullb-OR D. 10.76 Btu!lb·oK Thermodynamics - 76

SOLUTION: While the pressure remains constant at 689.5 Kpa the volume of a
system of air changes from 0.567 m 3 to 0.283 m3 • Find the change of
t>.W r = Pz VZ-P j V, internal energy.
A 389.68 KJ C. 678.68 KJ
t>.Wr = 620.36(0.017) - 103.4(0.0566)
8. 493.68 KJ D. 245.68 Kl
t>.W t = 4.69 K1
SOLUTION:
t>.h = t>.U + t>.W r
16.35 = t>.U + 4.69 t>.U = m c, (Tz - T, )
t>.U = 1 1.65 K.! P V P V
t>.U = me, (_2_2 - _1-')
t>.h mc p (T 2 - T)) mR mR

t>.U mC y
(T2 - T,) t>. U -- -(P2V2
Cv
- F;V1 )
R
16.35 cp 0.716
----- t>.U = --[(689.5)(0.567) - (689.5)(0.283)J
11.65 10.217 0.287

Cp = 14.33 KJ/kg- OK t>.U = 488.52 KJ

Thermodynamics - 75
Thermodynamics - 77 (ME Bd. Oct. 1996)
A perfect gas has a value of R = 58.8 ft-lbllb-oR and k = 1.26. If 20 Btu
are added to 5 Ib of this gas at constant 'volume when the initial If initial volume of an ideal gas is compressed to one-half its original
temperature is 90°F, find the final temperature. volume and to twice its original temperature, the pressure.
A. 100.76°C C. 10376°F A. doubles C. remains constant
B. 167.76°C D. 145.76°F B. quadruples D. halves

SOLUTION: ~. SOLUTION:

c; ~ Ri(k -I) <D e


I P2 T2
--
PIVI
---
o
GJ ITJ
~
T2 T,
c, =
c, =
58.8/778
1.26- 1
0.2906 Btu/lb-oR
t,=900F

..
VI I where: Vz
r,
P\TI
=
=
V,/2·
2T,
P2(V\/2)
v,
T,
P,
V,=V,I2
T,=2T,

Q = mcvCt z - t l )
10 TI 2T}
20 = 5(0.2906)(t z - 90) PI = Pz/4
tz = 103.76 OF f P2 = 4 PI

I
q.; Thermodynamics Thermodvnami. ., \ 1....;

59.7 P2
Thermodynamics - 78 (ME Rd. Oct. 1996)
540 590
P2 = 65.23 psia
What horsepower is required to isothermally compress 800 fe of air
per minute from 14.7 psia to 120 psia?
A. 13,900 Hp C. 256 Hp
B. 28 Hp D. 108 Hp
Thermodynamics - 80 (IVlE Bd. Oct. 1990)
SOLUTION:
Air is compressed polytropically so that the quantity PV I .4 is constant,
For isothermal process: If O.D2 m ' of air at atmospheric pressllre(101.3 Kpa) and 4°C are
compressed to a gage pressure of 405 KN/m 2 , determine the fTI"]lli
W = PIY I In(P/P 2) temperature of the air in "C,
A. 123.23°C C. 165.70°C
W = (14.7 x 144)(800) In(14.7/120) B. 187.23°C D. 28J4SoC

W = -355562 i .557 ft-Ib/min SOLUTION


n-j

W =
3555621.557 ft - Ib I min
---------
~~ ~~J~ 101.3kPa 405kPa
T1 - PI)
~
,33,000 ~--?
t,=4'C I' t,
W = 107.746Hp
-.21._=(405+ 101.3J-j-4
14-J
r- --J-i

~-~
4+273 IOU c........•.I
Thermodynamics .. 79 (Power-ME Bd Apr. 1997)
T2 ~ 438.7°K
All ideal gas at 45 psig and 80°F is heated in a closed container to t2 = 438.7 - 273
LWoF. What is the final pressure? t2 = /65.70 OC
A. 54 psia C. 75 psia
B. 65 psia D. 43 psia
Thermodynamics .. 81 (ME Rd. Oct. 1996)
SOLUTION
A refrigeration plant is rated at 20 tons capacity. How many pounds of
Pi = 45 + 14.7
PI = 59.7 psia CD o air per hour will it cool from 70 to 90°F at constant pressure.
A. 50,000 Ib/hr C. 52,000 lb/hr
P, = 45psig P,=?. B. 47,0001blhr D. 45,000 lb/hr
T[ = 80 + 460
T[ = 540 0 R t, = BO°F 1, =130°F
SOLlJrION:
T2 = 130+460
T2 = 590 0 R
mC p (t 2 - t \ )
Pj P2 Tons of Refrigeration
I 12,000
T\ T2

i
I

l l
~ ()
Til ermodyn ([1/1 ics TIJ erun II lvnmnic-; 4i

m(0.24)(90 ·70)
20 THERMODYNAlVIICS CYCLE
12 OOl)
,n = 50, (JOO lb/hr

Thermodynamics - 83 (Math-ME Bd Oct. 1998)


Fhermodvnarnics - 82 (ME Bd. Oct. 1996)
A steam engine operation hetween 150 0 e and 550°C. What is the
In constant temperature, closed system process, 100 Btu of heat is
3 theoretical maximum thermal efficiency?
transferred to the working fluid at 100°F. What is the change of A. 99'% C. 49%,
entropy of the working fluid? B. 27% D. 73%
A 0 18 KJ/oK c: 0.25 KJiOK
B. 0.57 KJ/oK D. 034 KJ/oK SOLUTION

SOLUTION: TH = 550 T 273


TH = 823°K
Q = 100(1.055)
Q = 1055KJ T[ = 150 T 273
TL = 423°K
T= 5/9 (100-32)' 273 TH - TL
C = ---
T ~- 310.78°K
TH
823 - 423
.6.s == QIT = 105.5/31075 c =
.6.s = 0.3395 KJlkg- D!( 823
e = 48.60'%

Thermodynamics - 84 (Math-ME Bd Apr. 1999)

An engine has a bore of 15 cm and stroke of 45 em. If the volumetric


compression is 2000 crn ', find the engine efficiency.
A. 46.2 C. 45.4
B 44.2 D. 40.3

SOLUTION:

v , = 2000 em"
V D =c (IT/4)(15i~ (45)
V D ~ 7952.156 em'
V,==V 2 + V D
V, = 2000 + 7952.156
V, - 9952.156em'
rl. V:,"y',
42 Thermodynamics Thermodynamics 43

rk = 9952.156/2000 \V = 81 Btu
rk = 4.976 W T H -T L 4 QA=130Btu 1
1 e = :;,
.

700°F
,i;

e = 1----- QA TH
(4.976) 14-1 W
81 1160 - T L
e 47.37%
-~--_.-

=
130 1160 .\ ~ (0;
T L = 437.23°R 3 QR=49 tu 2
t L = 437.23 - 460
Thermodynamics - 85 (Power-ME Bd Apr. 1997) t L = -22.77' F s
A Carnot engine requires 35 KJ/sec from the hot source. The engine
produces 15 kw of power and the temperature of the sink is 26°C.
What is the temperature of the hot source? Thermodynamics - 87 (Power-ME Bd Apr. 1997)
A. 245.57 C. 250 18
B. 210.10 D. 260.68
The maximum thermal efficiency possible for a power cycle operating
between 1200°F and 225°F is:
SOLUTION: A. 58~o C. 57.54%
B. 58.73% D. 57.40%
TL = 26 + 273
TL = 299°K
T "4' Q.=35KJ/s,
-~ (.1 ~
'1
.-;)
SOLUTION:
W TH - TL
e = W=15kw
QA TH T;, ~. 1200 + 460

12~OF f
15 TH - 299 26°C TH ~c 1660 0R

35 TH
.~.
~ TL
h
=
=
225 + 460
685°R
T'4[J'
T]-j = 523.25 -x s
523.25 - 273
t]-j =
tH = 250.25"C
T H -TL
e = -- ---- . 22SoF
.
Ttl 3 2
1660- 685 s
e=~--~--

Thermodynamics - 86 (Power-ME Bd Apr. 1997) 1660


e = 58.73%
A Carnot engine receives 130 Btu of heat from a hot reservoir at 700°F
and rejects 49 Btu of heat. Calculate the temperature of the cold
reservoir.
A. -21.9°F C. -20.8°F Thermodynamics - 88 (Power-ME Bd Oct. 1997)
B. -24.2°F D. -22.7°F
A heat engine is operated between temperature limits of ]370 DC and
SOLUTION: 260 DC. Engine supplied with 14,142 KJ per KWH. Find the Carnot
cycle efficiency in percent.
1H = 700 + 460 A. 70.10 C. 67.56
T H = 1160 0R B. 6505 D. 69.32
W = QA - QR
W = 130 - 49
44 Thermodynamics Thermodynamics 45

SOLUT10N
Thermodynamics - 90 (Power-ME Bd Apr. 1998)
T = 1370 + 273 T 4 •1
o-c 2
TeI
1643 "K A closed vessel contains air at a pressure of 160 KN/m gauge and

T;

e -
_ + 273
T-I = ?60

5
-
~
33"](

T,
1 - ~
-
Q137
3
:
W

260"C •
2
S
temperature of 30°C. The air is heated at constant volume to 60°C
with the atmospheric pressure as 759 mm A g • What is the final gauge
pressure?
A. 174
B. 169
C 167
D. 172
e zr: I - (53311643)
e - 6756%
. SOLUTION:

Patm = 759 (101.325/760)


Patm = I0 1.2 kpa
Thermodynamics - 89 (Power-ME Bd Oct. 1997) PI P2
-- ---

An Otto engine has clearance volume of 7%. It produces 300 kw TI T2


power. What is the amount of heat rejected in KW? (160 + 101.2) P
- - -2
A. 170 C 152 (30 + 273) (60 + 273)
B. 160 D 145 P2c~ 287 Kpa
P2 = 287 - 101.2
SOLUTION- P2 = 185.8 Kpag (No exact answer in choices)

I+c p' 3
Thermodynamics - 91 (Power-ME Bd Apr. 1998)
rk
c
14- 0.Q7 An air standard engine has a compression ratio of 20 and a cut-off
rk ---
= ratio of 5. If the intake air pressure and temperature are 100 kpa and
0.07
rk = 15.286 27°C, find the work in KJ per kg.
A. 2976 C. 2437
B. 2166 D. 2751
I
e = 1---
r kk-1 V
SOLUTION:

I p 2 ,3
e = 1----- IrK - I
(15.286) 14-1
e = l--{ c }
e 0.664
= r k k-I k(r c -I)
e W/QA
= ,4 '

0.664 = 300lQA I I (5) I 4 _ I l


QA = 452 kw e
W = QA - QR \ - (20FlI.4(5 - I) J 27'C
100Kpa
300 ~. 452 - QR e = 54.10%
QR = 152 kw T, = 27 + 273 V
4() Thermodynomics Thermodynamics 47

T, = 300 0 K 4320
Process I to 2 is Isentropic process: 0.35
lk 1
QA
T 1'--J
2 -
, V,
\/2
I QA ~. 12,342 K.J

T 2 = 300 (20)14-1
Thermodynamics - 93 (ME Bd, Oct. 1993)
T, = 994.J4°K

In an air standard Otto cycle, the clearance volume is 18% of the


Process 2 to J is constant pressure process:
displacement volume. Find the compression ratio and or thermal
T, V" efficiency.
_ c . = _ - =r
T V c A. 0.52 C. 0.53
2 1
To = 994.34 (5) B. 0.55 D. 0.60
To 4971.T'K
QA = m c p (T" - T 2) SOLUTION-
QA = (1)(49717-994.34)
QA = 4001.3 KJ/kg 1+ c
rk
c
W
e
QA 1 + 0.18
rk
018
W = (0541)(4001.3) rk 6.556
W = 21651U/kg I
e 1-----
(6.556) 14-1
e G.S3
Thermodynamics - 92 (Power-ME Bd Oct. 1997)

The thermal efficiency of a particular engine operating on an ideal


Thermodynamics - 94
cycle is 35'1<,. Calculate the heat supplied per 1200 watt-hr of work
developed in KJ.
A. 12,343 C. 14,218 The clearance of a diesel cycle is 10%. If initial temperature is 27 DC,
B. 10.216 D 11,108 determine the final tern peratu reo
A. 5lO De t 3 C. 540 D e
SOLUTION: B. 610De p 2 D. 1000 D e

W = 1200 w-hr SOLUTION:


W = 1.20 kw-hr I+c Vj
rk --.
W = 1.2 (3600) C V2 t,=27°C
W .~ 4320 KJ
] + 0.1
1
rk --- R>
w 0.1
lV,~ V
e rk = ] ]
()\
-- V,
---.J-
"'\'"
J Ii Ie rmotlynamics lh crnuulynumics 49

T'
.i ~,
'1
I (V .v.r' 1+0.15
l'\
T, 12: ~ 273) = (] If' 015
1'\ 7.667

r: - 782 85°K
c c
1- IlLI
1. 2 78=.,85 -173 (7.667)'
c: 47.89%
t2 ~ 509.85r

Thermodynamics - 97
T!l.:rmody!umll" - 95
'\11 ideal Otto cvcle. operating in hot air with k = 1.34 has compression
hn n n ideal diesel cycle with overall value of k = 1.33, compression r a t ro of 5. Determine the efficiency of the cycle.
is 15 and cu-off ratio of 2.1 ~ determine the cycle efficiency.
;'1 [iO A 52 45% C 64,27%
f\ 50.62% C 46.00';;0 B. 64.27% D. 36.46%
B. "290";0 D. 4900%
SOLUTION:
SOL UTION.
1
e I-~
1 r \..
e 1--- ---- { - '----}
r l, kl kir-1)
~ c

(21): - I e 1----
t: ---- ---_._----- (5) 134-1
I 1
(. 15)
1 ""
I
" '-' 1,';,7[-1
1.---, ••' \L., . .)
e 42.14%
~ .~. 0.529
e = 52.9% Thermodynamics - 98

An engine operates on the air standard Otto cycle. The cycle work is
ihcrmodYnarnics' 96
900 KJ/kg, the maximum cycle temperature is 3000°C and the
temperature at the end of isentropic compression is 600°C. Determine
~o ideal Otto cycle with 15"". clearance operates on 0.227 kg/sec of air
the engines compression ratio.
\ it h I, = 1.32. Determine the efficiency of the cycle. A 6.388 C. 867
/\. ';065% C 4 \32%
B. 10.45 D. 7.87
f3 4384"/0 D. 47.89%
SOLUTION:
SOUJTIClN

0\ IllqT; - T 2 )
I I "
I; ,000 + 273
r,
c 1;3273°](
I. 600 1- 273
so Thermodynamics I 'he rtnodyn am ie" 5I

T 2 = 873°K
QA/m = ciT) - T 2) Tlll:rlllol!v 11<1 III ics - I (Ill (I\U: Bd. Apr. 1(92)
QA/m = (0.716)(3273 - 873)
QA/m o~ 1718.4 KJ/kg -\ tIll',el cil~illl' is o[Jeraling on a -l-str okc cycle. has a heat rate of
e = W/QA II.JI5.6 KJ,K\V-hr brake. The compression ratio is 13. The cut-off
3
e = 900/1718.4 P r.ui» IS 2. [i S lI 1g K = 1.32. what is the brake engine efficiencv.
e = 0.5237 i\ I)-;.~, c. 735
1 r~ :' 1= D~5.3
e = 1---
k-l
rk
')()!!. ; I ( ) '"
1
0.5237 = 1 - -..
(r k ) r r,
______ I __ .s: .. I
rk 6.338 t,=600°C
v•
= ,I (

r, " k (r c - 1')

I'hermodynamics - 99 (ME Bd. Apr. 1995) (1) 13" _ 1


e = 1------
, (13)';2'
132(~> 1)
n an air standard diesel cycle, compression starts at 100 Kpa and
c 0:>010
OO°K. The compression ratio is 16 to 1. The maximum cycle
emperature is 2031 "K, Determine the cycle efficiency. (1 !<v,- hr) (3600)
A. 60.34% C. 65.98% e:
B. 56.23% D. 45.45% 1 Ul:5.6KJ
e,) 0:; 18
SOLUTION: '>
~, e, ,"
ec 0.318/0.5010
T 2/T 1 = (v/vd- ' c.. 63.5%
T 2/(300+273) = (16/1)14
0

1
2 3
T 2 = 1737.01 10 K P
Therrnodvna nucs - 101 (ME Bd. Apr. 1991)
T,=2031°K
rocess 2 to 3 is constant pressure: T,=3000K
V)V 2 = T)/T 2 4 P,=100okPa Determine the air-standard efficiency of an engine operating on the'
r, = V)N 2 = 2031/1737.011 diesel cvcle with clearance of 8'Yo when the suction pressure is 99.9:
r, cc 1.169 Kpa and the fuel is injected for 6'Yo of the stroke. Assume K = 1.4.
1 r k -I
e = I---{ c } 1 SOLUTION
r k k-l k(r -I)
c v
1 (1.169) J ( - 1 r k - i
e=I---{ }
(l6)IH 1.4(1.169-1) I', k- I kr r 1)
" ' (

\I, - V; = (J06V,)
e = 65.98% v. 008V"
\I, - 008V il - () Il() \"
v 014 V)
52 Thermodynamics
1 0.06V
v.v.
p
r Thermodynamics 53

rc =
VJ r
P·2·
(0). C;;
3
PURE SUBSTANCE
V2
0.14V O 4 P,=99.97
rc =
0.08V o
Thermodynamics - 103 (Math-ME Bd. Oct. 1999)
rc = 1.175
i + 0.08 ,1 ' Find the enthalpy at 100 psi and 97% quality, h r = 298.55 Btu/lb;
rk = - -
h rg = 889.119 Btu/lb.
r, =
0.08
13.5
IA~
$.08 Vp vp
LV A. 1,170 Btu/lb C. 1,734 Btu/lb
1 (1.75)~4 - 1 B. 1,161 Btu/lb D. 1,803 Btu/lb
e = 1- { }
(13.5)14-1 1.4(1.75-1) SOLUTION:
e = 60.02%
h = he + x h eg

Thermodynamics - 102 (Math-ME Bd Apr. 1996) h = 298.55 + 0.97(889.119)

A heat engine (Carnot cycle) has its intake and exhaust temperature of h = 1161 Btu/lb
157°C and 100°C, respectively. What is its efficiency?
A. 12.65% C. 15.35%
B. 14.75% D. r3.7'>%
Thermodynamics - 104 (Power-ME Bd. Oct. 1999)
SOLUTION:
180 grams of saturated water of temperature 95°C undergoes
T H = 157 + 273 ,v
evaporation process until all vapor completely vaporized. Determine
T'l~l
T H = 430 0K -c '1-
the changed in volume.
T L = 100 + 273 At 95°C, Vr = 0.00 I 0397 mvkg, V g = 1.9819m3/kg
T L = 373°K A. 0.1656 m
3
C. 0.2565 rrr'
Efficiency =
TH -TL
TH
430- 373
6J.•
j
2
SOLUTION:
B. 0.4235 m' D. 0.3656 m 3

Efficiency = s
430 Volume = Specific Volume x mass
Efficiency = 13.25% Volume = (v g - vr) x m
Volume = (1.9819 - 0.0010397)(0.18 kg)
Volume = 0.3565 m 3

s
Thermodynamics - 105

Five kilograms of saturated liquid at 120 Kpa is heated until its


moisture content is 5%. Find the work done for this process.
54 Thermodynamics 55
Thermodynamics

A. 813.59 KJ/kg C. 542.34 KJ(kg


Thermodynamics - 107
B. 643.23 KJ/kg D. 753.12 KJ/kg
SOLUTION:
Steam at 2 Mpa and 250°C in a rigid cylinder is cooled until the
quality is 50%. Find the heat rejected from the cylinder.
For constant pressure process,
At 2 Mpa and 250°C
W = P(V2 - VI) 3
V = 0.11144 m /kg
From steam table: At 120 kpa
u = 2679.6 KJ/kg
VI = vr at 120 Kpa (sat. liquid)
At 2 Mpa, (saturated)
v g = 1.4284 roJ/kg
vr = 0.0011767 rrr'zkg
VI = 0.0010473 mJ/kg
v g = 0.09963 mJ/kg
V2 = vr + x Vrg
Ur = 906.44
x = 1- Y
Urg = 1693.8
x = I - 0.05
5 A. -432.23 KJ/kgC. -834.55 KJ/kg
x ~ 0.95
B -926.26 KJ/kgD. 1082.34 KJ/kg
V2 0.00J0473 + 0.95(1.4284 - 0.0010473)
V, = 1.357
SOLUTION
Vi = 120(1.357 - 0.0010473)
W = 162.73 KJ/kg (5 kg)
Q = (U 2 - UI)
W = 813.59 KJ

U 1 = 2679.6
U. = U, + x Urg
Thermodynamics - 106
U 2 = 906.44 + 0.5(1693.8)
U2 1753.34 KJ/kg
Twenty kilograms of water at 40°C is confined in a rigid vessel. The
heat is supplied until all the water is completely vaporized. Find the Q = (175334 - 2679.6)
heat added in KJ. Q CC -926.26 KJ/kg
A. 45,422 KJ C. 45,252 KJ
B. 43,122 KJ D. 65,233 KJ
Thermodynamics - 108
SOLUTION:
Find the entropy in KJ/kg-K at 90% moisture of a IMpa steam-water
V=c
mixture?
For rigid vessel, (VI = v,)
me = 20kg At 1 Mpa:
t = 40°C Sg = 6.5865
Q = m (U 2 Ud -
Srg = 4.4478
UI = U, (saturated liquid)
A. 4.87 C. 2.583
U2 = U g (saturated vapor)

Q = m (U, - U r)
1 Q B. 6.34 D. 4.36

SOLUTION:
Q == m (U rg )
Q = 20 (2262.6) x I - 0.9
Q = 45,252 KJ
x 0.10
')6 Thermodynamics Thermodynamics 5'7

S ~' Sf + X Sfg SOLUTION:


Sfg = Sg - Sf
44478 = 6.5865 - Sf For isothermal process, t)= t2
Sf = 2.1387
Q = T (S2 - S) )
= 2.1387 + 0.10(44478) SI = Sf + X Sfg
S = 2.583 KJ/kg-"K SI = 18607 + 0.3(49606)
SI = 5.829
S2 = 7.0888
Q = (151.86 + 273)(7.0888 - 5.829)
Thermodynamics - 109 Q = 535.16 KJ/kg

At 1.3 Mpa, mixture steam and water has an entropy of 4 KJ/kg_°K.


Find the enthalpy of the mixture.
At 13 Mpa: Thermodynamics - I I I
sf~22515 hf = 8 14 .93
Sg = 6.4953 hfg = 1972.7 A tank contains exactly one kilogram of water consisting of liquid and
A. 1627.71 KJikg C. 1234.45 KJ/kg vapor in equilibrium at I Mpa. If the liquid and vapor each occupy
B. 1533.33 KJ/kg D 1734.45 KJ/kg one-half the volume of the tank, what is the enthalpy of the contents of
the tank?
SOLUTION: />. 644.40 KJ/kg C. 8331i0 KJ/kg
B. 774.40 KJ/kg D. 435.2lJ KJ/kg
S = Sf + X Sfg
4 = 2.2525 + x (6.4953 - 2.25 IS) At I Mpa Vf= 00011273 Vfg = 0.19444
x = 0.412
hIe 762.81 hfg " 20 I 53
h hf X hfg
~c
h = 814.93 + 0.412(1972.7) .--(Sat. Vapor)
SOLLJTlON:
h = 1627.71 KJ/kg
Let V ~. total volume of tan-, L:::: , rn, = 1 kg
(Sat. Liquid]
in! == Vl/Vl.
1/2 V

r -3
Thermodynamics - 110 mi.
00011273
mL 443.54 V
Mixture with 80% quality at 500 Kpa is heated isothermally until its
pressure is 300 Kpa. Find the heat added during the process.
m, v«. - -- '2_
1" :2 V m. iV'I.,v,'
r
At 500 Kpa m. me
.1..
2 v=;v
'
Sf' 1.8607 01944 - .4-_

Sfg = 4.9606 m, .' 572 V


At 300 Kpa and 151.86°C m
x -r-;

S = 70888 KJ/kg I'; \ +- ,n L


A 652.34 KJ/kg C. 983.44 KJ/kg
B. 535.16 KJ/kg D. 765.34 KJ/kg
58 Thermodynamics Thermodynamics 59

x
2.572V A. 156 c. ].672
2.572V + 443.542V B. 2.12 D. 3.230

SOLUTlON:
x 0.005765
=
h = h, + xh rg
From steam tables:
h = 762.81 + 0.005765(2015.3)
h = 774.43 KJ/kg At 70 bar(7 Mpa) and 65°C
VI = 0.001017 mJ/kg

At 50 bart S'Mpa) and 700 0K(427°C)

V2 = 0.06081 rrr'zkg
Thermodynamics - III (ME Bd. Oct. 1996)
ml = m2
J
A vessel with a volume of 1 m contains liquid water and water vapor 01 I v) = Q2 I V2
in equilibrium at 600 Kpa. The liquid water has a mass of 1 kg. Using
stea m tables, calculate the mass of water vapor. AxV\ A(]OO)
---
A.3.16kg C. 1.57kg 0.00 I0 17 0.06081
B. 0.99 kg D. 1.89 kg
VI = 1.672 m/sec

SOLUTlON:

From steam tables, at 600 Kpa: Thermodynamics - 114 (ME Bd. Oct. 1991)

~~i~~~
-'m __
1L Vv
v. > 0.001 ]01 mJ/kg Water substance at 70 bar and 65°C enters a boiler tube of constant
vg = 0.3157 m 'zkg inside diameter of 25 mm. The water leaves the boiler tube at 50 bar
m, t, and 700 0K
at velocity of 100 m/s. Calculate the inlet volume flow(li/sec)
Volume ofliquid = mL VL

Ef
A. 0.821 C. 0.344
Volume of liquid = ](0.001101) B.1.561 0.2.133
(Sat. Vapor)
Volume of liquid = 0.001101 m' m v
6S'C ' 700 0K

7Ob~ ~ba'
600Kpa
Volume of vapor = 1-0.001101 (Sat. Liquid) SOLUTION: -
J V, V,=100m/s
Volume of vapor = 0.998899 m
hom steam tables: :
Mass of vapor = 0.998899/0.3157 At 70 bar(7 Mpa) and 65°C
Mass of vapor = 3.164 kg 3
VI = 0.001017 m /k g

At 50 bar(5 Mpa) and 700 0 K ( 4 2 7 ° C )


3
V2 = 0.06081 m /k g
Thermodynamics - 113 (ME Bd. Oct. 1991)
mt = m2
Water substance at 70 bar and 65°C enters a boiler tube of constant QI I VI = Q2 I V2
ins'ide diameter of 25 mm. The water leaves the boiler tube at 50 bar
and 700 0 K at velocity of 100 m/s. Calculate the inlet velocity(m/sec)
Thermodynamics 61
60 Thermodynamics

AxV j A(lOO)
--- Thermodynamics - 120 (ME Bd. Apr. 1989)
0.001017 0.06081
V I = 1.672 mJsec Steam at the rate of 600 kg/hr is produced by a steady flow system
boiler from feedwater entering at 40°C. Find the rate at which heat is
Q\ = A x vet, transformed in Kcal per hour if enthalpy of steam is 660 Kcal/kg and
QI = rc/4 (0.025/(1.672) of the feedwater at 40 Kcal/kg.
Q\ = 0.8207li/sec A 372,000 C. 345,200
B. 387,000 D. 312.444

SOLUTION
Thermodynamics - 115 (ME Bd. June 1990)
Rate at which heat is transformed ~ ms(h, - h-)
One Ib (0.455 kg) of a mixture of steam and water at 160 psia(l 103.2
Kpa) is in rigid vessel. Heat is added to the vessel until the contents are Rate at which heat is transformed = 600(660 - 40)
at 560 psia (3861.2 Kpa) and 600°F (315.55°C). Determine the quantity
of heat in KJ added to the water and steam in the tank. Rate at which heat is transformed 372,000 Kcal/hr
A. 1423.70 C. 1562.34
B. 1392.34 D. 1294.45
Thermodynamics - 12] (ME Rd. Oct. 1988)
SOLUTION:
Steam leaves an industrial boiler at 827.4 Kpa and 171.6°C.·A portion
For a rigid vessel, the volume is constant: of the steam is passed through a throttling calorimeter and is
From steam tables: exhausted to the atmosphere when the calorimeter pressure is 10].4
At 1103.2 Kpa Kpa. How much moisture does the steam leaving the boiler contain if
vr ,~ 0.0011332 m
3/kg
J/kg
the temperature of the steam at the calorimeter is] ]5.6°C?
v g = 0.17704m A 3.78% C. 456%
U. = 780.65 KJ/kg B. 308% D. 2.34%
U rg = 1805.8 KJ/kg
~ I
r
At 3861.2 Kpa and 315 .55°C, SOLUTION: I Calorimeter
~ 101.4kPa
3/kg
V2 = 0.06378 m
E
U 2 = 2761.3 KJ/kg At 827.4 Kpa (171.6°C):Jl 115.6°C
Solving for the quality of mixture:
h; = 727.25 KJ/kg <i.i
Vj = V2
h fg = 20432 KJ/kg
vr+ XVrg = V2
From table 3:
0.0011332 + x(0.17704-0.00 11332) = 0.06378 At 101.4 Kpa and 115.6°C: 827.4kPa
x = 0.3561 = 35.61% h 2 = 2707.6 KJ/kg 171.6°C
Solving for U\: Let x = quality of steam entering the throttling
U = U r + xUfg
calorimeter.
U, = 780.65 + 0.3561(1805.8)
hi = h2
U\ = 1423.70 KJ/kg
hr + xh., = h2
Heat added = m(U 2 - U I)
727.25 + x(2043.2) = 2707.6
Heatadded = 0.455(2761.3 - 1423.7)
x = 0.9692
Heat added = 608.6 KJ
62 Thermodynamics Thermodynamics 63

y = moisture content SOLUTION:


y = I - 0.9692 V=O.058m'
I
y = 0.0308 VI = V 2 = 0.058/2 I
I
y = 3.08% VI = V2 = 0.029m3
T] ~ 27 + 273 v, /il'V,
137.8kPa :413.4kPa
T I = 300 0 K
Thermodynamics - 122 (ME Bd. Oct. 1995) T 2=I77f273 2r>C I 17r>C
T 2 ~ 450 0 K
Steam enters a throttling calorimeter at a pressure of 1.03 Mpa, The
calorimeter downstream pressure and temperature are respectively
ill I= PIV/RT 1
0.100 Mpa and 125°e. What is the percentage, moisture of the supply
ill I = (I 37.80)(0.029)/(0.287)(300)
steam? m, = 00464 kg
Properties of steam: ill 2 = P 2V 2/RT 2
P,Mpa h, htg hg
m, = (413.4)(0.029)/(0.287)(450)
103 2010.7 2779.25 m, = 0.0928 kg
Note At 0.100 Mpa and 125°C
h ~ 27266 Kl/kg
Heat loss = Heat gain
A. 2.62 C. 3. J 5
rn, C v2 (t 2 - tf) = rn, cv l (t f - t,)
8 5.21 D. 198
0.OlJ2S(0.7I 6)(1 77 - tf) = 0.0464(0.7I6)(tf - 27)
tf = 127°C
SOLUTION.
T, = 127 + 273
T r = 400 0K

h n = 2779.25 - 2010.7
h n ~ 768.55 KJ/kg 1.03,MPa :f. C.10MPa, 125°C L\.s = m c, In(T tiT1)
For throttling process: L\.SI = 0.0464(0.716) In(400/300)
hi = h2 L\.SI = 0.00956
h, + xh., = h2 L\. S 2 = 0.0928(0.716) 1n(400/450)
76855 -+- x(20 10.7) ~ 2726.6 L\.SI = -0.00783
x 09738 L\.s = 0.00956 - 0.00783
x 97.38% L\.s = 0.00173 KJ/"C
y .~ 100 - 97.38
Y = 2.62%
Thermodynamics - 125
Thermodynamics - 123 (ME Rd. Apr. 1996)
Using steam table, find the enthalpy of steam at 250 kpa if its specific
A vessel of 0.058 m 3 capacity is well insulated and is divided equally by volume is 0.3598 m 3/kg
a rigid conducting diaphragm. Initially both halves contain air at A. 1625.86 KJlkg
C. 1543.45 KJlkg
pressure of 137.8 Kpa and 413.4 Kpa and temperature of 27°C and B. 1785.34 KJlkg
D. 1687.55 KJlkg
177°C respectively. What is the increase of entropy of the system in
KJ/OC?
A 1.002 C. 0.00173 SOLUTION:
B. 0.5080 D. 0.1080

At 250 kpa:
t\~ Thermodynamics
Thermodynamics 65

h[= 535.37 Kl/kg


h,~ = 2181.5 KJ!kg
v,' = 0.0010672 m]/kg Thermodynamics - 127
v g = 0.7187 m]/kg
Steam enters an isothermal compressor at 400°C and 100 kpa, The
v = v, -+- XVlp exit pressure is 10 Mpa, determine the change of enthalpy.
03598 = 0.0010672 + x(0.7187 - 0.(010672)
A. 198 KJlkg C. 187 KJ/kg
x r--. 0.49988 B. 178 KJ/kg D. 182 KJ/kg

Solving for h:
h = h, -+- xhfg SOLUTION:
h = 53537 + 0.49988 (2181.5)
n /625.86 KJ/kg
At 100 kpa and 400 oe:
z;

h = 3278.2 KJ/kg

For isothermal process,


Thermodynamics - 126
t 2 = t\ ~~ 400°C

A throttling calorimeter is connected to the desuperheated steam line


At 400"C and 10 Mpa:
supplying steam to the auxiliary feed pump on a ship. The line
pressure measures 2.5 Mpa. The calorimeter pressure is 110 kpa and
h = 3096.5 Kl/kg
150"C: Determine the entropy of the steam line.
A. 6.8 KJ!kg-OK C. 6.6 KJlkg-OK /"h ". h, - h2
8. 7.2 KJ/kg-OK D. 7.5KJlkg-OK ,)h= 3278.2 - 3096.5
L\h = ] 81. 70 lU/kg
SOLUTION:

At 110 kpa and 150 oe: Thermodynamics - 128


h2 = 2775.6 Kl/kg
At 2.5 Mpa: Stearn enters an adiabatic turbine at 300°C and 400 kpa, It exits as a
h. = 962.11 Kl/kg saturated vapor at 30 kpa, Determine the work done.
hfg = 1841 KJ/kg A. 476.34 KJ/kg C. 436.33 KJlkg
Sf = 2.5547 B. 441.50 Kl/kg D. 524..34 Kl/kg
Sfg = 3.7028 SOLUTION:

n~
For throttling process: (h I = h2 )

hi = h2 h, + xh.,
Cc

2775.6 + 962.11 + x(1841)


At 300°C and 400 kpa:
b, .= 3066.8 Kl/kg J 300°0

). ._~~,~J.l~_ .~\--
x = 0.985 At 30 kpa and saturated vapor:
s I - Sf + XSfg
s\- 25547 + 0.985(37028)
h 2 =. h g = 2625.3 KJ/kg
W= hi - h2
"'.__ .._1 400Kpa

s\ 6.202 KJ/kg-"K W = 3066.8 - 2625.3


W 44/.51U/kg ----------to-
S

l
66 Thermodynamics Thermodynamin' 67

Thermodynamics - 129 Thermodynamics - 131

A 0.5 m ' tank contains saturated steam at 300 kpa. Heat is transferred A I kg steam-water mixture at 1.0 Mpa is contained in an inflexible
until pressure reaches 100 kpa. Find the final temperature. tank. Heat is added until the pressure rises to 3.5 Mpa and 400°C.
A. 934S"C C 99.63°C Determine the heat added.
B ~3::'3uC D 103.2'C A. 1378.64 KJ C. 1456.78 KJ
B. 1532.56KJ D.1635.45KJ
SOLl;TJO,\:
SOLUTION:

At 300 kpa: () 6058 m ' kg


VI \ - At 3.5 Mpa and 400°:

At 100 kpa, ~ 0 00 I 0432 m'/kg


V2 = 0.08453 KJ/kg-OK
U 2 = 2926.4 KJ/kg
v ~ ,.
Yf J' 694 m3/kg
At 1 Mpa:
vr = 0.0011273 m3/kg
Since v I is in between Vr and vb at I ()o kpa. then the vg = 0.19444 m3/kg
temperature is equal to the saturation temperature at U, = 761.68 m 3 lk g
I no kpa which is equal to 99.631!C. U rg = 1822 KJ/kg

for inflexible tank, VI = V2

VJ = V2 = Vr + XVrg
Thermodynamics - 130 0.08453 = 0.0011273 + x(0.1944 - 0.001127)
x = 0.4314
A 500 Ii tank contains a saturated mixture of steam and water at UJ = u,
+ x U rg
300"C. Determine the mass of vapor if their volumes are equal. U I = 761.68 + 0.4314(1822)
A. 1154 kg C 1345 kg U I = 1547.76 KJ/kg
8.]034kg D.1634 J,g Q=m(U 2 - U r)
SOLUTION Q = 1(2926.4 - 1547.76)
Q = 1378.64 KJ

At 300°C
vb = 002167 m'/kg
V[
mv -r- ~.

Thermodynamics - 132 (Math-ME Bd. Oct. 1999)


v
"
500/2 Atmospheric pressure boils at 212°F. At the vacuum pressure at 24 in
VI ---
1000 Hg, the temperature is 142"F. Find the boiling temperature when the
VI 025 m' pressure is increased by 45 psia.
111 v 0.25/0.02167 A. 342.34°F C. 479.13"F
mv 11.54 kg B. 526.34°F D. 263.45°F
68 Thermodynamics Thermodynamics 69

SOLUTION: HEAT CAPACITY

P2 = 14.7 + 45
P2 = 59.7 psia
PI = -24(14.7/29.92) + 14.7 Thermodynamics - 132 (Power-ME Bd Apr. 1998)
PI = 2.908
'"' What is the temperature in degree C of 2 liters of water at 30°C after
By interpolation: 500 calories of heat have been added to?
A. 35.70 C. 38.00
T, 14.7+45psi B. 30.25 D. 39.75
t2 59.7

SOLUTION:
30
0
e HEATER t,
212 14.7 212"F 14.7psi(atmospheric)

142 2.908 Q =m c p (t 2 - t.)


142°F -24"Hg Q = 500 cal
Q=500cal
t,-212 59.7-14.7 Q = 0.50 Kcal
0.50 x 4.187 = (2 x I) (4. I 87)(t 2 - 30)
t2 - 142 59.7 - 2.908 t2 = 30.25"C

t2 -212 = 0.7923tr 112.515


t 2 = 478.98°F Thermodynamics - 133 (ME Bd. June 1990)

A mass of 0.20 kg of metal having a temperature of lOODC is plunged


into 0.04 kg of water at 20 DC. The temperature of the water and metal
becomes 48 DC. The latent heat of ice at ODC is 335 KJ/kg-DK, and the
specific heat capacity of water is 4.19 KJ/kg-DK. Assuming no heat loss
to the surroundings, determine the specific heat capacity of the metal
in KJ/kg-DK.
A. 0.234 C. 0.754
water Metal
B. 0.564 .---"----p. OA51•r - - -•

SOLUTION: 0.04 kg 0.2kg


100 e
0

20°C

Mixture

Q
Heat loss by metal = heat gam by water
(m c p flt)metaJ = (m c p flt)water
70 Thermodynamics
Thermodynamics 71

0.20( c m )( 100 - 48) = 0.04(4.19)(48 - 20)


Heat loss by iron = Heat gain by water
c., = 0.451 KJ/kg- oK
m, cp,(t I - t3 ) = - t )
rn., c pw (t 3 2

where: t, = equilibrium temperature after mixing


Thermodynamics - 134 (ME Bd. Apr. 1996)
30(0.4)(220 - t) = 14.33(4.1 87)(t) - 10)
220 - t) = 5(t) - 10)
What is the total energy required heating in raising the temperature of t) = 45°C
a given amount of water when the energy applied is 1000 KWH with
heat losses of 25%?
A. 1000 C. 1333 i\.s = Q,
B. 1500 D. 1250 T]
i\.s ~ m, c p, (t 3 ~tl)
SOLUTION:
Q
--. HEATER 11000~Wh t3 + 273
Q - 0.25(Q) = 1000 30(0.4)(45- 220)
.6.5 = ------
Q = 1333 KWH
1 Loss=O.25Q i\.s =
45 + 273
-6.6 KJlkg

Thermodynamics - 135 (Power-ME Bd Oct. 1999)

A 30 kg iron was put in a container with 14.33 kg water. The water is


at looe and the iron has an initial temperature of 493°K until the iron
was in thermal equilibrium with water. Find the change in entropy.
(c, for iron = 0.4 KJ/kg°K)
A. -12.56 KJf'K C. -25.78 K!f'K
B. -43.58 KJf'K D. -6.6 KJf'K

SOLUTION: Iron Water

t, 30 kg 1, 14.33kg
220°c 20 0 e

'\: Mixture /

t,
t,
-r-

=
493 -273
220°C
D
72 Fuels & Combustion Fuels & Combustion 73

SOLLTION
FUELS AND COMBUSTION
Fuel .L Air ~ Product of Combustion

CIj!];O + 0, + 3.76N z ~ CO 2 -i- H20 +- 3.76 he

Fuels & Combustion - 1 C1JI!;u ~ 2150 2 + 21.5(376)N 7 ~ 14CO:.; 15H 20 i 21.5(3.76)N 2

A diesel power plant consumes 650 liters of fuel at 26°C in 24 hours f\ M~k~ Nt'; 1'., 21.5 + 21.5(376)
with 28°API. Find the fuel rate in kg/hr. ~. A . 0" )( l''J 71.- :"J') Theoretical A.T
~ 7e !1
f/'l I ~"-- o' -- ---

A 23.83 C. 22.85 r I Theoretical A/f = \02.34


B. 24.85 D. 26.85 t) lv\t1.~7"7 0t'1-~'j.~ ")

:7 'X" ""'" . 2.2:-:\_·~~ActuaJ A/F


102.34 (1.15)
i:"-Y\-\ Y)) Actual AlF 117.69 mol air/mol fuel
SOLlJT!ON:
141.5 )( ~ 7._--=~"-":i.2..

Sg.t 156°C ~~
131.5+° API
141.5 Fuels & Combustion - 3
Sgat1560C
131.5+28
A diesel power plant uses fuel that has a density of892.74 kg/m}
Sg.t156"c = 0.887
at 15.67°C. Find the heating value of fuel.
SG a1 W C = 0.887[ I - 0.0007(26 - 15.6)]
A. '(4.911() KJ/kg C. 43,000 K.J:'kg
SG at 26"C = 0.88
B. \9,301 Btu/lb D. 43562 KJ/kg
Density of fuel = 0.88(1 kg/Ii)
SOLUTiON:
Density of fuel = 0.88 kg/Ii

m Density of fuel
SG - - - - - _.. _----
w
v Density of water
V = 650/24 SG 892.74 /1000
V = 27.0833 li/hr SG 0.89274
0.88 = m/27.0833
m = 23.83 kg/hr 1..J 1.5
"API ------ - 131.5
0.89274
o.h,PI = 27

Fuels & Combustion - 2 Q ~. 41.130 r- 139PAPJ


Q =1-],130 -t- !39.6(27)
A boiler burns fuel oil with 15% excess air. The fuel oil may be Q cc 44,899.31 KJ/kg
represented by C I4 H 30 • Calculate the molal air fuel ratio.
A 14 C. 102.34
B 117.69 D.17.14
74 Fuels & Combustion Flld\ .e Combustion 7~

Fuels & Combustion - 4 Fuels & Combustion - (,

A certain coal has the following ultimate analysis: A diesel engine consumed 945 liters of fuel per day at 30°C. If the fuel
C = 69% N2 =5% was purchased at 15.5°C and 30° API at P5.00/li, determine the cost of
H 2 == 2.5% S = 7% Moisture = 8% fuel to operate the engine per day.
O 2 = 3.5% Ash = 5% A P5677.50 C P4888.90
Determine the heating value of fuel used. B. P4677.50 . O. P5000.0Q
A. 26,961.45 KJ/kg C. 25,002.4 KJ/kg
8. 45,256 KJ/kg D. 26,000 KJ/kg SOUTlON

1415
SOLUTION: SG 13 6 ' [ =
131.5 + 30
o SG I 3 6 C = 0.87616
Qh = 33,820C + 144,2l2(H --) + 9,304S SG,QcC = 0.87616 [I - 00007(30 - 156)J
8
0.035 SG w c = 0.8673
Oh = 33,820(0.69) + 144,212(0.025 - - - ) + 9,304(0.07) v ;O'C SG IS 6'C
8
Oh = 26,961.45 KJlkg VI' 6C SG JO'C
945 0.87616
-------

VIS 6"C 0.8673


Fuels & Combustion - 5 Vt3 6 C = 935.44 li
Cost = P500/li (935 44 Ii)
A diesel power plant uses fuel with heating value of 45,038.8 KJ/kg. Cost = P4,677.20
What is the density of fuel at 30°C?
A. 0.900 kg/Ii C. 0.850 kg/Ii
Fuels & Combustion - 7
B. 0.887 kg/li D. 0.878 kg/Ii
\. cylindrical tank 3 m long and 2 m diameter is used for oil storage.
SOLUTION:
How many days can the tank supply the engine having 27° API with
fuel consumption of 60 kglhr? 3m
Oh = 41,130 + 139.6 °API
A.484 C. 7.84, -
45,038.8 = 4],130 + 139.6(oAPI) B 5.84 O. 8.84 ~
API = 28 2m
SOLUTI.ON ~,"'Okglh
SG I5 6, C =
]41.5
---
I~~
131.5+0 API V ~ ;v4 0 2h · ..
SG I5 6, C = 0.8872 V - "1412/(3) ~
SG.1 30, C = 0.8872[ 1 - 0.0007(30 - 15.6)] V. 4.42478 m '
1415
'.. ~
SG ot 30'C = 0.8782 '-,( J t, f. (
Density of fuel = 0.8782(1 kg/Ii) 1]1.5~·27
Density of fuel = 0.8782 kglli
/0
Fuels & Combustion Fuels & Combustion 77
SG l 5 6 0
C 2. 0.8927
SOLUTION:
Density of fuel ~. 0.89274(1000 kg/m3)
Density of fuel = 892.74 kg/nr' A/F = 11.5C + 34.5(H - 0/8) + 9.3S
W = m/V A/F = 11.5(0.7) + 34.5(0.03 - 0.04/8) + 4.3(0.06)
892.74 = 60IV A/F ~~ 9.1705(1.25)
V = 0.0672 m 3/hr A/F = 11.46 kg air/kg fuel
Number of days = 9.42478/0.0672
Number of days = 140.23 hrs
Number of days = 5.843 days
Fuels & Combustion - 10 (ME Bd. Apr. 1991)

Fuels & Combustion - 8 :it A 650 Bhp diesel engine uses fuel oil of 28° API gravity, fuel
consumption is 0.65 lb/Bhp-hr. Cost of fuel is P7.95 per liter. For
Determine the minimum volume of day tank in continuous operation, determine the minimum volume of cubical day
nr' of 28° API fuel tank in em), ambient temperature is 45°C.
having a fuel consumption of200 kg/hr. 3
A. 10.43 m' l A. 5,291,880 em C. 5,491,880 em]
C. 6.87 rn' (
B. 5,391.880 crn' D. 5,591,880 crn'
B. 5.41 m' D. 7.56 m'

SOLUTION:

SOLUTION:
1415
SG 15 6°e =
131.5 + 28 141.5
SG 15 6°e = 0.887 SG\S6'C = --
131.5-t-28
SG!56'C = 0.887
Density of fuel = 0.887(1000) SG 45 C = SG1WC[l - 0.0007(t - 15.6)]
0

Density offue1 = 887 kg/rn ' SG 4 5 C = 0.887[1 - 0.0007(45 - 15.6)]


0

W = m/V
SG 4 5, C = 0.86Q
887 = 200N
Density of fuel =2 0.869(1 kg/li) ~
V = 0.22548 m 3/hr x 24 hrs/day ~

Density of fuel = 0.869 kg/li


V = 5.4/ m3
Solving for fuel consumption'
m. = 0.65(650)
Fuels & Combustion - 9 m. = 422.5 lb/hr
m- = 191.61 kg/hr
Given the following ultimate analysis: V r = 191.61/0.869
C = 70% O 2 = 4% V f = 220.495 li/hr
N2 = 5% ~ = 6% Moisture = 8%
H 2 = 3% Ash = 5% Volume of dav tank = 220.495 x 24 hrs
3/1000li
Using 25% excess air, determine the actual air fuel ratio Volume of day tank. = 5,291.88 Ii x Im
3
A. 11.46 C. 23.85 Volume of day tank. = 5.291.88 m' x (100)3 e m 3 1m
B. 24.85 D. 26.85 Volume of day tank. = 5,291,880 em'
Fuels & Combustion 79
78 Fuels & Combustion

S( l[l iT!ON:
Fuels & Combustion - 11 (ME Bd. Apr. 1991)
60°F = 15.6°C
A 650 Bbp diesel engine uses fuel oil of 28°API gravity, fuel
80°F = 26.6°C
consumption is 0.65 lblBbp-br. Cost of fuel is P7.95 per liter. For 1415
continuous operation, determine tbe cost of fuel per day at 45°C. SG 15 6"C = ---
A. P42,870.45 C. P42,570.45 131.5 + 30
B. P42,070.45 D. P42,170.45 SG I5 6"C 0.876
=
SG 26 6 "C = 0876[1 - 00007(2667 - 1556)]

SG 2 6 6 , C
= 0869

SOLUTION: At 26.6°C. m, = 250gal/24hrs x 3.7851i/gal


At 26.6°C. m. = 39.431lilhr
141j At 15.6°C, m. = 39.431(0.869/0.876)
SG 15.6o C = --- At 15.6°C,mr = 39.1161ilhr
131.5 + 28 Load = 2700/24
SG 1W C = 0.887 Load = 112.5 KW
SG w c = SG 1W C[1 - 0.0007(t - 15.6)] 39.l16xP3.00/li
SG w c = 0.887[1 - 0.0007(45 - 15.6)] Cost per KW-hr =
112.5
SG w c = 0.869 Cost per KW-hr = Pl.043/KW-h
Density offuel = 0.869(1 kg/li)
Density of fuel = 0.869 kg/li
Solving for fuel consumption:
m, = 0.65(650) Fuels & Combustion - 13 (ME Bd. Oct. 1981)
m, = 422.5 lb/hr
IDr= 191.61kglhr A logging firm in Isabela operates a Diesel Electric Plant to supply its
v, '" 191.61/\).869 electric energy requirements. During a 24 hour period, the plant
V r = 220.4951i/hr consumed 250 gallons of fuel at 80°F and produced 2700 KW-hrs.
Volume of day tank = 220.495 x 24 hrs Industrial fuel used is 30° API and was purchased at P3.00/li at 60°F.
Volume of day tank = 5,291.88 li Determine the overall thermal efficiency of the plant.
Cost of fuel per day = 5,291.88 li x P7.95/li A. 26.08% C 43.12%
Cost of fuel per day = P42,070.45 B. 34.23% D. 18.46%

SOLUTION:
Fuels & Combustion - 12 (ME Bd, Oct. 1981)
Qh = 41,130 + 139.6 x °API
A logging firm in Isabela operates a Diesel Electric Plant to supply its Qh = 41,130 + 139.6(30)
electric energy requirements. During a 24 bour period, tbe plant Qh = 45,3 18 KJ/kg
consumed 250 gallons of fuel at 80°F and produced 2700 KW-hrs.
60°F = 15.6°C
Industrial fuel used is 30° API and was purchased at P3.00m at 60°F.
80°F = 26.6°C
Determine cost of fuel to produce one Kwb
A. P3.043/KW-h C. P1.043/KW-h
B. P4.043/KW-h D. n.043/KW-h

I
ISU Fuels & Combustion Fuels & Combustion 81

141.5 Mass ot Iuel -r- 24.933 kg/hr


SG 1S6 o C = --
131.5+ 30 Qh ~, 41,1:10 + l396CO API)
SG 1W C = 0.876 Qh = 41,13 0' 139.6(2 8)
SG 26 6 C = 0.876[1 - 0.0007( 26.67 - 15.56)]
0
Qh = 45,039 KJlkg
SG 26 6 , C = 0.869
Power Output
At 26.6°C, IDr = 250gaV 24hrs x 3.7851i/gal Overall Efficien cy ------
At 26.6°C, IDr = 39.431l ilhr mrQ h
Load == 2700/24
Load = 112.5 KW 82.5
Overall efficien cy
IDr = 39.431l ilhr x 0.869 kg/li x Ihr/360 0sec (24.933 ! 3600)(4 5.039)
IDr = 0.00952 kg/sec
Power' Output Overall efficien cy 26.47%
Overall Efficiency = ----- ---=-
IDfQ h

112.5 -----_._---
Overall efficiency = Fuels & Combu stion - 15 (ME Rd. Oct. 1991)
0.00952(45,318)
Overall efficiency A circula r fuel tank 45 feet long and 5.5 feet diamet er
= 26.08% is used for oil
storage . Calcula te the numbe r of days the supply tank
can hold for
continu ous operati on at the followi ng conditi ons:
Stea m now = 2000 Ibs!h r
Fuels & Combu stion -14 (ME Bd, Oct. 1981) Steam dry and saturat ed at 200 psia
Fecdw ater temper ature = 230°F
A diesel electric plant in one of the remote provinces in the South Boiler Efficien cy = 75%
utilizes diesel fuel with an °API of 28 at 15.6°C. The plant consum Fuel oil = 34° API
es C 13.45
680 liters of diesel fuel at 26.6°C in 24 hrs, while the power genera A. 1234
ted B 1758 D. 2344
for the same period amoun ts to 1,980 KW-hr s. Determ ine overall
therma l efficiency of the plant
A. 26.47% SOLUT ION:
C. 23.45% 45ft
B. 12.34% D. 34.34% From steam tables:
SOLUT ION: At 200 psi( 1380 Mpa), h, =c 2789.6 KJ1<g rr-""" ""'''''
5.5ft

141.5
At 2JO°F( 1 10°C), h F = 461.3 KJ/kg ~ m,=2000 Iblh
,---
SG 15 6 , C = --- J2~Opsia
131.5+ 28 Qh 41,130- + 139.6(3 4) In, •BOILER
SG I5 .6 , C = 0.887
SG 26 6, C = 0.887[ 1 - 0.0007( 26.6 - 15.6)]
OJ,
In,
c 45.876 KJlkg
2000/2 205 ·, 75%
,1\ 230°F

Y
+;
SG 26 6, C = 0.88 111, = lJ07 kg/hr Yv~-

Density = 0.88 x 1 kg/Ii 1ll,(h s - h F ) -'. ~


Density = 0.88 kglli '1l> = - - - - - FURNAC E
mfQIJ
Mass of fuel = 680 (0.88)/2 4
82 Fuels & Combustion Fuels & Combustion 83

907(2789.6 - 461.3)
0.75 = - - - - - - - ' -
m f(45,876) Fuels & Combustion - 17 (ME Bd. Apr. 1'987)
me = 61.376 kg/hr
A steam generator burns fuel oil with 20% excess air. The fuel oil may
141.5
SG J5 .6 , C = - - - be represented by C 14 H lOo The fuel gas leave the preheater at 0.31
131.5+ 34 Mpa. Determine partial pressure of H10.
SG J5 6"C = 0.855 A. 23.34 kpa C. 35.7 kpa
Density = 0.855(1000 kg/nr') B. 29.34 kpa D. 32.34 kpa
Density = 855 kg/m?
SOL,UTION:
Volume of tank = 1[/4 (5.5/3.28i)2(45/3.281)
Volume of tank = 30.297 m3 . Fuel + Air -; Product of combustion

Total weight of fuel = 30.297 m 3 x 855 kg/m" C 14 H30 + O2 + 3.76N2 - ; CO 2 + H20 + 3.76 N z
Total weight offuel = 25,904 kg C I4H30 + 21.50z + 21.5(3.76)Nz -; 14 COz + 15HzO + 21.5(3.76)Nz
25,904
Number of Days = - - - -
61.376(24) Combustion reaction with 20% excess air:
Number of Days = 17.58 days C 14H30 + 1.20(21.5)02 + 1.2(21.5X3.76)Nz -; 14 CO 2 + 15 HzO
+ 1.2(21.5)(3.76)N z + 0.20(21.5)Oz

Total mols of product = 14 + 15 + 1.2(21.5X3.76) + 0.2(21.5)


Fuels & Combustion - 16 (ME Bd. Apr. 1987) Total mols of product = 130.308 mols

A steam generator burns fuel oil with 20% excess air. The fuel oil may Partial pressure of HzO = (15/130.3 08)( 0.3 1)
be represented by C 14 HJ(Io The fuel gas leave the preheater at 0.31 Partial pressure of H20 = 0.0357 Mpa
Mpa. Determine the actual air-fuel ratio in kg air per kg fueL Partial pressure ofHzO = 35.7 kpa
A. 17.89 C. 12.34
B. 15.67 D. 19.45

SOLUTION: Fuels & Combustion - 18 (ME Bd. Oct. 1985)

Fuel + Air • Product of combustion ~xit temperature


In the boiler design, it is desired to have the flue gas
C 14H30 + O 2 + 3.76N2 - ; CO2 + H20 + 3.76 N 2 above the dew point. Estimate the dew point temperature of the flue
gas produced by combustion having the gravimetric analysis of:
C J4H30 + 21.502 + 21.5(3.76)N 2 -; 14 C~ + 15H20 + 21.5(3.76)N2 N 2 = 71.84%
CO 2 = 20.35%
.. 21.5(32) + 21.5(3.76X28) O 2 = 3.61%
Thoeritical A/F = H20 = 4.20010
14(12) + 30(1)
A. 32°C C. 45°C
Thoeritical A/F = 14.91 kg air/kg fuel
B. 23°C D. 39°C
Actual AIF = 14.91(1.2)
Actual AfF = 17.89 kg airikg fuel

\
i
t
84 Fuels & Combustion Fuels & Combustion 85
SOU 'II< Ii'-..
Fuel and Combustion -20 (ME Rd. Apr. 1984)
Converting the given analysis to volumetric:
N2 0.7184/28
-r- 0.02565714 The dry exhaust gas from oil engine has the following gravimetric
CO; = 0.2035/44 0004625 analysis:
O2 = 00361/32 = 000112812 CO 2 = 21.6% O 2 = 4.2% N 2 = 74.2%
H/) = 0.042118 = 0.00233333
Specific heats at constant pressure for each component of the exhaust
gas in Kcal/kg °C are:
Total mols of product 0.025657 +- 0.00462 -t- 0.00 I] 28 + 000233 CO 2 = 0.203 O 2 = 0.2]9 N 2 ;= 0.248
Total rnols or product 003374359
Calculate the gas constant in J/kg_°K.
A. 272 C. 274
Partial Pressure ofH,O ~. (0.0023333/0.0337436)(10\.325)
C. 276 D. 278
Pallial Pressure ofH·O= 7.006 Kpa .
From steam table: At 0.007006 Mpa, tsar -~ 39°C SOLUT]ON
Dew po mt temperature = 39 '-'C
Converting the gravimetric analysis to volumetnc:
'.
Fuels & Combustion - ]9 (ME Bd. Oct. 1995) CO 2 0.2\6 0.216/44 0.004909
O2 0042 0042132 0.00]3]2
A flue gas has the following volumetric analysis: N2 0.742 0.742/28 ~QI6500
CH 4 = 68% C 2 H 6 = 32% 0.03272] mols/kg-mol
Assume a complete combustion with ]5% excess air at 101.325 Kpa, Molecular Weight ~ 1/003272]
what is the partial pressure of water vapor in Kpa. Molecular Weight = 30.56 kg/kg-mol
A 15.95 C. 12.45
B.23.12 D.27.34 R = 8.3 ]4/M
R = 8.3 ]4/30.56
SOLUTION
R = 0.27206 KJ/kg-OK
R = 272.06 Jlkg- OK
Fuel +- Air ~ Product of Combustion

(}(lSCfI.j 1360, +- 136(376)N, ~0.68CO" + I 36H 2 0 i \.36(3.76)N)


T
Fuel and Combustion - 2] (ME Rd. Apr. 1984)
(U::'C,!-fc,' 112Q~ +LI2(376Y~" ~ 0.6~_CO~ +- Q.96H 2Q+-1.12(3.76)N" ,r
i lULL 2480, + 248(3.76)N, ~\.32CC}, +232H 20+2.48(376)N 2
T
The dry exhaust gas from oil engine has the following gravimetric
analysis:
Considering 15% excess air: CO 2 = 21.6% O 2 = 4.2% N 2 = 74.2%
I Fuel « 1.15(24)0 2 [ 1.15(2.48)(3.76)N 2 ~ ] 32C0 2 -t- 2.32H 20 " Specific heats at constant pressure for each component of the exhaust
1.I5(2.48)(376)N 2 +- 0.] 5(2.4)0 2 gas in Kcal/kg °C are:
CO, = 0.203 O 2 = 0.219 N 2 = 0.248
Total rnols of product = 1.32 + 2.32 i 2.48(3.76)(115) + 0.15(248) Calculate the specific gravity if the molecular weight of air is
Total mots of product = 14.735 mols 28.97 kg/kg-mol
A. 0.98] CLOSS
Partial pressure of H 20 (2.321] 4.73 5)( 10 I.325 ) B ] .244 D. 0.542
Partial pressure of H 20 15.95 Epa
86 Fuels & Combustion Fuels & Combustion 87

SOLUTION: A. 2.870 C. 2.274


B. 3.120 D. 6.233
Converting the gravimetric analysis to volumetric:
SOLUTION
CO, 0.216 0216/44 0.004909
O2 0.042 0042/32 0.001312 Thea. A/F 11.5C + 34.5(H - 0/8) + 4.3S
N, 0.742 0.742/28 0.026500 Thea. A/F 11.5(0.715) + 34.5(0.05 - 0.07/8) + 4.3(0.036)
0.032721 mols/kg-rnol Thea AIF 9.8 lb air/lb coal

Molecular Weight = 1/0032721 O 2 in air by weight = 232%


Molecular Weight = 3056 kg/kg-mol
SG = 30.56/2897 Therefore:
SG ~ 1.055
Theoretical weight of O 2 0.232(9.8)
Theoretical weight of O, 2.274 1Mb coal

Fuel and Com bustion - 22 (ME Bd. Apr. 1984)

The dry exhaust gas from oil engine has the following gravimetric Fuel and Combustion - 24 (ME Bd. Oct. 1996)
analysis:
CO 2 = 21.6% O 2 = 4.2% Nz = 74.2% There are 20 kg of flue gases formed per kg of fuel oil burned in the
Specific heats at constant pressure for each component of the exhaust combustion of a fuel oil C 12 H 26 • What is the excess air in percent?
gas in Kcal/kg °C are: A. 26.67% C. 12.34%
CO 2 = 0.203 O 2 = 0.219 N2 = 0.248 B. 18.34% D. 20.45%
Calculate the specific heat ofthe gas in KJ/kg_°K.
A. 0.872 C. 0.452 SOLUTION:
B. 0.992 D. 0673
C I , H 26 + 18.5 O 2 + 18.5(3.76)N 2 --+ 12 CO 2 + 13H 20 + 18.5(3.76)N z
SOLUTION:
18.5 + 3.76( 18.5)
cp = 0216( ruo]) +- 0.042(0.219) + 0.742(0.248) Thea. A/F
cp = 0.237 K(;,.' ll~b-oC x 4.187 I
Thea. A/F 88.06 mol/mol
cp = O. Q92 KJ/kg- 't:"
88.06(28.97)
Thea. A/F in kg/kg = -----
12(12)+26(1)
Thea. A/F in kg/kg = 15 kg air/kg fuel
Fuel and Combustion - 23 (ME Bd. Oct. 1986) Mass of air = 20 kg flue gas - I kg fuel gas
Mass of air = 19 kg air
A bituminous coal has the following composition: Actual A/F = 19 kg air/kg fuel

C = 71.5% H = 5.0% 0 = 7.0% N = 1.3% Actual A/F = Thea. A/F ( I + e)


S = 3.6°/;, Ash = 8.2% W = 3.4% 19 = 15( I + e)
Determine the theoretical weight of Oxygen in lb/lb of coal e = 0.2667
e = 26.67%
88 Fuels & Combustion Fuels & Combustion 89

Combustion reaction with 125% theoretical air:


7108C -+- 7050H z -+ 0.0165 -t- O.004N 2 ~. 1.25(l0649)02 +
Fuel and Combustion ~ 25 (ME Bd. Apr. 1984)
l.25(l0649)(376)N?·c 7108C0 2 + 7.050H 20 + 0.0 16S0 2 i- 50054N 2 .
+ 0.25(10.649)0 2
A gaseous fuel mixture has a molal analysis:
Ill! = total 1110ls in product
H 2 = 14% CRa =: 3% CO = 2Tlfo rn- = 7108 i 7.050 + 0.016 + 50.054 + 2.662
O 2 ~ 0.6% CO2 = 4.5% N2 = 50.9% m- = 66.89 mols
Determine the air~fuel ratio for complete combustion on molal basis. Partial pressure of H2 0 = (7.050/66.890)( 170)
A. 2.130 C. 1.233 Partial pressure of H 20 = /7.92 Kpa
B. 3.230 D. 1.130

SOLUTION:
Fuel and Combustion - 27
Chemical reaction with Oxygen:
0.14H2 + 0.0700 2 = 0.14H20 Calculate the theoretical air needed for the complete combustion of
O.03C}-4 + 0.0600 2 = 0.03C02 + 0.06H20 ethane C 2 H 6 for 20 kg fuel.
0.27COz + 0.1350;\ = 0.27C02 A. 432.23 kg C. 234.45 kg
B. 28745 kg D. 320.32 kg
0.265 O2
Actual O 2 in product = 0.265 O 2 - 0.006 O 2 SOLUTION:
Actual O2 in product = 0.259 Oz
0.259 + 0.259(3.76)
Molal AJF C 2 Hc, + O 2 + (3.76)N" -,) CO 2 -t- I-hO -i- (3.76) N?
1
Molal NF = 1.233 mots air/mol a/fuel Balancing the equation:

C 2 Hr. " 350 2 -L 3.5(3.76)N z --~ 2C0 2 + 3HP + 3.5(3.76) N z

Fuel and Combustion - 26 (ME Bd, Apr. 1995) 3.5(32) + (3.5)(3.76)(28)


Theoretical A/F = .,
(2xI2) + (lx6)
A steam generator burns fuel oil that bas the foUowing chemical
Theoretical A/F = 16.016 kg airlkg fuel
analysis by mass in percent:
Mass of air needed ~ 16.016(20)
C = 85.3 H 2 = 14.1 S = 0.5 N 2 = 0.1
Mass of air needed = 320.32 kg
A. 19.85 C. 17.93
B. 11.l4 D. 14.20

SOLUTION: Fuel and Combustion - 28

Converting the given mass analysis to molal analysis: A fuel oil is burned with 50 percent excess air, and the combustion
characteristics of the fuel oil are similar to en H 2• • Determine the
C 85.3/12 7.108 volumetric (molal) analysis of CO 2 in the product of combustion.
Hz 14.1/2 7.050 A. 9.34°'u C. 6.34%
S 0.5/32 0.016 B. 8.66'% D. 7.45%,
Nz 0.1/28 = 0.004
l)O Fuels & Combustion Fuels & Combustion 91
SOLUTiON it 13/12
-
N) balance C 83.5
-c-

Oo-N,ratlo c/b=376
C"H i 6 " 0::' (376l N 2 ~ CO 2 + n.o t (3.76) N 2 83.5/b = 3.76
b 222
-r;

Balancing the equation: Hi balance: 2d = (13/]2)(26)


d ]4.08
r-r-

C,l-l!(, + (I 85)0 2 + 18.5(3.76)N 2 ~ 12C0 2 + 13IhO + 18.5(3.76) N 2


Divided the equation by a to determine the combustion equation for I mole
Considering the 50% excess air, of fuel.
C. 11", + 20.500 ~ 7708N 2 '-c> I J .8C07 +- 0 I &eO + 3.230 2 T 7708N 2
Cloth" (15)(185)0 2 + (1.5)(185)(376)N 2 ~ :12C02 + 13H 20 ~ 13H 2 0
. (1.5)(18.5)(37 11 ) N 2 ~ (0.5)( 185 )0 2
(20.5)(32) T (77.08)(28)
011 = total mols of product Actual A/F - - - ._-- ---"---------
..

][] 2 ( J 2) + 26(1)]
rn- -z: 12 -t- 13 + 1.5(18.5)(3.76) T (0.5)(18.5)
Actual AT = 16 5 kg air/kg fuel
mT = 138.55 mols
% CO 2 = 12/138.55
The balance equation for 100 percent theoretical air is
% CO 2 = 0.0866
°lr, CO 2 - 8.66% C, Hi,~' 1850 2 + 18.5(3.76)N 2 ~ ]2CO, + 13H 20 + 18.5(3.76)N 2

Theoretical .'\/F- (18.5)(32) + (! 8.5)(376)(28)


1[12(12) + 26(I)J
Fuel and Combustion - 29 Theoretical A.T' ]4.93 kg air/kg fuel

Fuel oil, C l 2 H 2o , is burned in air at atmospheric pressure. The Orsat 16.5 - J4.93
analysis of the products of combustion yields E~Cess air
CO 2 : 12.8%
14,93
Excess air = lO.52%
O2 : 3.5%
CO : 0.2%
N2 : 83.5%
Determine the percent excess air.
Fuel and Combustion - 30
A. 12.34?/o C. 10.52'}"
B. 8.34% D. ! 8.45~o
An unknown fuel has the following Orsat analysis:
CO 2 ; 12.5%
SOLUTION
CO : 0.3%
O2 : 3.1 '%
N2 : 84.1 %
Applying the conservation of mass on each reactants:
Determine the actual air-fuel ratio in kg air per kg fuel.
A 17.13 C i923
aC t2 HOb i b0 2 + eN, -) 12.8C0 2 + 0.2eO + 83.5N 1 + dH 2 0
B. 1234 D. 23.23
C balance: 12a 12.8 T 0.2
92 Fuels & Combustion
Fuels & Combustion 93
SOLUTION:
Theoretical O?/F ~ Weight of Oxygen
Weight of fuel
C.Ht. + C02 + dN 2 ~ 12.5C02 + 0.3CO + 3.102 + 84.1N2 + eR20 6.5(32)
Theoretical 0 21 F = - - - -
12(4)+ 1(10)
C balance: a =12.5 + 0.3 Theoretical Oj/F = 3.586
a =12.8
N 2 balance: d = 84.1 Mass of O 2 = 3.586(1000)
O 2 - N 2 ratio: die = 3.76 Mass of O 2 = 3586 kg
84.llc = 3.76
c = 22.36
O 2 balance: 2236 = 12.5 + 0.3/2 + 3.1 + el2
e = 13.2 Fuel and Combustion - 32
b = 2e
b = 2(13.2)
A volumetric analysis of a gas mixture is as follows:
b = 26.4
CO 2 : 12%
C l2 .s H26.4 + 22.3502 + 84.1N2 ~ 12.5C02 + O.3CO + 3.102 + 84.1 N 2 + O2 : 4%
13.2H20 N2 : 82%
CO : 2%
Actual AfF = (22.36 + 84.1X28.97) What is the percentage of CO on a mass basis?
1[12(12.8) + 1(26.4)J A. 1.0% C. 1.2%
B. 1.5% D. 1.9%
Actual AIF = 17.13 kg airlkgfuel
Converting to mass basis:

CO 2 ~ 0.12 x 44 = 5.28

Fuel and Combustion - 31


o? = 0.04 x 32 = 1.28
N? ~ 0.82 x 28 = 22.96
CO ~ 0.02 x 28 0.56
-c-

What mass of liquid oxygen is required to completely burned 1000 kg Total mass of product = 5.28 + 1.28 + 22.96 + 0.56
of liquid butane, C 4 H 10• on a rocket ship? Total mass of product = 30.08 kg
A. 4568 C. 3586 % mass of CO = 0.56130.08
B. 2746 D. 6345 % mass of Ct) = 1.9%

SOLUTION:
Fuels and Combustion - 33 (ME Rd. Apr. 1998)

C 4 RIO + O 2 + (3.76)N2 ~ CO 2 +J{20 + (3.76) N 2 What is the percent theoretical air for a combustion process to which
the fuel and combustion gas analysis are known as follows:
Balancing the equation: Fuel: % by volume
CO 2 : 12.4%

C 4 RIO + 6.502 + 6.5(3.76)N2 ~ 4C02 + 5H20 + 6.5(3.76) N 2 CO : 27%


94 Fuels & Combu stion VAR IABL E LOA D PRO BLE MS
11., : 2.2%
N-~ : 58.4%
Combu stion ga~ % by volume
CO z 24.6%
Variab le Load Problem s - ]
Oz : 1.0%
Hz : 74.4% a load
A. III C 116 A 50 MW fL"er plant has an averag e load of31,50 0 KW and
D. 126 factor of 70%. Find the reserve over peak.
B. 121 C. 5 MW
A. 4 MW
B. 3 MW D.6M W
SOLUT ION
SOLUT ION:

Combu stion reaction with theoreti cal air: Ave. Load


Load Factor = - - - -
() 124CO: + I) 27CO + 0022H : -r o 584N 2 + 0.1460" O. J 46(3.76)
N;o Peak Load
--> C:i94CO o 0022H 2 0 ~ o
T 584N 2 ;- O.146(3 76)N? 31,500
O. 70 C~
Peak Load
Combu stion with excess air:
Peax Load = 45,000 Kw
~ Peak Load = 45 Mw
0.] 24CO;o + onCO t 0022H 2 + o 584N 2 + (I +x)O
1460,
Reserve over peak = 50 - 45
(I ix)OI4 6(176) N 2 --> o 394CO , -t 0.022H 20 + o 584N 2 + Reserve over peak = 5 Mw
0146(3 .76)(1 +-x)N 2 + x(O 14\))0,

water:
Express ing the percent age of oxygen in the product s excludi ng the
Variab le Load Problem s - 2
0.01
KW-
The daily energy produc ed in a certain power plant is 480,000
x = 0.11 hrs, What is the daily averag e load?
Percent age Theoret ical air 0= 1 + 0.11 A. 10 MW C. 25 MW
Percent age Theoret ical air = III % B. 15 MW D. 20 MW

SOLUT ION:
Energy Pr oduced
Averag e Load
No. of hours
A verage Load 480,000 /24

A verage Load 20,000 Kw

A verage Load 20Mw


96 ~ 'ariable Load Problems Steam Cycles 97

Variable Load Problems - 3 STEAM CYCLE


The annual energy produced in a 100 MW power plant is 438,000,000
KW-hrs. What is the annual capacity factor of the plant? Steam cycle - 1 (ME Bd Oct. 1999)
A. 40% C. 35~'o
B. 50% D. 60% In a Rankine cycle steam enters the turbine at 2.5 Mpa (enthalpies &
entropies given) and condenser of 50 Kpa (properties given), what is
SOLUTION: the thermal efficiency of the cycle?
At 2.5 Mpa: h g = 2803.1 KJ/kg Sg = 6.2575
Annual Energy Pr oduced At 50 kpa: Sr = 1.0910 Srg = 6.5029
Annual Capacity Factor = h, = 340.49 h rg = 2305.4
Plant Capacity x 8760
Vr= 0.0010300
438,000,000 A. 25.5~~~~ C. 34.23%
Annual Capacity Factor =
100,000x 8760 B. 45.23% D. 12.34%
Annual Capacity Factor ~ 50%
SOLUTION: T

h, = 2803.1 KJ/kg
Variable Load Problems - 4
Solving for hz:
A power plant has a use factor of 50% and capacity factor of 44%. s = Sr + x Srg
How many hours did it operate during the year? 6.2575 = 1.0910 + x(6.5029)
A. 7700 hrs C. 7709 hrs x = 0.7945
B. 7800 hrs D. 7805 hrs h z = h, + xh rg
h z = 340.49 + 0.7945(2305.4)
SOLUTION: hz = 2172.13 L-·----------
s""·
Annual Energy Pr oduced h, = 340.49 KJ/kg
Annual Capacity Factor h, = h r + vr(P z - PI)
Plant Capacity x 8760 h, = 340.49 + 000 I 03(2500 - 50)
Annual Enerzv Pr oduced h, = 342.98
0.44 = ~-
Plant Capacity x 8760 (hi -h z)-(h 4 -h,)
Efficiency = -------
Energy Produced = 3854.4(Plant Capacity) (h j - h 4 )
(2803.1 - 2172.11)- (342.98 - 340.49)
Energy Pr oduced Efficiency =
Use Factor = -----'=------- (2803.1 - 342.98)
Plant Capacity x t Efficiency = 25.55%
Energy Pr oduced
0.50 ~
Plant Capacity x t
Steam cycle - 2
Energy Produced = 050(Plant Capacity)t ,
In an ideal Rankine cycle, the steam throttle condition is 4.10 Mpa and
0.50(Plant Capacity)t = 3854.4(Plant Capacity)
440°C. If turbine exhaust is 0.105 Mpa, determine the pump work in
t = 7708.8 hrs.
KJ per kg.
98 Steam Cycles
Steam Cycles 99
A. 6.34 C. 4.17
B. 5.34 SOLUTION:
D. 2.12

SOLUTION: Solving for h.:


At 410 Mpa and 440°C (Table 3)
Solving for h.: hi = 3305.7 KJ/kg
h, =h fatO.105Mpa s\ = 6.8911 KK/kg-OK
h- = 423.24 KJ/kg Saving for h 2: CD
V3 = 0.0010443 mvkg At 0.105 Mpa(Table 2)
Solving for h.: sf=1.3181 h r=423.24
Sfg = 6.0249 h rg = 2254.4 @
Using pump work equation:
h, V3(P4 - P3) + h 3.
-c;
s I = S2 = Sr + XSfg
V3 = 00010443 m
3/kg 6.8911 = 1.3 I 81 + x(6.0249)
h, = 0.0010443(4100 - lOS) + 423.24 x = 0.925 G») • 1

h, ~ 427.412 KJ/kg h2 = h, + xh.,


W p = h4 - h3 h 2 = 423.24 + 0.925(2254.4)
W p = 427.412 - 423.24 h 2 = 2508.54 KJ/kg s
W p = 4.172 KJ/kg Solving for h.;
h, = h-at 0.105 Mpa
h, = 423.24 KJ/kg
Steam cycle - 3 Solving for h.:
Using pump work equation:
A thermal power plant generates 5 MW has also 300 KW power h, = Vi(P 4 - P 3 ) + h,
3/kg
needed for auxiliaries. If the heat generated by fuel is 13,000 KJ/sec, V3 = 0.0010443 m

determine the net thermal efficiency. h4.= 0.0010443(4100 - 105) + 423.24


A. 35.78% C. 30.56% h, = 427.412KJ/kg
B. 36.15% D. 3367%
QA = h. - h,
SOLUTION: QA = 3305.7 - 427.412
5,000 - 300 QA = 2878.29 KJ/kg
11 net =
13,000
WT = hi - h2
Wr = 3305.7 - 2508.54
11 net = 36.15% WT = 797.16 KJ/kg
Wp = h, - h,
Wp = 427.412 - 423.24
Steam cycle - 4 w, = 4.172 KJ/kg

In an ideal Rankine cycle, the steam throttle condition is 4.10 Mpa and W ne t = W T - W p
440°C. If turbine exhaust is 0.105 Mpa, determine the thermal W net = 797..16 - 4.172
efficiency of the cycle. W n e t = 79299 KJ/kg
A. 20.34% C. 34.44% 11, = W nc,lQA
B. 27.55% D. 43.12% 11, = 792.99/2878.29
11t = 27.55%
100 Steam Cycles 101
Steam Cycles
Steam cycle - 6 (ME Bd. Oct. 1989)
Steam cycle - 5 (ME Bd. Oct. 1991)
1-- A steam generating plallt has two 20 MW turbo-generators. Steam is
supplied at 1.7 Mpa and 320°C. Exhaust is at 0.006 Mpa. Daily
In a Rankine cycle, saturated liquid water at 1 bar is compressed
isentropically to 150 bar. Fir..t by heating in a boiler, and then by average load factor is 80%. The steam generating units operate at
superheating at constant pressure of 150 bar. the water substance is 70% efficiency when using bunker fuel having a heating value of
brought to 750°K. After adiabatic reversible expansion in a turbine to 31,150 'KJlkg and an average steam rate of 5 kg steam/K'W-hr.
1 bar, it is then cooled in a condenset to saturated liquid. What is the Calculate the Mtons of fuel oilfbunker fuel required per 24 hours.
thermal efficiency of the cycle (%)? A. 515 C. 6.17
A. 23.45% C. 34.24% B. 432 D. 762
B. 16.23% D. '18.23%

SOLUTION: SOLUTION:
CD
At 150 bar(l5 Mpa) and 750 0 K ( 4 7 7 ° C )
h, = 3240.5 KJ/kg Ave. Load
Load Factor = I I I

SI = 6.2549 KJ/kg-OK Peak Load


Av/;;. Load ®
At I bar(O.IO Mpa) 0.8 = - - - - ~
Sf = 1.3026 h f = 417.46 20,000 x 2
Sfg = 6.0568 h fg = 2258.0 Ave. Load = 32,000 KW m, @
vr = 0.001043 s
SI = S2 = Sf + xSsg From Steam Tables: PM .- J

6.2549 = 1.3026 + x(6.0568) h, = 3077 KJ/kg Pw


x = 0.8176 h, =, 151.53 KJ/kg
3/kg
x = 81.76% V3 = '0.0010064 m

hz = 417.46+0.8176(2258) Solving for 14:


hz = 2263.6 KJ/kg 14 = V3(P 4 - P 3) + h)
14 = V3(P 4 - P 3) + h3 h, = 0.001 0064( 1700 - 6) + 151.53
14 = 0.0010432(15,000-100)+417.46 h, = 153.23 KJ/kg
14 = 433 KJ/kg m, = 5(32,000)
Wp = 14 - h, m, = 160,000 kg/hr
Wp = 433 -417.46 m s (h l - h 4 )
Wp = 15.54 KJ/kg llb =
mfQ h
Wr = h, - h z 160,000(3077 - 153.23)
Wr = 3240.5 - 2263.6 0.70 = ------'------'-
Wr = 976.9 KJ/kg m f(31,150)
m, =.21,454 kg/hr
976.9 - 15.54 21,454(24)
Efficiency = Fuel needed for 24 hours operation =
(3240.5 - 433) 1000
Efficiency = 34.24% Fuel needed for 24 hours operation = 514.9 Mtons
102 Steam Cycles Steam Cycles 103

value of 6,388.9 Kcal/kg and the steam generator efficiency is 86%.


Steam cycle - 7 (ME Bd. Oct. 1994) What is the net station efficiency of the plant in percent?
A. 30% C. 33%
A back pressure steam turbine of 100,000 KW serves as a prime B. 25% D.38%
mover in a cogeneration system. The boiler admits the return water at
a temperature of 66°C and produces the steam at 6.5 Mpa and 455°C. SOLUTION:
Steam then enters a back pressure turbine and expands to the
pressure of the process, which is 0.52 Mpa. Assuming a boiler Net Output = 1000 - 0.09( 1000) ®
efficiency of 80% and neglecting the effect of pumping and the Net Output = 910 MW
pressure drops at various location, what is the incremental heat rate Net Output = 910,000 KW
for electric? Heat generated = m, Qh Qh
The following enthalpies have been found; turbine entrance = 3306.8 m
KJ/kg, exit = 2700.8; boiler entrance = 276.23 KJ/kg, exit = 3306.8. 9800(907) I pw J

A. 21,504 KJIKW-hr C. 23,504 KJIKW-hr Heat generated = (6,388.9 x 4.187)


24 x 3600
B. 22,504 KJIKW-hr D.24,504 KJIKW-hr
Heat generated = 2,752,001 KW

Station efficiency = Net output/Heat input


Station efficiency = 910,000/2,755.00 I
SOLUTION:
Station efficiency = 33.07%
WT = turbine work
WT = m(h, - h2)
WT = m(3306.8 - 2700.8)
Steam Cycle - 9 (ME Bd. Oct. 1995)
WT 606 m KW
A superheat steam Rankine cycle has turbine inlet conditions of 17.5
(m x 3600)(h) - h 4)
QA Mpa and 530°C expands in a turbine to 0.007 Mpa. The turbine and
llbo pump polytropic efficiencies are 0.9 and 0.7 respectively, pressure
losses between pump and turbine inlet are 1.5 Mpa. What should be
........... m
(m x 3600)(3306.8 - L.71.J0.~) • , the pump work in KJ/kg?
QA A. 17.3 C. 37.3
0.8 D. 47.3
B. 27.3
QA 13,637,565m KJ/hr

13,637,565m ~l I SOLUTION:
Heat rate = • • »;> V 3 (P4 - P3)
606m m
IIp
Heat rate = 22,504 KJIKW-h,
where: Using density of water = 1000 kg/rrr'
V3 = 1/1000
3/kg
V3 = 0.001 m
Steam Cycle - 8 (ME Bd. Oct. 1994)
P4=17.5+1.5
A coal-fired power plant has a turbine-generator rated at 1000 MW P4 = 19 Mpa
gross. The plant required about 9% of this power for its internal P4 = 19,000 Kpa
operations. It uses 9800 tons of coal per day. The coal has a heating P3 = 0.007 Mpa
104 Steam Cycles Steam Cycles 105
P3 = 7 Kpa QA = h) - h 4
IIp = 0.70
T]b
0.001(19,000 - 7)
W = ------ 3187.1- 274.14
p 0.70 QA =' - - - - -
Wp = 27.1 KJ/kg 0.8
QA = 3641.2 KJfkg
QT+Q R
Cogeneration efficiency =
:.;- Steam Cycle - 10 (ME Bd, Oct. 1995) QA
361.55 + 2156.81
A steam plant operates with initial pressure of 1.70 Mpa and 370°C Cogeneration efficiency =

temperature and exhaust to a heating system at 0.17 Mpa. The 3641.2


Cogeneration efficiency = 69.16%
condensate from the heating system is returned to the boiler at 65.5°C
and the heating system utilizes from its intended purpose 90% of the
energy transferred from the steam it receives. The T]T is 70%. If boiler
efficiency is 80%, what is the cogeneration efficiency of the system in Steam Cycle - 11 (ME Bd. Apr. 1996)
percent. Neglect pump work.
Steam properties:
In a cogeneration plant, steam enters the turbine at 4 Mpa and 400°C.
At 1.70 Mpaand 370°C: h = 3187.1 KJ/kg S = 7.1081 One fourth of the steam is extracted from the turbine at 600 Kpa
At 0.17 Mpa: h f = 483.20 Sf = 1.4752
pressure for process heating. The remaining steam continues to
hfg = 2216.0 Sfg = 5.7062
expand to 10 Kpa, The extracted steam is then condensed and mixed
At 65.soC: h, = 274.14 with feed water are constant pressure and the mixture is pumped to the
A. 78 C. 91.24 boiler pressure of 4 Mpa. The mass flow rate of steam through the
B. 102.10 D. 69 boiler is 30 kg/sec. Disregarding any pressure drops and heat losses in
the piping, and assuming the turbine and pump to be isentropic, how
SOLUTION: much process heat is required in KW?
Steam properties:
h, = 3187.1 KJ/kg At 4 Mpa and 400°C: h = 3213.6 KJlkg, s = 6.7690
Solving for h2 : At 600 Kpa: hr = 670.56 Sf = 1.9312
SI = S2 = Sr + XSrg h rg = 2086.3 Sfg = 4.8288
7.1081 = 1.4752+x(5.7062) A. 15,646.8 C. 1.9312
x .~ 0.9871
B. 2,468.2 D. 1,027.9
h z = h, + xh rg
hz = 483.20 + 0.9871(2216)
hz = 2670.60 KJ/kg SOLUTION:
h, = h, = 274.14 KJ/kg

WT = (h, - hz)lh
®
51 =
52 = St· + xSrg
W 1 = (3187.1-2670.60)(0.70) 6.7690 = 1.9312 +x(4.8288) .-.
WT = 361.55 KJ/kg .. • 1I J. ®
x = 1.00(saturated vapv" - ~ 6bj'i,Ij.;1
OR = 0.90(hi - h 3)
QR = 0.90(2670.6 - 274.14) hz = h r+ xh rg ®
QR = 2156.81 KJ/kg hz = 670.56 -I. 1.00(2086.3) p).. • y
106 Steam Cycles Steam Cycles 107
h2 == 275/,)."7 11 5 ~c 251.13 KJ/kg
h, == h-at 600 Kj . vr == 0.0010172 m3lkg
h, == 670.56 h, == V5(P6 - P 5) + hs
Q == m p (h, - h 3 ) h, == 0.0010172(8000-19.94)+25I.13
Q == (30/4)(2756.9 - 670.56) h, == ·259.25 KJ/kg
Q == 15,647.5 KW
QA == (hi - h 6) + (h, - h2)
QA =' (3348.4 - 259.25) + (3379.5 - 2963.145)
QA == 3505.501 KJ/kg
WT · == (hi - h 2) + (h, - ~)
Steam Cycle - 12 WT == (3348.4-2963.145)+(3379.5-2411.41)
WT == 1353.345 KJ/kg
In an ideal Reheat cycle, the steam throttled condition is 8 Mpa and QR == h, - h,
480°C. The steam is then reheated to 2 Mpa and 460°C. If turbine QR == 2411.41 - 25I.13 2.8MPa
exhaust is 60°C, determine cycle efficiency. 540°C
QR == 2160.28 KJ/kg
A. 38.3% C. 34.3% W p == h, - h,
B. 24.3% D. 45.2% w, =259.25-25I.13
W p == 8.12KJlkg
SOLUTION: W net = W T - Wp
W net == 1353.345 - 8.12
At 8 Mpa and 485°C(Table 3) 8ft1Pa W net == 1345.225 KJ/kg
hi == 3348.4 KJ/kg 4l10°C<D@ Tj == Wne/QA
51 == 6.6586KJ/kg-OK 2MPa
Tj == 1345.225/3505.505
- ' I ' - 4 l1 460°C
Tj == 38.37% ®
At 2 Mpa(Table 3), (51 == 52)
P)Oll
®
• y

6.6388 2952.3
5,=5,
6.6586 h2 Steam Cycle - 13 (ME Bd. Apr. 1991)
6.6828 2976.4
By interpolation, h 2 == 2963.145 KJ/kg A reheat steam cycle has 13,850 kpa throttle pressure at the turbine
At 2 Mpa and 460°C (53 == 54) 5 inlet and a 2800 Kpa reheat pressure, the throttle and reheat
h 3 == 3379.5 KJ/kg temperature of the steam is 540°C, condenser pressure is 3.4 Kpa,
53 == 7.3147
engine efficiency .of high and low pressure is 75% find the cycle
thermal efficiency
At 60~C (19.94 kpa) -Table 1
5r == 0.8312 h r == 25I.13
5fg == 7.0784 h fg == 2358.5 SOLUTION:

53 == 54 == 5r + X5rg At 13.85 Mpa and 540°C,


7.3147 == 0.8312+x(7.0784) hi == 3434 ..1 KJ/kg(interpolated)
x == 0.916 51 == 6.53553 KJlkg-OK(interpolated)
h, == 25I.13 + 0.916(2358.5) At 2.8 Mpa(Table 3) and 51 == 52 == 6.53553
h, == 2411.41 KJ/kg h 2 == 2974.914 KJ/kg
h, == h, at 60°C At 2.8 Mpa and 540°C,

l
108. Steam Cycles Steam Cycles 109
h, = 3548.5 KJlkg 51 = 7.2206 KJlkg-OK
5] = 7.3810 KJlkg-OK At 0.60 Mpa and s, = s- (Table 3)
At 0.0034 Mpa: SI = 52
5f = 0.384 hr = 109.84 7.1816 _ _.2957.2
Sfg ~~ 8.1488 hlg = 2439.5 7.2206 _ _ h2
v; = 0.0010032 7.2214 2978.2
53 = 54 = Sr + XSrg h2 = 2977.78 KJlkg
7.3810 = 0.384 + x(8.1488) At 31°C (Table I)
x = 0.8586 Sf = 0.4507 hr = 129.97
14 = 109.84 + 0.8586(2439.5) Srg = 7.9822 hrg = 2428.1
h, = 2204.426 KJlkg Vr == 0.0010046
h s = h, at 0.0034 Mpa s SI = S2 = S3 = Sf + XSfg
h, = 109.84 KJlkg 7.2206 = 0.4507 + x(7.9822)
h, = vs(P 6 - Ps) + h, x = 0.848
h, = 0.0010032(13850 - 3.4) + 109.84 h, = 129.97 + 0.848(242~.l)
h, = 123.73 KJlkg h, == 2189.30 KJlkg
Considering the engine efficiency: 14 == hr at 31°C
W T = (h, - h 2 )T] stl + (h, - 14)T]st2 14 = 129.97 KJlk 6
WT = (3434.1 - 2974.9)(0.75) + (3548.5 - 2204.5)(0.75) h, == V4(PS -P 4) + 14
WT = 1,352.4 KJlkg P, = Psat at 31°C
Wp = h, - h, P, == 0.004496 Mpa
Wp = 123.73 - 109.84 P, = 4.496 Kpa
Wp = 13.89 KJlkg h, = 0.0010046(600 - 4.496) + 129.97
QA = (hi - ~) + (h, - h2) h, == 130.56 KJlkg
QA = (3434.1 - 123.73) + (3548.5 - 2974.9) hs == hrat 0.6 Mpa
QA = 3,883.97 KJlkg h, == 670.56 KJlkg
WOe l = 1352.4 - 13.89 V6 == 0.0011006 m3lkg
Wnet = 1338.51 KJlkg h- = V6(P 7 -P 6) + h6
Efficiency = 1338.51/3883.97 h7 = 0.0011006(5400 - 600) + 670.56
Efficiency = 34.46% h- = 675.84 KJlkg
By heat balance in Regenerative heater
Steam Cycle - 14 mh, + (l-mjh, = 1~

h 6 - hs
Steam is delivered to turbine at 5.4 Mpa and 600°C. Before m=
h2 - hs
condensation at 31°C, steam is extracted for feedwater heating at 0.6
Mpa. For an ideal regenerative cycle, find the thermal efficiency. 67056 - 130.56
m
A. 23.45% C. 28.34% 2977.78-13056
B. 34.34% D. 44.14% m == 0.1896

SOLUTION:
QA == 1(h1 - h-)
QA == 1(3663.3 - 675.84)
e
QA == 2987.46 KJlkg
s
At 5.4 Mpa and 600°C(Tab1e 3) WT = 1(h1 - h2) + (I-m)(h2 - h3)
h, = 3663.3 KJlkg WT == 1(3663.3 - 2977.78) + (I - 0.1896)(2977.78 - 2189.30)
110 Steam Cycles Steam Cycles III

WT = 1324.504 KJ/kg Calculate the mass flow rate of subcooled liquid if steam flow rate is
QR = (1 - m)(h 3 -14) 0.865 kg/s.
QR = (1- 0.1896)(2189.30 - 129.97) Steam Properties are:
QR = 1668.88 KJ/kg At 7 bar, saturated vapor:
W PT = W P J + Wrz h g = 2763.5 KJ/kg
W PT = (1 - m)(h s -14) + l(h? - 11,;) At 7 bar and 25°C:
W PT = (1 - 0.1896)(130.56 - 129.97) + 1(675.84 - 670.56) h r = 105.5 KJ/kg
W PT = 5.758 KJ/kg At 7 bar, saturated liquid:
W Oe l = W T - W TP h, = 697.22 KJ/kg 7 bar
W Oe l = 1324.504 - 5.758 A. 2.725 C. 2.286 m,=O.865kg/s
W Oe l = 1318.746 KJ/kg B. 3.356 D. 3.948
TJ = Woell QA (Sat. liquid)
TJ = 1318.746/2987.46 SOLUTION: m, • 7 bar
• HEATER 7 bar,

TJ = 44.14%
Heat Absorbed
Efficiency = Heat Supplied
Steam Cycle - 15 (ME Rd. Apr. 1998)
m L (h 3 - h 2 )
A steam condenser receives 10 kg/s of steam with an enthalpy of 2570
Efficiency = m (h. - h )
KJ/kg. Steam condenses into a liquid and leaves with an enthalpy of s 3
160 KJ/kg. Cooling water passes through the condenser with
temperature increases from 13 degrees C to 24°C. Calculate the water m L (697.22 -105.5)
flow rate in kg/s. 0.90 = --"'---------
0.865(2763.5 - 697.22)
A. 533 C. 523
mL = 2.725 kg/s
B. 518 D. 528

i SOLUTION:

13°C

m=10kg/s
By heat balance in the condenser:

Heat rejected by steam = Heat absorbed by water


m s (hI - h 2) = mw c p (t 2 - t.)
Steam Cycle - 17 (ME Rd. Oct 1997)

Steam expands adiabatically in a turbine from 2000 kpa, 400°C to 400


_ 10(2570 - 160) = m w (4.187)(24 - 13) kpa, 250°C. What is the effectiveness of the process in percent
24°C m., = 523.2 kg/s assuming an atmospheric pressure uf 15°C. Neglect changes in kinetic
and potential energy.
Steam Properties are:
At 2000 Kpa and 400°C.
Steam Cycle - 16 (ME Rd. Apr. 1998) h = 3247.6 KJ/kg
s = 7.1271 KJ/kg-K
In an open feedwater heater for a steam plant, saturated steam at 7 At 400 Kpa and 250°C.
h = 2964.2 KJ/kg
bar is mixed with subcooled liquid at 7 bar and 25°C. Just enough
steam is supplied to ensure that the mixed steam leaving the heater s = 7.3789 KJ/kg-K
will be saturated liquid at 7 bar when heater efficiency is 90%. A. 82 C. 80
B. 84 D. 86
112 Steam Cycles Steam Cycles 113
SOLUTION:

2000Kpa
Steam Cycle - 19 (ME Rd. Oct. 1997)
Q = (h, - h 2 )
400°C
Q = 3247.6 - 2964.2 Steam enters the turbine of a cogeneration plant at 7.0 Mpa and
Q = 283.4 KJlkg 500°C. Steam at a flow rate of7.6 kg/s is extracted from the turbine at
Qs = T (S2 - s.) 600 Kpa pressure for process heating.• The remaining steam continues
Qs = ('" + 273)(7.3789 - 7.127 400Kpa to expand to 10 kpa. The recovered condensates are pumped back to
Qs = 72.5 KJlkg 250°C the boiler. The mass flow rate of steam that enters the turbine is 30
283.4 kg/so Calculate the cogeneration efficiency in percent.
Effectiveness =
283.4 + 72.5 Steam properties:
Effectiveness = 79.60% At 7.0 Mpa and 500°C:
s h = 3410.3
7Mpa
s = 6.7976 500°C
Steam Cycle - 18 (ME Rd. Oct. 1997) At 600 Kpa: CD
h r = 670.56 m.=30kg/s
Steam enters the superheater of a boiler at a pressure of 25 bar and h rg = 2086.3 e
dryness of 0.98 and leaves at the same pressure at a temperature of Sf = 1.9312

370°C. Calculate the heat energy supplied per kg of steam supplied in Srg = 4.8288
the superheater. At 10 Kpa:
Steam Properties: h, = 191.83
At 25 bar and 370°C: h rg = 2392.8 Q)
h = 3]71.8 KJIkg Sr = 0.6493

At 25 bar Srg = 7.5009


s
hf = 962.11 KJIkg A. 60 C. 65
hfg = 1841.0 KJ/kg B. 50 D. 5'5
A. 407.46 C. 405.51
B. 408.57 D. 406.54 SOLUTION:

SOLUTION: 370°C WT = mj(h l - h 2) + mih 2 - h,)

h = hr + xhrg
J 25~ar 51
6.797
= 52 = 5r+
=
X5rg
1.9312 + X2 (4.8288)
X2 = 1.0
hi = 962.11 + 0.98(1841.0) h2 = hr + X2 hig
BOILER h- =.670.56 + 1.0(2086.3)
Q 25bar.
h, = 2766.3 KJlkg h 2 = 2756.86 KJ/kg
(x=98%)
Sj = 52 = :>3 = Sf + X3 Sig
Q = h2 - h, 6.7976 = 0.6493 + x3(7.5009)
x3=0.8196
Q = 3171.8 - 2766.3 h, = hfJ + xh fg
h, = 191.83 + 0.8196(2392.8)
Q = 405.5 KJ/kg .. h, = 2152.96 KJlkg
WT = 30(3410.3 - 2756.89) + (30 - 7.6)(2756.86 ·2152.96)
WT = 33,129.66 kw
114 Steam Cycles
Steam Cycles 115
QR = (rn. - m2l (h, - 14)
h, = hc at 10 Kpa
14 = 191.83 KJ/kg Steam Cycle - 21
QR = (30 - 7.6)(2152.96 - 191.83)
QR = 43,929.312 Kw Pump work of Rankine cycle is 15 KJ/kg. Density of water entering
QA = ml(h l-hc4) the pump is 958 kg/rn", If condenser pressure is 100 Kpa, what is the
QA = 30(3410.3 - 191.83) pressure at the entrance of tlie turbine?
QA = 96,554.1 Kw A. 14.47 Mpa C. 15.67 Mpa
B. 20.48 Mpa D. 17.77 Mpa
33,129.66+43,929.312
Cogeneration efficiency = - - - - - - - - SOLUTION:
96,554.1
Cogeneration efficiency = 79.81% W p = v(P l - P 4 )
(No exact answer in the choices) W p = (l/w)(P 1 - P4 )
15 = (I/958)(P[ - 100)
P = 14,470 Kpa
P = 14.47 Mpa
Steam Cycle - 20 (ME Bd. Oct. 1997)

A heat exchanger was installed purposely to COOl U.50 kg of gas per


second. Molecular weight is 28 and k = 1.32. The gas is cooled from Steam Cycle - 22
150°C to 80°e. Water is available at the rate of 0.30 kgls and at a
temperature of rz-c, Calculate the exit temperature of the water in In a Rankine cycle the turbine work is 1,000 KJ/kg and pump work of
"C. A. 48 C. 46 13 KJ/kg. If heat generated by generator is 2800 KJ/kg, what is the
B. 42 D. 44 efficiency of the cycle?
A. 35.25% C. 38.65%
B. 40.75% D. 30.25%
SOLUTION:

r~
SOLUTION:
R = 8.314/28
R = 0.2969 KJ/kg-K 1000-13
kR 14. 180°C 11t =
cp = 2800
HEAT
k-I ::l t:XCHANGE
11t = 35.25%
1.32(0.2969)
cp = I • ~ 1SOoC
1.32- 1
cp = 1.2247 KJ/kg-K ~

By heat balance:
t,,=12 c
Q

t m w=O.3kgls
Steam Cycle - 23

Qgain = Qloss
In a Reheat power plant the difference in enthalpy at the entrance and
m; cp (t, - to) = fig cpg (t2 - t l ) exit is 550 KJ/kg for first stage and second stage is 750 KJ/kg. If both
(0.30)(4. 187)(tb - 12) = (0.5)( 1.2247)(150 - 80) stages has an efficiency of92% and heat added to boiler is 3,000
t b = 46.125"C KJ/kg. Determine the plant cycle efficiency neglecting the pump work.
A. 30% C. 40%
B. 35% D. 45%
Steam Cycles 117
116 Steam Cycles
water required because of high pressure exit velocity if the steam flow
SOLUTION: rate is 38 kg/s and the cooling water temperature rise is iz'c with an
inlet condition of 30°C.
A. 11.23 kg/s C. 13.23 kg/s
W n + Wn B. 17.23 kg/s D. 21.78 kg/s
llT =
QA
0.92(550) + 0.92(750) SOLUTION:
llT =
3000 If the exit velocity is not considered: 38 kg1s
15Kpa (90%quality)
llT = 39.87% At 15 Kpa:
h, = 225.94 KJ/kg
h fg = 2373.1 KJ/kg 30°C
hi = h f + X h fg
Steam Cycle - 24 h, = 225.9 + 0.9(2373.1)
hi = 2361.73 KJ/kg
An adiabatic feed pump in a steam cycle delivers water to a steam h2 = h, at 15 kpa m.
generator at a temperature of 200°C and a pressure of 10 Mpa. The h2 = 225.94 KJ/kg h,
water enters the pump as a saturated liquid at 180°C. If the power
supplied to the pump is 75 kw, determine the mass flow rate. Qgain = Qloss
A. 6.23 kg/s C. 7.39 kg/s m; (4.187)(12) = 38(2361.73 - 225.94)
B. 8.34 kg/s D. 9.12 kg/s m; = 1615.38 kg/s

If exit velocity is considered:


SOLUTION: (1)(240)2
h, = 2361.73 + [(1/ 2) ]

At 180°C:
~.

10 Mpa
p...
200"C ~ 180"C
h, = 2390.53 KJ/kg
1000

h, = 763.22 KJ/kg
vr =
3
0.0011274 m /kg 75 Kw IDw (4.187)(12) = 38(2390.53 - 225.94)
P sat = 1.0021 Mpa m; = 1637.1 kg/s

h2 = vr (P2 - PI) + hi Mass difference = 1637.1 - 1615.38


h2 = 0.0011274(10,000 - 1002.1) + 763.22 Mass difference = 21.78 kg/s
h2 = 773.36 KJ/kg

W p = m s (h2 - hi)
75 = m s(773.36 - 763.22) Steam Cycle - 26
rn, = 7.39 kg/s
A steam generator has an exit enthalpy of 3195.7 KJfkg at the rate of
10 kg/s. The enthalpy available at the turbine inlet is 3000 KJfkg.
Steam Cycle - 25 Determine the heat lost between boiler outlet and turbine inlet.
A.1957kW C.1873kw
A condenser receives steam from a turbine at 15 kpa, 90% quality, B. -1957 kw D. -1873 kw
and with a velocity of 240 m/s. Determine the increase in circulating
118 Steam Cycles Steam Cycles 119
SOLUTION: h, = 289.23 + 0.0010223 (2000 - 30)
h, = 291.24 KJ/kg
By mass balance in the heater:
Q = m (h z - h.) Assume supply steam = Ikg
Q = 10(3000 - 3195.7) m\(h z) -t (I - m.jh, = I h,
Q=-1957kw h, = h-at 2 Mpa
h6 = 908.79 KJ/kg
ml(2902.5) + (I - m l)(291.25) = 1(908.79)
Steam Cycle - 27 m, = 0.2364 kg extracted steam/kg supply

A Rankine cycle has a turbine unit with available enthalpy of 800


KJ/kg. The pump has also 10 KJ/kg energy available. Find the net
cycle output of the plant if mass now rate is 5 kg/so Steam Cycle - 29
A. 2619 kw C. 8745 kw
B. 3950 kw D. 4234 kw An adiabatic turbine in a steam generating plant receives steam at a
pressure of 7.0 Mpa and 550°C and exhausts at 20 kpa. The turbine
inlet is 3 m higher than the turbine exit, the inlet steam velocity is 15
SOLUTION: m/s and the exit is 300 m/s. Calculate the turbine work in KJ/kg.
A. 1297.45 C. 1093.45
B. 1197.10 D. 1823.45
W net = m, (W T - W p)
W net = 5(800 - 10) SOLUTION:

I C"--
W net = 3950 kw
At 7.0 Mpa and 550°C: 550°C
h = 3530.9 KJ/kg . V'==15mi~ ~

s = 6.9486 KJ/kg-OK
Steam Cycle - 28 At 20 kpa:
~"" w
h, ~~ 251 ,4 .~ L'-"il • •
In a Regenerative cycle, the steam is extracted from the turbine at 2
Mpa and 250°C for feedwater heating and it is mixed with condenser
exit at 30 kpa after pumping. Find the fraction of vapor extracted
hhg - 23:J8.3
Sr = 0.8320
Srg = 7.0766 ~

1
from the turbine. s = Sr + XSrg \61 20 Kpa
A. 0.23464 C. 0.5632 6.9486 = 0.8320 + x(7.70766)
B. 0.19338 D. 0.3855 x = 0.864 V,= 300 m/s
h, = 251.4 + (0.864)(2358.3)
SOLUTION: h2 = 2288.9 KJ/kg
2 Mpa
At 2 Mpa and 250°C: m, T 250°C
W T = (hi - h2 ) + 1/2 m (v/ - v/) + (PEl - PEz)
hz = 2902.5 KJ/kg
At 30 kpa,
h, =
Vr =
289.23 KJ/kg
0.0010223 m3/kg
-
30 Kpa
1
HEATER 50 Kpa
1-m, W T = (3530 -2288.9) +
15
2-300 2

2(1000)
+
3(1 x9.81)
1000
hs = 14 + V4(PS - P4) WT = 1197.IOKJ/kg
120 Steam Cycles Steam Cycles 121

Steam Cycle - 30 Steam Cycle - 31

A steam power plant operates on the Rankine cycle. The steam enters A Carnot cycle uses steam as the working substance and operates
the turbine at 7 Mpa and 550°C with a velocity of 30 m/s. It discharges between pressures of. 7 Mpa and 7 kpa. Determine the cycle thermal
to the condenser at 20 kpa with a velocity of 90 m/s. Calculate the net efficiency.
work in kw for a flow of 37.8 kg/s. A. 44.17% C. 34.23%
A. 23.23 Mw C. 34.22 Mw B. 54.23% D. 59.44%
B. 53.34 Mw D. 46.54 Mw
SOLUTION:

SOLUTION: At 7.0 Mpa:


t = 285.88°C
At 7 kpa:
At 7.0 Mpa and 550°C: t = 39°C T
2 285.B80C 1
. h = 3530.9 KJ/kg T H = 285.88 + 273

At 20 kpa:
S = 6.9486 KJ/kg-OK

hf = 251.4
w
T H = 558.88°K
T L = 39 + 273
TL = 312
[;]
3 5,-5, 4
hhg = 2358.3 TI-l -TL
Sf = 0.8320 e =
5
Sfg = 7.0766 TI-l
S=Sf+XSfg 558.88 - 312
e =
6.9486 = 0.8320 + x(7.70766) 558.88
x = 0.864 e = 44.17%
h 2 = 251.4+(0.864)(2358.3)

h2= 2288.9 KJ/kg Steam Cycle - 32


h,= h f at 20 kpa
h]= 251.4 KJ/kg A supercritical power plant generates steam at 25 Mpa and 560"C.
v]= 0.001017 m3 /k g The condenser pressure is 7.0 kpa.. Determine the exit quality of steam
h,= h, + V3 (P4 - P 3 ) if it expands through a turbine in this power plant.
h,= 251.4 + 0.0010 17(7000 - 20) A. 45.66% +
C. 56.56%
h,= 258.5 KJ/kg B. 68.45% rI D. 74.26% '1'
W25MPa
30 2 _ 90 2
WT = (3530.9 ·2288.9) + - - - 560°C
SOLUTION:
2(1000)
WT = 1238.4 KJ/kg
WOe l = 1238.4 - (258.5 - 251.4)
WOe l = 1231.3 KJ/kg (37.8 kg/s)
At 25 Mpa and 580°C: / 7Kpa r
WOel = 46,543.19 kw /- @
h = 3430.5
WOe l = 46.54 Mw S = 6.2897
5
122 Steam Cycles Boilers 123

At 7 kpa:
Sc = 0.5592
BOILERS
SCg = 7.7167
S = Sc + XSCg
6.2897 = 0.5592 + x(7.7167)
x = 74.26% Boiler - 1

The heating surface area of water tube boiler is 200 m2 , what is the
equivalent rated boiler horsepower?
A. 217 Hp C. 200 Hp
Steam Cycle - 33
B. 2365.93 Up D. 219.78 Hp
Steam enters a turbine at 1.4 Mpa and 320?C. The turbine internal
efficiency is 70%, and the total requirement is 800 kw. The exhaust is SOLUTION:
to the back pressure system, maintained at 175 kpa. Find the steam
Rated Boiler horsepower = RS.!0.91
flow rate.
C. 3.23 kg/s Rated Boiler horsepower = 200/0.91
A. 2.62 kg/s
D. 5.34 kg/s Rated Boiler horsepower = 219.78 Hp
B. 4.23 kg/s

SOLUTION:
Boiler - 2
W=800kw
At 1.4 Mpa and 320°C:
h, = 3084.3
--. The rated boiler horsepower of a fire tube boiler is 500 Up. What is
SI = 7.0287
the heating surface area of the boiler?
A. 500 m2 C. 400 m
2
At 175 Kpa: 2 2
B. 300 m D. 550 m
Sc = 1.4849
SCg = 5.6868
SOLUTION:
hc = 489.99
V,= 90 m/s
hcg = 2213.6
Rated Boiler horsepower = H. S:! 1.1
Solving for the quality:
7.0287 = 1.4849 + x (5.6868)
x = 0.9748 500 = H.S.! 1.1
h 2 = 489.99 + 0.9748(2213.6) U.S, = 550m 2
h 2 = 2647.93 KJ/kg

WT = IDs (hi - h 2)(llT)


Boiler - 3
800 = ills (3084.3 - 2647.93)(0.70)
ills = 2.62 kg/s
A water tube boiler has a heating surface area of 500 m2 • For a
developed boiler hp of 825. Determine the percent rating of the boiler.

A. 120.15% C. 150.15%
B. 160.15% D. 300.15%
Boilers 125
124 Boilers
Rated boiler Hp = 281.32 Hp
SOLUTION' Dev. Boiler Hp
%R = xIOO%
Rated Boiler Hp
Rated boiler horsepower = 500/0.91
Dev. Boiler Hp
Rated boiler horsepower = 549.45 hp 2 = -----
Dev. Boiler Hp 281.32
%R = xIOO% Developed Boiler Hp = 562.64 Hp
Rated Boiler Hp
ills (2257 x 1.08)
%R = 825/549.45 x 100% 562.64 = - - - - - " - - - - -
%R = 150.15% 35,322
ID, = 8153.02 kg/hr

Boiler - 4
Boiler - 6
The factor of evaporation of a boiler is 1.1 and a steam rate of 0.79
kg/sec. What is the developed boiler horsepower? The actual specific evaporation of a certain boiler is 10. Factor of
A. 300 C. 869 evaporation is 1.05. If the heating value of fuel is 30,000 KJ/kg, find
B. 200 D. 250 the boiler efficiency.
A. 60% C. 70%
SOLUTION: B.65% D.79%

h, - h F SOLUTION:
FE = ---
2257 ills(hs-h F )
h, - h, = 2257 x 1.1 l1b =
ills(hs-h F ) IDrQ h
Developed Boiler hp
35,322 IDs
10
(0.79 x 3600)(2257 x 1.1) ID r
Developed Boiler hp
35,322 1O(2257xI.05)
l1b =
Developed Boiler hp 199.89 Hp 30,000
l1b = 79%
Boiler - 5

The percent rating of water tube boiler is 200%, factor of evaporation Boiler - 7
2
of 1.08 and heating surface area is 256 m • Find the rate of
evaporation. The AS ME evaporation units of h boiler is 24.827,000 KJ/hr. The
A. 8,153.02 kg/hr B. 7,200 kg/hr boiler auxiliaries consumes 1.5 MW. What is the net boiler efficiency if
B. 5,153.02 kg/hr D. 8,500.46 kg/hr the heat generated by the fuel is 30,000 KJ/hr?
A. 64.75% C. 62.76%
SOLUTION: B. 68.94% D. 68.54%

Rated boiler Hp = 256/0.91


127
lLO Boilers Boilers

SOLUTiON. SOLUTiON:

ms(h s - h F ) - Boiler Aux. Theo A/F = 11.5C + 345(H - 0/8) T 4.3S


11ner = Theo. NF = 11.5(0.705) + 34.5(0.045 - 0.06/8) + 4.3(0.03)
mrQ" Theo. A/F = 9.53 kg air/kg fuel
(24,827 ,000 I 3600) - (L5xl 000) Actual A/F = 9.53(1.3)
llne'
(30,000,000 I 3600) Actual A/F = 12.389 kg air/kg fuel
11net ,= 64.75%
h, - h F
FE~ --
2257
(h, - h F ) = FE x 2257
(h, - h f ) = 1.1 x 2257
Boiler - 8 Qh = 33,820C + 144,212(H - 0/8) + 9304S, Kl/kg
Qh = 33,820(0.705) + 144,212(0.045 - 0.06/8) + 9,304(0.03)
A 100,000 kg of coal supplied two boilers. One has a capacity of 150 Qh = 29,930 KJ/kg
kg/hr. How many days to consume the available fuel if the other boiler
consumes 200 kg/hr? m s (h s - hF)
A. S days C. 15 days llb =
mrQ h
B. 7 days D. 12 days 175,000( 1.1 x 2257)
0.70 = ------
SOLUTION: m r (29,930)
m, = 21,018.456 kglhr
m mrl + mf2
= rna = 21,018456(12389)
m = ISO + 200 rna = 260,397.651 kg/hr
m = 350 kg/hr PV = mRT
101.325(V) = 260,397.651(0.287)(15.6 ~ 273)
3/hr
No. of days = 100,00/350 V = 212,830 m
No. of days = 285.71 hrs
No. of days = 11.9 days

Boiler - 10 (ME Bd. Apr. 1981)


Boiler - 9 (ME Bd. Apr. 1981)
The following coal has the following ultimate analysis by weight:
The following coal has the following ultimate analysis by weight: C = 70.5% Hz = 4.5% Oz = 6.0% N z = 1.0%
C = 70.5% Hz = 4.5% O, = 6.0% N z = 1.0% S = 3.0% Ash = 11 % Moisture = 4%
S = 3.0% Ash = 11 % Moisture = 4% A stocker fired boiler of 175,000 kglhr steaming capacity uses this
3/hr
A stocker fired boiler of 175,000 kg/hr steaming capacity uses this coal as fuel. Volume of air in m with air at 60°F and 14.7 psia
3/hr
coal as fuel. Calculate volume of air in m with air at 60°F and 14.7 pressure. Weight in metric tons of coal needed for 24 hours operation
psia pressure if boiler efficiency is 70% and FE = 1.10. at rated capacity if boiler efficiency is 70% and FE = 1.10.
3/hr
A. 212,830 m C. 213,830 rrr'rhr A. 503.443 Mtons C. 502443 Mtons
B.. 2 J 5,830 mY/hr D.. 214.830 m'zhr B. 508.443 Mtons D. 504,443 Mtons
128 Boilers
Boilers 129
SOLUTION:
(h, - h-) = 1.2 x 2257
Theo. A/F 1I5e i 34.5(H - 0/8) + 4.3S
Theo. A/F- 115(0.705) j 34.5(0.045 - 0.06/8) + 4.3(0.03) m s ( h s - h f- )
Theo AiF 953 kg air/kg fuel
llb = mrQ"
ActuaiA/F = 9.53(13)
Actual A/F·~ 12.389 kg air/kg fuel
1591( 1.2 x 2257)
0.65 = - - - - - -
h, - h F rn f ( 7590 x 4.l 87) 1591 kg/h B1
FE = - -
2257 mfI= 208.605 kg/hr
(h, - hr ) = FE x 2257:
.m Boiler#1
(h, - hr) = 1.1 x 2257 For Boiler No.2: 911,)00 kg
On = 33.820C + 144,212(H - 0/8) + 9304S, KJrKg (hs-hF ) = FE x 2257
Oh = 33.820(0.705) -i- 144,212(0.045 - 0.06/8) + 9,304(0.03) (h, - hr)= 1.15 x 2257
Bunker
Oil = 29.930 KJ/kg ms(hs-h F ) B2
ms\h s - h F ) llb = 1364kgfh
llb = mrQ h
mrQ h 1364(1.15 x 2257) Boiler#2
0.60 = - - - ' - - - - -
175,000(1.1x2257)
0.70 = m r (7590 x 4.187)
rn r (29,930) mf2 = 185.673 kg!hr
rn, = 21,018.456 kg/hr rn- = total fuel consumed
mT = rnfl + rnf2
21,018.456(24) m- = 208.605 + 185.673
Coal needed in 24 hrs = m- = 394.278 kg!hr
1000 .
Coal needed in 24 hrs = 504.443 Mtons No. of hours = 91,000/394.278
No. of hours = 230.8 hrs

Boiler - 11 (ME Bd. Oct. 1982)


Boiler - 12 (ME Bd. Oct. 1984)
Two boilers are operating steadily on 91,000 kg of coal contained in a
bunker. One boiler is producing 1591 kg of steam per hour at 1.2 A steam generating plant consisting of a boiler, an economizer and
factor of evaporation and an efficiency of 65% and another boiler superheater generates superheated steam at the rate of 50 tons /hr,
produces 1364 kg of steam per hour at 1.15 factor of evaporation and Feed water enters the boiler at 5 Mpa and 120°C. Steam leaves the
an efficiency of 60%. How many hours will the coal in the bunker run superheater at 4.5 Mpa and 320°C. If the coal used has a heating
the boilers if the heating value of coal is 7590 Kcal/kg? value of 30,000 KJ/kg, calculate the number of tons of coal fired per
A. 220.8 hrs C. 230.8 hrs hour for a gross efficiency of 85%.
B. 256.2 hrs D. 453.3 hrs A.4.89 C. 5.34
SOLUTION: B. 6.34 D. 45.5
SOLUTION:
For Boliler No. I:
(h, - h r) = FE x 2257 At 4.5 Mpa and 320°C(Table 3),
h, = 3000.6 KJ/kg
Boilers 131
130 Boilers
At 5 Mpa and 120°C(Table 4), coal that could be used in order to ensure the generation of required
h F = 507.09 KJ/kg steam.
A. 28,464 C. 23,556
B. 29,977 D. 30,976
ms(hs-h F ) 11kg/c:m'
_ m, =50000kglhr
llb = mrQ h,
h SOLUTION:
h.
(50)(3000.6 - 507.09) p = 11 kg/cm 2 x 101.325/1.033
0.85 = BOILER
_,I r (30,000)
P = 1079.1 Kpa
p = 1.0791 Mpa
m, = 4.889 tons/hr a.,..30000kJ/k
m, h, From Table 2, hs =h g at 1.0791 Mpa
h, = 2781 KJ/kg
From Table 1, hF = hr at 80°C
Boiler - 13 (ME Bd. Oct. 1986) hF = 334.91 KJ/kg

A water tube boiler has a capacity of 1000 kg/hr of steam. The factor
of evaporation is 1.3, boiler rating is 200%, boiler efficiency is 5%,
ms(h s - h F )
heating surface area is 0.91 m 2/boiler Up, and the heating value of llb =
fuel is 18,400 Kcal/kg. The total coal available in the bunker is 50,000 mrQ h
kg. Determine total number of hours to consume the available fuel 50,000(278-1 - 334.91)
A. 533.45 C. 634.34 0.85 = -------
4,800Q h
B. 743.12 D. 853.26
o, = 29,977 KJ/kg
SOLUTION:

(h, - h F) = FE x 2257
(h, - h F)= 1.3 x 2257 Boiler - 15 (ME Bd. Apr. 1984)
ms(hs-h F )
llb = A boiler operating at 11 kg/cm 2 is required to generate a minimum of
aC.
mrQ h 50,000 kg/hr of saturated steam. Feed water enters the boiler at 80
1000(1.3 x 2257) The furnace is designed to fire coal at an average rate 4,800 kg/hr and
0.65 = ----'--------'----
m r (18,400x4.l87) boiler efficiency is 85%. Compute the developed boiler horsepower.
A. 3462.56 hp C. 2345.67 hp
m, = 58.592 kglhr
B. 1234.56 hp D. 4233.34 hp ~'m.
No. of hours = 50,000/58.592
No. of hours = 853.36 hrs SOLUTION:

P = 11 kg/crni x 101.325/1.033
Boiler - 14 (ME Bd. Apr. 1984)
P = 1079.1 Kpa
P = 1.0791 Mpa
A boiler operating at 11 kgicm 1 is required to generate a minimum of
From Table 2, h, = h g at 1.0791 Mpa
50,000 kg/hr of saturated steam. Feed water enters the boiler at 80°C. h, = 2781 KJlkg
The furnace is designed to fire coal at an average rate 4,800 kg/hr and
From Table 1, h F = h, at 80°C
boiler efficiency is 85%. Compute the minimum heating value of local
Boilers 133
132 Boilers
At 145°C: h, 610.63
z;

hF - 334.9i ~.J!h:g
A.65 C. 88
B. 95 D. 78
50,000(2781- 334.91)
Developed Boiler Hp
35,322 SOLUTION:
Developed Boiler Hp == 3,462.56 Hp
ms(hs-h F )
Tlb
mfQ h

Boiler - 16
23.5(3195.7 - 610.63)
Tlb =
A steam boiler generating 7.1 kg/s of 4.137. Mpa, 426.7°C steam is 2.75(25,102)
continuously blown at the rate of 0.31 kg/sec. Feed water enters the Tlb = 88%
economizer at 148.9 0C. The furnace burns 0.75 kg coal/sec of 30,470.6
KJ/kg higher heating value. Calculate the overall thermal efficiency of
steam boiler.
A. 76.34% C. 82.78%
'* Boiler - 18 (ME Bd. Apr. 1997)

B. 84.23% D. 88.34% A steam boiler on a test generates 885,000 Ib of steam in a 4-hollr


period. The average steam pressure is 400 psia, the average steam
temperature is 700°F, and the average temperature of the feedwater
SOLUTION: supplied to the boiler is 280°F. If the boiler efficiency for the period is
82.5%, and if the coal has a heating value of 13,850 Btu/lb as fired,
From Steam Table: find the average amount of coal burned in short tons per hour.
h, = 629.87 KJ/kg At 400 psia and 700°F, h, == 1362.7 Btu/lb
h, = 3274.1 KJ/kg At 2BO°F, hI = 249.1 Btu/lb
h, = 109702 KJ/kg A. 9.84 short tons per hour
7.1(3274.1) + 0.31(1097.02) - 7.41(629.87) B. 10.75 short tons per hour
Tlb C. 12.05 short tons per hour
0.75(30,470.6)
D. I 1.45 short tons per hour
Tlb == 82.78%
SOLUTION:

Boiler - 17 rn, = 885,000/4


m, = 221,250 lb/hr
23.5 kg of steam per second at 5 Mpa and 400°C is produced by a ms(hs-h F ) I400 p.sia
700~F m, = 885,0001b
steam generator. The feedwater enters the economizer at 145°C and
leaves at 205°C. The steam leaves the boiler drum with a quality of
Tlb =
mfQ h
r.-....:..--.
98%. The unit consumes 2.75 kg of coal per second as received having 221,250(1,362.7 - 249.1)
0.825
a heating value of 25,102 KJ/kg. What would be the overall efficiency m r (13,850) BOILER
of the unit in percent? mf= 21,563 Ib/hr
Steam properties: mf= 2 1,563/2000 Q. = 13,850 Btull~ I 280°F
At 5 Mpa and 400°C: h = 3195.7 KJ/kg mf- 10.78 short tons per hr
At 5 Mpa: h; = 1154.23 hfg = 1640.1
At 205°C: h, = 875.04
134 Boilers Steam Engine 135

Boiler-19
STEAM ENGINE
A steam boiler generates 401,430 kg of steam in a 4-hour period. The
steam pressure is 2750 kpa and 370°C. The temperature of the water
supplied to the steam generator is 138°C. If the steam generator
efficiency is 82.5% and the coal has a heating value of 32,200 KJ/kg, Steam Engine - 1
find the average amount of coal burned per hour.
A. 9771 C. 9563 A steam engine have 10% brake thermal efficiency and delivers 750
B. 8734 D. 7354 kglhr steam. The enthalpy of engine entrance is 2800 KJ/kg and
condenser exit is 450 KJ/kg. Determine the brake power of the engine.
A. 46 KW C. 49 KW
SOLUTION: B. 47 KW D. 48 KW

SOLUTION:
At 2750 kpa and 370°C:
h, = 3166.9 KJ/kg Brake Power
At 138°C: lltb =
hF = hf at 138°C ID s ( h s - hf2 )
hF = 580.54 KJ/kg Brake Power
0.10 = --------
401,430 (750 /3600X2800 - 450)
IDs
4 x3600 Brake Power = 48.96 KW
IDs = 27.877 kg/s
IDs(hs-h F )
llb =
IDFQ h Steam Engine - 2
27.877(3166.9 - 580.54)
0.825
ID F (32,200)
The indicated efficiency of a steam engine is 60%. The engine entrance
is 2700 KJ/kg and exit is 2000 KJ/kg. if steam consumption is 800
IDF = 2.714 kg/s (3600)
kg/hr and mechanical efficiency is 90%, what is the brake power of the
IDF = 9770.77 kg/hr
engine?
A. 55 KW C. 65 KW
B. 84 KW D.70KW

SOLUTION:

Indicated Power
llei
ffi s ( h \ - h2 )
Indicated Power
0.60 =
(800/3600)(2700 - 2000)
136 Steam Engine Steam Engine 137

Indicate d Power 93.33 KW V 0 = 2[11:/4 (030):' (045) (220/60 )]


11m = BP/IP V o = 0.23326 rnzsec
0.90 = BP/93.3 3 Indicate d Power .~ 392.4(0 .233263 3) = 91.53 KW
BP = 84 KW Indicate d Power = 91.53/0 .746
Indicate d Power = 122.7 Hp

Steam Engine - 3

g at
A steam engine has bore and stroke of 300 mm x 420 mm runnin Steam Engine - 5
ed pressur e of the engine is 400 Kpa.
250 rpm has mean indicat
Determ ine the indicat ed power. A steam engine develop s 60 Bhp with dry saturat ed steam at
1034.25
A. 100 KW C. 65 KW consum ption is 736.36
Kpa absolut e and exhaus t at 124.11 Kpa. Steam
B 50KW D. 99 KW engine efficien cy based on 90%
kg/hr. Calcula te the indicat ed
mechan ical efficien cy.
SOLUT ION A. 34.23% C. 45.23%
B. 54.23% D. 66.74%
2
VD = 2[11:/4 D L N]
VD = 2[rc/4 (0.3)2 (0.42)(2 50/60)]
V::J = 0.247 m' SOLUT ION:
Indicate d Power = P mi X V 0
Indicate d Power = 400 x 0.247 hi = h g at 1.03425 Mpa
Indicate d Power = 98.96 KW hi = 2779.4 KJ/kg (interpo lated)
S I = Sg at 1.03425 Mpa
5\ = 6.5748 KJlkg-O K(interp oiated)

At 0.12411 Mpa:(B y interpol ation)


Sf = 1.17165 h f = 443.43
Steam Engine - 4
Sfg = 5.9152 h fg = 2241.56
The crank shaft of a double acting steam engine rotates at
220 rpm. SI =. S2 = Sf + XSfg

and stroke of the steam engine is 300 mm x 450 mm, and the 6.5748 = 1.37165 j x(5.915 2)
The bore
, Find the x = 0.8796
mean effectiv e pressur e acting upon the piston is 4 kg/ern"
indicat ed horsep ower develop ed in the cylinde r. h2 = 443.43 + 0.8796( 2241.56 )
A. 122.7 hp C. 143.2 hp h 2 = 241.5.16 Kl/kg
B. 110.3 hp D. 176.3 hp
Indicate d Power = (60/0.9 ) x 07'+6
Indicate d Power = 49.733 KW
SOLUT ION:
Indicate d Power
VD 11el =
Indicate d Power Pm' X
m,(h j -0 h2 )
49.733
Pm' = 4 kg/em" x 101.325 /1.033 11"
2
Pm' = 3924 KN/m (736.36 /3600)( 2779.4 - 24\51)
VD = 2[11:/4 [)2 L N] rtei = 66.74%
138 Steam Engine Steam Engine 139

indicated mean effective pressure is 600 Kpa, determine brake thermal


Steam Engme - 6
efficiency.
A. 23.34% C. 14.66%
A steam engine develops 60 Bhp with dry saturated steam at 1034.25 B. 18.34% D. 27.34%
Kpa absolute and exhaust at 124.]] Kpa. Steam consumption is 736.36
kg/hr. Calculate the thermal efficiency of equivalent Rankine engine. SOLUTION:
A. 15.59% C. 12.45%
B 34.23% D. 21.34% VD = 2[71:/4 (0.35)2 (0.45) (260/60)]
VD = 0.37522 m 3 /sec
SOLUTION:
Indicated Power = Prru X V D
Indicated Power = 600 x 0.37522
hi = hg at 1.03425 Mpa Indicated Power = 225.13 KW
hi = 2779.4 KJ/kg (interpolated)
SI = Sg at 1.03425 Mpa
Brake Power = Indicated Power(Y]rn)
SI = 6.5748 KJlkg-OK(imerpolated) Brake Power = 225.13(0.88)
Brake Power = 198.11 KW
At O. ] 2411 Mpa:(By interpolation)
Sf = 1.37165 h, = 443.43
At 2 Mpa and 230°C(Table 3)
Sfg = 5.9152 h rg = 2241.56 h, = 2849.6
S 1"= S: = Sf + XSfg SI = 6.4423
6.5748 = 1.37165 + x(5.9152)
x = 0.8796
At 0.1 Mpa:
s, = 1.3026 h, = 417.46
h, ~ 443.43 + 0..8 796(2241.56)
Sfg = 6.0568 h rg = 2258
hz = 2415.16 KJ/kg
hf2 = h, at 0.12411 Mpa
hf2 = 417.46 KJ/kg
1,'2 = 443.43 KJ/kg
Brake Power
m s (hI - h 2) Y]tb
Y]R = m s (h J - h f2 )
m s (h l - h f 2 )
2779.4 - 2415.1
Y]R = ----- 198.11
2779.4 - 443.43 Y]tb =
(2000/3600)(2849.6 - 417.46)
Y]R = 15.59%
Y]tb = 14.66%

:!/.:: Steam Engine - 7


Steam Engine - 8
A 350 mm x 450 mm engine running at 260 rpm has an entrance steam
A 350 mm x 450 mm engine running at 260 rpm has an entrance steam
condition of 2 Mpa and BO°C and exit at 0.] Mpa. The steam
consumption is 2,000 kg/hr and mechanical efficiency is 88%. If condition of 2 Mpa and BO°C and exit at 0.1 Mpa. The steam
consumption is 2,000 kg/hr and mechanical efficiency is 88%. If

l
140 Steam Engine 14'1
Steam Engine
SOLUTION:
indicated mean effective pressure is 600 Kpa, determine indicated
thermal efficiency.
A. 16.66% C. 12.34%
VD = piston displacement
B. 34.23% D. 21.23%
VD = 2[(71:/4)D 2LN]
SOLUTION:
VD = 2[(71:/4)(10/12)2(12/12)(300)]
V D = 2[rc/4 (0.35)2 (0.45) (260/60)]
VD = 327.25 felmin
V D = 0.37522 m 31sec
Indicated Power P rru V D
Indicated Power = Prru X VD
Indicated Power = 600 x 0.37522
Indicated Power = 225.13KW Indicated Power = (120)(144)(327.25)
33,000
At 2 Mpa and 230°C(Table 3)
hi = 2849.6 Indicated Power = 171.40 hp
Sl = 6.4423

At 0.1 Mpa:
Sf = 1.3026 hf = 417.46
Sfg = 6.0568 hfg = 2258

hf2 = 417.46 KJ/kg

Indicated Power
11tl
m s ( h J - h f2 )

225.13
11 ti
(2000/3600)(2849.6 - 417.46)
116 16.66%

-t Steam Engine - 9 (ME Bd. Apr. 1997)

Steam is admitted to the cylinder of an engine in such a manner that


the average pressure is 120 psi. The diameter of thepiston is 10" and
the length of stroke is 12". What is the engine when it is making 300
rpm?
A. 171.5 C. 173.2
B. 175 D. 174.4

I
1-.+2 .\·1t'1If11 Turbine 143
Steam Turbine

STEAM TURBINES
Stca m Turbine - 2 (ME Rd. Apr. 1987)

An industrial power plant requires 1,5 kg of dry saturated steam per


second at 165°C for heating purposes. This steam may be supplied
Steam Turbine - 1 from an extraction turbine which receives stearn at 4 Mpa and 380°C
and 'is exhaust to condenser at the rate of 0.8 kg steam per second at
A steam turbine receives 5,000 kglhr of steam at 5 Mpa and 400°C and 0.0034 Mpa while rejecting 1400 KW to the cooling water. If
velocity of 25 m/sec. It leaves the turbine at 0.006 Mpa and 85% mechanical- electrical efficiency is 95% and that the he-at loss in the
quality and velocity of 20 rn/sec. Radiation loss is 10,000 KJ/hr. Find turbine casing is 10 KW, calculate the power generated by 'he plant.
the KW developed. A. 2,126.44 Kw C. 3,123.34 Kw
A. 1273.29 C. 1373.29 B. 1,556.5 Kw D. 4,344.33 Kw
C. 2173.29 D. 7231.29
SOLUTION:

SOLUTION: From Steam tables:


h, = 3165.9 KJ/kg
10i,{'.~
At 5 Mpa and 40QoC hz = hg at 165°C
4--~Gen
hi = 3195.7 KJ/kg
Sl = 6.6459 ---
5MPa.400oC
m7=5000kg/h
hz = 2763.5 KJ/kg
h, = h, at 0.0034 Mpa
h) = 109.84 KJ/kg ° . ~<;> @~. r».
! ~~) OutP':lt

At 0.006 Mpa w By mass balance: ~,~~~5~gIS . 1Tl,=O.8kgls


h r = 15 I. 53 and hrg = 2415.9 TURBINE rn, = 1.5 + 0.8 f 1,400kw
hz = hr +- xhrg m, = 2.3 kg/sec-: h
hz = 151.53 +- 0.85(2415.9) By heat balance: 't
hz = 2205.045 KJ/kg 7=20m/s m.h, = 1.5h z +- O.Sh) + 10 +- 1400 +- \'1
(x=85%) 2.3(3165.9) = 1.5(2763.5) + 0.8(109.84) +- 10 -t- 1400" W
O.006MPa W = ]638.45 KW
KE 1 = l/2 m yZ
KE 1 = 1/2 (5,OOO/3600)(25)z Generator Output = 1638.45(0.95)
KE 1 = 434.03 W Generator Output = 1,556.5 KW
KE 1 = 0.43403 KW -------~------------
KE z = 1/2 m yZ Steam Turbine - 3
KE z = l!2 (5,000/3600)(20)z
KE z = 277.78 W A steam turbine with 90% stage efficiency receives steam at 7 Mpa
KE z = 0.2778 KW and 550°C and exhausts as 20 Kpa. Determine the turbine work.
A.. 117 KJ/kg C. 123 K.l/kg .
By energy balance: B. 132 KJ/kg D. 143 KJ/kg
KE, -t- mh I = KE z -i- mh, +- Q -i- W
W = (KE 1 - KE z) +- m(h, - h z) - Q SOLUTION:
5000 10,000
W = (0.43403 - 0.2778) -i- ( - - )(3195.7 - 2205.045) - - - At 7 Mpa and 550°C
3600 3600 hi = 3530.9 KJ/kg
W = 1373.29 KW
s\ = 6.9486
)44 Steam Turbine .~ .
Steam Turbine 145
At 20 Kpa(0.020 Mpa)
3530.9 - h
Sr = 0.8320 h, = 251.40 090 ~
2a
----
Srg = 7.0766 h,g = 2358.3 3530.9 - 2289.78
s I = S2 = Sf + XSfg h 2> .= 2413.89 KJlkg
6.9486 = 0.8320 + x(7.0766) h2> = h, + xhrg
x = 0.8643 2413.89 = 25.\.40 + x(2358.3)
h2 = 251.40 + 0.8643(2358.3) x = 0.9167
h2 = 2289.78 KJ/kg x == 91.67%
p
hi - h Za
llsT =
h, - h z
j~30.9 - h
0.90 = Za Steam Turbine - 5
3530.9 - 2289.78
h2• = 2413.89 Kl/kz A small steam turbine power plant of 5,000 KW capacity has a full
WT = hi - h2 • load steam rate of 6 kg steam per KW-hr. No load steam consumption
W T == 3530.9 - 2413.89 5
may be taken as 10% of the full load steam consumption. Write the
W T = ]]7.01 KJ/kg equations of WILLANS LINE of this turbine and at 60% of rated
load, calculate the hourly steam consumption of this unit.
A. 19,100 kg/hr C. 19200 kg/hr
B. 19,300 kg/hr D. 19,400 kg/hr
Steam Turbine - 4
SOLUTION:
A steam turbine with 90% stage efficiency receives steam at 7 Mpa
and 550°C and exhausts as 20 Kpa. Determine the quality at exhaust. Full load steam consumption == 6(5000)
A. 87.45% C. 82.34% Full load steam consumption = 30,000 kg/hr
B. 76.34% D. 91.690/,
No load steam consumption 0.10(30,000)
p
No load steam consumption 3,000 kg/hr
SOLUTION:
By Two point slope formula: m.
(kg) FL
At 7 Mpa and 550°C
h, = 3530.9 KJIkIl 30,000 - 3,000
ms - 3,000 == (L - 0) ,(sooo, 30000)
SJ = 6.9486 5,000 - 0 ,
At 20 Kpa(0.020 Mpa) ,,
ms-3,000 == 5.4L ,
Sf = 0.8320 h, = 251.40 5 rn, = 5.4 L + 3,000 Nl~~30-0-00)-- -- :
Sfg = 7.0766 hfg = 2358.3
SI == S2 = sr+ XSrg L(kw)
rn, at 60% load 5.4(0.6 x 5000)
2 + 3000
6.9486 = 0.8320 + x(7.0766) m, = 19,200 kg/hr
x = 0.8643
h2 = 251.40 + 0.8643(2358.3) Steam Turbine - 6
h2 = 2289.78 KJ/kg
hl-h Za Steam flows into a turbine at the rate of 10 kg/sec and 10 KW of heat
llsT =
hi - h, are lost from the turbine. Ignoring elevation and kinetic energy effects,
146 Steam Turhine Steam Turbine 147

calculate till' power output from the tur hine. Inlet enthalpy is 2739
KJ/kg ami exhaust enthalpy is 2300.5 KJ/kg. rn, = 55(42ll0) I 3.000
A. 4605 KW C 4375 KW
B. 4973 KW D. 4000 KW rn, ~ 26,100 kg/hr

SOLUTION:

W = ffi(h 1 - h2 ) - Q Steam Turbine - 9

W = 10(2739 - 2300.5) - 10 A steam turbine has an entrance enthalpy of 3050 KJ/kg. The exit has
2500 KJ/kg. Determine the actual enthalpy after isentropic expansion
W = 4375 KW if stage efficiency is 90%.
A 1255 KJ/kg C. 2500 KJ/kg
B. 2555 KJ/kg D. 2000 KJ/kg
Steam Turbine - 7
SOLUTION:
Steam entering the turbine has a rate of 10 kg/sec with enthalpy of
3400 KJ/kg and 2600 KJ/kg at the exhaust. If 100 KW of heat is h, - h 1 a
rejected from turbine casing, what is the turbine work? "lsr
h, - h 1
A. 7900 KW 'c. 5600 KW
B. 7700 KW D. 5400 KW
3050 - h 1 a
0.90
SOLUTION: 3050- 2500

W = (h, - h-) - Q
ffi s h1a = 2555 KJ/kg
W = 10 (3400 - 2600) - 100
W = 8000 -100
W = 7900KW
Steam Turbine - 10

Steam Turbine - 8 Steam enters the turbine at the rate of2.5 kg/sec with enthalpy of 3200
KJ/kg and exhaust enthalpy of 1100 KJ/kg. Steam is extracted from
A steam turbine of 6 MW capacity has a Willan's line equation of rn, the turbine at the rate of 1 kg/sec for heating purposes with enthalpy
= 5.5L + 3,000, kg/hr. Determine the steam consumption at 70% load. of 2750 KJ/kg. What is the turbine work.
A. 3564 kg/hr C. 26,100 kg/hr A. 2000 KW C. 3000 KW
B. 3546 kg/hr D. 58,000 kg/hr B. 2500 KW D. 3600 KW
SOLUTION:

At 70% load, SOLUTION:


ffi\ = ffi2+ ffi3
L = 0.7(6,000) 2.5 = 1+ m,
rn- = 1.5 kg/s
L = 4200 KW
Ic\X Stemn Turbine Geothermal Power Plant 149

W = 2~(1:()()) - i(2750)-1.5(1100)
GEOTHERMAL POWER PLANT
W = 36()() 1\W

Geothermal Power Plant - 1


Steam Turbine - 11
Mass flow rate of ground water in a geothermal power plant is
A steam turbine has an entrance enthalpy of 3400 KJ/kg and 2500 1,500,000 kg/hr and the quality after throttling is 30%. Determine the
KJ/kg at exit. If generator generates 2430 KW and has 90% efficiency, brake power of turbine if the change of enthalpy of steam at inlet and
what is the mass of steam entering the turbine? outlet is 700 KJ/kg.
A 10,400 kg/hr C. 10,700 kg/hr A. 68.5 MW C. 64.5 MW
B. 10,600kglhr D.l0,800kg/hr B. 87.5 MW D. 89.5 MW

SOLUTION: SOLUTION:

W T = m, (hi - h2 ) ID, = x mg
243010.90 = TIl, (3400 - 2500) m, 0.3(1,500,000)
=
m, = 3 kg/sec x 3600sec/hr ID, = 450,000 kg/hr
m, = }O,800 kg/hr ID, = 125 kg/sec
WT = m, (h, - h4 )
W T = 125(700)
W T = 87,500 KW
Steam Turbine - 12 W T = 87.5 MW

Steam turbine in Rankine cycle has an exhaust enthalpy of 2650 KJ/kg


and delivers 0.8 kg/sec of steam. Determine the heat rejected from the
condenser if enthalpy at exit is 200 KJ/kg. Geothermal Power Plant - 2
A. 1960 KW C. 1995 KW
B. 1940 KW D. 1909 KW Ground water of geothermal power plant has an enthalpy of 700
KJ/kg and at turbine inlet is 2,750 KJ/kg and enthalpy of hot water in
SOLUTION: flash tank is 500 KJ/kg. What is the mass of steam flow entering the
turbine if mass flow of ground water is 45 kg/sec?
OR = mlh 2 - h3 ) A. 3.27 kg/sec C. 4.27 kg/sec
B. 2.27 kg/sec D. 9.27 kg/sec
OR = 0.8(2650 - 200)
SOLUTION:
QR = 1960 KW
h2 = hi + x(h, - hf)

700 = 500 + x(2750 - 500)


x = 0.0888
!:'() Geothermal Power Plant Geothermal Power Plant 151

ills x fIl g
ill, 0.888(45) Geothermal Power Plant - 5
ill, 4 kg/sec
A 16,000 KW geothermal plant has a generator efficiency and turbine
efficiency of 90% and 80%, respectively. If the quality after throttling
Geothermal Power Plant - 3 is 20% and each well discharges 200,000 kg/hr, determine the number
of wells are required to produce if the change of enthalpy at entrance
The enthalpy entering the turbine of a geothermal power plant is 2750 and exit of turbine is 500 KJ/kg.
KJ/kg and mass rate of 1 kg/sec. The turbine brake power is 1000 KW A. 4 wells C. 6 wel1s
condenser outlet has enthalpy of 210 KJ/kg. If temperature rise of B. 5 wells D. 8 wells
. cooling water in condenser is 8°C, what is the mass of cooling water
requirement? SOLUTION:
A. 44 kg/sec C. 46 kg/sec
B. 45 kg/sec D. 47 kg/sec W T = ill s(h3 - h 4 )
16,000
SOLUTION: - - = i l l s(500)
0.9(0.8)
W T = ill,(h 3 - h 4 ) ills = 44.44 kg/sec
1000 = 1(2750 - h4 ) ills = 160,000 kg/hr
h, = 1750 KJ/kg 160,000 = 0.20 illg
QR = Qw illg = 800,000 kg/hr
ills(~
- h s) = ill w Cp(t2 - t.) No. of wel1s = 800,000/200,000
1(1750 - 210) = ill w(4.187)(8) No. of wells = 4 wells
m., = 45.97 kg/sec

Geothermal Power Plant - 4 Geothermal Power Plant - 6 (ME Bd. Apr. 1988)

In a 12 MW geothermal power plant, the mass flow of steam entering A geothermal power plant draws pressurized water from a well at 20
the turbine is 26 kg/sec. The quality after throttling is 25% and Mpa and 300°C. To produce a steam water mixture in the separator,
enthalpy of ground water is 750 KJ/kg. Determine the overall where the un flashed water is removed, this water is throttled to a
efficiency of the plant. pressure of 1.5 Mpa. The flashed steam which is dry and saturated
A. 7.4% C. 5.4% passes through the steam collector and enters the turbine at 1.5 Mpa
B. 9.6% D. 15.4% and expands to 1 atm. The turbine efficiency is 85% at a rated power
output of 10 MW. Calculate overall plant efficiency
SOLUTION: A.7.29% C. 9.34%
B. 12.34% D. 19.45%
ills = x illg SOLUTION:
26 = 0.25 illg
illg = 104 kg/sec
12,000 At 1.5 Mpa (Table 2)
f]o h, = 2792.2 KJ/kg
104(750) S3 = 6.4448
f]() 15.38% At I arm (100°C)
152 Geothermal Power Plant Geothermal Power Plant 153

Sf = 1.3069 h, 4 I9.04 hr '= 640.23


Sfg = 6.048
S3 = S4 = Sf -t X4Sf~
64448 = 1.3069 + X4 (6.0480)
hf.~ - 2257 hrg = 2108.5
h, = h g at 0.50 Mpa
h, = 2748.7 KJ/kg
[d:>
m.
1500kPa

X4 ~ 0.8495 h, = n2 '= hr + xzhrg .


h, = h r + xh fg 697~22 = 640.23 + Xz (2~108.5)® I" I
.h, = 419.04 + 0.8495(2257) X2 - 0.027 I :
h, = 2336.4 KJ/kg -
m.- I I
m, = xm g \-'
Wr = ms(h 3 " h4)Tlr m, = 0.027(29.6) ,29.6 kg/s '!®
10,000 = m,(2792.2 - 2336.4)(0.85) m, = 0.80 kg/sec -IJG) +""" +@ 6t;"10°C
rn, = 25.81 kg/sec From Mollier Diagram: j ~

At 20 Mpa and 300°C(Table 4) 14 = 2211 KJ/kg


h,= 1333.3 KJ/kg Power produced = m s(h 3 - ~)

At 1.5 Mpa: Power produced = 0.8(2748.7 - 22 I I)


hr = 844.89 Power produced = 430.16 KW
hfg = 1947.3
h, = h2 = h, + X2 hfg
1333.3 = 844.89+X2(1947.3)
X2 = 0.25 Geothermal Power Plant - 8 (ME Bd. Oct. 1985)
m, = Xl fig
(25.81 X 3600) = 0.25(m g \ A flashed steam geothermal power plant is located where
fig = 37 I ,664 kg/hr underground hot water is available as saturated liquid at 700 Kpa.
10,000 s The well head pressure is 600 Kpa. The flashed steam enters a turbine
at 500 Kpa and expands to 15 Kpa, when it is condensed. The flow rate
(371,644/3600)(1333.3)
from the wdl is 29.6 kg/sec. Determine the cooling water flow in kg/sec
110vel'all = 7.26%
if water is available at 300 e and a lODe rise is allowed through the
condenser.

SOLUTION:
Geothermal Power Plant - 7 (ME Bd. Oct. 1985)

A flashed steam geothermal power plant is located where


h, = h-at 0.70 Mpa
underground hot water is available as saturated liquid at 700 Kpa.
h, = 697.22 KJ/kg (i)
The well head pressure is 600 Kpa. The flashed steam enters a turbine
at 500 Kpa and expands to 15 Kpa, when it is condensed. The flow rate e-t><:I--+-

from the well is 29.6 kg/sec. Determine the power produced in KW


! .. 430.13 kg/s C. 370.93 kg/s 1#.':
B. 540.23 kg/s D. 210.34 kg/s i! ® ll.t=10°C
SOLUTION:

h, = h f at 0.70 Mpa
m, = xm g
h,- 697.22 KJ/kg
m, = 0.027(29.6)
At 500 Kpa:
It
154 Geothermal Power Plant Geothermal Power Plant 155

m, = 0.80 kg/sec P4 .~ 0.016932 Mpa


h, = hrat 15 kpa(0.015 Mpa) From Mollier diagram:
h, = 225.94 KJ/kg 14 = 2085 KJ/kg
m s(14 - h s) = m; c p (At) Shaft Power = (480,915/3600)(2796 - 2085)( I - 0.015)(0.75)
0.8(2211-225.94) = m w(4.187)(10) Shaft Power = 70,167 K W
m; = 37.93 kg/sec Plant Output = 70,167(0.97)
Plant Output = 68,062 kw
Plant Output = 68.075 MW

Geothermal Power Plant - 9 (ME Rd. Oct. 1981)


Geothermal Power Plant - 10
In a certain geothermal area, studies show .that 1,500,000 kglhr of
pressurized ground water is available at 2,500 psia and 620°F. The A geothermal power plant has an output of 16,000 KW and combined
water will be throttled to 250 psia to produce wet steam and this efficiency of 80%. The pressurized ground water at 175 bar and 280°C
mixture will be passed through a water separator to remove the water leaves the wells to enter the nash chamber maintained at 14 bar. The
droplets so that saturated steam at 250 psia is available at the entrance steam collected enters the turbine at 14 bar and exhaust at 1 atm. If
of steam turbine for the proposed power plant. Other power plant one well discharges 180,000 kg/hr of hot water, how many wells are
data are as follows: required?
Discharge pressure of turbine = 25 in Hg vacuum A. 2 C. 3
Turbine engine efficiency = 75% B. 4 D. 5
Mechanical Loss = 1.5% of shaft power
Generator Efficiency = 97% SOLUTION: ~ "
l~
Assume atmospheric pressure to be 30 in Hg. Determine the maximum
amount of power in Kilowatts that the plant can generate. h, =h g at 14 bar(1.4 Mpa) m, @
A. 60 MW C. 66 MW h, = 2790 KJ/kg I ~-.§.,
B. 68 MW D. 74 MW S3 = Sg at 1.4 Mpa H<D +®
S3 = 6.4693
SOLUTION: At I atm(100°C): ~175bar

Sr = 1.3069 h r = 419.04
At 2500 psi(17.232 Mpa) and 620°F(326.667°C) Srg = 6.0480 h rg = 2257
hi = 1490.9 KJ/kg S3 = S4 = Sr + X2Srg
At 250 psi(I.7232 Mpa) 6.4693 = 1.3069 + ~(6.0480)
hr = 875 KJ/kg X4 = 0.8536
h rg = 1921 14 = 419.04 + 0.8536(2257)
h, = h g at 1.7232 Mpa 14 = 2345.55 KJ/kg
h, = 2796 KJ/kg 16,000/0.80 = m.(h3 -14)
h, = h 2 = h r + X2hrg . 20,000 = m s(2790 - 2345.55)

1490.9 = 875 + X2(1921)


X2 = 0.3206
~il2?.
;j
... I®
+ @25 in Hg
m, = 45 kg/sec
At 1.4 Mpa:
rn, = X2 m,.g '1'-' vacuum h r = 830.30 h rg = 1959.7
'I
m, = 0.3206(1 ,500,000) ~ At 175 bar(17.5 Mpa) and 280°C(Table 4)
rn, = 480,915 kg/hr 'J 2500 psi hi = 1231 KJ/kg
P 4 = (30 -25) x 0.101325/29.97 hi = h 2 = hr + X2hrg
156 Geothermal Power Plant Geothermal Power Plant 157

1231 = 830.30 t- xl(l959.?) hi h2 he + xhcg


X2 = 0.2046 870<; I - 770.38 + xz(2009.2)
m, = Xl m g Xl 0049836
(45 X 3600) = 0.2046mg rn. .~ xm~
m g = 791699.25 kg/hr 121.8 = 0.049836m g
No. of wells = 791699.25/180,000 m g = 2,444 kg/sec
No. of wells = 4.3984
No. of wells « 5 wells

Geothermal Power Plant - 11 (ME Rd. Oct. 1995)

A liquid dominated geothermal plant with a single flash separator


receives water at 204°C. The separator pressure is 1.04 Mpa. A direct
contact condenser operates at 0.034 Mpa. The turbine has a polytropic
efficiency of 0.75. For a cycle output of 50 MW, what is the mass flow
rate of the well-water in kg/s?
A. 2871 C. 186
B. 2100 D. 2444
At 204°C:
he = 870.51 KJ/kg
At 1.04 Mpa:
h c = 770.38 hcg = 2009.2
h g = 2779.6 5 g = 6.5729
At 0.034 Mpa:
hc = 301.40 hcg = 2328.8
Sc = 0.9793 Srg = 6.7463

SOLUTION:

h, = h. at 1.04 Mpa <i>


h, 2779.6 KJ/kg
= -
Solving for h.t: 1m•
S3 = S4 =' Sr + xSrg
6.5729 = 0.9793 + )4(6.7463) ~<D
@
)4 = 0.829
h, = 301.4 + 0.829(2328.8) U 0c
h, = 2232.3 KJ/kg 204
W T = m, (h, - h.t)
50,000 = m..(2779.6 - 2232.3)0.75
m, = 121.8 kg/sec
Solving for X2: (hi = h2)
158 Diesel Power Plant Diesel Power Plant 159

A 1000 KW C. 968 KW
DEISEL POWER PLANT B. 775 KW D. 588 KW

SOLUTION:
Diesel Power Plant - I
Indicated Power
Determine the indicated mean effective pressure of an engine in psi 110 =
having a brake mean effective pressure of 750 Kpa and 80% mrQ h
mechanical efficiency. Indicated Power
0.44 =
A 136 psi C. 138 psi 0.05(44.000)
B. 137 psi D. 140 psi Indicated Power = 968 KW

SOLUTION:

11m = PmWPml Diesel Power Plant - 4

0.80 =750/P m t A 750 KW diesel electric plant has a brake thermal efficiency of 34%.
14.7 If the heat generated by fuel is 9,000,000 KJ/hr, what is the generator
PIT" = 937.5---
101.325 efficiency?
Pm' = 136 psi A 85.33% C. 75.55%
8. 65.88'% D. 88.23%

SOLUTION:
Diesel Power Plant - 2
Brake Power
Determine the friction power of an engine if the frictional torque 11tb
mrQ h
developed is 0.3 KN.m running at 1200 rpm.
A. 40.6 KW C. 36.5 KW
Brake Power
B. 37.7 KW D. 50.3 KW 0.34
9,000,000/3600
SOLUTION:
Brake Power = 850 KW
Friction power = 2 IT T N 110 = 750/850
Friction power = 2 n(O.3)(l200/60) 110 = 88.23%
Friction power = 37.70 KW

Diesel Power Plant - 5

Diesel Power Plant - 3 A ) 6-cylinder V-type diesel engine is directly coupled to a 5000 KW
AC generator. If generator efficiency is 90%, calculate the brake
What is the power developed in the cylinder if indicated horsepower of the engine,
thermalefficiency is 44%, the engine uses 0.05 kg/sec fuel with heating A 7447 Hp C. 8542 Hp
value of 44,000 KJ/kg? B. 6468 Hp D. 7665 Hp
160 Diesel Power Plant Diesel Power Plant 161

SOLUTION: °AP1 27=

141.5
SG.= - - -
(icn. Output 131.5+°API
llCi -
Brake Power 141.5
SG
0.9 = 5000/Brake Power 131.5+ 27
Brake Power = 5555.55 KW (1/0.746) SG = 0.8927
Brake Power = 7447.12 Hp

Diesel Power Plant - 8


Diesel Power Plant - 6
A 500 KW diesel engine operates at 101.3 Kpa and 27°C in Manila. If
Determine the brake power of the engine having a brake thermal the engine will operates in Baguio having 93 Kpa and 23°C, what new
efficiency of 35% and uses 25° API fuel with fuel consumption of 40 brake power will developed if mechanical efficiency is 85%?
kg/hr. A. 600 KW C. 459 KW
A. 165.84 KW C. 173.52 KW B. 754 KW D. 971 KW
B. 173.52 KW D. 160.67 KW
SOLUTION:
SOLUTION:
WI = P/RT
Qh = 41,130 + 139.6°API WI = 101.325/(0.287)(27 + 273)
Qh = 41,130+ 139.6(25) WI = 1.1765 kg/rrr'
Qh = 44,620 KJ/kg W2 = P/RT
Brake Power W2 = 93/(0.287)(23 + 273)
11tb = W2 = 1.0947 kg/rrr'
mfQ h
Brake Power
0.35 = - - - - - - llm = BP / IP
(40/ 3600)(44,620) 0.85 = 500/ind. Power
Brake Power = 173.52 KW Ind. Power = 588.23 KW

Friction Power = IP - BP
Diesel Power Plant - 7 Friction Power = 588.23 - 500
Friction Power = 88.23 KW
Determine the specific gravity of fuel oil having a heating value of 1P2 w2
44,899.2 KJ/kg. IP\ wj
A. 0.90 C. 0.877
IP2 1.0947
B. 0.80 D. 0.893 -----
58823 1.1765
SOLUTION:
IP 2 = 547.336 KW
Qh = 41,130 + 139.6°API BP 2 = 547.336 -8823
44,899.2 = 41,130 + 139.6°API BP 2 = 459.106 KW
16:::' Diesel Power Plant Diesel Power Plant 163

Diesel Power Plant - 9 Diesel Power Plant - I I

What is the displacement volume of 300 nun x 400 mm, 4-stroke, 1200 Determine the output power of a diesel power plant if the engine and
rpm, 8 cylinder diesel engine? generator efficiency is 83% and 95%, respectively. The engine uses
A. 0.243 m1/sec C. 5.75 kg/sec 25° API fuel and has a fuel consumption of 0.08 kg/sec.
B. 2.262 m]/sec D. 1.25 m] /sec A. 2795 KW C. 9753 KW
B. 8642 KW D. 2815 KW
SOLUTION:
SOLUTION:
Yo = n/4D 2 L N c
Qg = mrQh
1200 Qg 0.08[41,130 + 139.6(25)]
=
Yo (n/4)(0.3)2(OA)(--) (8) Qg = 3569.6 KW
2 x60
Generator Output = 35696(0.83)(0.95)
Generator Output = 2814.63 KW
Yo = 2.262 m'rsec

Diesel Power Plant - 12

Diesel Power Plant - 10 Determine the piston speed of a 250 mm x 300 mm diesel engine
running at 1200 rpm.
What is the friction horsepower of a 300 KW diesel engine having a A. 6 m/sec C. 18 rn/sec
mechanical efficiency of 86%. B. 12 mlsec D. 5 m/sec
A. 86.5 Hp C. 90.5 Hp
B. 87.5 Hp D. 65.5 Hp
SOLUTION:

SOLUTION: Piston Speed 2LN


Piston Speed 2(030)( 12(0/60)
Piston Speed J 2 rn: sec

Brake Power
11m
Indicated Power Diesel Power Plant - 13

300 Determine the speed of a 6-pole generator of 60 hz.


0.86 A. 600 rpm C. 1200 rpm
Indicated Power B. 1000 rpm D. 3600 rpm

Indicated Power = 348.84 KW SOLUTION:

Friction Power = 348.84 - 300 120 f


Friction Power = 48.84 KW N =
Friction Power = 65.46 Hp P
164 Diesel Power Plant Diesel Power Plant 165

120(60)
N Diesel Power Plant - 16 .
6
N 1200 rpm
An engine-generator rated 9000 KV A at 80% power factor, 3 phase,
4160 V has an efficiency of 90%. If overall plant efficiency is 25%,
what is the heat generated by the fuel.
A. 18,800 KW C. 7500 KW
Diesel Power Plant - 14 B. 28,800 KW D. 20,000 KW
A 50 Bhp blast furnace engine uses fuel with 10 felBhp-hrs. The
heating value of gas is 33,500 KJ/m 3 • Determine the brake thermal SOLUTION:
efficiency.
A.80% 25.76%C: Gen. Output = pfx KVA
B. 50% D. 28.31% Gen. Output = 0.8 x 9000
Gen. Output = 7200 KW
SOLUTION: Gen Output
11 overall
lOft] 1 1 Qg
Qg = [ x 50Bhp x - - x , ](33,500) 0.25 = 7200/Qg
Bhp - hrs 3600 (3.281)-
Qg = 28,800 KW

Qg = 131.73 KW
50 x 0.746
Qg =
131.73 Diesel Power Plant - 17
Qg = 28.31%
A 4-stroke, 8 cylinder diesel engine with bore and stroke of 9 in x 12
in, respectively and speed of 1000 rpm has brake mean effective
pressure of 165 psi. Determine the engine brake horsepower.
Diesel Power Plant -15 A. 1753.34 Hp C. 1950.34 Hp
B. 1850.34 Hp D. 1272.34 Hp
The heat generated by fuel is 2500 KW If the jacket water loss is 30%. ~

Determine the mass of water circulated in the engine if the


temperature rise is soc. SOLUTION:
A. 20.5 kg/sec C. 58.5 kg/sec
B. 22.4 kg/sec D. 12.3 kg/sec Vo = 1t/4D2LNc
v: = 1t/4(9)2 (12) (1000/2)(8)
SOLUTION: V0 = 3053628.059 in3/roin

Jacket water loss = 0.3 Qg Brake power = Pmb X V0


Jacket water loss = 0.3(2500) Brake Power = 165(3053628.059)
Jacket water loss = 750 KW Brake Power = 503848629.8 in-lb/min x 1/12
Jacket water loss = row c p (At) Brake Power = 41987385.82/33,000
750 = m., (4.187)(8) Brake Power = 1272.34 hp
row = 22.39 kg/sec
I ()() Diesel Power Plant Diesel Power Plant 167

SOLUTION:
Diesel Power Plant - 18
141.5
What torque is developed by the JOn KW engine running at 900 rpm? SG
A. 2.65 KN.m C. 3.18 KN.m 131.5+ 25
B. 6.85 KN.m D. 5.65 KN.m SG 0.904

Density of fuel 0.904( I)


SOLUTION: Density of fuel 0.904 kg/Ii

Time to run the engine 90013000


KW = 211: TN
Time to run the engine 0.3 hr
Time to run the engine 1080 sec
300 .~ 211: T (900/60)

T = 3.18 KN.m
v = 240/1080
V = 0.2221i1sec

m = 0.904(0.222)
m = 0.2 kg/sec
Diesel Power Plant - 19

What is the mechanical efficiency of a diesel engine if friction power is


30 KW and brake power of 150 KW?
Diesel Power Plant - 21
A. 85.55% C. 65.44%
B. '83.33% D. 75.88%
The indicated thermal efficiency of a two stroke diesel engine is 50%.
If friction power is 3% of heat generated, determine the brake thermal
efficiency of the engine.
SOLUTION: A.33% C. 36%
B. 34% D.37%
11m BP/IP
=
IP BP + FP
=
SOLUTION:
IP = 150 + 30
IP = 180 kw
1111 = IP/Qg
11m = 150/180 0.5 = IP/Qg
11m = 83.33% IP = 0.5 Qg

BP = IP - FP
Diesel Power Plant - 20 BP = O.5Qg - O.13Qg
BP = 0.37Qg
A 3MW diesel engine consumes 240 liters of 25° API fuel and generates
900 KW-hr. Determine the rate of fuel consumed by the engine. 11tb = BP/Qg
A. 0.2 kg/sec C. 0.6 kg/sec
11tb = 0.37QglQg
B. 0.4 kg/sec D. 0.8 kg/sec
11tb = 37%
168 Diesel Power Planl Diesel Power 1'/11"1 169

Dlcsel Power Plant - 24


Diesel Power Plant - 22
What maximum power that can be delivered by the 2000 KW engine
During the dynamometer test of an engine for 1 hr steady load, the at JOOO ft elevation considering the pressure effect alone?
engine consumes 40 kg fuel having 43,000 KJ/kg heating value. If the A. 1600 KW C. 1900 KW
torque developed is 2.5 KN-m during the test at 600 rpm, what is the B. 1700 KW D. :300 KW
brake thermal efficiency of the engine?
A. 31.22% C. 55.77% SOLUTION:
B. 32.88% D. 25.99%
Solving for the pressure at higher elevation:
SOLUTION: B = 29.92 - (3000/1000)
B = 26.92 in Hg
m. = 40/3600 B
m; = 0.0 III kg/sec
P, = Ps ( - - )
29.92 .
P, = 2000(26.92/29.92)
BP = 2nTN r, = 1799.46 KW
BP = 2n(2.5)(600/60)
BP = 157.08 KW

157.08 Diesel Power Plant - 25


11tb
0.0 IIII( 43,000)
A 1000 KW diesel engine operates at an altitude of 1500 m elevation.
11'b = 32.88%
Considering the pressure effect alone, find the power developed by the
engine at higher elevation?
A. 753 KW C. 345 KW
B. 823 KW D. 983 KW
Diesel Power Plant - 23

A waste heat recovery boiler generates 250 kg/hr with h, -h F = SOLUTION:


23GOKJ/kg. What is the exhaust loss from the engine if the actual heat
developed is 35%?
A. 492.6 KW C. 456.4 KW 3.6
T = 520 - - - x (l500x3.281)
B. 365.7 KW D. 845.4 KW 1000
T = 502.28

Ps~ 5:0
SOLUTION:
Pe =
Waste heat recovered = rn, (h, - hF)
Waste heat recovered = 25W3600 (2300)
Waste heat recovered = 159.72 KW J502.28
P, =1000 - -
520
Exhaust Loss 159.72/0.35 P e = 982.82 KW
Exhaust Loss 456.35 KW

'.,
Diesel Power Plant 171
: "jl I /)/I'S/'! Power Plant
SOLUTION:
Diesel Power Plant - 26
BP = 2 rt TN
The piston speed of an engine running at 1200 rpm is 12 m/s. Find the BP = 2 rt (5)( 1800/60)
stroke of the engine in inches. BP = 94248 KW
A. 15.74 mm C. 300 mrn
B. 16 in D. 15.75 in Tllb= BP/Qg
0.31 = 942A8/Qg
Qg = 3040.25 KW
SOLUTION:

Piston Speed = 2 L N
12- 2L(l200/60) Diesel Power Plant - 29
L ~ 0.3 rn
L ~ 30n mm In a diesel engine the fuel is injected at 6.5% of the stroke, the
clearance volume is 10% of the stroke. Find the cut-off ratio.
A. 1.80 C. 1.70
B. 1.65 D. lAO
Diesel Power Plant - 27

The density of air entering the engine is 1.176 kg/m'' has a volume flow SOLUTION:
rate of 0.375 m 3/sec. If the air fuel ratio is 21, find the mass flow rate of
fuel. Cut-off ratio Y3 I V 2
A. O. J 0 kg/sec C. 0.30 kg/sec
B. 0.02 kg/sec D. OAO kg/sec Cut-off ratio = YI +0.065Y D
YI
SOLUTION:
YI = O.IOYD
m, == W Y
0.10Y D + O.065Y D
m, =." 1.176(0.375) Cut-off ratio
m. = 0441 kg/sec O.IOY D
A/F = m, / m, Cut-off ratio = 1-, 65
21 = OA41/mr
m, = 0.0211 kg/sec

Diesel Power Plant - 30 (ME Bd. Oct. 1989)

Diesel Power Plant - 28 A 373 KW (500 Hp) internal combustion engine has a brake mean
effective pressure of 551.5 Kpa at full load. What is the friction power
A diesel engine develops a torque of 5 KN-m at 1800 rpm. If the brake if mechanical efficiency is 85%?
thermal efficiency is 31 %, find the heat generated by the fuel. A. 88.23 hp C. 56.34 hp
A. 3050.25 KW C. 3000.25 KW B. 97.33 hp D. 76.23 hp
B. 304025 KW D. 5000 KW
17.' Diesel Po wer Plant A. 31.53% C 45.34%
B. 27A5% o 54.23%
S()I>UTlON:

Tlm Brake Power/Indicated Power SOLUTION:


0.85 SOO/Indicated Power
Indicated power = 588.23 Hp
Friction Power = Ind. Power - Brake power m, 0.26 kg/KW-hr x 150 KW
=
Friction Power = 588.23 - 500 rn, =39 kg/hr
Friction Power = 88.23 Hp rn,= 0.010833 kg/sec
Brake Power
Brake thermal efficiency
mrQ h
Diesel Power Plant - 31 (ME Bd. June. 1990) 150
Brake thermal efficiency
0.0 I0833( 43,912)
A 4-stroke 394 mm bore and 534 mm stroke single acting diesel engine
Brake thermal efficiency 31.53%
with four cylinders is guaranteed to deliver 350 Bhp at 300 rpm. The
engine consumed 66.8 kg/hr of fuel with a heating value of 44,251
K.J/kg. Calculate indicated mean effective pressure in Kpa if
mechanical efficiency is 89%.
Diesel Power Plant - 33 (ME Bd. Oct. 1986)
A. 450.8 kpa C. 345.6 kpa
B. 234.5 kpa D. 643.2 kpa
A four-stroke, 8 cylinder Diesel engine with bore and stroke of 9
inches and 12 inches, respectively and speed of 950 rpm has a brake
SOLUTION:
mean effective pressure of 164 psi. The specific fuel consumption is
0.39 Ib/bhp-hr and the fuel heating value is 18,500 Btu/lb. Determine
Vo = rr/4 D 2 L N c
thermal efficiency.
300 A. 35.27% C. 38.23%
Vo = rr/4 (0.394)2(0.534)(--)( 4)
2x60 B. 45.23% D. 54.23%
Vo = 0.651 m 3/sec
Brake power = Pmb X V 0 SOLUTION:
350(0.746) = P mbxO.651
Pmb = 40I.2 Kpa PmbLANc
Bhp = ----"'-''-------
Tlm = Brake Power/Indicated Power
33,000
11m = P mb / P ml
A = n/4 (9/12/
0.89 = 401.2/P m,
A = OA418ft 2
P mt = 450.8 Kpa
(164x144)(l2/ 12)(0.4418)(950/2)(8)
Bhp
33,000
Diesel Power Plant - 32 (ME Bd. Apr. 1990) Bhp 1,201AHp

A 305 mm x 457 mm four stroke single acting diesel engine is rated at rn; = 0.39 lb/Bhp-hr x 120 I A Bhp
150 KW at 260 rpm. Fuel consumption at rated load is 0.26 kg/KW-hr m, = 468.546 lbs/hr
with a heating value of 43,912 KJ/kg. Calculate brake thermal
efficiency.
174 Diesel Power Plant Diesel Power Plant 175

120 l~i2545)
A. 234 C. 242
Thermal efficiency = 468.546( 18,500) B. 873 D. 292

SOLUTION:
Thermal efficiency = 35.27%
Note: 1 Hp = 2545 Btu/hr
280
VD = (rc/4)(0.4)z(0.60)(--)(8)
2x60
VD = 1.4074 mJ/sec
Diesel Power Plant - 34 (ME Bd. Apr. 1988)
Indicated Power = (130 x 101.325/14.7)(1 4074)
A dynamometer test was done for one hour-at steady load on a 6
Indicated Power = 1,261.16 KW
cylinder diesel engine. It was found to use 42 kg of fuel having Qh =
Indicated Power = 1,690.56 Hp
42,000 J/g. Cylinder is 22.8 em x 27.2 em, 4-cycle type. Speed, 550 rpm
mr = 0.6(1,690.56)
arid dynamometer torque at 27000 kg-em. Determine the brake
m, = 1,014.3361bslhr
thermal efficiency.
Heat supplied = m-Q, = 1,014.336(19,100)
A. 31.13% C. 35.34%
Heat supplied = 19,373,817.6 Btu/hr
B. 43.22% D. 45.32%
Heat gained by water = 0.25(19,373,817.6)
Heat gained by water = 4,843,454.4 Btu/hr
SOLUTION:
Heat gained by water = 80,724.24 Btu/min
Heat gained by water = m w cp (t z - t.)
T = 27,000 icg-cm x 0.00981 x 1/100
80,724.24 = rn., (1)(40)
T = 2.6487 KN-m
m; = 2,018.106 Ib/min
2,018.106 x 7.481
Brake Power = 2 rc TN V
Brake Power = 2rc(2.6487)(550/60) 62.4
Brake Power = 152.552 KW V = 241.9 gal/min

152.554
Brake thermal efficiency =
(42/ 3600X42,000)
Brake thermal efficiency = 31.13% Diesel Power Plant - 36 ~ME Bd. Oct. 1985)

When the pressure is 101.325 Kpa and temperature is 27°C, a diesel


engine has the full-throttle characteristics listed:
Brake power = 275 KW
Diesel Power Plant - 35 (ME Bd. Oct. 1992) Brake specific fuel consumption = 0.25 kg/KW-hr
Air fuel ratio = 22
A certain diesel engine with the following classification, 8 cylinder, 400 Mechanical efficiency = 88%
mm x 600 mm, four stroke cycle has a fuel consumption of 0.6 Ibs/hp- If the engine is operate at 84.5 Kpa and temperature of 15.5°C, find
hr based on 19,100 Btu/lb. Engine speed is 280 rpm with an indicated brake specific fuel consumption kg/kwh.
mean effective pressure 130 psi. If the jacket water carries away an A. 0.294 C. 0.423
estimated 25% of the heat supplied, find its capacity(gpm) required if B. 0.862 D. 1.08
the allowable rise is 40°F.
176 Diesel Power Plant Diesel Power Plant J77

SOLUTION: 36
T = S20-(----XI9ln2x3.181)
1000
At 103.3 Kpa and 2JOC: T = 496.6°R
Density = P/RT r-
BIT
Density = 101.325/(0.287)(27 + 273) Po = P, (--)( 1-)
Density = 1.1765 kg/rrr' 29.92 ~ 520
Indicated power = 275/0.88 23.42 .. !496.6-
Indicated power = 312.5 KW P, = 2.5(-j(,/"-)
Friction power = 312.5 - 275 29.92 V 520
Friction power = 37.5 KW(constant)
P, = 1.912 KW
mn = 0.25(275)
mn = 68.75 kg/hr (constant)
mal = 68.75(22)
3/hr Diesel Power Plant - 38 (ME Rd. June 1990)
mal = 1512.59 m
Val = 1512.5/1.1765
Val = 1285.59 m 3/hr (constant)
A 1119 KW, six cylinder, 589 mm x 711 mm, 225 rpm, four stros,e
diesel engine has a fuel consumption of 0.23 kg/kWh based on 44,fJi,H i
At 84.5 Kpa and 15.5°C:
KJ/kg fuel (heating value). A waste heat recovery boiler recovers 3S'''-o
Density = P/RT
of the exhaust loss. Jacket water 105s are estimated at 30%. Assume
Density = 84.5/(0.287)(15.5 +273)
losses due to friction, etc. at 8%. Calculate tilt: quantity of 136 KP3
Density = 1.0205 kg/rrr'
Indicated power = 312.5(1.0205/1.1765) steam that can be produced in kg/hr, if jacket water at 70"C is used for
Indicated power = 271.06 KW boiler feed.
A. 439.4 kg/hr . C. 543.32 kg/hr
Brake power = 271.06-37.5
B. 623.4 kg/hr D. 984.45 kgihr
Brake power = 233.56 KW
Brake spec. fuel consumption = 68.75/233.56
Brake spec. fuel consumption = 0.294kgIKWh SOLUTION:

m, =0.23(1119)
m, =257.37 kg/hr
Heat generated by file! = m, Qh
Diesel Power Plant - 37 (ME Rd. Apr. 1992)
Heat generated by fuel = (257.37/3600)(44,099)
Heat generated by fuel > 3152.7 K\V
Find the power which a 2.5 MW natural gas engine can develop at an
By heat balance ill the engine:
altitude of 1981.2 meters taking into consideration the pressure and
Heat generated = Jacket water less + Brake power -I- Exhaust
temperature change
C. 2.56 MW loss + friction and radiation loss
A. 2.34 MW
3152.7 cc 0.3(3152.7)-+ 1119+Exhaustloss+O.08(3152.Ti
B. 1.912 MW D. 1.67 MW
Exhaust loss = 835.7 KW
Heal utilized in the boiler = 0.35(835.7)
SOLUTION:
Heat utilized ;n the" 292.495 KW
I
Using Steam table:
B = 29.29---(1981.2x3.281) h, '.~ h, at 70°C
1000 hr 292.98 KJ/kg
h, h,. at i36 Kpa
B = 23.42 in Hg h, "IiR91 K.Lkg
178 Diesel Power Plant Diesel Power Plant 179

ills = mass of steam


ills(hs - hd = heat utilized in the boiler Diesel Power Plant - 40 (ME Rd. Oct. 1995)
ills(2689.1 - 292.98) = 292.495
ills = 0.12207 kg/sec A waste heat recovery boiler produces 4.8 Mpa(dry saturated) steam
ills = 439.4 kg/hr from 104°C feedwater. The boiler receives energy from 5 kg/sec of
954~C dry air. After passing through a waste heat boiler, the
temperature of the air is has been reduce to 343°C. How much steam
in kg is produced l'er second? Note: At 4.80 Mpa dry saturated, h =
Diesel Power Plant - 39 (ME Rd. Oct. 1995) 2796.
A. 1.3 C. 2.1
A 2000 KW diesel engine'unit uses I bbl oil per 525 KWH produced. B. 0.92 D. 3.4
Oil is 25°API. Efficiency of generator 93%, mechanical efficiency of
engine 80%. What is the thermal of engine based on indicated SOLUTION: (Steam)
power(%)? 4.8MPa
A. 31.69 C. 39.60 hr = approximate enthalpy offeedwater fOry
T ~ sat.)
B. 29.47 D. 35.60 hr = cp t I i

h r = 4.187(104) I ENGINE
SOLUTION: BOILER
h r = 435.45 KJlkg ~." .."'. »». .•, ,••.•
x'·' '.' ',' '.' '.' '.'
Heat loss = Heat gain .... '" '" '"104°C
~t I
1 bbl = 42 gallons illg Cp (t 1 - t z) = ills (hs - h F) \~eeCJ water)
.- . 1343°C
5( 1.0)(954 - 343) = ills(2796.0 - 435.45)
141.5 ills = 1.3 kg/sec
SG =
25 + 131.5
SG = 0.904
Diesel Power Plant - 41 (ME Rd. Oct. 1995)
w = 0.904(1 kg/Ii)
w = 0.904 kg/li A diesel electric plant supplies energy for Meralco. During a 24-hour
period, the plant consumed 200 gallons of fuel at 28°C and produced
rn, = V xw 3930 KW~hr. Industrial fuel used is 28°API and was purchased at
m, = (42 x 3.785)(0.904) P5.50 per liter at 15.6°C. What is the cost of fuel be to produce one
m, = 143.724 kg KW-hr?
A. Pl.05 C. PI.069
Qh = 41,130 + 139.6(25) B. PLIO D. P1.00
Qh = 44,620 KJlkg
SOLUTION:
Indicated work = 525/(0.93)(0.80)
Indicated work = 705.645 KW-hr 141.5
SG 15 6C =
705.645(3600) 131.5+ 28
TJti SG I5 6C = 0.887
143.724( 44,620) Density at 15.6°C = 0.887(1 kg/li)
TJti 39.6% Density at 15.6°C = 0.887 kg/li
SG Z8°C = 0.887[ I - 0.0007(t - 15.6)]
180 Diesel Power Plant Diesel Power Plant 181
SG 28C = OJ!7<)
Density at 28°(' o 879( I kg/li) Diesel Power Plant - 43 (ME Bd. Apr. 1995)
Density at 28')C 0.879 kg/li
.\ supercharged six-cvlinder four stroke cycle diesel engine of 10.48 em
V 280C / V \5 6 .~ SG 15 6'C I SG 28 0e
cC
bore and 12.'7 cm stroke has a compression ratio of 15. When It IS
200 I V IWC = 0.887/0.879 tested on a dynamometer with a 53.34 ern arm at 2500 rpm. the scale
reads 81.65 kg. 2.86 k.g of fuel of 45,822.20 KJ/kg heating value are
V 15 e-c 198.196 gallons x 3.785 Ii/gal burned during a 6 min test, and air metered to the cylinders at the rate
V 15 6C 750.171861i of tl.I 82 kg/sec. Find the brake thermal efficiency.
A. U327 C 0.307
(5.5)(750.172) B 0.367 D. 0.357
Cost
3930
Cost P1.05/Kw-hr SOLUTION

T~· (81.65xO.0098J)(05334)
T= 04~72S KN-m

Diesel Power Plant - 42 (ME Bd. Apr. 1996)


Brake Power 2nTN
A single-acting, four-cylinder, 4 stroke cycle diesel engine with a bore Brake Power 2n(0.42725 )(2500/60)
x stroke of 21.59 x 27.94 cm, operating at 275 rpm, consumes 8.]89 Brake Power 111.854 KW
kg/h of fuel whose heating value is 43,961.4 KJ/kg. The indicated mean
effective pressure is 475.7 Kpa. The load on the brake arm, which is 2..86
mj
93.98 cm is ] 13.4 kg. What is the brake mean effective pressure in 6x60
Kpa? m, = 0.00794 kg/s
A. 415.20 C. 319.95
B. 124.17 D. 645.33
I \ 1.854
llib
SOLUTION: (0.00794)( 45,822.20)
lltb 3070%
275
VD = n/4 (0.2159)2 (0.2794)(--)(4)
2x60
3
VD = 0.09376 m /sec
Diesel Power Plant - 44 (M E Bd. Oct. 1998)
T = (1 13.4 x 0.00981)(0.9398)
T = 1.045 KN-m In a double acting, 2 stroke compression ignition engine. 8-q linder,
the diameter of the cylinder is 700 mm, stroke is 1350 mm and the
Brake power = 2nTN piston rod diameter is 250 mrn. When running at 108 rpm, the
Brake power = 2n(1.045)(275/60) indicated mean effective pressure above and below the pistons are 5.80
Brake power = 30 KW bar and 4.90 bar respectively. Calculate the brake power of the engine
with a mechanical efficiency of 80% in kilowatts.
BP=PmbXV D
A. 6050 C. 6010
30 = P mb X 0.09376
Pmb = 319.97 Kpa B. 6030 D. 6070
Diesel Power Plant 183
182 Diesel Power Plant
Vw = 3.0455(7.481)
SOLUTION:
Vw = 22.80 gpm

At head end:
Diesel Power Plant - 46 (ME Bd, Apr. 1997)
V» =(11:/4)0 2 LNc
Vo = (n/4)(070)ZC1.35)(108/60X8)
3 A six cylinder, four stroke diesel engine with 76 mm bore x 89 mm
Vn = 7.481 m /s Brake Power stroke was run in the laboratory at 2000 rpm, when it was found that
Indicated Power = Pmi X V0
the engine torque was 153.5 N-m with all cylinders firing but 123 N-m
Indicated Power = (5.80 x 100)(7.481)
when one cylinder was out. The engine consumed 12.2 kg of fuel per
Indicated Power = 4,339 Kw
hour with a heating value of 54,120 KJIkg and 252.2 kg of air at 15.6'C
At crank end: per hour. Determine the indicated power.
Vo = (1t/4)(02 - d
2)LNc
A. 32.1kw C. 23.3kw
V D = (11:/4)[(0.7i - (0.25)2]( 1.35)(1 08/60)(8) B. 38.4 kw D. 48.3 kw
V D = 6.527 m 3/s
Indicated Power = (4.9 x 100)(6.527) SOLUTION:
Indicated Power = 3,198 Kw
Total Indicated Power = 4,339 + 3,198 Brake Power = 2 11: r N
Total Indicated Power =' 7,537 Kw
Brake Power = 21t (0.1535)(2000/60)
Brake Power = 7,537(0.8) Brake Power = 32.15 kw
Brake Power = 6,030 kw

Friction power per cylinder = 32.15(5/6) - 21t(0.123)(2000/60) Friction


power per cylinder = 1.031 kw Indicated Power
Diesel Power Plant - 45 (ME Bd. Apr. 1997)
Total friction power = 1.031 (6)
In a test laboratory, it was found out that of the 80 Bhp developed by
Total friction power = 6.19 kw
an engine on test, 45 Hp are absorbed by the cooling water that is
pumped through the water jacket and the radiator. The water enters Brake
Indicated power = 32.15 + 6.19
the top of the radiator at 200°F. At that temperature, enthalpy of the Indicated power = 38.34 kw I , I
water is 168.07 Btu/lb. Water leaves the bottom of the radiator at
190°F and with an enthalpy of 158.03 Btu/lb. What is the water flow Friction Power
rate for a steady-state operation?
A. 25 gpm C. 23 gpm
Diesel Power Plant - 47
B. 20 gpm D. 24 gpm
A waste heat recovery boiler receives energy from 10 kg/s of 950'C hot
SOLUTION:
80 bhp gases from a diesel engine. The exit temperature of hot gases (c p =
1.0) has been reduced to 250°C. Steam is produced at 5 Mpa (dry
Qh saturated) from 95°C feedwater. How much steam in kg is produced
Q = m., (hi - h2 )
200°F per hour? At 5 Mpa, h g = 2794.3 KJIkg
45(42.4) = rn., (168.07 - 158.03) 190°F
A. 10,515 C. 11,055
m; = 190.04 Ib/min
B. 10,155 D. 11,515
V w ,~ 190.04/62.4
VYo = 30455 ft3/ min
] 8·1 Diesel Power Plan' Diesel Power Plant 18"
(Exhaust)

SOLUTION: d~ 10 kg/s After 15 minute test:


rn. = 6.72 x 10'3 (IS x 60)
Using approximate value of hr: fir = 6.048 kg
h r = cp t 141.5
h, = 4.187(95) Engine SG =
131.5+28
h, = 397.765 KJ/kg SG = 0.887
Boiler
By heat balance in recovery boiler: 95°C
6.048138
m c p (t 2 - t 1) = m, (h, - h F ) Feedwateri 1 I v
(10)(1)(950 - 250) = m, (2794.3 - 397.765) 250°C
(0.887 x I)
ills = 2.92 kg/s (3600) V = 6.82 liters
rn, = 10,515 kg/hr

Diesel Power Plant - 48 Diesel Power Plant - 49

A SIX cylinder, four stroke cycle diesel engine has an engine thermal A 3 MW diesel electric power plant uses 3700 gallons in a 24 hours
efficiency of 34%. The engine was tested on a dynamometer with a 23 period. Oil is 25°API. What is the thermal efficiency of the engine
in arm, running at 1800 rpm, the scale reads 210 Ibs. During the 15 based on indicated power if the generator is 90%. efficient and 95°/~)
min test, the engine uses fuel with 28° API and air metered to the mechanical efficiency is assumed?
cylinder at the rate of 0.201 kg/so Find the fuel consumption during A.55% C. 65%
rhe 15 min test. B.60% D.70%
A. 6.33 liters C. 6.83 liters
B. 5.97 liters D. 8.97 liters SOLUTION:

SOLUTION:
m,=O.201kg/s ~ ,..... 141.5
SG
131.5 + 25
T F x distance SG = 0.904
T 210 (23!l2)
T 402.5 ft-lb Density of fuel = 0.904(1000)
27t (402.5)(1800) Density of fuel = 904 kg/m?
p
31,000
P 137.94 hp
= 3700x3.785
rn, = 904( )
P = 102,906 kw
1000
Qh = 41,130 + 139.6(28) m, = 12,661AI kg
Qh = 45,038,8 KJikg
Brake Power Pi = power input
TIth = 3,000
mrQ h Pi
102.906 (0.9)(0.85)
0.34 =
PI 3921.57 kw
=
m r (45,038.8)
Qh = 41,130+ 139.6(25)
mr = 6.72 x 10,1 kg/s
Qh c= 44.620 KJ/kg.
IXh Diesel Power Plant Diesel Power Plant 187

3921.57 SOLUTION:
1111 -
12,661.41
( )(44,620)
24 x 3600 . 2000
60%
VD (n/4 )(0.076)L(0.089)(--)( 6)
Ill, 2 x60
V D = 0.0404 m 3/s

(137.4 ,I 3600)(0.287)( 15.5... 273)


Diesel Power Plant - 50 Va
101.325
A single acting, 8 cylinder, 4 stroke cycle diesel engine with a bore to
stroke of 15.24 em x 22.86 ern, operates at 1200 rpm. The load on the Va 0.0312 m3/s
brake arm which is 101.60 ern is 120 kg. What is the brake mean
effective pressure in kpa? Va
A. 4505 kpa C. 445.5 kpa 11"
VTJ
B. 455.5 kpa C. 495.5 kpa
11" = 0.0312/0.0404
SOLUTION:
11, = 77.23%
J200
V D = (n/4)(0.1524)2(0.2286)(--)(8)
2 x60
V D = 0.3336 m 3/s
T = Force x distance
T = (120 x 0.00981)(1.016)
T ~~ 1.196 KN-m
P = 2rrTN
P ~c 2 7t (I. 196)( 1200/60)
P = ISO.298Kw
P =, Pmb X V D
150.298 = Pmb (0.3336)
Pmb = 450.5 Kpa

Diesel Power Plant - 51

A six cylinder four stroke diesel engine with 76 mm bore and 89 mm


stroke was run in the laboratory at 2000 rpm. The engine consumed
137.4 kg of air at IS.SoC per hour. Determine the volumetric efficiency
of air only.
A. 69.84% C. 92.54%
B. 88.32% D. 77.23%
Gas Turbine Plant 189
188 Gas Turbine Plant
14-1
] 123
GAS TURBINE POWER PLANT 583 =(f p ) 14

fp = 9.9]9
e = ] _ I
14-1
Gas Turbine - 1
(9.919) 14
An air-standard Brayton cycle has a pressure ratio of 12. Find the e = 48.08%
thermal efficiency of the cycle.
A. 34.23% C. 56.32%
B. 50.&3% D. 65.23%
Gas Turbine - 3
SOLUTION:
An air-standard Brayton cycle has a pressure ratio of 8. The air
properties at the start of compression are 100 kpa and 25°C. The
f" = 1- k-j maximum allowable temperature is 1l00°C. Determine the net work.
(f p ) k A. 373.24 KJ/kg C. 321.34 KJ/kg
B. 283.45 KJ/kg D. 398.23 KJ/kg
I
e = I - --1-4-1
SOLUTION:
(12) 14

e = 50.83% P2
-=8
PI
T 1 = 25+273
T 1 = 298°K
Gas T.urbine - 2 k-I

An air-standard Brayton cycle has an air leaving the high-temperature


heat exchanger at 850°C and leaving the turbine at 310°C. What is the ~:t:r
r.
2c1311000C
thermal efficiency? T 14-1
A. 42.21% C. 48.08% _1 =(8r1.4 • Wnet
° 4
B. 23.34% D. 56.34% 298
T 2 = 539.81°K . 1 °100Kpa,2SoC

c:J0
SOLUTION:
k-I

T3
T3
T4
=
=
=
850 + 273
I ]23°K
310+273
r.
2 3 850°C ~: =[:J'
T 3 = 1100 + 273
T4 = 583 OK
Wnet
T 3 = 1373°K
1 ° ° 4 310°C 14-1
k-I 1373 ._-
T --=(8) 14
- 3 = (r ) k
T4 P 5 T4
T 4 = 757.95°K
190 Gas Turbine Plant Gas Turbine Plant 191

We = m Cp (1'2 - Tj )
We = m(1 )(539.81 - 298) Gas Turbine - 5
We 1m = 241.81 KJ/kg
Wr = m Cp (1' J - 1'4) The compressor for an actual gas rurbine requires 300 KJ/kg of work
Wr = m (1)(1373 - 757.95) to quadruple the inlet' pressure. ..,he inlet air temperature is 100°C
W r 1m = 615.05 KJ/kg Determine the compressor air exit temperature.
Wiler = 615.05 - 241.81 A. 234°K C. 653°K
Wnet = :; 73 .24 KJ/kg B. 542°K D. 673°K

Gas Turbine - 4 SOLUTION:

The air-standard Brayton cycle has a net power output of 100 kw. The
working substance is air, entering the compressor at 30°C, leaving the 1', = 100+273
high-temperature heat exchanger at 750°C and leaving the turbine at 1'1 = 373°K
300°C Determine the mass now rate of air. We = m Cp (1'2 - Tj)
A. 1698 kg/hr C. 1543 kg/hr We 1m = Cp (1'2' - Tj)
B. 1756 kg/hr . D. 2344 kg/hr 300 = 1 (1'2' - 373)
1'2' = 673°K
SOLUTION:

1'1 = 30 + 273 Gas Turbine - 6


1'1 = 303°K
1'3 = 750+273 The compressor for an actual gas turbine requires 300 KJ/kg of work
1'3 = 1023°K to quadruple the inlet pressure. The inlet air. temperature is 100°C
1'4 = 300 + 273 Determine the compressor efficiency.
1'4 = 573°K A. 34.56% C. 60.42%
1'2 1'3 Power B. 53.23% D. 76,34%
--- Output
100kw
1'1 1'4
1'2 1023 SOLUTION:
----
303 573
1'2 = 540.96°K P 2 = 4 PI
We =, m Cp (1'2 - 1'1) rp = P2 I PI
We = m (1)(540.96 - 303) rp = 4P 1IP j i~OKJlkg
We = 237.96 m 1'3 = 100+273
W r = mc p (1' J - 1' 4 ) 1'3 = 373 OK
W r = m(1)(1023 - 573) k-l
W r = 450 m
WOe l = Wr - We
100 = 450 m - 237.96 m
rp = 4
l'
2
1'1
(
=l~] -
k
P
1'2 14-1 1
m = 0.4716 kg/r (3600)
m = 1697.79 kg/hr 373 =(4)--;-4
192 Gas Turbine Plant Gas Turbine Plant 193
T 2 = 554.27°K A. 20 kgls C. 30 kg/s
We/m = cp(T 2 - T 1) B. 25 kg/s D. 35 kg/s
We/m = 1(554.27 - 373)
We/m = 181.27 KJ/kg
11e = 181.27/300 SOLUT~ON:

l1e = 60.42%

T, = 280 0 K
Gas Turbine - 7 PI = 100 kpa
P 3 = 1000 kpa
An ideal gas turbine operates with a pressure ratio of 8:1 and T 3 = 1167
temperature limits of 20°C and 1000°C. The energy input in the high W T = 11,190 kw
temperature heat exchanger is 200 kw. Determine the air flow rate.

.
W T = ill cp (T 3 - T4 )
A. 560 kg/hr C. 873 kg/hr P4 = PI = I 00 kpa
B. 970 kglhr D. 453 kg/hr r p = P3/P4
T
rp = 1000 I 100 3 ~~~~!<:a

O
SOLUTION: P31P4 = 10
2 11,190Kw
k-J
( 1-- • Wnet •
rp ~ 8 T3 P,I j k
. 4

QA = 200 kw
QA = m cp (T 3 - T 2)
T4 = l'P 4
1 ·100Kpa, 280 0K
S

k-I 14-1
( '\- 1167
T2 I P2 I k -=(10) 14
T4
~=lI\ j T 4 = 604.44°K
T 14-1
_--=2,--- = (8) ~

20+ 273 11,190 = ill (1)(1167 - 604.44)


T 2 = 530.75°K m = 19.89 kg/s
T] = 1000+273
T 3 = 1273°K
200 = m (I) (1273 - 530.75)
m = 0.26945 (3600) \.was 1 urbine -9
m = 970 kg/hr
An air-standard Brayton cycle has air enter the compressor at 27°C
and 100 kpa, The pressure ratio is 10 and the maximum allowable
temperature in the cycle is 1350°K. Determine the cycle efficiency per
Gas Turbine - 8 kilogram of air.
A. 48.20% C. 45.23%
In an air-standard Brayton cycle the compressor inlet conditions are B. 51.34% D. 65.23%
oK.
100 kpa and 280 The turbine inlet conditions are 1000 Kpa and
1167°K. The turbine produces 11,190 kw. Determine the air flow rate.
194 Gas Turbine Plant Gas Turbine Plant 195
SOLUTION:
85% and there is a 27 kpa drop between the compressor discharge
and the turbine inlet. Determine thermal efficiency of the cycle per
T 1 = 27+273
kilogram of air
T} = 300 K
A 34,23~'o C 3050%
I'
P2
= - = 10 B 43,23% D. 47.23%
p P
1
T 2 = T 1 (fpik-I)1k SOLUTION:
T 2 = 300 (10)(14-1)/14
T 2 = 579.6 K PI = 100 kpa
P}=P2=100kpa TI = 300 K
T] = 300(10)14-1/14
T 3 = T",ax = 1350 OK
k-I T] = 579.6 K

2i=[~]k -
579,6 - 300
T 0.85

~
~ 1 3 500 K
T P 1, - 300
4 4
1c .~ 628,goK
1.4-1
1350 P. 10(100) - 27
--=(10) 1.4 ~. Wnel • _ r p=10
=
P3 = 973 kpa
T4 (1~. 0 41 T4 = 1350(100/973)(14-1)/14
T 4 = 698.8°K _ 7 C, 100Kpa, T4 = 704,3°K
We = ill Cp ( T2 - T 1) S
We = 1(1)(579.6-300) 1350-T4
T'lt = 0,85 = ------'
We = -280.9 KJ/kg 1350 -704.1
Wt = ill Cp (T 3 - T 4 ) T4 = 801.0 OK
WT = 1(1)(1350 - 698.8) We = mCp(T] - T 1)
W T = 651.2 KJ/kg We 1(1)(628.9 - 300)
=
W net = 651.2 - 280.9 We 328,9 KJ/kg
=
W net = 370.3 KJ/kg W T = m cp(T 4 ' - T,)
QA = (T 3 - T 2)
ill C p W T = I ( I )(1350 - 80 I)
QA = 1(1)(1350 - 579.6) W T = 549 KJ/kg
QA = 770.4 QA = I1lC p (T 3 - T]')
Wnel QA = 1(1)(1350-628,9)
e =--
QA = 724.5 KJ/k~
Q" W ne, = 549 - 328.9
e = 370.41770.4
W nc ! = 220.1 KJ/kg
e = 48.06%
e = 220.11724,5
e = 30.38%

Gas Turbine - 10
Gas Turbine - 11 (ME Bd Oct. 1997)
An air-standard Brayton cycle has air enter the compressor at 27°C
A gas turbine working on an air standard Brayton cycle has air enter
and 100 kpa. The pressure ratio is 10 and the maximum allowable
oK. into the compressor at atmospheric condition and 22°C. The pressure
temperature is 13S0 The compressor and turbine efficiencies are

~.
Gas Turbine Plant 197
196 Gas Turbine Plant
785.9
ratio is 9 and the maximum temperature in the cycle is 1077"e. Combustor Efficiency
Compute for the cycle efficiency per kg of air in percent. 984.7
A. 44.85% C. 41.65%
B. 43.92% D. 46.67% Combustor Efficiency 79.8%

SOLUTION:
Gas Turbine - 13 (ME Rd. Oct. 1995)

Efficiency 1--- - T Air enters the compressor of a gas turbine at 100 Kpa and 300
0K
with
k-I 310noc
a volume flow rate of 5 m 3/sec. The compressor pressure ratio is 10
(r p) k
and its isentropic efficiency is 85%. At the inlet to the turbine, the
r.=9 pressure is 950 Kpa and the temperature is 1400°K. The turbine has
4 an isentropic efficiency of 88% and the exit pressure is 100 Kpa. On
Efficiency 1 - 14-1
(1 the basis of air standard analysis, what is the thermal efficiency of the
(9) 14 cycle in percent?
s A. 42.06 C. 31.89
Efficiency 46.62% B. 60.20 D. 25.15

SOLUTION:

Gas Turbine - 12 (ME Rd. Apr. 1996) Solving for the mass flow rate:
PV = mRT ":l 950Kpa
In a gas turbine unit, air enters the combustion chamber at 550 kpa, 100(5) = m(0.287)(300) T 1400
0K

277°(' and 43 m/s. The products of combustion leave the combustor at m = 5.81 kg/s
511 kpa, 1004°C and 140 m/s. Liquid fuel enters with a heating value Solving for T 2 :
T 2/T 1 = (p/PI)k.11k 100Kpa
of 43,000 KJ/kg. For fuel-air ratio of 0.0229, what is the combustor
T 2/300 = (10)'4.1/14
efficiency of the unit in percent?
Air Fuel T 2 = 579°K
43kg/s
SOLUTION: -;OKpa 0,=43,000 K Jlkg Solving for T 4: s
27°C T}/T 4 = (PiP 4)k.11k
Heat supplied by fuel = m, Qh 1400/T4 = (950/100)14.1/14
Heat supplied by fuel = 0.0229(43 00) T 4 = 736°K
Heat supplied by fuel = 984.7 KJ/k air Pro uct
Combustor
5111Kpa We = mC p(T2 - T 1)
100 4°C We = 5.81(1.0)(579 - 300)
Q = heat absorbed by fuel
2
Q/m = cp (T 2 - T I) + 1/2 (V 2 - V 12) We = 1621 KW
We' = 1621/0.85
We' = 1907 KW
Q/m (1.0)(1004-227) + 1/2 [(140)2 _(43)21 WT = mCp(T} - T 4 )
. 1000 J W T = 5.81(1.0)(1400 -736)
Q/m = 785.9 KJ/kg air W T = 3858 KW
WT ' = 3858(0.88)
W T ' =3395 KW
198 Gas Turbine Plant Hydro-electric Power Plant 199
WI'<' c WT ' - We'
WN ' 3395-1907 HYDRO-ELECTRIC PLANT
W N ' ~ 1488 KW
QA = mCp(T3 - T z)
QA = 5.81(1.0)(1400 - 579)
QA = 4770 KW
Efficiency = 1488/4770
Efficiency = 31.19% Hydro-electric Plant - 1

A hydro-electric power plant consumes 52,650,000 KW-hrs per


annum. Expected flow is 1665 m3/min and overall efficiency is 65%.
What is the net head?
A.30m C. 32 m
B. 31 m D. 34 m

SOLUTION:

Gen. Output
11 net =
Water Power
52,650,000/8760
0.65 = ------
Water Power
Water Power = 9246.575 KW
Water Power = wQh
9246.575 = 9.81 (1665/60)h
h = 33.966 m

Hydro-electric Plant - 2

In a hydro-electric power the tail water level fixes at 480 m. The net
head is 27 m and head loss is 4% of the gross head. What is the head
water elevation?
A. 500.34 m C. 456.34 m
B. 508.12 m D. 567.34 m

SOLUTION:

h ~ h~ + hL
27 hg + 0.04h g
hg 28.125 m
200 Hydro-electric Power Plant Hydro-electric Power Plant 201

h g = H.W.Elev - T.W.r::lev. h = 33.25 ill


28.125 = H.W.Elev - 480 Water Power = wQh
H.W.Elev = 508.J25 ill Water Power = 9.81(10)(33.25)
Water Power = 3261.825 KW

Hydro-electric Plant - 3
Hydro-electric Plant - 5
The available flow of water is 25 illJ/sec at 30 m elevation. If a hydro-
electric plant is to be installed with turbine efficiency of 85% and The flow of a river is 20 mJ/sec and produces a total brake of 6,000
generator efficiency of 90%, what maximum power that the plant KW. If it is proposed to install two turbines each has 85% efficiency,
could generate? what is the available head?
A. 4658.5 KW C. 5628.5 KW A. 35 ill C. 39 ill
B. 3478.5 KW D. 4756.5 KW B.37m D. 36 ill

SOLUTION SOLUTION

Water Power = wQh Water Power ~ 6,000/0.85


Water Power = 9.81(25)(30)
Water Power = 7357.5 kw Water Power = 7058.82 KW

Gen. Output 7357.5 (0.85)(0.9) Water Power = wQh


Gen. Output 5628.49 kw
7058.82 = 9.81(20)h

h = 35.98 m
Hydro-electric Plant - 4

For a proposed hydro-electric plant, the tail water and head water
elevation is 160 m and 195 m, respectively. If available flow is 10 Hydro-electric Plant - 6
nr'zsec and head loss of 5% of water available head. What is the water
power? Two turbines generates a total brake power of 5000 KW. If one unit is
A. 3261.8 KW C. 5874.5 KW thrice the capacity of the other, find the capacity of smaller unit.
B. 4254.6 KW D. 2456.5 KW A. 1250 KW C. 2456 KW
R 3450 KW D 5763 KW

SOLUTION: SOLUTION'

hg = H.W.E1ev - T.W.Elcv WT WI + W,
-r-;

hg = 195 - 160 W:= 3W,


h, ~, 35 ill 5000 c. \\/,t 3\\.
h = hg - h L WI = 1250 KW
h = 35 - 0.05(35)
202 Hydro-electric Power Plant Hydro-electric Power Plant 203

Hydro-electric Plant - 7 Hydro-electric Plant - 9

In a hydro-electric plant the brake power is 1800 KW running at 450 For a generator running at 300 rpm and 60Hz, find the number of
rpm and net head of 30 m. Determine the specific speed of the turbine. generator poles.
A 6029 rpm C. 75.29 rpm A. 24 poles C. 18 poles
R 65.29 rpm D. 71.29 rpm B. 8 poles D. 20 poles

SOLUTlON:
SOLUT£ON:
Hp cc 1800 x I Hp/O.746KW
n, = 24]2.87 Hp 120 f
N
N,
NJHP p
h 5/4
450J2ill.S7 120(60)
300 --
N. = (30x3.281)5!4 p
N, = 71.29 rpm
P 24 poles

Hydro-electric Plant - 8
Hydro-electric Plant - 10
The specific speed of turbine is 75 rpm and running at 450 rpm. If the
The penstock of hydro-electric plant is 0.5 x 0.5 m with velocity of 5.5
head is 20 m and generator efficient) is 90%, what is the maximum
mlsec has a head of 20 m. What is the output of the turbine if turbine
power delivered by the generator.
efficiency is 87%?
A. 4505 KW C. 650.5 KW
A. 845.32 K\V C. 654.56 KW
B. 354.5 KW D. 7805 KW
B. 789.34 KW D. 234.56 KW
SOLUTION
SOLUTION:

N.JHP Q A xv
'" -
=
"'s' h 5/ 4 Q (0.5 x 0.5)(5.5)
=
Q = 1.375 m 3/sec
450!HP
Water Power = w Q h
Water Power = 981(1.375)(20)
Water Power = 269.775 KW

Generator Output (%X92 x 0.746)(0.9) Turbine Output - 269.775(0.87)


Generator Output 65():,\ KW Turbine Output 234.70 KW
204 Hydro-electric Power Plant Hydro-electric Power Plant 205

Hydro-electric Plant - II Hydro-electric Plant - 13

A 410 x 0.510 channel has water velocity of 5 m/sec. If the head is 100 '" turbine has a mechanical efficiency of 95%, volumetric efficiency of
m, what is the annual energy produced if overall efficiency is 70%? and total efficiency of 80%. If effective head is 40 rn, find the
l)"1('l'"

A. 40,456,000 KW-hrs C. 60,154,920 KW-hrs total head.


B. 34,456,345 KW-hrs D. 54,867,234 KW-hrs A. 48.72 m C. 46.72 m
B 4072 m D. 34.72 m

SOLUTION: SOLU'ION

Q Ax v
= TlT = llmllhllv
Q (4 x 0.5)(5)
= '0.8 =0.9S(Ylil)(0.97)
3
Q
= 10 m (sec Tlh 0868
=
Water Power c , w Q h Total head = hllh
Water Power 0' 9.81(10)(100) Total head ~. 40(0.868)
Water Power" 9810 KW Total head 'c 3472 m
Annual Energy Produced = 981O(8760hrs)(0.7)
Annual Energy Produced = 60,154,920 KW-hrs
Hydro-electric Plant - 14

A Pelton type turbine has 30 m head friction loss of 4.5 m. The


coefficient of friction head loss(from Moorse) is 0.00093 and penstock
Hydro-electric Plant - 12 length of 80 m. What is the penstock diameter?
A. 1234 mrn C. 677 mm
In Francis turbine, the pressure gage leading to the turbine casing B. 3476 mm D. 1686 rnm
reads 380 Kpa and center of spiral casing is 3 m above the tailrace. If
velocity of water entering the turbine is 8 rn/sec, what is the net head SOLUTION:
of the turbine?
A. 30 m C. 40 m y ~2gh
B. 35 m D. 45 m
h 30 - 4.5
h 25.5
SOLUTION:
y ~2(9.81)(25.5)
y 2237 m/sec
P v
h = -+z+- 2 f L y2
w 2g hl =
gD
380 8) 2( 0.00(93)(80)(223 7) 2

I
I
i
h = -+3+--
9.81 2(981)
4.5

D I .(,l~6 IT1
9.81 D

i h = 45 m D I M{h mITI
1

l
206 Hvdro-electric Power Plant Hydro-electric Power Plant 207

A. 58.960 KW-hl's C. 65,960 K).V-hrs


Hvdro-electric Plant - 15 B. 60,960 KW-hl's D 70,960 Kw-hrs

From a height of 65 m water flows at the rate of 0.85 m 3/s and is SOLUTION:
driving a turbine connected to 160 rpm generator. If frictional torque
is 540 N-m, calculate the turbine brake power. Plant Capacity 8,000(0.88 )(24i
A. 533 KW C. 455 KW Plant Capacity 168,960 KW-hrs
B. 677 KW D. 488 KW
Secondary Power 168.960 - 110.000
SOLUTION: Secondary Power 58.960 K\N-Im.

WT = w Q h - Friction power

Friction Power = 2 IT T N Hydro-electric Plant - 18 (ME Bd. Oct. 1986)


Friction power = 2 IT (0.054)(160/60)
Friction power = 9.047 kw At a proposed hydroelectric plant site, the average elevation of
WT = 9.81(0.85)(65) - 2n(0.54)(160/60) headwater is 600 m, the tailwater elevation is 480 ill. The average
WT = 532.95 KW annual water flow is determined to be equal to that volume flowing
through a rectangular channel 4 m wide and 0.5 m deep and average
velocity of 5.5 m/sec. Assuming that the plant will operate 350
Hydro-electric Plant - 16 days/year, find the annual energy in Kwh that the plant site can
develop if the hydraulic turbine that will be used has an efficiency of
What is the turbine output of 100 m head and delivering 1 m
3/sec
in a 80% and generator efficiency of 92 % • Consider a headwork loss of 4%
hydro-electric power plant if turbine efficiency is 88%? of the available head.
A. 863.28 KW C. 167.34 KW A. 76,854,851 C. 75,234,45'5
B. 734.28 KW D. 492.34 KW B. 65,234,556 D. 82,456,677

SOLUTION:
SOLUTION:
WT = (wQh)(llT)
hg =H.W.Elev - T.W.Elev
WT = [(9.81)(I)(l00)J(0.88) hg =600 - 480
hg = 120 m

WT = 863.28 KW h = 120 - 0.04(120)


h = 115.2 In
Q = Axv
Q = 4(0.5)(5.5)
Q = II mJ/sec
Hydro-electric Plant - 17
Generator Output = (w Q h)IlTf)C;
In an 8,000 KW hydro-electric plant the over-all efficiency is 88% and Generator Output = (9.81 x I i x 1152)(OR)(092)
the actual power received by the customer is 110,000 KW-hrs for that Generator Output = 9,149.39 KW
day. What is the secondary power could this plant deliver during the Annual Energy 9,149.39(24)(350)
-c;

entire day? . Annual l.ncrgy = 76,854,851 KW-hrs


208 Hydro-electric Power Plant 209
Hydro-electric Power Plant
v = 2.368 m/sec
Hydro-electric Plant - 19 (ME Rd. Apr. 1992) 2
2f Lv
hL =
In Maria Cristina Hydroelectric Project, the available head is 140 10; gO
the water flow is one cubic meter per second; efficiency of the turbine 2( 0.00093)(80)(22.368)
is 95%; efficiency of the generator is 95%, three phase, 60 cycles, 4.5 =
voltage delivered is 4160 V. Determine yearly income of the plant if the 9.810
cost of the generated electric energy is PO.60 per Kwh. o = 1.686 m
A. P5,234,567 C. P6.514,778 Q = Axv
B. P7,385,678 O. P9,354,677 Q = [n/4 (1.686)2](22.368)
Q = 49.94 rrr'zsec
Power = w Q h = 9.81(49.94)(25.5)
SOLUTION: Power = 12,492.74 kw

Generator Output = (w Q h)llT llG


Generator Output = (9.81 x 1 x 140)(0.95)(0.95)
Generator Output = 1239.49 KW Hydro-electric Plant - 21 (ME Rd. Apr. 1989)
3/sec
Annual Energy = 1239.49(8760) From a height of 65 10, water flows at the rate of 0.85 m and is
Annual Energy = 10,857,963.06 KW-hrs driving a water turbine connected to an electric generator revolving at
Yearly Income = 10,857,963 x PO.60 160 rpm. Ca lculate the power developed by the turbine in KW if the
Yearly Income = P6,514,777.80 total resisting torque due to friction is 540 N.m and the velocity of the
water leaving the turbine blades is 4.75 m/sec.
A. 753 C. 635
B. 523 O. 833
Hydro-electric Plant - 20 (ME Rd. Apr. 1987)

A Pelton type turbine was installed 30 10 below the head gate of the SOLUTION:
penstock. The head loss due to friction is 15 percent of the given
elevation. The length of penstock is 80 10 and coefficient of friction is (4.75)2
0.00093. Determine the power output in KW. h 65----
~~
2(9.81)
A. 12,273 C. 12,345
h = 63.85 m
B. 13.2:3'1 O. 12,493
Water Power = w Q h
Water Power = 9.81(0.85)(63.85)
Water Power = 532.41 kw
SOLUTION:
1~:-

r
Friction Power 2nTN
hL 0.15(30)
= L JQ Friction Power
r;

~~ 2 rt (0.54 )(160/60)
hL = 4.5 m Om
h = 30 - 4.5 Friction Power = 9 KW
h = 25.5 m
Turbine Power = 532.4 - 9
v = ~2gh
Turbine Power = 532.40 kw
v = ~2(9.81)(25.5)
210 Hydro-electric Power Plant Hydro-electric Power Plant 211

Hydro-electric Pian! - 22 (ME Bd, Oct. I 985) Hydro-electric Plant - 23 (ME Bd, Apr. 1990)

A proposed hydro-electric power plant has the following data: A Francis turbine is installed with a vertical draft tube. The top of the
Elevation of normal headwater surface = l 94 m draft is 1.0 m below the center line of spiral casing while the tail race
Elevation of normal tailwater surface = 60 m water level) is 3.0 m from the top of the draft tube. There is no velocity
Loss of head due to friction = 6.5 m of whirl at the top or bottom of the draft tube and leakage are
Turbine discharge at full gate opening = 5 m 3/sec negligible. The elevation of water from the reservoir to the center line
Turbine efficiency at rated capacity = 90% of the turbine spiral casing is 50 m, and water velocity at the inlet is 5
Turbine is to be connected to a 60 cycle AC generator. m/sec. Discharge is 2.5 m 3/sec, hydraulic efficiency is 86%, overall
Find the num ber of poles of the generator efficiency of 83%. Determine the reading of a pressure gage(in psi) if
A. 6 C. 8' one is installed at the penstock just before the water inlet to the
B. 10 D. 12 turbine.
A. 69.35 C. 56.34
B. 74.33 D. 92.45
SOLUTION:

h = (194 - 60) - 6.5 SOLUTION:


h
h
=
=
127.5mx3.281
418.2 feet
h =
P
-+-
yl 1I,!-
'= s
I - om
~~r;;y
r: _
Brake Power = (w Q h}rh
W 2g i
1m+=: _ '"_~._
Brake Power = (9.81 x 5 x 127.5)(09) P <;2 i)lt,
Brake Power = 5,628.5 KW
50 :.' - ' - + --.--
9.81 2(9.81)
4-- -
From MRII charts and tables: P = 478 Kpa x 14.7/101.325
For 418.2 feet net head, using Francis Turbine P = 69.35 psi
N, = 33
NJHP
Ns h 5/4
N.J5628.5 I 0.746 Hydro-electric Plant - 24 (ME Bd. Apr. 1990)
33 _.~~~--

(418.2)5/4
H.W. Elev. = 194m A Francis turbine is installed with a vertical draft tube. The top of the
draft is 1.0 ill .below the center line of spiral casing while the tail race
N = 718.5 rpm water level) is 3.0 m from the top of the draft tube. There is no velocity
of whirl at the top or bottom of the draft tube and leakage are
Brake
120f Power negligible. The elevation of water from the reservoir to the center line
N
p of the turbine spiral casing is 50 m, and water velocity at the inlet is 5
m/sec. Discharge is 2.5 m 3/sec, hydraulic efficiency is 86%, overall
120(60) T.W. Elev. =60m efficiency of 83%. Determine the mechanical efficiency of the turbine
718.5 = A 76.23% C. 96.51 %
P B. 83.45% D. 64.34%
P = 10.02 poles say 10 poles
Hydro-electric Power Plant 213
212 Hydro-electric Power Plant
SULUTION Hvdro-electric Plant - 26 (ME Rd. Oct. 1989)

A hydro-electric plant has a 20 MW generator with an efficiency of


111 llh 11m 11v 96%. The generator is directly coupled to a vertical Francis type
where: 11v = 1.0 (ifnot given) hydraulic turbine having an efficiency of 80%. The total gross head on
0.83 = 0.86( 11rn)( 1.0) the turbine is 150 m while the loss of head due to friction is 4% of the
11m = 96.51% gross head. The runaway speed is not to exceed 750 rpm. Determine
the flow of water through the turbine in cfs,
A. 651 C. 763
B. 534 D. 827

Hydro-electric Plant - 25 (ME Rd. Apr. 1984)


SOLUTI0N:
A remote community in Mountain Province plans to put up a small
hydro-electric plant to service six closely -located barangays estimated h = hg - h L
to consume 52,650,000 KW-hrs per annum. Expected flow of water is h = 150 - 0.04(150)
1665 m 3/min. The most favorable location for the plant fixes the tail h = 144 m
water level at 480 m. The manufacturer of turbine generator set have Generator Output = (w Q h) 11T 11G
indicated the following performance data: turbine efficiency - 87%; 20,000 = (9.81 x Q x 144)(0.8)(0.96)
3
generator efficiency is 92%; loss in head work not exceed 3.8% of Q = 18.435 m3/sec x 35.3 I ft 1l m'
available head. In order to pinpoint the most suitable area for the Q = 650.94 cfs
dam, determine the head water elevation
A.234m C. 345 m
C. 842 m D. 509 m
Hydro-electric Plant - 27 (ME Rd. Oct. 1989)

SOLUTION: A hydro-electric plant has a 20 MW generator with an efficiency of


96%. The generator is directly coupled to a vertical Francis type
Annual Energy Output = (w Q h)l1T11o x 8760 hydraulic turbine having an efficiency of 80%. The total gross head on
52,650,000 = (9.81 x 1665/60 x h)(0.87)(0.92)(8760) the turbine is 150 m while the loss of head due to friction is 4% of the
h = 27.583 m gross head. The runaway speed is not to exceed 750 rpm. Determine
the rated speed of the turbine.
h = hg - h L A. 300 C. 400
h = hg - 0.038h g B. 500 D.600
h = 0.962 h g
27.583 = 0.962 hg
h g = 28.67 m SOLUTION:

Head water elevation 480 + 28.67 h = h g - hL


Head water elevation 508.67 m h = 150 - 0.04(150)
h = 144 m
From MRII charts and tables, for 144 m(472 ft) head, is
2J4 Hydro-electric Power Plant Hydro-electric Power Plant 215

within the range or francis turbine with N, 29


20,000 Hydro-electric Plant - 29 (ME Bd. Oct. 1995)
HP
096(0746)
A hydroelectric generating unit station is supplied from a reservoir of
HP 27,927 hp
capacity 6,000.000 m'' at a head of 170 m. Assume hydraulic efficiency
Ns =
NM of 80% and electrical efficiency 90%. The fall in the reservoir level
h 5/4 after a load of 15 MW has been supplied for 3 hours, if the area of the
NJ27j27 reservoir is 2.5 sq. Km is closet to:
29 = A S39cm C 5.98 em
(472)5/4
B. 4.32 em D. 4.83 em
N = 382 rpm, use 400 rpm (standard)
SOLUTION
A = 2.5 krn
r-. -:
Ac :2.5 x 10 m '
Hydro-electric Plant - 28 (ME Bd. Oct. 1995)
Generator Output = (w 0 h)11 r 11G
15,000' (981 x Q x 170)(0.80)(U90)
Water flows steadily with a velocity of 3.05 mls in a horizontal pipe Q .zz: 12.492 mJisec
having a diameter of 15.24 em. At one section of the pipe, the A her 3 hours,
temperature and pressure of the water are 21°C and 689.3 Kpa, Q ~ I:2A92(3x3600)
respectively. At a distance of 304.8 m downstream, the pressure is C) = 134.914 m 3
516.9 Kpa. What is the friction factor? Volume =c Area x Height
A. 0.134 e.0.0189 134,914 = (2.5 x 10") H
B. 0.0050 D. 0.641 H = 0.0539 m
H = 5.39 ern

SOLUTiON:
Hydro-electric Plant - 30 (M E Bd. Oct. 1995)
p - P
hL ~
~
_d_ _s

w A pelton wheel is to be designed to run at 300 rpm under an effective


689.3-516.9 head of 150 m. The ratio of the nozzle diameter of the pitch circle is
hL = 1/12. Assuming efficiency of 84%, what is the size of the wheel in
9.81
h L=17.574m meters. Assume a speed ratio of 0.45.
A. 1.05 C. 1.55
2 B 2.00 D. 2.86
f Lv
hL =
2gD SOLUTION:
nDN
q)= ~-~

[(3048)(3.05/ ~2gh
7.574 =
2(9.81)(0.1524) n D (300 / 60)
OA'i
f = 0.0 185 )2(9.81)(150)
I) I"''i In
216 Hydro-electric Power Plant Hydro-electric Power Plant 217

---_.- -- 275 (035)2


Hydro-electric Plant - 31 (ME na. Oct. 1995) h -+---
9.81 2(9.81)
A Francis turbine is installed with a vertical draft tube. The total head
to the center of the spiral casing at the inlet is 38 m and velocity of h = 28.039 m
water at the inlet is 5 m/sec, The discharge is 2.1 m3isec. The hydraulic
~iJJI' Turbine
efficiency is 0.87 and overall efficiency is 0.84. The top of draft tube is nON
1 m( water) below the center line of the spiral casing while the tailrace <1>= J2gh Tail race
(water) level is 3m from the top of the draft tube. Neglected velocities
of whirl and leakage losses. What is the total head on the turbine in 1[ (0.55)(520/60)
meters? <1> = ----;=====
A 34.72. C.,55.20 J2(9.81)(28.039)
B. 43.27 D 48.12 «1> = 0.638

SOLUTION:
Hydro-electric Plant - 33 (ME Bd, Apr. 1998)
h = total head
p V 2_V 2
A hydraulic turbine receives water from a reservoir at an elevation of
h = - + Z+ A B LOO meters above it. What is the minimum water now in kg/s to
W 2g
produce a steady turbine output of 50 Mw?
5 2 - 02 A. 50,247 C. 50,672
h 38 + (1 + 3) + 2(9.81) B. 50,968 D. 50,465
h = 43.27 m
SOLUTION:

Water Power wQh Reservoir


Hydro-electric Plant - 32 (ME Bd, Apr. 1998) .~.

100m
A hydro-electric plant discharges water at the rate of 0.75 m3/s and 50Mw
enters the turbine at 0.35 mps with a pressure of 275 Kpa. Runner
Q = 50.968 m 3/s
inside diameter is 550 mm, speed is 520 rpm and the turbine efficiency
is 88%. Find the turbine speed factor.
A. 0.638
m = 50.968 (lOOJ)
C. 0.368
B. 0.386 D. 0.836
m = 50,968 kg/s
SOLUTION:

Hydro-electric Plant - 34 (ME Bd, Apr. 1997)


P v2
h -+-
W 2g A hydro-electric plant having 50 sq. km reservoir area and 100 m head
is used to generate power. The energy utilized by the consumers
whose load is connected to the power plant during a five-hour period is
218 Hydro-dearie Power Plant 219
Chimney
6
13.5 X 10 kwh. The overall generation efficiency is 75%. Find the fall
in the height of water in the reservoir after the 5-hour period. CHIMNEY
A. 2.13 m C. 3.21 m
B. 1.32 m D. 0.53 m

SOLUTION: Chimney - I

Energy Output = Power x time A boiler uses 2500 kg of coal per hour and air required for combustion
Energy Output = (w Q h)11 x time h I.:. .... _.. _.. __ .. is 16 kg per kg coal. If ash loss is 10%, determine the mass of gas
6
13.5 x 10 = 9.81(Q)(100)(0.75)(5) Reservoir entering the chimney.
Q = 3669.725 m 3/s s: 42,250 kg/hr C. 85,452 kg/hr
B 78,300 kg/hr D. 33,800 kg/hr
Volume after 5 hrs = 3669.725(5 x 3600)
Volume after 5 hrs = 66,055,050 m ' SOLUT10N:

Volume = A x height rng rna 1- m, - mash


66,055,050 = (50 x 106 ) h
h = 1.321 rn A/F= rna / m-
16 =- rna / m.
m a = 16 rnf

Hydro-electric Plant 35 - (Math-ME Bd. Oct. 1997) mg = l6mr+mr-O.lInf


mg = 16.9 m,
A reaction turbine develops 500 bhp. Flow through the turbine is 50 rn g = 16.9(2500)
cfs, Water enters at 20 fps with a 100 ft pressure head. The elevation mg = 42,250 kg/hr
of the turbine above the tailwater level is 10 ft. Find the effective head.
A. 130 ft C. 110 ft
B. 120 ft D. 116.2 ft
Chimney - 2

SOLUTION: The gas density of chimney is 0.75 kg/m'' and air density of 1.15 kg/rn ',
. If the driving pressure is 0.25 Kpa, determine the height of chimney,
P V 2_V 2 A. 54.6 m C. 74.6 In
h = -+z+ A B B. 63.7 m D. 68.5 In
W 2g 10ft

100+ 10 + (20)2_ 0 2 SOLUT10N:


h
2(32.2) h., = H(d. - dg )

h 116.21 ft 0.25 = H( 1.15 - 0.75)(0.00981)

H = 63.7l In
220 ( 'h nnncy 22\
Chimney
rn, TIn, InA~Jl t In l',

Chimney - 3 46,500 . 3000 o 11(3000) +- mg


m~ = 49,170 k~hr

The actual velocity of gas entering in a chimney is 8 m/sec, The gas PV = mR'1
temperature is 25°C and pressure of 98 Kpa with a gas constant of 98.2(V g) (49.170/3600)(0.277)(285 -t- 273)
V g = 21.498 m 'zsec
0.287 KJ/kg_°K. Determine the chimney diameter if mass of gas is
50,000 kg/hr. Let 0 = diameter of chimney
A. 1.57 m Q = Axv
C. 3.56 m
B. 1.81 m O. 1.39m Q = ( rc/4 02)V
21.498 = (rc/4 0 2)(7 5)
SOLUTION: D = 19\ m

Pg V s = mg Rg T g

98(V g) = (50,000/3600)(0.278)(25 + 273) Chimney - 5 (ME Bd, Apr. 1990)

Vg = 12.12m3/sec A power plant situated at an altitude having an ambient air of 96.53


Kpa and 23.88°C. Flue gases at a rate of 5.0 kg/sec enter the stack at
Vg = A x v 200°C and leaves at 160°C. The flue gases graVimetric analysis are
18% COz, 7% O, and 75% N z. Calculate the height of stack necessary
11.74 = rc/4 0 2 (8) for a- driving pressure of 0.20 Kpa.

D = 1.39 m SOLUTION:

Solving for the molecular weight and gas constant of the flue gas:

Chimney - 4 (ME Bd. Apr. 1981) CO 2 18% 0.18/44 = 0.00409


O2 7% 0.07/32 = 0.00219
A coal fired steam boiler uses 3000 kg of coal per hour. Air required N2 75% 0.75/28 = 0.02678
for combustion is 15.5 kg per kg of coal at barometric pressure of 98.2 0.03306
Kpa. The flue gas has temperature of 28SoC and an average molecular
weight of ~O. Assuming an ash loss of 11 % and allowable gas velocity Mg = 110.03306
of 7.S mlsec, find the diameter of chimney. M, = 30.25
R g = 8.314/30.25
SOLUTION: R g = 0.275
T g = (200 + 160)/2
Amount of air required = 15.5(3000) T g = 180°C
Amount of air required = 46,500 kglhr mf FURNACE
• 285°C d g = P/RT
96.53
dg =
R = 8.3141M (0.275)(180 + 273)
R = 8.314/30 d g = 0.775 kg/m'
R = 0.277 KJlkgOK d. = P/RT
By mass balance: m(ash)=11 %mf o
-,"'-:.
Chimney
Chimney 223
l)().'i 3
do h., = 26.30 01 of flue gas
(02S7)(23.88 + 273)
d•.= I.! 33 kg/m' v. = 'ihl9
-\..8\ V'\ 76 '")
- .~.
Draft lIZ d. - d g )
0

v cc 22.716 m/sec
0.20 = H(1I33 - 0.775)(0.00981)
H = 56.95 m Actual velocity 40% v
Actual velocity 0.40(22.716)
Actual velocity 9.1 In/sec
Q = Ax v
Chimney - 6 (ME Bd. Apr. 1990) (5/0.775) = (11'/4 D
2)(91)

D = 0.95 m
A power plant situated at an altitude having .an ambient air of 96.53
a
Kpa and 23.88 C. Flue gases at a rate of 5.0 kg/sec enter the stack at
a
200 C and leaves at 160 a C. The flue gases gravimetric analysis are
18% CO z, 7% O 2 and 75% N z• Calculate the diameter of stack in Chimney - 7 (ME Ed. Apr. 1995)
meters for a driving pressure of 0.20 Kpa.
A steam generator with economizer and air hearer has an O'\"IC)'g H draft
SOLUTTON: loss of 21. 78 em of water. If the stack gases are at l7i"C and jf the
atmosphere is at 101.3 Kpa and 26°C, what theoretical height of stack
Solving for the molecular weight and gas constant of the flue gas: in meters is needed when no draft fan arc used? Assume that th~' gas
constant for the flue gases is the same as that for air.
CO 2 18% 0.18/44 = 0.00409 A. 565 C. 545
O2 7% 0.07/32 = 0.00219 B. 535 D. :'50
N2 75% 0.75/28 = 0.02678
0.03306
SOLUTION:
M g = 1/0.03306
M g = 30.25 YV :.-=- P'"E.T
R g = 8.314/30.25 101.325
R, = 0.275 do
200+ 160 Z0.287)(26 + 273)
Tg d. = I.! 80 kgn-: 3
2
T g = 180°C 101.3
dg = P/RT dg
96.53 (0.28 7 )0 T' -i- 2n)
dg dg 0.784 kg im3
(0.275)(180 + 273)
d g ~~ 0.775 kg/rn ' Draft O.2178( lOOO)
v = theoretical velocity Draft == 217.8 Kg/rn'
v = ~2gh", Draft = H(d a - dwl
0.20 217.R H(Ll8 - 0.78"+)
hw = !1 5~(\ m
0.775(OOO9RI)
224 Chimney Chimney 225

A.46 C. 40
Chimney - 8 (ME Bd, Oct. t 995) B. 50 D. 56

If the actual draft required for a furnace is 6.239 em of water and the SOLUTION:
frictional losses in the stack are 15% of the theoretical draft, calculate
the required stack height in meters. Assume that the flue gas have an
t o =12°C I I

average temperature of 149°C and molecular weight of 30. Assume air


temperature of 21°C
A. 215 C. 220
m,~9,OOOkg/hr
i i
I
IH
B. 230 D. 210
I

SOLUTION ---+
hw " total draft
h, = 6.239 ~. O.ISh"
Ill' =, 7.34 em water h; = hxw
h; = 00734(9.81) h; = (0.02286)(994.78)
h, = 0.72 Kpa h; = 22.74 kg/rrr'
p
d'l -c. PRT d =--
101325 • RaTa
d,
(0.287)(21 +273) P = 760 mm Hg
d, = 1.2 kg/m ' P = 101.325 kpa
dg ~ P/RT 101.325
da - -
101325 (0.287)(32 + 273)
d =
g (8.314/30)(149+273) d, = 1.157 kg/rrr'
d g = 0.867 kg/m ' P
ds =
h",= H(do - d g) R g Tg
0.72 = H(1.2 - 0.867)(0.00981)
H ~c 220 m 101.325
ds = --

(0.287)(260 + 273)
d g = 0.662 kg/rrr'
h... = H(d a - d g )
Chimney - 9 (ME Bd, Apr. 1998) 22.74 = H(!.!57 - 0.662)
H = 46m
A steam boiler plant consumes 9,000 kg of coal per hour and produces
20 kg of dq flue gases per 'kg of coal fired. Outside air temperature is
32"C, average temperature of the flue l::as entering the chimney is
343°C and average temperature of dry flue gas in the chimney is
260"C. The gage fluid density is 994.78 kg per m' and the theoretical
draft of 2.286 em of H 20 at the chimney base is needed when the
barometric pressure is 760 mrn Hg. P .terrnine the height of the
chimney,
226 Machine Foundation Machine Foundation 227

MACHINE FOUNDATION
Machine Foundation - 5

The machine foundation must have a factor of safety of _


Machine Foundation - I A.3 C.5
B.4 D.6
All heavy machinery should be supported on of sufficient
mass and base area to prevent or minimize the transmission of The answer IS: C
vibrations.
A. steel foundations C. solid foundations
B wooden foundations D. soil foundations Machine Foundation- 6

The answer is: C Foundations should be isolated from floor slabs or building footings at
least _ _ mm around its perimeter to eliminate transmission of
vibration.
Machine Foundation - 2 A. 20 C. 30
B 25 D. 35
Foundations mass should be from times the weight of the
machinery it is supposed to support. The answer is: B
A. 3 to 5 C. 4 to 6
B. 5 to 7 D. I to 2
The answer is: A Machine Foundation - 7

Machine Foundation - 3 Foundations are preferably built of concrete in the proportion of

If the unbalanced inertia forces produced by the machine can be A.I:2:3 C. 1:2:5
calculated, a mass weight equal to times the forces should be B. 1:3:3 D. 1:2:4
used to dampen vibration.
A. 2 to 4 C. 10 to 20 The answer is: D
B. 5 to 6 D.l0tol5

The answer is: C


Machine Foundation - 8

Machine Foundation - 4 The machine should not be placed on the foundation until 7 days have
elapsed or operated until another "_ days have passed.
For stability, the total combined engine, driven equipment, and A 6 C. 8
foundation center of gravity must be kept _ _ the foundation's top. B.7 D. 10
A. above C. I m
B. below D. none of these The answer is: B
The answer is: B
"'2X 111acllllll' loundation Machine Foundation 22Y

Machine Foundation - 9 Machine Foundation - 13

Concrete foundations should have steel bar reinforcements placed No foundation bolts shall be less than mm diameter
both vertically and horizontally, to avoid . A. 10 C. 14
A. damaging C. superheating B. 12 D. 16
8 thermal cracking D. superposing The answer is: B

The answer is 8

Machine Foundation - 14

Machine Foundation - 10 Machine should be leveled by driving wedges between the machine's
base and concrete foundation and with the aid of a _ _
The weight of reinforced steel in a foundation should be from of A. hose and water C. spirit level
the weight of the foundation. B. try square D. level hose
A. l/2% to 1% C. 2% to 3%
B. 1% to 2% D. 3% to 4% The answer is: C

The answer is: A

Machine Foundation - 15

Machine Foundation - 11 Grouting all spaces under the machine bed with a thin mixture of one
part cement and _ _ part sand.
Foundation bolts of specified size should be used and surrounded by a A. I C. 3
pipe sleeve with an inside diameter of at least _ _ times the diameter B. 2 D. 4
of the anchor bolt.
A. I C.3 The answer is: A
B. 2 0.4

The answer is: C


Machine Foundation - 16

For foundation of stacks, the maximum pressure on the soil is _ _ to


Machine Foundation - 12 the pressure due to the weight and the wind movement.
A. more than C. equal
Foundation bolts length should be at least times the diameter of B. less than D. none of the above
the anchor bolt.
A. 12 C. 16 The answer is: C
B. 14 D. 18

The answer is: 0


230 Machine Foundation 231
Machine Foundation

Machine Foundation - 17 Machine Foundation - 22

Guyed stacks seldom exceed 1.83 m diameter and _ _ m high. The steam turbine foundation should be designed to support the
A. 32.45 C. 43.23 machine load plus _ _ for impact, condenser load, floor loads and
B. 36.34 D. 30.48 dead loads.
The answer is: D A.10% C. 25%
B. 15% D. 50%·
The answer is: C

Machine Foundation - 18

The angle betwee.i the stacks and the guy wire is usually _ _ degrees. Machine Foundation - 23
A. 45 C. 75
B. 60 D. 90 For diesel engines foundation the concrete mixture must be !:2:4, and
The answer is: B the broken stone or gravel must have a size of mm maximum.
A. 30 C. 50
B. 40 D. 60
Machine Foundation - 19 The answer is: C

The angle between wires in a set is _ _ degrees for a set of three.


A. 90 C. 110
B. 100 D. 120 Machine Foundation - 24
The answer is: D
In pouring a concrete mixture for foundation of diesel engine, it should
be poured _ _ time .
Machine Foundation - 20 A. 1 C.3
B. 2 D.4
The maximum unit pressure of turbine and generator on the The answer is: A
reinforced concrete should not exceed _ _ kg/cm 2 •
A. 17.62 C. 19.34
B. 15.34 D. 21.34
The answer is: A Machine Foundation - 25

After pouring the foundation of diesel engine, the tops should be


covered and wet down twice daily until the forms are removed at the
Machine Foundation - 21 end of _ _ day.
A. third or fourthC. fourth or fifth
For steam turbine foundations the concrete mixture should be B. second or third D. none of these.
A. 1:2:3 C. 1:3:5
B. 1:2:4 D. 1:2:2 The answer is: A
The answer is: B
Machine Foundation 233
232 Machine Foundation

Machine Foundation - 26 Machine Foundations - 30 (ME Bd. Oct. 1995)

The diesel engine should not be place on the foundation until 10 days What is the required base area of the foundation to support an engine
have elapsed, nor operated until after another _ _ days. with specified speed of 1200 rpm and weight of 9,000 kg? Assume
A. 7 C. 9 bearing capacity of soil as 47.867 kpa. Use e = 0.11.
o 2
B. 8 D. 10 A 5.57 m" C. 7.75 m
2
The answer is: 0 B. 8.87 m 2 D. 10.5 m

Machine Foundation - 27 SOLUTION:

The safe soil bearing pressure of diesel engine foundations is


A. 4,600 kg/ern' C. 5,633 kg/ern" WF = weight of foundation
B. 4,890 kg/ern" D. 2600 kg/ern" WF = exWex.,fN
WF 0.1 1(9,000).J1200
The answer is: B Wr 34,295 kg

We + W F
Sb =
A
Machine Foundation - 28 (9,000+ 34,295)(0.00981)
47.867
The foundation depth may be taken as a good practical rule, to be A
_ _ times the engine stroke. A = 8.87 m2
A. 2.3 to 4.5 C. 3.2 to 5.2
B. 3.2 to 4.2 D. 5.2 to 6.2
The answer is: B
Machine Foundations - 31

A foundation measures 3 m x 4 m x 5 m. Find the yd:' of stone needed


Machine Foundation - 29 for a concrete mixture of 1:2:4.
A. 65 C. 78
Anchor bolts should be embedded ;'1 concrete of at least times B. 56 D. 69
the bolt diameter.
A. 10 C. 30 SOLUTION:
B. 20 0.40

The answer is: C v 3x4x5


V 60 m3
(3.281)3
V = 60----
(3)3
V ~ 78.48 yd'
For 1 yu 3 of concrete, it needs 0.88 yd'' of stone.
234 Machine Foundation
Machine Foundation 235
Therefore.
SOil :T10N:
78 .48 _vd ' concrete - 088-yd 'stone
Volume of stone .. _. - _.- J --
yd concrete
(b l+b 2)h
A =
Volume of stone 69 yd' of stone 2
(2 + 3)( 1.5)
A =
2
A = 3.75 m 2
Machine Foundations - 32 V = A L
V = 3.75(4)
A foundation measures 10 ft x I2 ft x 15 ft. Find the number of sacks V = 15 rrr'
of cement needed for 1:2:4 mixture. V = 19.622 yd 3
A 300 C. 350
B. 400 D. 450 For I yd'' concrete, it needs 0.44 yd' of sand. Therefore,

SOLUTION: Volume of sand 0.44(19.622)


Volume of sand 8.633 yd 3 of sand

V = 10 x 12 x 15

1 d3 Machine Foundations - 34
v 1800 ft' (---t-)
3 ft J
j

A foundation has a weight equal to 4 times the weight of the engine.


v = 66.67 yd' of concrete The safe bearing pressure of soil is 60 kpa and the foundation has a
base dimension of 2 m x 4 m. Find the maximum weight of the engine
For every 1 yd ' concrete, it needs 6 sacks of cement. to be mounted.
Therefore, A 8,234 kg C. 9,736 kg
B 6,455. kg D. 12,344 kg
No. of sacks = 6 (66.67)
No. of sacks = 400 sacks SOLUTION:

F PA
=

Machine Foundations - 33 F 60 (2 x 4)
=
F = 480 KN
A machine foundation has a trapezoidal cross-section with bases of 2
m and 3 m. The height is 1.5 m and foundation length of 4 m. Find IF, -;-; 0
J
the required yd of sand to be used for 1:2:4 mixture. We +- We = F
A. 7.33 C. 9.34 but W f = 4 We
B 12.4 D. 863 We j 4 We = 480
W, 96 KN(l/O 00981)
W, ().7:1631 ,,!C
Heat Transfer 237
236 Machine Foundation
HEAT TRANSFER
Machine Foundations - 35

A rectangular foundation cross-section has a bed plate dimension of 4


ft x 8 ft. The uniform clearance on each side is 1 ft. The height of
foundation is 2.5 ft. If the weight of the steel bar reinforcements
needed is 1/2% of weight of foundation, find the weight of steel bars.
Heat Transfer - 1
lise concrete density of 2400 kg/m 3 •
A. 51 kg C. 65 kg
A 15 em tbick wall bas a tbermal conductivity of 5 W/m-°K. If inside
B. 45 kg D. 76 kg
and outside surface temperature of tbe wall are 200°C and 30°C,
respecnvely, Determine tbe beat transmitted.
SOLUTION
A. 5.67 KW/m 2 C. 8.87 KW/m 2
2
B. 4.68 KW/m D. 6.87 KW/m 2
A = (4+2)(8+2)
A = 60 ft2
SOLUTION:
V = Ah
V = 60(2.5)
kA(t z - t])
V=150fe Q =
V = 4.247 nr' x o
W = wV
200 e .l. .l. 30 e
0

W = (2400)(4,247) Q 5(200 - 30)


Q
W = 10,192.57 kg A 0.15
Weight of steel bars (1/2%) W F
Weight of steel bars = 0.005(10,192,57) Q/A = 5666.67 W/m 2 K,
Weight of steel bars = 50.96 kg
Q/A 5.6667 KW/m 2
=
Jscm).

Heat Transfer - 2

Two walls of cold storage plant are composed of an insulating material


(k = 0.25 KJlhr-m·oC), 100 mm thick at the outer layer and material
(k = 3.5 Kl/hr-m-e'C), 15 em thick at inner layer. If the surface
temperature at the cold side is 30°C and hot side is 250°C, find the
heat transmitted per square meter.
2 2
A. 0.138 KW/m C. 0.025 KW/m
B. 0.450 KW/m D. 0.065 KW/m 2
2
238 Heat Transfer Heat Transfer 239
SOLUTION:
Heat Transfer - 4
A(t 2 -tI)
Q =
Xl 2 X {,=Z50·C f t t,=30'C Determine the thermal conductivity of a material that is use a 2 m 2 test
--+-
ki k2 panel, 25 mm thick with a temperature difference of 10.8 OF between
250 - 30 Q the surfaces. During the 5 hours test period, the heat transmitted is
Q/A = 200 ~J.
(0.15 I 3.5) + (0.1 I 0.25) k 0.045 W/m-oK C. 0.023 W/m-oK
oK D. 0_370 W/m-oK
B. 0.560 W 1m-
2
496.77 KJ I hr - m
Q/A =
SOLUTION:
3600 JX,J X,
Q/A = 0.138 KW/m 2
~oC I ~oF = 5/9

~oC/lO.8 = 519
Heat Transfer - 3
~oC = 6°C
Sea water for cooling enters a condenser at 27°C and leaves at 37°C. kA(t 2 -t I)
Q = --=----''-

The condenser temperature is 45°C, what is the log mean temperature x


difference? 200 k(2)(6)
---
A. 50. 56°C C. 37.82°C 5 x 3600 0.D25
B. 12.33°C D. 80.54°C
k = 2.3148 X 10- 5 KW/m-oC
SOLUTION: k = 0.023148 W/m-oC

~tA = 45 - 27
Heat Transfer - 5
~tA = 18°C 45'C 45'C
~tB = 45 - 37 At, A steam pipe having a surface temperature of 200°C passes through a
~tB = 8°C .6.t.
I
room where the temperature is 27°C. The outside diameter of pipe is
~t A - ~t B
~7'~ 80 mm and emissivity factor is 0.8. Calculate the radiated heat loss for
~t1l1e.n 27'C 3 m pipe length.
~tA A. 1434.47 W C. 2756.57 W
In - -
B. 3746.35 W D. 3546.45 W
~tB

27°C
18-8
~tll1eon SOl UTION:
In 181 8
A, = 7tDL
~t1l1e.n 12_33°C Ao = 7t(0.08)(3)
Ao = 0.7539 m 2
Solving for heat due to radiation:
Heat Transfer 24\
{i
240 lIeul Transfer
C. 56.80 ft2
I,
~
A. 51.80 ft2
20,408.4 x 10-8AJ:,('l14 - 1'21 ) , J/hr
·1
OR B. 37.30 ft2 D. 15.80 ft2
T1 = 200 + 273
T1 =473°K SOLUTION:
T 2 = 27 + 273
T7 = 300 0 K o= AU L1t
OR = 20,408.4 x 10-8 (0.8)(0.7539) [(473)4 - (300)4]
OR = 5164079.866 JIhr x Ihr/3600sec 11 = A(0.5)(l5)
OR = 1434.47 W
2 2!lm2
A = 1.467 m 2 x 3.281 ft

2
A = 15.79 m
Heat Transfer - 6 I
A counter flow heat exchanger is designed to heat fuel oil from 30 0 e to
\
90 0 e while the heating fluid enters at 140 0 e and leaves at 105°C.
Determine the arithmetic mean temperature difference.
Heat Transfer - 8 I
,1
3/hr i\i

A. n.5°e c. 45.5°C Brine enters a circulating brine cooler at the rate of 60 m at


B. 62.5°e D.67.5°C lOoe and leaves at -16°C. Specific heat of brine is 1.072 KJ/kg-OK and
specific gravity of 1.1. Determine the tons of refrigeration.
SOLUTION: A. 53.5 TR C. 33.5 TR
B. 65.3 TR D. 44.5 TR

L1t A 105-30
= SOLUTION:
L1tA 75°C
=
Density of brine = 1.1(1000 kg/m')
L1t B = 140 - 90
Density of brine = 1100 kg/rrr'
L1tB = 50 0 e
A _ L\t A - L\tB 11OO( 60)
,-,tnean - L\
tA m=
In-- 3600
L\tB m = 18.33 kg/sec
75- 50
L1tnean =
In75/50 o m c L\t
= p

L1t",ean = 6I.66°e
o =18.33(1.072) [-10 -(-16)1
o = 117.92 KW ,~
TR = 117.92/3.516 'i~
,'11
I
TR = 33.54 Tons of refrigeration
Heat Transfer - 7

A heat exchanger has an overall coefficient of heat transfer of 0.50


KW/m 1 _ 0 C. Heat loss is 11 KW and the mean temperature difference
is 15°C. What is the heat transfer area in fe?
242 Heat Transfer 243
Heat Transfer
Q/A = 230(1145 - 545)
Heat Transfer - 9
t,=1145°C t~ t,=45°C
QIA = 138,000
A heat exchanger has a hot gas temperature of 300°C and surface
A(t! -t 2 ) Q
conductance on hot side is 200 W/m 2. oK. If heat transmitted is 1000 Q = 1 x 1
W/m 2• what is the surface temperature on the wall at hot side? --+-+-
A. 295°C C. 234°C h. k h2 h, h,
k
B 465°C D. 354°C (t l - t 2 )
Q/A
SOLUTION:
1
-+-+-
h, k h,
x 1
LJ
Q = A (ho)(!!t) (1145 - 45)
138,000 = ----'--------'-----
(1 1230) + (x I 115) + tl 1290)
Q/A = (ho)(!!t) x = 0.020115 m
x = 20.115 mm
1000 = 200 (300 -tt)

t l = 295°C
Heat Transfer - 11 (ME Rd. Oct. 1985)

An uninsulated steam pipe passes through a room in which the air and
Heat Transfer - 10 (ME Bd, Oct. 1986) walls are at 25°C. The outside diameter of the pipe is 70 mm, and its
surface temperature and emissivity are 200°C and 0.80, respectively. If
A heat exchanger is to be designed for the following specifications: the coefficient associated with free convection heat transfer from the
Hot gas temperature; 1145°C surface to the air is 15 W/m 2 -oK , what is the rate of heat loss from the
Cold gas temperature. 45°C surface per unit length of pipe? 2
Unit surface conductance on the hot side, 230 W/m 2 .oK A. 998 w/m
2
C. 762 w/m
2
Unit surface conductance on the cold side. 290 W/m 2. oK B 872 w/m 2 D. 422 w/m
Thermal conductivity of the metal wall, 115 W/m-oK
Find the maximum thickness of metal wall between the hot gas and the
cold gas, so that the maximum temperature of the wall does not SOLUTION:
exceed 545°C.
A. 10 mm C. 20 mm Qc = heat transmitted by convection
B 30mm D.40mm Qc = h, Ao (t, - t 2 )
Qc '" 15[1t (0.07) LJ(200 - 25)
Qc I L = 577.268 W/m
SOLUTION: QR = heat transmitted by radiation
4
QR = 20,408.4 X 10,8 Ao Fe (T ]4 - T z ) , Jihr
where: Ao = n(0.07)L
Q = hi A (t] - ta) T I '" 200+273
T. = 473°K
Q/A = hi (t, - (2) T 2 = 25 + 273
Tz '" 298°K
244
Heat Transfer Heal Transfer 245
8
OR = 20,408.4 x 10- [n(0.07)L](0.8)[(473)4 _ (298)41 the compressor is 145°C and a counter flow air cooler reduces the air
OR/L = 1,514,032 J/hr x 1/3600 temperature to 45°C before it goes to the engine suction header.
OR/L = 420.564 Wlm Cooling water enters air cooler at 30°C and leaves at 38°C. Calculate
OIL = Oc + OR the quantity of cooling water in mJ/h,. required to cool the total air
OIL = 577.268 + 420.564 requirements of the engine at rated load and speed.
OIL = 997.832 Wlm A. 144 C. 123
B. 136 D. 165

Heat Transfer - 12 (ME Bd. Oct. 1981)


SOLUTION:
A turbo-charged, 16 cylinder, Vee-type dieset engine has an air
consumption of 3,000 kg/hr per cylinder at rated load and speed. This Heat gained by water = heat lost by air
air is drawn in through a filter by a centrifugal compressor directly (m cp ~t)w.ter = (m cp ~t).il
connected to the exhaust gas turbine. The temperature of the air from
the compressor is 145°C and a counter flow air cooler reduces the air m w (4 .187)(38 - 30) = 48,000(1.0)(145 - 45)
temperature to 45°C before it goes to the engine suction header. m., = 143,301 kg/hr
Cooling water enters air cooler at 30°C and leaves at 38°C. Calculate
the log mean temperature difference. Ave. temp of water = (30 + 38)/2
A. 4re e. 3~e Average temperature = 34°e
B. 87°e D. 65°e
From steam table,
SOLUTION: vr at 34°e = 0.0010056 mJ/kg

MA = 45 - 30 145°C Volume flow 143,301 (0.001 0056)


144.1 m 3 fhr

4~

~tA = 15°e Volume flow


~tB 145 - 38
~tB iov-c
~t
'mean
= ~tA - ~tB
~tA
~8.C
Heat Transfer - 14 (ME Bd. Apr. 1983)
In-- 30'C
~tB An oil heater heats 100 kg per minute of oil from 35°C to tOO°C in a
107 -- 15 counter flow heat exchanger. The average specific heat of the oil is 2.5
~t.nean
In(l07 I 15) KJ/kg_°C. Exhaust gases used for heating enter the heater with an
~t.ne.n = 46.82°e average specific heat of 1 KJ/kg-OC, a mass flow rate of 250 kg/min
and an initial temperature of 200°C. The overall heat transfer
coefficient is 75 W/m 2 _ 0 C. Determine heating surface in square meters
Heat Transfer - 13 (ME Bd. Oct. 1981) A. 30 C. 40
B. 63 D. 36
A turbo-charged, 16 cylinder, Vee-type diesel engine has an air
consumption of 3,000 kg/hr per cylinder at rated load and speed. This SOLUTION:
air is drawn in through a filter by a centrifugal compressor directly
connected to the exhaust gas turbine. The temperature of the air from Heat gained by oil = Heat lost by gas
(m c p ~t)"ll= (m c p ~t)g.s
Heat Transfer 247
246 Heat Transfer
11.36 - 3.36
100(2.5)(100 - 35) = 250(1)(200 - to) tmean
to = 135°C In( 11.36/3.36)
tmean 6.56°C x 1.8
Heat transferred = (100/60)(2.5)( I 00 - 35) tmean 11.82°e
Heat transferred = 270.83 K W
t.t A = 135 - 35
t.t A = 100°C
M B = 200 - 100 Heat Transfer - 16 (ME Bd. Apr. 1983)
L\tB = 100°C
A surface condenser serving a 50,000 KW steam turbo-generator unit
Since f.t A = t.tB, use the av~rage value receives exhaust steam at the rate of 196,000 kg/hr. Vacuum in
100+ 100 condenser is 702 mm Hg. Sea water for cooling enters at 29.5°C and
t.t m leaves at 37.5°C. For steam turbine condenser design, manufacturers
2 consider 950 Btullb of steam turbine condensed as heat given up to
t.t m = 100°C cooling water. Calculate the required quantity of cooling water in
cubic meters per hour.
Q = U A t.t m A. 10,374 C. 11,345
270.83 = (0.075) (A) (100) B. 12,445 D. 13,509
A = 36.11 m 2

SOLUTION:
Heat Transfer - 15 (ME Bd, Apr. 1983)
Heat absorbed by cooling water = 950(196,000)(2.205)
Heat absorbed by cooling water = 410,571,000 Btulhr
A surface condenser serving a 50,000 KW steam turbo-generator unit
receives exhaust steam at the rate of 196,000 kg/hr. Vacuum in
AverageSG of sea water is 1.03 and cp of gas is 0.93 Btu/lb-vl­
condenser is 702 mm Hg. Sea water for cooling enters at 29.5°C and
leaves at 37.5°C. For steam turbine condenser, manufacturers ill cp t.t = 410,571,000
consider 950 Btu/lb of steam turbine condensed as heat given up to m(0.93)(37.5 - 29.5)(1.8) = 410,571,000
cooling water. Calculate logarithmic mean temperature difference in ill = 30,657,930.11 Ibsfhr
of.
A. 10 C. 12 v = 30,657,930.11/(62.5)(1.03)
B. 14 D. 16 V = 477,003 ft3fhr
V = 477,003/35.31
SOLUTION:
V = 13,509 m 3lhr

Condenser pressure = 101.325 - (702 x 10 1.3251760)


Condenser pressure = 7.733 Kpa
Heat Transfer - 17 (ME Bd. Oct. 1994)
From steam table, at 0.007733 Mpa, t Sa l = 40.86°C
t.tA = 40.86 - 29.5 Calculate the energy transfer rate across 6" wall of firebrick with a
t.tA = 11.36°e temperature differences across the wall of 50°C. The thermal
t.t B = 40.86 - 37.5

t.tB = 3.36°e

248 Heat Transfer 249


Heat Transfer
conductivity of the firebrick is 0.65 Btu/hr-It-Pf" at the temperature v = 0.0011334 m2/sec
interest. 0.508(5)
A. 285 W/m 2 C. 112 W/m 2 NR =
B. 369 W/m 2 0.0011334
D. 429 W/m 2
NR = 2241
SOLUTION:

t a - tb = 50(9/5)
Heat Transfer - 19 (ME Rd. Oct. 1995)
t. - tb = 90°F'

kA(t -t b )

Q = a The hot combustion gases of a furnace are separated from the ambient
x air and its surrounding, which are at 25°C, by a brick wall 0.15 m
thick. The brick has a thermal conductivity of 1.2 W/mfK and surface
0.65(90) emissivity of 0.8. Under steady state conditions and outer surface
Q/A
(6/12) temperature of lOO°C is measured. Free convection heat transfer to
Q/A = 117 Btu/hr-fr' the air adjoining this surface is characterized by a convection
Q/A 117 Btu/hr-ft' X 1055J/Btu X Ihr13600sec X 10.76ft2/m2 coefficient of 20 W/m 2 -oK. What is the brrck inner surface
Q/A 369 W/m 2 temperature in DC?
A. 623.7 C. 461.4
B. 352.5 D. 2563

Heat Transfer - 18 (ME Bd. Oct. 1994) SOLUTION:

Water is flowing in a pipe with radius of 25.4 em at a velocity of 5

mlsec at the temperature in the pipe. The density and viscosity of the Qc = convection heat transfer

water are as follows: density = 997.9 kg/sec viscosity = 1.131 Qc = Ah, (t, - t2)

Pa-s. What is the Reynolds Number for this situation? Qc 1A = 20(100 - 25)

A. 2241 C. 3100 o, = 1500 W/m


2

B. 96.2 D. 1140 Qr = radiated heat loss .


8 4 2
Qr = 20,408.4 X 10 Fe (T 1 - T/) ,1/hr-m
8
SOLUTION: Qc = 20,408.4 X 10. (0.8)[(100+273)4 - (25+273)4]
Qc = 1,872,793 l/hr-m 2 X (1/3600)
nv, Qr = 520 W/m
c
R Q = Qc + Q,
v
where: Q = 1500 + 520
D diameter
=
Q = 2020 W/Ill'
D 2(25.4)
= k(t a -thY
Q
D = 50.8 em X

D = 0.508 m 1.2(t a - lOa)

V o = velocity 2020
0.15
V o = 5 m/sec
t, = 352.5°C
v = kinematics viscosity

v = 1.131/997.9

L.JV
Heat Transfer
Heat Transfer 2' I

Heat Transfer - 20 (ME Bd. Oct. 1996) 2668.6 = 914.52 + x(l885.5)


x = 93%
Steam initially saturated at 2.05 Mpa, passes through a 10.10 cm
standard steel pipe for a total distance of 152 m, The steam line is
insulated with a 5.08 cm thickness of 85% magnesia. For an ambient
temperature of 22°C, what is the quality of the steam which arises at Heat Transfer - 21(ME Bd. Apr. 1999)
its destination if the mass flow rate is 0.125 kg steam per second?
Properties of steam: Compute the amount of condensate form during 10 minutes warm-up
Pressure Temperature Enthalpy of 150 meter pipe conveys the saturated steam with enthalpy
2.05 213.67 hf~ 914.52 hfg = 1885.5 vaporization h rg = 1,947.8 LJ/kg. The minimum external temperature
hg = 2800.0 of pipe is. 2°C. The final temperature of pipe is 195°C. The specific
k for 85% magnesia = 0.069 W/m-oK heat of pipe material is 0.6 KJ/kg_°C. The specific weight is 28 kg/m.
f, for still air = 936 W/m-oK A. 249.69 kg C. 294.54 kg
A.93% B. 982.45 kg D. 42345 kg
C. 84%
B.98% 0.76%
SOLUTION:
SOLUTION:
m, = mass of pipe
rl 10.10/2 1o=2°C jQ
m, = 28(150)
rl
'=

5.05
'=

r2 '= 5.08 + 5.05


(Steam~
~
C.
"I"
~ h,

~m.
rn, = 4200 kg
Heat loss by steam = Heat loss from pipe
rz'=10.13cm
• 15h m, (h, - h.) = mp cp (tz - t.)
m,(l947.8) '= (4200)(0.6)(195 - 2)
A, = 21trzL

m, = 249.69 kg
A, = 21t(O.I013)(152)

A, '= 96.746 m Z

Heat Transfer - 22 (ME Bd. Oct. 1999)


(t , -to) 2
Q What is the heat transfer in the glass surface area of 0.7 m having an
In( r 2 I r l ) I inside temperature (room) of 2SoC and 13°C outside temperature
---+-­
21tkL Aoh o (surrounding). The thickness of glass surface is 0.007 m. The thermal
eonductivity is 1.8 W/m-°K.
A. 5.8 kw C. 6.2 kw
(213.67 -22)
Q B. 3.6 kw D. 2.34 kw
In(O.lOI3/0.0505) I
+
21t(0.069XI52) 96.746(9.36)
Q = 16,427.4 W SOLUTION:
Q = 16.427 KW
Q = m,(h l - hz)

16.4274 = 0.125(2800 - hz)

hz = 2668.6

h '= hr + xhfg

252 Heat Transfer Air Compressor 253

kA(t 2 -t 1 ) AIR COMPRESSOR


Q=
x

(1.8XO.70)(25 - 13)
Q = --'-----'------'---------'-- Air Compressor - 1
0.007

Q = 2160 watts An air compressor takes air at 100 Kpa and discharges to 600 Kpa. If
the volume flow of discharge is 1.2 mJ/sec, determine the capacity of
Q = 2.16 kw air compressor.
A. 432 m 3/sec C. 6.85 m
3/sec
3
B. 3.33 m /sec D. 7.42 m3/sec

SOLUTION:

PI V l n = P2 V 2n
25°e 13°e
a n = 1.4 (for standard air)

-h-+
- IOO(V I)L4 = (600)(1.2)14

VI = 4.315 m 3/sec

Air Compressor - 2

The discharge pressure 01 an air compressor is 5 times the suction


pressure. If volume flow at suction is 0.1 mJ/sec, what is the
compressor power assuming n = 1.35 and suction pressure is 98 Kpa?
A. 21.67 KW C. 25.87 KW
B. 19.57 KW D. 10.45 KW

SOLUTION:
n-I
nP V P -~
W = _ 1 - 1 [(_2) n _ I]
n-I PI

135-1
- 1.35(98)(0.1)[(5P / PI) 135 - 1]
W - 1.35-1 I

W = 19.57 kw
255
Air Compressor
2:l4 Air Compressor
1
'.,
Air Compressor - 3 Air Compressor - 5
3/sec
An air compressor has a suction volume of 0.25 m at 97 Kpa and
A 10 Hp motor is use to drive an air compressor. The compressor discharges to 650 Kpa. How much power saved by the compressor if
efficiency is 75%. Determine the compressor work.
A. 5.0 KW C. 7.6 KW
there are two stages? C. 3.86 KW
A. 8.27 KW
B.6.5KW D.5.6KW D. \0.0 KW
B. 6.54 KW
SOLUTION:
SOLUTION:
W
YJc =
Brake Power' For single stage:
W n-I
0.75 = --­ P -~-

10xO.746 n PI VI ((_2) n - l]
W
W = 5.59 KW n -1 PI
I~ -1
\.4(97)(0.25) ((650 197) 14 -lJ
W~
1.4 - \
Air Compressor - 4 W= 6\.28 KW

The initial condition of air in an air compressor is 98 Kpa and 27°C For two stages:
and discharges air at 400 Kpa. The bore and stroke are 355 mm and
381 mm, respectively with percent clearance of 5% running at 300 Px = ~
rpm. Find the volume of air at suction. r, = ~97(650)
3
A. 600 rrr'zhr C. 620 m /hr
3 P, = 251.097Kpa
B. 610 m /hr D.630m3/hr
n-l
p ­
2nP1Vl((~)n _I]
SOLUTION: W
n- 1 PI

14-1
YJy = 1 + C _ c(.!2)I!n
PI
2(\.4)(97)(0.25) [(25\.097 /97) 14 - 11
W=
YJy = 1 -i- 0.05 - 0.05(400/98)1114 \.4 - 1
YJy = 0.913 W= 53 KW

power Saved = 61.28 - 53


VD = nl4 D2 L N
power Saved = 8.27 KW
VD n/4 (0.355/(0.381 )(300160)
=
VD 0.1885 m3/sec
=
VI = 0.1885(0.913)
3!sec
VI = 0.17215 m
3
VI = 619.75 m /hr
257
256 Air Compressor
Air Compressor

Air Compr essor - 6


A.5%
R 7%
C. ]5%
D. 11% 1i
I
and 0.2 SOLUT ION:
The suction conditi on of an air compre ssor is 98 Kpa, 27°C
3/sec. the frce air
m If surrou nding air is 100 Kpa and 20°C, determ ine
3/sec. 11v = 1 +c _c(pz/P j)!fn
capacit y in m
A. 0.15 C. 0.25 0.87 = 1 + c - C(3P 11P1) l f 14
B. 0.]9 D. 0.23
c = 10.9]%

SOLUT ION:

Pr,-V P,V s
- -F - - -
IF Ts Air Compr essor - 9
piston
100(V F ) 98(0.2) The compre ssor work of an air compre ssor is 100 KW. If the
speed is 15 m
3/min,
determ ine the mean effectiv e pressur e.
(20 + 273) (27 + 273) A. 200 Kpa C. 400 Kpa
3isec
V F = 0.]914 m B. 300 Kpa D. 600 Kpa

SOLUT ION:

Air Compr essor - 7 W=Pm xV o

A 355 mm x 381 mm air compre ssor has a piston displac ement of 100 = Pm ( ] 5/60)
3/sec.
0.1885 m Determ ine the operati ng speed of the compre ssor.
A. 250 rpm C 350 rpm Kpa
Pm = 400
B. 300 rpm D. 600 rpm

Air Compr essor - 10


SOLUT ION:

2 A double acting air compre ssor has 16 in x 7 in, 600 rpm has what
Yo = nl4 D L N
volume displac ement?
0.1885 = rr/4 (0.355) 2(0.38] )N 3
A. 688 ft /min C. 488 /min
3
fe
N = 5 rev/sec x 60sec/m in B. 'i55 ft3/m in D. 977 ft /m i n
N ~ 300 rpm
SOLUT ION:
2LN]
V o=2(r rD
Air Compr essor - 8

87%
v, = 2(rr/4 (16/12)2 (7112) (600)]
Determ ine the percen t clearan ce of an air compre ssor having
cy and compre ssor air pressur e to be thrice the 3
'Ivolumetric efficien Vo = 977.38 ft /m in
suction pressur e.
258 Air Compressor Air Compressor 259

Air Compressor - 11
Air Compressor ~ 13
A two-stage air com pressor has a suction pressure of 14 psi and The piston speed of an air compressor is 140 mlmin and has a volume
discharge pressure of 130 psig. What is the intercooler pressure in displacement of 0.2 m 3/sec. Determine the diameter of compressor
Kpag.
cylinder,
A. 209 Kpag C. 477 Kpag C. 467 mm
A. 500 mm
B. 600 Kpag D. 300 Kpag O. 246 mm
B. 358 mm

SOLUTION:
SOLUTION:
2
VD = rei4 0 L N
P2 ]30+]4.7

Piston Speed = 2 L N
P2 ]47.5 psia

140 = 2 LN
LN = 70 m/min
Px Jp j P2
Px J14(l447) 0.2 = 7[ / 4 0 2 (70/60)
D = 467,19 mm
P, = 45 psi x 1Ol.325/14.7
P, = 310.24 Kpaa - ]0l.325
P, = 208.91 Kpag
Air Compressor - 14

An air compressor piston displacement is 5000 em:' when operates at


Air Compressor - 12 900 rpm and volumetric efficiency of 75%. Determine the mass now
of air at standard density. .
A two stage air compressor has an intercooler pressure of 3 kg/ern". A. 365,3 kg/hr C. 386.4 kg/hr
What is the discharge pressure if suction pressure is 1 kg/cm/? B. 243,5 kg/hr D. 465,2 kg/hr
A. 3 kg/ern" C. 12 kg/ern"
B. 9 kg/ern' D. 15 kg/ern:' SOLUTION:

VI
SOLUTION: 11v
Vo
VI
Px=~ 0.75
5000
V, 3750 cm 3 (900)
p/= P (P2) 1
V, 3375000 cmvrnin

32 = 1(P 2 ) 1,2 kg/m' (at standard)


w
m 1.2(3375000/1 00 3 )
P 2 = 9 kg/em/
m 4.05 kg/min
m = 243 kg/hr
260 Air Compressor Air Compressor 261

Air Compressor - 15
0.4 = 2[7(/4 (D)2(D)(300/60)]

A two-stage compressor air at 100 Kpa and 22°C discharges to 690 D = 0.37067 m
Kpa, If intercooler intake is 105°C, determine the value of n. L = 0
A 1400 C. 1.345 L = 370.67 mm
8. 1.358 D. 1.288

SOLUTlON:
Air Compressor - 1i
Px = ~PI P2
P, = JlOO(690) An air compressor takes air at 97 Kpa at the rate of 0.5 m/sec and
P, C~ 262.68 Kpa discharge 500 Kpa, If power input to the compressor is 120 KW,
n-) determine the heat loss in the compressor,
r-;;­
T
x
~= ~j
l
(p
x I
A 26.85 KW
B. 18.55 KW
C. 30.45 KW
D. 22.36 KW

n-I SOLUTION:
(l 05 + 273) = ( 262.68.J--;;
(22 + 273) l 100 n-]
n-I P -­
W n PI VI [(_2) n _ I]
1.281 = (2.6268) n
n -I PI
n-- I In 1.28\ 14-1

n In 2.6268 W 1.4(97)(0.5) [(500 / 97) 14 -I]


n - I = 0.2564n 1.4 - 1
n = 1.345 W 101.45 KW

Heat Loss 120-101.45


Heat Loss 18.55 KW
Air Compressor - 16

The piston displacement of a double acting compressor running at 300


rpm is 0.4 mJ/sec. If bore and stroke are unity, determine the length of Air Compressor - 18 (ME Bd. Apr. 1983)
stroke.
A. 350 mm C. 371 mm A single acting air compressor has a volumetric efficiency of 87%,
8. 380 mm D. 400 mm operates at 500 rpm. It takes in air at 100 Kpa and 30°C and
discharges it at 600 Kpa. The air handled is 6 mJ/min measured at
discharge condition. If compression is isentropic, find mean effective
SOLUTION: pressure in Kpa
A ]82 C. ]98
V D =2(nI4D 2LN) B. 973 D. 204
L o (for unity)
262 Air Compressor 263
Air Compressor
SOLUTION. Vo = 0.45 m 3/min
PI = 90 - 5
k
PI V l = P2 V/ P,=85kpa
P2 = 600 -t- 10
IOO(V 114) = 600(6)14 P2 = 610 kpa
Tlv = I + c - c(P 2/p,)lfn
VI = 21.58 rrr'zrnin 1']v = 1+0.10-0.10(610/85)11128
1']v = 0.633684
V0 = 21.58/0.87
V I = 0.45(0.633684)
Vo = 24.8 m3/min
V I = 0.285 mJ/min
m = PV/RT
. n~l
nP V P ­ m = 85(0.285)/(0.287)(29.3 + 273)
W = _'_I [(_2) n _ I] m = 0.2792 kg/min
n- I PI m ~ 16.76 kg/hr

I 4~ I
1.4(100)(21.58) [(600/ 100) 14 _ I]
W = 1.4-1 Air Compressor - 20 (ME Bd, Apr. 1986)

A single acting air compressor operates at 150 rpm with an initial


w= 5,049 KJ/min
W = Pm X Vo condition of air at 97.9 Kpa and 27°C and discharges the air at 379
5,049 = Pm X 24.8 Kpa to a cylindrical tank.. The bore and stroke are 355 mm and 381
Pm = 203.6 Kpa mm, respectively, with 5% clearance. If the surrounding air is at 100
Kpa and 20°C while the compression and expansion process are PVI.J
= C, determine free air capacity, m 3/sec
A. 0.0818 C. 1.23
Air Compressor - 19 (ME Bd. Oct. 1984) B. 2.13 O. 4.23

A single acting reciprocating air compressor has a clearance volume of SOLUTION:


10%. Air is received at 90 Kpa and 29.3°C and is discharged at 600 2LN
Kpa. The compression and expansion are polytropic with n = 1.28. The Vo =1t/40
pressure drop is 5 Kpa at suction port and 10 Kpa at the discharge Vo = 1t/4 (0.355)2 (0.381) (150/60)
3/sec
port. The compressor piston displacement is 500 cnr' when operating Vo = 0.094278 m
11n
at 900 rpm. Determine the mass of compressed air in kg/hr 1']v = I + c - c(P 21Pd
A. 16.76 C. 98.33 1'], = 1 + 0.05 - 0.05(379/97.9)1113
B. 20.45 O. 28.23 1']v = 0.908
V I = 0.908(0.09427tn
VI = 0.085604 mJ/sec
SOLUTION:
Solving for free air capacity:

v, = (rr/4 n 2 L) N PFV
- -F - - -
PlV l

VD = (500)(900) TF T]
Vo = 450,000 crrr'zmin
264 Air Compressor Air Compressor 265

I OO( VF ) 97 .9( 0.085604) D = 0.45 m


D = 450 mm
(20 + 273) (27 + 273) D = L = 450 mm
VF = 0.081851 m
3/sec

Air Compressor - 23 (ME Bd. Nov. 1983)


Air Compressor - 21 (ME Bd. Apr. 1987) A 2-stage, double acting, L-type air compressor 16" x 10" x 7", 600
rpm, has a free air unloader at each end for capacity control. It is
The piston displacement of a double acting compressor is 0.358 m 3/sec, driven through V-belts by a 150 Hp electric motor, 460 V, 3 phase, 60
delivers gas from 101.325 Kpa and 300 0 K to 675 Kpa at the rate of Hz, 1200 rpm. Barometric pressure is 125 psi gage. Calculate piston
0.166 m 3/sec at 150 rpm. Value of n for compression and expansion is displacement in m 3/hr
1.33. Find the compressor percent clearance..' A. 562 m 3/hr
3
C. 649 m !hr , 1661 m 3 /hr
k 16.96 C. 12.34 3/hr 3/hr, 3
B. 762 m O. 833 m 724 m !hr
B. 14.23 O. 18.44

SOLUTION: SOLUTION:

llv=V}/V D Piston displacement of first stage:


nv = 0.166/0.358
llv = 0.4637 VD = 2[11:/40 2 L N]

llv = 1 + c - c(pZ/Pj)l/n VD = 2[11:/4 (16/12)2 (7/12)(600)]


0.4637 = 1 + c - c(675/10 1.325)1/133
VD = 977.384 cfm
c = 0.1696
c = 16.96% VD = 977.384(60)/35.31
VD = 1,661 m 3/hr

Piston disp lacement of second stage:


Air Compressor - 22 (ME Bd. Apr. 1987)
V D = 2[11:/4 (10/12)2(7/12)(600)]
The piston displacement of a double acting compressor is 0.358 m3/sec, V D = 381.791 cfm
delivers gas from 101.325 Kpa and 300 0 K to 675 Kpa at the rate of V D = 381.791(60)/35.31
0.166 m3/sec at 150 rpm. Value of n for compression and expansion is 3
V D = 648.75 m /hr
1.33. Find the bore and stroke assuming bore = stroke.
A. 300 mm C. 350 mm
B. 400 mm O. 450 mm Air Compressor - 24 (ME Bd. Apr. 1984)

SOLUTION: A two-cylinder single-acting air compressor is directly coupled to an


electric motor running at 1000 rpm. Other data are as follows:
VI) = 2(11:/4 D 2 L N) a. size of each cylinder = 150 mm x 200 mm
b. clearance volume = 10% of displacement
0.3 58 = 2[11:/4 (0)2 (D) (150/60)] c. exponent (n) for both compression and re-expansion
266 Air Compressor Air Compressor 267

process = 1.6 SOLUTION:


d. air constant = 1.4
e. air molecular mass M = 29 VOl = 94,390 cml/sec
Calculate the volume rate of air delivery in terms of standard air for a Vol = 0.09439 m
3/sec
delivery pressure 8 times ambient pressure under ambient air V I = Vo l(11,) = 0.09439(0.85)

3/sec
conditions of 3000K
and 1 bar. VI = 0.0802315 m
3 3 PI = 14.5 psi x 101.325/14.7
A. 2 m /hr C. 3 m /hr
3
B. 4 m /hr D. 5 m 3/hr PI = 99.946 Kpa
P, = (30 + 14.7)(101.325/14.7)
P, = 308.11 Kpa
SOLUTION: P 2 == (100 + 14.7)(101.325/14.7)
P 2 = 790.61 Kpa
Vo = 71:/4 D2 L N Solving for the mass flow rate:
V0 = 71:/4 (0.15)2 (0.2) (1000) PI VI = m R r,
Vn = 7.068 m
3/min 99.946(0.0802315) = m(O.287)(22 + 273)
11, = 1 + c - c(P2/P I ) 11n m = 0.09471 kg/sec
11, = 1 +0.10-0.10(8P/P\)IIL6 Solving for polytropic exponent n:
(T, 1 T 1) = (P, 1 Pj)",lfn
11, = 0.733 n-l
V I = 0.733(7.068)
105 + 273 =(308.11 )---;;-­
VI = 5.181 m 3/min
Standard air is at 70°F(21.11 "C) and 14.7 psi(10 1.325 Kpa) 22 + 273 99.946
1.2813 = (3.08277)",l/n
r; v. PI Vi
----- n -1 In 1.2813
r, r,
n In3.08277
101.325(V s ) 100(5.181)
n -~ 1.2824
(21.11 + 273) (300) Compressor power of the first stage:
V, = 5.013 m 3/min n-l
n(mRT) P~ ­
W = [(--) n - 1]
n- 1 PI
L28-1
Air Compressor - 25 (ME Bd. Apr. 1991) 1.2824(0.0947)(0.287)(22 + 273) [(308.11/99.946) -'28 _ 1]
W = 1.2824-1
A two stage compressor with first stage piston displacement of 94390 W = 10.245 kw
cnr'rsec is driven by a motor. Motor output is 35 Hp, suction
temperature 22°C, volumetric efficiency is 85%. Mechanical efficiency Compressor power of the second stage:
128-1
is 95%, the intercooler pressure is 30 psi gage. Air temperature in and
1.2824(0.0947)(0.287)(44 + 273) [(790.611/308.11) 128 _ 1]
out of the intercooler are lO5°C and 44°C. Final discharge pressure is
W = 1.2824-1
100 psi gage, suction estimated 14.5 psi. Find the compression
efficiency W = 8.956 kw
A. 77.60%' C. 87.34%
B. 63.34% D. 98.23% WT == 10.245 + 8.956
W T = 19.20 kw
268
Air Compressor Air Compressor 269

19.20
Compression efficiency =
(35 )(0.746 )(0.95) Air Compressor - 27 (ME Rd. Apr. 1995)
Compression efficiency = 77 AI %
A single stage air compressor handles 0.454 mJ/sec of atmospheric
pressure, 27°C air, and delivers it to a receiver at 652.75 Kpa. Its
volumetric efficiency on an isothermal basis is 0.85 and its mechanical
. Air Compressor - 26 (ME Rd. Apr. ~995) efficiency is 0.90. If it operates at 350 rpm, what power in KW is
required to drive it?
An air compressor is to compress 8.5 mJ/min from 98.56 Kpa to 985.6 A. 95 C. 120
Kpa, Assuming conditions ideal, and with n ;=: 1.3, what will be. the B 112 D. 100
saving in work due to two staging?
A. zero SOLLTIO~
C. 5.6 KW
B. 4.6 K'W D. 3.5 KW
For Isothermal process.
SOLUTION:
W = Pi VI In(P 2fP 1)
For single stage compressor:
n-1 \V = 101.3(0.454) In(652.75/l01.3)
nP V P. -
W = _'_I [(_2) 0 _ IJ
n -I Pj \\ = 85.685 KW
13-1 85.685
1.3(93.56)(8.5 / 60) --- Power Input
W = [(985.6/98.56) 13 - IJ 085(0.90)
1.3 - I Power Input 112KW
W = 42043 KW

For two stage-compressor:

Air Compressor - 28
r, = .J98.56(985.6)
P, = 311.67 Kpa A two stage compressor receives 0.50 kg/s of air at 120 kpa and 300"K
and delivers it at 7 .\1 pa, Find the heat transferred in the intercooler.
0-1
P - A. 118]4 kw C. 233.23 kw
w 2nP J VI [(.2_) n -1] B. 134.55 kw D ) 87.34 kw
n- I P,
13-1
W 2(1.3)(98.56)(8.5/60) [(311.67 / 98.56) 1.3 -1] SOU;T10~
1.3 - 1 P P,
W = 36.83 KW r-'
Pi 7 Mpa
:::L
rl
T
Pi 7f)()() kpa m=O.Si'ljlc,
Power Saved = 42.43 - 36.83
P, intercooler pre~SUTI

P, j(T2(Jj7~(Kif!) c, c,
Power Saved = 5.6 KW
P, 91n515 kpa
,~
270 Air Compressor Air Compressor 271
r
n-l and 3000K
and discharged it at 377.1 kpa. Determine the isothermal

Tx =r.~]-;- efficiency.
A 67.34% C. 81.89%
T] \ PI B. 76.34% D. 67.48%
1.4-1

~=(916.515)~
(300) \ 120 SOLUTION:
r, = 536.28 "K
Q = m cp (T x - T I)
Q = 0.5( I)(536.28 - 300) For isothermal compression:
Q = 118.14kw
W = PI V , ln(P2 / Pd

W = 101.4(0.189) In(377.11101.4)
Air Compressor - 29
W = 25.17kw
An air compressor is tested and it is found that the electric motor used
Isothermal Efficiency = 25.17/37.3
37.3 kw when the compressor handled 0.189 m3/s of air at 101.4 kpa
and 300 0 K and discharged it at 377.1 kpa, Determine the adiabatic Isothermal Efficiency = 67.48%
efficiency.
A. 67.34% C. 81.89%
B. 76.34% D. 92.34%
Air Compressor - 31
SOLUTION:
Calculate the volumetric efficiency of a single-cylinder, double acting
pII compressor with a bore and stroke of 0.45 x 0.45 m. The compressor
n-I is tested at 150 rpm and found to deliver gas from 101.3 kpa and
P - 3/s
300 0 K to 675 kpa at a rate of 0.166 m when n = 1.33 for expansion
w n PI VI [(_2) n _ I]
n-I PI and compression process.
]4-1 A. 46.39% C. 48.34%
1.4(101.4)(0.189) [(377.1/101.4) -1] B. 56.23% D. 74.23%
w L4
1.4 - I
w 30.54 kw
SOLUTION:
Adiabatic Efficiency = 3054137.3
Adiabatic Efficiency = 81.89%
2
Vo 2(nI4)d L N
=
V0 = 2[(nI4)(0.45)2(0.45)(l50/60)]
3/s
Vo = 0.3578 m
Air Compressor - 30 n. = V [I v«
n, = 0.166/0.3578
An air compressor is tested and it is found that the electric motor used llv = 46.39%
37.3 kw when the compressor handled 0.189 m 3/s of air at 101.4 kpa
Air Compressor 273
272 Air Compressor
V D = 0.1227
V I = 0.1227 (0.5289)
Air Compressor - 32 3/s
VI = 0.0649 m

A reciprocating compressor with a 3% clearance receives air at 100 n-I


kpa and 300 0K and discharges it at 1.0 Mpa. The expansion and nP V P --
W = _1_1 [(_2) n - 1]
compression are polytropic with n = 1.25. There is a 5% pressure n-l PI
drop through the inlet and outlet valves. Find the volumetric 1 3-1
efficiency. 1.3(95)(0.0649) --"
W = [(2000/95) 1-, -1]
A. 76.23% C. 98.33% 1.3 - I
B. 82.50% D. 65.33% W = 27.25 kw
100kPa 1000kPa
SOLUTION:
Q---t=f Air Compressor - 34
PI = 100 (1 - 0.05)
PI ~ 95 kpa 3/s
A water-jacketed air compressor handles 0.143 m of air entering at
P2 = 1,000 (1 + 0.05) 965 kpa and 21°C and leaving at 480 kpa and 132°C; 10.9 kg/h of
P2 ~ 1050 kpa cooling water enters the jacket at ISoC and leaves at 21°C. Determine
P2 I/n
the compressor power.
T]v =

n, =
1 + c - c(-)
PI
1+ 0.03 - 0.03(1050/95)1/125
Compressor
.. A. 26.163 kw
D. 17.23 kw
C. 34.44 kw
D. 19,33 kw

T]y = 82.50% SOLUTION:


~
W
n-I 21°C r-;
( '1- -..I

Air Compressor - 33 T
2
=l~j n -""Q Radiator
T\ PI
A reciprocating compressor has a 5% clearance with a bore and
stroke of 25 x 30 em. The compressor operates at 500 rpm. The air = r 'j'-;
n-I
Compressor r 1-S oC
-
enters the cylinder at 27°C and 95 kpa and discharges at 2000 kpa. If
132 + 273
21+273 \96.5
480
I Compressor Power
n = 1.3, determine the compressor power.
n-llnl.377
A. 12.34 kw C. 27.25 kw
B. 18.45 kw D, 34.23 kw n In4.974
n = 1.249
i 249-1
SOLUTION:
1.249(96.5)( 0.143) ._._-
W = [( 480/96.5) 1.249 -1]
P2 lin 1.249 - 1
Ilv = 1 + c - c(-) W = 26.087 kw
PI Q -z: heat loss
T]y ~ 1 + 0.05 - 0.05(2000/95)1111
Q = me p(t 2 - t.)
Tly = 52.89% Q = (10.9/3600)(4.187)(21 - 15)
VD ~ (11:/4)(0.25)2(0.3)(500/60)
274 Air Compressor Air Compressor 275 .
,'.

'.~
Q 0.076 kw
= 41.63-37.45 .;'.

Compressor power = W + Q % Increase =


37.45 'il
Compressor power =, 26.087 + 0.076 %Increase = //./6% }!
Compressor power = 26.163 xw h

~
:.1",
d

Air Compressor - 36 (ME Bd Oct. 1997)


Air Compressor - 35 (ME Bd. Apr. 1998)
An ideal-single stage air compressor without clearance takes in air at
A 2-stage compressor operates between constant pressure limits of 100 kpa with a temperature of 16°C and delivered it at 413 Kpa after
98.6 kpa and 1.103 M pa. The swept volume of the low pressu re piston isentropic compression. What is the discharge work done by the
is 0.142 m'', Due to failure of the cooling. water supply to tbe compressor in KJ/kg?
intercooler , air is passed to the high pressure cylinder without A. -59.22 C -54.75
reduction in temperature. Using PV I. 2 = C, determine the percentage B. -52.43 o -5613
increase in power.
A. 26 C. 11
SOLUTION: 100Kpa 4 - J. pe
~
B. 21 D. 16

SOLUTION:

~PIP2
c, c,
W
kmRTilP2
1- k
r'( ,-­
l PI /
k-I

Jk I LJ
P,
P,
=
= ';'-98-.6-(-11-03-) P,
Two-stage r 14-'
W = -1.4(0.287)(16+273)1(413)-;-4-_
P, = 329.8 kpa 98.6Kpa y I 1

Solving for power due to two staging:


I i m 1- 1.4 l100

n-I c, W/m = -/45 KJ/kg (no answer in the given choices)


W = 2nP,V\ ~ --;;--1
[() ] Single stage
n- 1 PI
1.2-1
2(1.2)(98.6)(0.142) ­
W = [(329.8/98.6) 12 - 1]
1.2 - 1
W = 37.45 KJ

Solving for power due to single stage:


n-I
nP V P ­
W = _1_' [(_2 ) n _ 1]

n -1 PI

1.2-1

1.2(98.6 )(0.142) ­
W -----'-'- -[(1103/98.6) 1.2 - I]

1.2 - 1

W 41.63 KJ
276 Pumps 277
Pumps
h, ~. :2.5 + 2 + 0.8
PUMPS h, = 5.3 m

Pumps -1 Pumps - 3

A double suction centrifugal pumps delivers 70 fe/sec of water at a A centrifugal pump requires 40 ft head to deliver water from low level
head of 12 m and running at 1250 rpm. What is the specific speed of to higher level. If pump speed is 1600 rpm, determine the impeller
the pump? diameter of the centrifugal pump.
A.5014rpm C. 2345 rpm A. 185 mm C. 154 mm
B. 6453 rpm D. ~968 rpm B. 160 mm D. 176 mrn

SOLUTION'
SOLUTION:
v J2gh
=
N.JQ ---_.
N, =-~ v = J2(9.81(40/3.281)
h v = 15.466 m/sec
Q = 70/2 fe/sec x 7.481 gal/1 ft3 x 60sec/l min v=nDN
Q = 15710.10 gal/min
70ft'/s 15.466 = nD(l600/60)
h = 12 x 3.281

D = 0.]8461 m
h = 39.37 ft

~-- D = 184.61 mrn


1250.J15710.10
Ns = (39.37)3/4 35ft'/s
-----.,
~
....... I ill s~ft'/S

N, = 9968.4 rpm
Pumps - 4

Pumps - 2 The suction pressure of a pump reads 2 in. of mercury vacuum and
discharge pressure reads 130 psi is use to deliver 100 gpm of water
The pump centerline of a centrifugal pump is located 2.5 m above with specific volume of 0.0163 fellb. Determine the pump work..
from the high tide level. The sea. water varies two meters from high A. 4.6 KW C. 7.4 KW
tide to low tide level. If friction loss at the suction is 0.8 m, determine B. 5.7 KW D. 8.4 KW
the total suction head. r: .... rr®D= 4
A. 5.30 m C. 6.30 m SOLUTION:
B. 2.30 m D. 8.23 m
PI = -2 in Hg x 101.325/29.9:­
PI = - 6.773 Kpa
H~~~ Tide Level P2 = 130 psi x 101.325/14.7
SOLUTION:
P 2 = 896.071 Kpa
h, = total suction head w = l/v
w = 1/.0163
w = 61.35 Ib/ft; x 9.81/62.4
w = 9.645 KN/m>
278
Pumps Pumps 279
P2 - PI
h
w T\combtned 0.85(0.7)
896.071- (-6.773)
h = -----'----....:...
9.645 11combined 59.50%
h = 93.075 m
Q = 100 gal/min x 3.785Ii1lgal x Im 3/1000li x 1/60
Q = 0.006308 m 3/sec
P = wQh Pumps - 7
P = 9.645(0.006308)(93.075)
P=5.69KW In a boiler feed pump, the enthalpy at the entrance is 765 KJ/kg. If
pump has a head of 900 m, what is the exit enthalpy of the pum p.
A 897 KJ/kg C. 774 KJlkg
B. 465 KJ/kg D. 864 KJlkg
Pumps - 5
SOlCTION
A pump is to deliver 150 gpm of water at ahead of 120 m. If pump
efficiency is 70%, what is the horsepower rating of motor required to m(h:? - hi) = m x h x 0.00981
drive the pump?
A. 40.44 Hp h:?-765 = 900 x 0.00981 h, h,
C. 38.44 Hp
B. 25.66 Hp D.2111Hp ~~.\
h:? = 773.83 KJlkg
SOLUTION: h=900m

W, = wQh Pumps - 8
W, = 9.81 (150gal/min x 0.003785m 3/lgal x 1/60)(120)
w, = 11.139 KW A submersible pump delivers 350 gpm of water to a height of 5 ft from
BP = 11.139/0.7 the ground. The pump were installed 120 ft below the ground level and
BP = 15.913 KW a draw down of 8 ft during the operation. If water level is 25 ft above
BP = 21.33 hp the pump, determine the pump power.
A. 7.13 KW C. 7.24 KW
B. 4.86 KW D. 864 KW

Pumps - 6
SOLUTION:
A motor is used to drive a pump having an efficiency of 85% and 70%
respectively. What is the combined efficiency of pump and motor?
A. 59.50% C. 62.50% h = 5+120-(25-8)
B. 61.50% D. 65.50% h = 108/3.281
h = 32.916 m
SOLUTION: Q = 350 gal/min x 0.003785m 3/gal x Imin/60sec
Q = 0.02246 m 3/sec
11combtned = 11p 11m W, = wQ h
DiU Pumps Pumps 281

WI' 9.81 (0.02246)(32.916) BP = 32846/0.65


w, 725 KW BP = 505.32 KW

~I
Pumps - 9 Pumps -11

Determine the number of stages needed for a centrifugal pump if it is What power can a boiler feed pump can deliver a mass of 35 kg/s
used to deliver 400 gal/min of water and pump power of 15 Hp. Each water at a head of 500 m? II
impeller develops a head of 38 ft. A. 356.56 KW C. 456.64 KW ,.~
·1
.Jv·
A. 6 C. 8 B. 354.54 KW D. 171.67 KW -,·r
f}. 4 D.7
SOLUTION:
SOLUTION:
P = mxhxO.00981
W, = w Q h
P = 35 x 500 x 0.00981
15 x 0.746 = 9.81 (400 galimin x 0.00785m J /gal x 1/60)h
p = 171.675KW
h = 45.20 m x 3.281 film
h = 148.317 ft

Number of stages ~ 148.317/38 Pumps - 12


Number of stages = 3.903 stages
N umber of stages "" 4 stages A pump running at 100 rpm delivers water against a head of 30 m. If
pump speed will increased to 120 rpm, what is the increase in head?
A. 43.2 m C. 34.6 m
B. 13.2 m D. 56.3 m

Pumps - 10
SOLUTION:
A boiler feed pump receives 50 Ii/sec of water with specific volume of
0.00112 mJ/kg at ahead of750 m. What is the power output of driving
motor if pump efficiency is 65%? h, N2 2
A. 505.32 KW -=(-)
C. 785.56 KW
hi N1
B. 643.54 KW D. 356.45 KW
h, 120 2
-" = ( - )
SOLUTION: 30 100
h 2 = 43.2 m
Increased = 43.2 - 30
W, = w Q h Increased = 13.2 m
w, = (1/0.001 12x 0.00981)(0.050)(750)
w, = 32846 KW
282 Pumps Pumps 283

Pumps - 13 Pumps - 15

A pump is used to deliver 50 Ii/sec of sea water at a speed of 120 rpm. A certain pump is used to deliver 150 gpm of water having a density of
If speed will increased to 135 rpm, determine the increase in pump 61.2 Ib/fe. The suction and discharge gage reads 4 in Hg vacuum and
capacity. 25 psi, respectively. The discharge gage is 2 ft above the suction gage.
A. 56.25 li/sec C. 87.54 Ii/sec What is the brake power ofthe motor if pump efficiency is 75%?
B. 34.56 li/sec D. 6.260 IiIsec A. 3.24 Hp C. 5.45 Hp
B. 2.67 Hp D. 6.89 Hp
SOLUTION:
SOLUTION:

Q2--2 N Pd - P,
-
h = +z
Q1 N\ W
Ps = - 4 in Hg x 14.7/29.92
Q 135 P, = -1.965 psi
2- -
- Pd = 25 psi
50 120 25 - (-1.965)
Q2 = 56.25 Ii/sec h = [ ](144) + 2
61.2
Increased = 56.25 - 50 h = 65.45 ft
Increased = 6.25 Ii/sec BP = w Qh
(61.2)(150/7.481)(65.45)
BP = ...:...----'--'----...:....:...--....:...
33,000(0.75)
BP = 3.24 Hp
Pumps - 14

A 15 KW motor running at 350 rpm is used to drive a pump. If speed


will changed to 370 rpm, what is the increase in power? Pumps - 16
A. 2.72 KW C. 56.45 KW
B. 17.72 KW D. 5.67 KW The discharge pipe of a pump is 400 mm in diameter delivers 0.5
mJ/sec of water to a building which maintains a pressure of 100 Kpa at
SOLUTION: a height of 30 m above the reservoir. If equivalent head is 2 m, what
power must be furnished by the pump?
A.21IKW C.340KW
P2 No 3 B. 480 KW D. 240 KW
- = ( - ")
p\ N,
P2 370 3
SOLUTION:
-=(-)
]5 350 Q = Axv
P2 = 17.72 KW 0.5 = (re/4 x 0.4 2) V
Increased = 17.72 - IS V = 3.9788 mlsec
Increased = 2.72 KW
284 Pumps Pumps 285

A 1265 KW C. 14.17 KW v,
v P
-+-
r
h = --+-+Z B. 23.54 KW D. 45.35 KW
2g w A Pd I
.=~~~_.

(3.9788) 2 100 SOLUTION: 151i/s


110m
h = +-+(30+2) I 12m
2(8.81) 9.81
n = 43 m Wp = w Q h
W p = wQh h = (72 - 10) + 0.1
W p = 9.81(0.50)(43) h = 62.15 ill +- I
W p = 210.92 KW W p = 9.81(0.015)(62.15)
w, = 9.145 KW
Power input = 9.145/0.65
Power input = 14.07 KW
Pumps -17

A centrifugal pump is designed for 1800 rpm and head of 61 m.


Determine the speed if impeller diameter is reduced from 305 mm to Pumps - 19
254 mm.
A. 1000 rpm C. 1500rpm The elevation of suction reservoir is 5 m above the pump centerline
B. 1250 rpm D. 1600 rpm and delivers to 85 m elevation tank which maintain 150 Kpa. If 1.5
mJ/sec of water is used to deliver a total head of 3m, determine the
SOLUTION: power needed by the pum p.
A. 1446 KW C. 4675 KW
h, D2 2 B. 2567 KW D. 3456 KW
-=(-)
h) D1
SOLUTION:
h, 254 2
-=(-)
61 305 h = (85 - 5) + 3 + 150/9.81
h 2 = 42.30 m h = 98.29 m
h2 N2 2 P = w Qh
-=(-)
hI N] P = 9.81(1.5)(98.29)
42.3 N2 2
P = 1446.34 KW
-=(-)
61 1800
N 2 = 1499 rpm
Pumps - 20 (ME Bd. Oct. 1989)

Pumps - 18 Water from a reservoir is pumped over a hill through a pipe 900 mm
in diameter and a pressure of one kg/crrr' is maintained at the pipe
discharge where the pipe is 85 m from the pump centerline. The pump
Water from a reservoir A 10 m elevation is drawn by a motor driven
have a positive suction head of 5 m. Pumping rate of the pump at 1000
pump to an upper reservoir B at 72 m elevation. Suction and discharge
rpm is 1.5 mJ/sec. Friction losses is equivalent to 3 m of head loss.
head loss are 0.15 rn, respectively. For discharge rate of 15 li/sec, find
What amount of energy must be furnished by the pump in KW?
the power input to the motor if overall efficiency is 65%.
286 Pumps
Pumps 287
A. 1372 kw C. 1234 kw
B. 1523 kw D. 1723 kw 0.5
Yd = ----c:-
(1t 14)(0.45)2
v. Yd = 3.144 mlsec
SOLUTION: I, :41 Pd = 1.0 kg/ern' x 101.325/1.033
•Yd 5 II
1.em' Pd ~~ 98.088 Kpa
= Q/A Q=1.5m'/s _------.J

1.5 5m 85m 2_y 2


p_p y
Yd
(1t/4)(0.9)2 I ':=-=0 h (zd-zs)+( d s)+(hLs+h Ld)+ d s
w 2g
Yd = 2.358 mlsec
98.088-0 (3.144)2 -(0»)
h (30) + ( ) + 1.5 + ...:....------'-------'--
Pd = 1 kg/ern" x 101.325/1.033 . 9.81 2(9.81)
P d = 98.088 Kpa h 42m
P 5 = O(open to atmosphere)
Water power = w Q h
p_p y 2_y 2
Water power = 9.81(0.5X42)
h = (zd -zs)+( d s)+(hLs+h Ld)+ d s
Water power = 206 kw
w 2g
. 98.088 - 0 (2.358)2 - (0)2
h = (85 - 5) + ( ) + 3 + ...:....-------:---=---..:.-
9.81 2(9.81)
h = 93.28 m Pumps - 22 (ME Rd. Apr. 1988)

Water Power = w Q h = 9.81(1.5)(93.28) Water from an open reservoir A at 8 m elevation is drawn by a motor-
Water Power = 1372.6 KW driven pump to an open reservoir R at 70 m elevation. The inside
diameter of the suction pipe is 200 mm and 150 mm for the discharge
pipe. The suction line has a loss of head three times that of the velocity
head in the 200 mm pipe. The discharge line has a loss of head twenty
Pumps - 21 (ME Bd. Oct. 1986) times that of the velocity head in the discharge pipeline. The pump
centerline is at 4 m. Overall efficiency of the system is 78%. For a
. Water from a reservoir is pumped over a hill through a pipe 450 mm discharge rate of 10 Ii/sec, find the power input to the motor and the
in diameter and a pressure of 1 kg/cm 2 is maintained at the summit. pressure gage readings installed just at the outlet and inlet of the
Water discharge is 30 m above the reservoir. The quantity pumped is pump in Kpag.
0.5 m'/sec. Frictional losses in the discharge and suction pipe and A. 3.34 kw C. 6.59 kw
pump is equivalent to 1.5 m head loss. The speed of the pump is 800 B. 5.45 kw D. 7.84 kw
rpm what amount of energy must be furnished by the pump, KW? v.;- --
A. 202 C. 204
B. 206 D. 208
SOLUTION: r~ ~
I
10lils
y, = Q/A 8m
SOLUTION:
v, =0.010, ~J
®
Yd = Q/A v, = 0.31831 mlsec :
Yd = Q/A
Pumps 289
288 Pumps
0.1782
0.010 v,
Vd (rr / 4)( 5 / 12) 2
(re /4 )(0.15)2
Ys 1.307 fils
Yd = 0.566 m/sec 0.1782
(0.31831)2 Yd =
h Ls = 3( ) (n I 4)(4 / 12) 2
2(9.81) = 2.043 fils
Yd
hLs = 0.01549 m From Steam Table, at 150 psig(l64.7 psi) and 140°F,
(0.566)2 w = 61 .424 Ib/fi l
h Ld = 20( ) p_p y 2_y 2
2(9.81) h=(d s)+(d s)
h Ls = 0.32642 m w 2g

Pd - Ps Y/ - Ys2 150(l44)-(-2xI4.7xI44) (2.043)2 -(1.307)2


h (Zd~Zs)+( )+(hLs+hLd)+·~-~- h =( +[ ]
w 2g 61.424 2(32.21)
h = 354 ft
h = (66_4)+0+(0.01549+0.3264)+[(0.566)2 -(0.31831)2
(61.424)(80 17.481)(354)
2(9.81) ] Water Power
h = 62.35 m 33,000
Water Power = w Q h Water Power = 7.05 hp
Water Power = 0.010(9.81)(62.353)
WaterPower = 6.12KW Brake horsepower = 7.05/0.7
20wer Input = 6.12/0.78 Brake horsepower = 10.07 Hp
Power Input = 7.84 KW Power Input of motor = 10.07/0.80
Power Input of motor = 12.59 Hp
Power Input of motor = 9.39 KW

Pumps - 23 (ME Rd. Apr. 1982)


Pumps - 24 (ME Bd. Apr. 1986)
A pump is to deliver 80 gpm of water at 140°F with a discharge
pressure of 150 psig. Suction pressure indicates 2 inches of mercury Determine the water horsepower and the mechanical efficiency of a
vacuum. The diameter of suction and discharge pipes are 5 inches and centrifugal water pump which has an input of 3.5 Up if the pump has
4 inches, respectively. The pump has a efficiency of 70%, while the an 8 inches nominal size suction and 6 inches nominal size discharge if
motor efficiency is 80%. Determine power input to the drive motor it handles 150 gpm of water at 150°F. The suction line gage shows 4"
A. 7.23 kw C. 8.34 kw Ug vacuum and the discharge gage shows 26 psi. The discharge gage is
B 2.34 kw D. 9.39 kw located 2 feet above the center of the discharge pipe line and the pump
inlet and discharge lines are at the same elevation.
SOLUTION: A. 2.52 hp C. 4.23 hp
B. 6.33 hp D. 8.34 hp
Q = (80)/(7.481 x 60)
Q = 0.1782 fills
290 Pumps 291
Pumps
SOLUTION:
Pumps - 26 (ME Bd. Oct.l984)
Q 150 gal/min x I fel7.48gal x IminJ60sec
=;
A boiler feed pump receives 40 liters per second at 180°C. It operates
Q 0.334 ft3/ sec
=
against a total head of 900 m with an efficiency of 60%. Determine
P, = - 4 in Hg x 14.7/29.92
power output of the driving motor in KW.
P, = -1.965 psi A. 453.23 C. 983.45
Vs = velocity at suction
C. 623.34 D. 523.27
v. > Q/A
v, = 0.334/[n/4 (8112)2]
v, = 0.957 ft/sec SOLUTION:
Vd = Q/A

Vd = 0.334/[n/4 (6!12i] From steam table(table 4), at 4 Mpa and 180°C,


Vd = 1.701 ft/sec h, = 764.74 KJ/kg
From steam table, at 150°F, 3/kg
VI = 0.00112484 m
w = 1/0.01634 Density = 1/0.00112484
w = 61.2 lb/ft' Density = 889.015 kg/rrr' (Im
3/1000kg)

(26xI44)-(-1.965xI44) (1.701)2 -(0.957)2 Density = 0.889 kg/li


h=( +2+[ ] Waterpower = (40xO.889015)(900)(0.0098!)
61.2 2(32.2)
Water power = 313.964 kw
h = 67.83 ft
Power output of motor = 313.964/0.60
(61.2)(0.334)(67.83)
Water Hp = Power output of motor = 523.273 kw
33,000

Water Hp = 2.521 .Hp


Pumps - 27 (ME Rd. Oct. 1984)

A boiler feed pump receives 40 liters per second at 180°C. It operates


Pumps - 25 (ME Rd. Oct. 1984) against a total head of 900 m with an efficiency of 60%. Determine
discharge pressure in Kpa for a suction pressure of 4 Mpa.
A boiler feed pump receives 40 liters per second at 4 Mpa and 180°C. A I J ,850 kpa C. 12,566 kpa
It operates against a total head of 900 m with an efficiency of 60%. B. 13,455 kpa D. 14,233 kpa
Determine the enthalpy leaving the pump in KJ/kg
A. 783.45 C. 756.23 .
B. 773.57 D. 765.23 SOLUTION:

SOLUTION: 180°C, -4MPa 2


-401ils ~®_ Pump Work = m(hz-h\) = mxhxO.00981
----.....' • . p "
,1 ' (h z - 764.74) = 900 x 0.00981
Pump Work = m(h z - hi) = m x h x 0.00981 h, - h.. gOOm h, . h 2 = 773.57 KJ/kg
(h- - 764.74) = 900 x 0.00981 h 2-h l = Vl(P 2 - P \)
hz = 773.57 KJ/kg 773.57 - 764.74 = 0.001 12484(P2 - 4000)'
P 2 = 11,850 Kpa

'"
29::' Pumps Pumps 293

(260/7.481)(62.4)(226)
Pumps - 28 (ME Bd, Apr. 19(0) Water power
33,000
Water power 14.85 Hp
A boiler feed pump receives 45li/sec of water at 190°C and enthalpy of
839.33 KJ/kg. It operates against a head of 952 m with efficiency of
Brake Hp = 14.85/0.70
70%0 Estimate the water leaving temperature assuming that the
Brake power = 21.21 Hp
temperature rise as due to the inefficiency of the input energy
A. 191°C C. 123°C
B. 143°C D. 165°C

SOLUTION: Pumps - 30 (ME Bd. Apr. 1985)


Let m, mass flow rate. kg/sec
Cc

A submersible, multi-stage, centrifugal deep well pump 260 gpm


Pump work = rn., x h x 0.00981
capacity is installed in a well 27 feet below the static water level and
mw (h 2 - h.) = m; x h x 0.00981
running at 3450 rpm. Drawdown when pumping at rated capacity is
h- - 839.33 = 952 x 0.00981
10 feet. The pump delivers the water into a 25,000 gallons capacity
11 2 = 848.67 KJ/kg
m - h ) overhead storage tank. Total discharge head developed by pump,
_,,~.",----_I -m",(h 2 -hl)=m c w
-t ) including friction in piping is 243 feet. Calculate the diameter of the
P(t 2 l
1"1 p impeller of this pump in inches if each impeller diameter developed a
head of 38 ft.
(84867 - 839.32>__ (848.67- 839.33) = (4.187)( t 2 - 190) A 3.28 C. 4.23
0.70 B. 5.33 D. 6.34
12 = 191°C

SOLUTION:

Pumps - 29 (ME Bd, Apr. 1985)


Let 0 = diameter of impeller
A submersible, multi-stage, centrifugal deep well pump 260 gpm
capacity is installed in a well 27 feet below the static water level. v = nDN
Drawdown when pumping at rated capacity is 10 feet. The pump
delivers the water into a 25,000 gallons capacity overhead storage
V = ~2gh

tank. Total discharge head developed by pump, including friction in


piping is 243 feet. Calculate brake horsepower req uired to drive the 1t 0 (3450/60) == ~2(322)(3'6)
pump if pump efficiency is 70%.
A. 3l.31 C. 21.21 D ~c 0.2738 ft
B. 41.41 D. 51.51 D = 3.28 inches

SOLUTION:
Pumps - 31 (ME Bd. Oct. 1981)

Total dynamic head 243 - (27 - 10) A double suction, single stage, centrifugal pump delivers 900 m 3/hr of
Total dynamic head 226 ft sea water(SG = 1.03) from a source where the water level varies two
meters from high tide to low tide level. The pump centerline is located
294 Pumps Pumps 295

2.6 meters above the surface of the water at high tide level. The pump Water Power wQh
discharges into a surface condenser, 3 m above pump centerline. Loss 62.4(0.0557)(245)
Water Power
of head due to friction in suction pipe is 0.8 m and that in the 550
discharge side is 3 m. Pump is directly coupled to a 1750 rpm, 460 V, 3 Water Power = 1.548 Hp
phase, 60 Hz motor. Calculate the specific speed of pump in rpm.
A. 3131 rpm C. 4141 rpm Motor size 1.548/0.64
B. 5151 rpm D. 6161 rpm Motor size 2.42 Hp

Therefore, use 3 Hp motor


SOLUTION:

Total suction head = 2 + 2.6 + 0.8 Pumps - 33 (ME Bd. Oct. 1996)
Total suction' head = 5.4 m
Total suction head = 17.71 Water is pumped at I mJ/sec to an elevation of 5 m through a flexible
0=900m'/h
Total discharge head = 3 + 3 hose using a 100% efficiency pump rated at 100 KW. Using the same
Total discharge head = 6 m I, n length of hose, what size of motor is needed to pump I mJ/sec of water
Total discharge head = 19.686 ft ,I·: to a tank with no elevation gain? In both cases both ends of hose are at
Q = 900/2 (double suction) atmospheric pressure. Neglect kinetic energy.
Q = 450 m 3/hr +1--- A. 51 KW C. 43 KW
Q = 1981 gal/min 0/2 Fl~ 012 B. 18 KW D. 22 KW
h = 17.71 + 19.68
h = 37.392 ft SOLUTION:
I750 v"l98! 2.6m
Ns At 5 m elevation:
(37.3 92) 3/4 I iHighT'Ide level
cbts=.._-
Water Power = w Q h
Ns 5151 rpm :r
100 = 9.81(1)(h)
2m r
I
I ilow Tide level h = 10.194 m
~
-' Ifthere is no elevation:
Pumps - 32 (ME Bd. Oct. 1996)
h = 10.194 - 5
h = 5.194 m
A pump driven by an electric motor moves 25 gal/min of water from
reservoir A to reservoir B, lifting the water to a total of 245 feet. The 9.81(1)(5.194) .
Power
efficiency of the pump and motor are 64% and 84% respectively. Power 51 KW
What size of motqr(HP) is required?
A. 5 Hp C. 4 Hp
B. 3 Hp D. 7.5 Hp Pumps - 34 (ME Bd. Apr. 1996)
SOLUTION:
A vacuum pump is used to drain a flooded mine shaft of 20°C water.
The pump pressure of water at this temperature is 2.34 Kpa. The
Q = 25 gal/min x 1min/60sec x 1ft317.481 gal
pump 1\ incapable of lifting the water higher than 10.16 m. What is the
Q = 0.0557 ft3/sec
atmospheric pressure?
L'Jb
Pumps Pumps 297
A. 1o» C. 98
B. 112 D. 101.9 Pumps - 36 (ME Bd. Apr. 1985)
SOLUTION: !:.
The rate of flow of water in a pump installation is 60.6 kg/sec. The
intake static gage is located 1.22 m below the pump center line and
Using Bernoulli's Theorem:
reads 68.95 Kpa gage; the discharge static gage is 0.61 m below the
2 pump centerline and reads 344.75 Kpa gage. The gages are located
PI VI P2 V22 close to the pump as much as possible. The area of the intake and
-+--+Zl =-+--+Z2
W 2g w 2g discharge pipes are 0.093 m 2 and 0.069 m 2, respectively. The pump
efficiency is 70%. Take density of water equals 1000 kg/rrr', What is
the hydraulic power in KW?
PI P 2 V2 2-V t 2
A. 17.0 C. 31.9
-=-+ +(Z2- ZI )
w w 2g B. 24.5 D. 15.2

~ 2.34
9.81 = 9.81 + 0 + 10.16 SOLUTION:
,
Q = 60.6/l 000
PI == 101.9 Kpa
Q = 0.0606 m3/sec
Ys == 0.0606/0.093
Pumps - 35 (ME Bd. Oct. 1995) Ys = 0.652 m/s
Vct = 0.0606/0.069
Yct == 0.878 m!s
It is desired to deliver 5 gpm at a head of 640 m in a single stage pump
having specific speed not to exceed 40. If the speed is not to exceed
1352 rpm, how many stages are required? . p_p y 2_y 2
h (
d s) +z+(--~-)
d s
A. 3 C. 5 w 2g
B.4 D.2
344.75- 68.95 (0.878)2 - (0.652)2
h ==( )+(-0.61+1.22)+[ ]
SOLUTION: 9.81 2(9.81)
h == 28.742 m
N == N.jQ
s
h 3/4 Hydraulic power == w Q h
Hydraulic power == 9.81(0.0606)(28.742)
40 = 1352..[5 Hydraulic power == 17.10 KW
h 3/4

h = 319.54 ft (head per stage)


Number of stages == 640/319.54 Pumps - 37 (ME Bd. Apr. 1996)
Number of stages == 2 stag.es
Water in the rural areas is often extracted from underground water
source whose free surface is 60 m below ground level. The water is to
be raised 5 m above the ground by a pump. The diameter Of the pipe is
298 Pumps 299
Pumps
10 em at the inlet and 15 em at the exit. Neglecting any heat
SOLUTION:
interaction with the surroundings and frictional heating effects, what
is the necessary power input to the pump for a steady now of water at
the rate of 15 Ii/sec in KW if pu mp efficiency is 85%?
H = total head

A. 9.54 C. 7.82
P

B. 5.343 D. 11.23 H = z + ­
w

SOLUTION: 137

H = 8 +­ 8m
9.81
Q 15 Ii/sec Q=28Jlps
H = 21.96 ill

Q 0.015 rrr'zsec Power = w Q H

Power = (0.283)(9.81)(21.96)

0.015 Power = 61 kw

v,
(n /4)(0.10)2
v, 1.91 m1s
0.015
Vd Pumps - 39 (ME Bd. Apr. 1998)
(n /4)(0.15)2
Vd 0.85 m/s A pump receives 8 kg/s of water at 220 Kpa and 110·C and discharges
it at llOO kpa. Compute for the power required in kilowatts.
V 2 _ V 2 A. 8.126 C. 7.041
h (Zd- Z )+ ( d s) B. 5.082 D. 6.104
s . 2g

SOLUTION:
h = 5-(-60)+ (0.85)2 -(1.91)2
2(9.81) Power = ill X h x 0.00981
h = 6485 ill
1100-220 m=8kgls
h =
Water Power = w Q h 9.81

Water Power = 9.81(0.015)(64.85) h = 89.704 m

Water Power = 9.54 KW


Power = (8)(89.704)(0.00981)
Power input = 9.54/0.85 Power = 7.04 kw
Power input = 11.22 KW

Pumps - 38 (ME Bd. Apr. 1998) Pumps - 40 (ME Bd. Apr. 1998)

A pump lifts water at a rate of 283 Ips front a lake and force it into a A fuel pump is delivering 10 gallons per minute of oil with a specific
tank 8 m above the level of the water at a pressure of 137 kpa. What is gravity of 0.83. The total head is 9.14 m, find how much energy does
the power required in kilowatts, ' the pump consumes in KJ per hour.
A. 71 C. 61 A. 169 C. 189
B. 41 D. 51 B. 199 D. 179
301
Fans & Blowers
300 Pumps
SOLUT ION:
SO!JJT ION:

20,000
3.785(60) Q =
Q = 10x--- 1000(60)
1000 3/s
3/hr
Q = 0.33 m
Q = 2.271 m
PI 127 (101.325/760)
Power = (0.83 x 9.81)(2.271)(9.14) PI 16.93 kpa
Power = 169 KJ/hr 0.33
Yj
(rr 14)(0.40)2
YI = 2.626
Pumps - 41 (ME Bd, Apr. 1998) 0.33
Y2 =
(rr 1 4)(0.35)2
of 75
A pump dischar ges 150 liters per second of water to a height Y2 = 3.429 m/s
efficien cy is 75% and the speed of the pump is 1800
meters. If the p_p y 2_y 2
what is the torque in N-m to which the drive shaft is subject ed? (d s)+Z+ (d s)
rpm, h
A. 771 C. 791 W 2g
B 781 D. 681 (3.429)2 -(2.626 )2
75-(-1 6.93) ]
h =[ ]+(0.45 +0.075 )+[
SOLUT ION: 9.81 2(9.81)
h = 10.05 m

Power = w Q h

Power = (0.150)(9.81)(75) Brake Power


Power = I] 0.4 kw Pumps - 43 (ME Bd. Oct. 1997)
Brake power = 110.4/0.75
to a
Brake power = 147.2 kw A centrif ugal pump deliver s 300,000 liters per hour of water
Power = wQh of water is 5
pressur ized tank whose pressur e is 280 kpa. The source
suction pipe is 300 mm and
Brake Power = 2 rt T N m below the pump. The diamet er of the
driving
147.200 = 2rrT(l8 00/60) the dischar ge pipe is 250 mm. Calcula te the kw rating of the
T = 781 N-m motor assumi ng the pump efficien cy to be 72%.
A. 41.75 kw C. 43.28%
B. 35.75k w D.38.1 6kw
.Pumps - 42 (ME Bd. Oct. 1997)
SOLUT ION:
er
A pump with a 400 mm diamet er suction and a 350 mm diamet
per minute of 15.6°C water.
dischar ge pipe is to deliver 20,000 liters
below 300,000
Calcul ate the pump head in meters if suction gage is 7.5 em Q =
127 mm Hg vacuum and dischar ge gage is
pump centerl ine and reads 1000(3600)
45 cm above the pump centerl ine and reads 75 kpa. Q= 0.0833 m
3/s

A. 15 m C. 20 m
B. 5 m D. 10m
302 Pumps Pumps 303

0.0833 Water Power


VI
Pump Efficiency
(rr /4)(0.3)2 Brake Power
VI 1.178 mls (9.81)(0.8)h
70 = ------'--'-----'-
00833 0.74
V2
(rr /4)(025)2 h = 66 m
Vl 1.697 mls
p_p v 2_v 2
h=(d s)+Z+(d 5)
W 2g Pumps - 45 (ME Bd, Apr. 1997)
280 - 0 (1.697)2 -(1.178)2
h =(--) + 5 + [ . ] A pump delivers 500 gpm of water against a total head of 200 ft and
9.81 2(9.81) operating at 1770 rpm. Changes have increased the total head to 375
h=33.62m ft. At what rpm should the pump be operated to achieve the new head
Water Power at the same efficiency?
Pump Efficiency
Brake Power A. 2800 rpm C. 3434 rpm
B. 3600 rpm D. 2424 rpm
Water Power = w Q h
Water Power = (9.81)(0.0833)(33.62)
Water Power = 27.473 kw
SOLUTION:
27.473
0.72 =
Brake Power
Brake Power = 38.16 kw (motor rating)
hI NI 2
-==(-)
h, N2
200 1770 L
Pumps - 44 (ME Bd. Oct. 1997) -=(-)
375 N2
A centrifugal pump delivers 80 liters per second of water on test. N 2 = 2424 rpm
Suction gage reads 10 mm Hg vacuum and 1.2 meters below pump
centerline. Power input is 70 kw. Find the total dynamic head in
meters.
A. 66 C. 62
B. 60 0.64

SOLUTION:

Q = 80/1000
Q = 0.08 mJ/s

Using the typical pump efficiency of 74%.


304 Fans & Blowers Fans & Blowers 305

SOLUTION:
FANS AND BLOWERS
P
wa -
RT
101.325
Fans & Blowers - 1 (ME Apr. 1997) wa =
0.287(25 + 273)
A fan whose static efficiency is 40% bas a capacity of 60,000 fe/hr at Wa 1.18 kg/m'
60°F and barometer of 30 in Hg and gives a static pressure of 2 in of h w «;
water column on full delivery. What size of electric motor should be h, =
Wa
used to drive the fan?
(00254)( I 000)
A. 1/2 Hp C. 2 Hp hs =
B. 1 Hp D. 1 '1/2 Hp U8
hs = 2 1.52 m

SOLUTION: 1.42
v =
s (n/4)(OJ)2
~ v, = 20.09 mls
h W
h=
s ~
wa
f 1 2in
Vd =
(n
1.42
4)(0.275)-
1

~s:;;:~: ';:{!;)~ =~~~~


(2 112)(62.4)
hs = Vd = 23.9 mfs
wa 30 in Hg (239)-1 - (20.09)"'
h, =' 10Alw. h = ------
v 2(9.81)
Air Power ~ wa Q h h, = 8.5--1 m 300mmO

W a (60,00 160)(10.4 I W
a) ~ a
Air Power = -''------------=---
33,000
h = h, + h,
h = ~IS~ + 8.54
-c
760rnrnHg
L:"'=J-4T~-'----+

~}54,m
Air Power = 0.315 Hp h = 30.0C' 111 25°C

Therefore: Use I hp motor (standard) w.\Qh


'11
Hi>
(1.18 x 0.00981XI.42X30.06)
0.70 =
BP
Fans & Blowers - 2 (ME Bd. Oct. 1997) BP = 0.7058 kw

A fan draws 1.42 m J per second of air at a static pressure of 2.54 cm of I]


wnQ h,
= ---
water through a duct 300 mm diameter and discharges it through a , BP
duct of 275 mm diameter. Determine tbe static fan efficiency if total p.1 S:\ O.00981)(l.42X21.52)
fan mechanical is 70% and air is measured at 25"C and 760 mm Hg. 1], -=
0.7058
A. 50.1 1% C. 65.67%
1], c 50113%
B. 54.34% D. 45.34%
306

Fans & Blowers - 3


Fans & Blowers Fans & Blowers
b, = 125 rn
Air power = wQh
Air Power = (1.2 x 0.00981)(90,000/3600)(125)
307

I
*
",;

Find the air horsepower of an industrial fan that delivers 25 rolls of air Air Power = 36.78 KW
Ihruu~h a 900 mm by 1200 mm outlet. Static pressure is 127 mm of Size of motor = Brake Power
~lIll'r gal-:l' and air density is 1.18 kg/nr', Size of motor = 36.78/0.65
A. 65.35 Hp C. 60.35 Hp Size of motor = 56.59 KW
B. 52.35 Hp D. 70.35 Hp

,t II LIT/ON:
Fans & Blowers - 5
Q=Axv
25 = (0.9 x 1.2) v At 1.2 kg/m 3 air density a fan develops a brake power of 100 KW. If
v = 23.15m1sec operates at 98 Kpa and 32°C with the same speed, what is the new
h, = v 2/2g brake power of the fan?
h, = (23.15)2/2(9.81) A. 68.4 KW C. 67.5 KW
h, = 27.315 m B. 36.7 KW D. 93.3 KW
h, = hw(dw/d a )
h, = 0.127(1000/1.18) SOLUTION:
h, = 107.63 m
h = h, + h, WI = 1.2 kg/rn'
h = 107.63 + 27.315 W2 = P/RT
h = 134.94 m W2 = 98/(0.287)(32 + 273)
Air Power = w Q h W2 = 1.1195 kg/m'
Air Power = (1.18 x 0.00981)(25)(134.94)
Air Power = 39.052 KW BP 2 w2
----
Air Power = 52.35 Hp
BPI WI
BP1 J.I195
---_.--
Fans & Blowers - 4 100 1.2
BP2 = 93.29 KW
A boiler requires 90,000 ml/hr of standard air. The mechanical
efficiency of fan to be installed is 65%. Determine the size of driving
motor assuming fan can deliver a total pressure of 150 mm of water
gage. Fans & Blowers - 6
A. 56.6 KW C. 45.5 KW
B. 78.5 KW D. 23.5 KW A fan has a suction pressure of 30 mm water vacuum with air velocity
of 3 m/sec. The discharge has 150 mm of water gage and discharge
SOLUTION: velocity of 7 m/sec, Determine the total head of fan if air density is 1.2
3 .
kg 1m .
For Standard air, w = 1.2 kg/m ' A. 150 m C. 154 rn
h, = hwCdw/d.) B. 152 m D.156m
h, = 0.15(10001l.2)
301l Falls & Blowers Fans & Blowers 309
SOLUTION: A. 150 mm water gage C. 24 J mm water gage
B. 180 mm water gage D. 456 mm water gage

(h w 2 - h W1 )d w SOLUTION:
h,
da
[0.15 - (-0.03)J(I 000) WI 1.2 (standard air density)
hs Wz = PIRT
1.2 735( 10 1.325/ 760)
hs 150 m Wz
v 2 _ V 2 0.287(93 + 273)
_ct_ _s
hv Wz 0.933 kg/rn '
2g h2 w2
2
7 _ 32
h, hi WI

2(9.81) h, 0.933
- - - -

h, = 2.038 m 310 1.20


h = 150 + 2.038 11 sz = 24 J mm water gage
h = 152.038 m

Fans & Blowers - 9


Fans & Blowers - 7
The total head offan is 185 m and has a static pressure of 210 mm of
water gage, what is the velocity of air flowing if density of air is 1.15
A 50 KW motor is used to drive a fan that has a total head of 110m. If kg/rn''?
fan efficiency is 70%, what is the maximum capacity of the fan using A. 6.85 m/sec C. 4.76 m/sec
standard density of air?
J B. 3.45 m/sec D. 8.54 m/sec
A. 27 m /sec C. 3 I rrr'/sec
B. 29 mJ/sec D. 33 m'zsec SOLUTION:
SOLUTION: h, =0.21(1000/].J)
h, = 182.6J m
Air power = 50(0.7) h = h, + h,
Air power = 35 kw J85 = 182.61 + h,
Air power = w Q h h, = 2.39J m
35 = (1.2 x 0.00981)(Q)(110) hv = y 2/2g
Q = 27.02 m'zsec 2.3J = YZ/2(9.8J)
v =6.85 m/sec

Fans & Blowers - 8 Fans & Blowers - 10

A fan using standard air condition can developed a static pressure The volume flow of air delivered by fan is 20 mJ/sec and 180 mm water
head of 310 mm water gage. If fan will operate at 93°C and 735 mm of gage. The density of air is 1.185 kg/rn'' and the motor power needed to
Hg, find the new static pressure required. drive the fan is 44 KW. What is the fan efficiency?
j/u
Fan.\ .e Blowers Fans & Blowers 31 ]
A. 70.26% C. 75.26% Static pressure (0.9329/J .2)(310)
B. 80.26% O. 90.26% Static pressure 24] mm water gage
SOLUTION:

h, = 0.18(1000/1.185) Fans & Blowers - 12 (ME Bd. Oct. 1994)


h, = ]51.89m
Air power = w Q h Air enters a fan through a duct at a velocity of 6.3 mls and an inlet
Air power = (1.185 x 0.00981)(20)(151.89) static pressure of 2.5 em of water less than atmospheric pressure. The
Airpower = 35.316 KW air leaves the fan through a duct at a velocity of 11.25 rn/s and a
T] = 35.3 ]6/44 discharge static pressure of 7.62 cm of water above the atmospheric
T] = 80.26% pressure. if the specific weight of the air is 1.20 kg/m' and the fan
delivers 9.45 m3;sec. what is the fan efficiency when the power input to
the fan is 13.75 KW at the coupling?
A 7181% C. 81.34%
Fans & Blowers - 11 (ME Bd. Apr. 1984) B. 91.23% D. 61.34~·{'

A fan is listed as having the following performances with standard air:


Volume discharged - 120 m3/sec SOLUTION:
Speed - 7.0 rps
Static pressure - 310 mm water gage
Brake power required - 620 KW h = h, + h,
The system duct will remain the same and the fan will discharge the p_p y 2 _ y2
3/sec h=(d s)+(d s)
same volume of 120 m of air at 93°C and a barometric pressure of w 2g
735 mm Hg when its speed is 7.0 rps. Find the brake power input and
the static pressure required.
A. 23] rum water C. 24] mm water 0.0762 - (-0.025) (11.25) 2 - (6.3)2
h =[ ](1000)+[ ]
B. 251 mm waterO. 261 mm water 1.20 2(9.81)
h = 88.76 m
SOLUTION:
Air power ,~ w Q h
For standard air, w = 1.2 kg/rrr' Air power = (1.2 x (\.0098])(9.45)(88.761)
Air power = 9.874 KW
Solving for the density at 93°C and 735 mm Hg Fan Efficiency = 9.874/13.75
w = P/RT
Fan Efficiency c= 71.81 %
73S(101.J25 I 760)
w
0.287(93 + 273)
w = 0.9329 kg.nr' Fans & Blowers - 13 (ME Bd. Apr. 1995)
Using fan laws: l
A fan delivers 4.7 m 3/sec at a static pressure of 5.08 em of water when i
'J
Brake power input = (0.9329/] .2)(620) operating at a speed of 400 rpm. The power input required is 2.963
Brake power input = 482 kw KW. If 7.05 m3/sec are desired in the same fan and installation, find
the pressure in cm of water.
312 Fans & Blowers Fans & Blowers 313

A. 7.62 C. 11.43 BPI WI T2


--=-=-
B. 17.14 D. 5.08
BPz w2 T]
SOLUTION: 6.5 65 + 273
Q N, P2 2 1+ 273
-1 - -
Q2 N 2 P2 = 5.68 KW
4.7 400
7.05 N z Fans & Blowers - 15 (ME Bd. Oct. 1996)
N 2 = 600 rpm
h, NJ z Air is flowing in a duct with velocity of 7.62 m/s and static pressure of
-==(-)
h, N2 2.16 em water gauge. The duct diameter is 1.22 m, the barometric
pressure 99.4 Kpa and the gage fluid temperature and air temperature
5.08 400 2
-==(-) are 30°e. What is the total pressure against which the fan will operate
hz 600 in em of water?
h1 = 11.43 em of water A. 3.25 C. 3.75
B. 2.5 D. 1.25

Fans & Blowers - 14 (ME Bd. Apr. 1995) SOLUTION:


2
V
A fan described in a manufacturer's table is rated to deliver 500 hv '[I
fa
m3/m in at a static pressure (gage) of 254 cm of water when running at
I
~b

250 rpm and requiring 3.6 KW. If the fan speed is changed to 305 rpm (7.62)2
h = --- 11
and air handled were at 65°C instead of standard 21°C, find the power v 2(9.81)
in KW.
h, = 2.959 m
A. 3.82 C. 4.66
w = P/RT
B. 5.08 D. 5.68
w = 99.4/(0.287)(30+273)
w = 1.143 kg/nr'
SOLUTION:
Solving for the velocity head in terms of em of water
h, = 2.959(1.14311000)
At 305 rpm and 21 0C:
h, = 0 0034 m of water
PI/P z = (N,/N 2 ) 3
h, = 0 34 em of water
3.6/P2 = (250/305)3
h = h, + h,
P1 = 6.5 KW
h = 216 + 0.34
At 305 rpm and 65°C: h = 2.5cmofwater
w = P/RT
WI PI RT,
----
\V
2 P/RT2
\V 1 T2
---
w2 T]
314
Fluid Mechanics
Fluid Mechanics 315
FLUID MECHANICS
Fluid - 2 (Math-ME Bd Oct. 1997} .

What is the expecten nead loss per mile of a closedcircular pipe (17
Fluid - I (Math-ME Bd Oct. 1997)
in inside diameter), friction factor of 0.03 when 3300 gal/min of water
now under pressure?
A perfect venturi with throat diameter of 1.8 inches is placed A. 38 ft C. 0.007 ft
horizontally in a pipe with a 5 in inside diameter. Eighty (80) Ib of B.3.580ft D. 0.64 ft
water now through the pipe each second. What is the difference
between the pipe and venturi throat static pressure? SOLUTION:
z
A. 29.91b/in C. 5020lb/in z
z
B. 34.81b/in D. n.3lb/in z Velocity ~- QIA
3300/7.481
SOLUTION Velocity
(n ! 4 )(17 / 12) 2 (60)
VI ~ QIA I Velocity = 4.66 ft/s

VI
80/62.4
CD -..::c..~":.:c _ _ ® _
L = 5280 ft (I mile)
f L v
2 ..Y.J 12:] Q=33~Ogal/min
Head loss =
(rr 14)(5/12)2 I --."""::"-.... -:.~J.~::-:'-=---
2 g 0 L=1mile
m=80lb/s V
VI 9.4 fils --+ \ 5" ................8"1 P ~ 0.03(5280)(4.66) 2
Vz QIA z
V, p,(,L~i~:,:.~~.,.;"=·~l~_~< "-~ c ) Head loss
2(32.2)(17/12)
80/62.4 Head loss = 37.7/1
Vz
(n 14)(1.8/12)2
Vz = 72.549 fils
Fluid - 3 (Math-ME Bd Apr. 1997)
Apply 109 Bernoulli's Equabon:
A rigid container is closed at one end and measures 8 in diameter by
2 ] 2 in long. The container is held vertically and is slowly moved
PI VI P2 V/ downward until the pressure in the container is 15.5 psia. What will
-+--+Zt =-+--+Z
w 2g . w 2g be the depth of the water surface measure from the free. water
surface'? A. 22 in C. 12 in
For horizontal, z, = Zz
B 9.2 in D. 9.8 in
Water Surface
2_V 2
V_2_ SOLUTION: - - --
--- - ---- -
PI - P2 _1
---_
-.-
- -

w 2g Pabs = P~age + Patn. 8"16


h
PI - P2 (72.549)2 _(9.4)2 15.5 = Pgage + 14.7
Pgage = 0.8 psi Air
62.4 2(32.2) Pgage = w h 15.5psia
12"¢
PI - Pz = 5014.281b/tY x I1144 0.8(144) = 62.4(h) -
PI - P2 = 34.82 psi h = 1.846 ft x 12
h = 22.15 in
316 Fluid Mechanics Fluid Mechanics 317

PI -P2 22 _ V 2 )
V '+(Z2- Zj
Fluid - 4 (Math-ME Bd Apr. 1997) w 2g
w = density of air
The pressure drop across a turbine is 25 psi. The flow rate is 55 3
w = 0.075 Ib/ft (standard)'
gallons per minute. Calculate the power output of the turbine.
6.24 V/-O
A. 0.802 hp C. 1.05 hp --= +0
B. 0.41 hp D. 2.54 hp 0.075 2(32.2)
V z = 73.2 ft/sec (3600/5280)
SOLUTIOW V 2 = 49.9mph

h P/w
25 x 144 Fluid - 6 (Math-ME Bd, Apr. 96)
h
62.4 ~TUrblne~gpm
h 57.69 ft P,-P,=25psi What is the density of a stone that weighs 19.9 Ib (88 N) in air and 12.4
Ib (55 N) in water?
Q = 5517.4ISl A 2,651.2 kg/rn' C. 2,578.2 kg/rrr'
3D. 3
Q = 7.35 ft 3/min B. 2,612.5kg/m 26,700kg/m
w.s.
Turbine power = w Q h ~ --../::

7.35( 62.4 )(57 .69) SOLUTION:


Turbine power = BBN BBN
33,000
Turbine power = 0.802 hp
Let V = volume of stone

If body floats. then the weight of the object is


t
equal to the Bouyant force.
Fluid - 5 (Math-ME Bd Apr. 1997) W = BF 55N
BF is also the difference in weight of object in air
Two tubes are mounted to the roof of a car. One tube points to the and in water.
front of the car while the other point to the rear. The tube are BF = 19.9 - 12.4
connected to a manometer filled with fluid of specific gravity 0.60. BF = 7.51bs
When the height difference is 2 inches, what is the car's speed? BF = wV
A. 46 mph C. 50 mph 7.5 = (62.4) V
B. 43.8 mph D. 62 mph V = 0.1202 ft3
Density of stone = 19.91b/0.1202 ft3
SOLUTION: Density of stone = 165.561b/ft3 (1000/62.4)
Rear rront
Density of stone = 2,653.2 kg/m'
For the liquid +v;-
PI-Pz=wh
PI - P2 = (0.6 x 62.4)(2/12)
PI - Pz = 6.24 Ib/ft 2
By applying the Bernoulli's equation:
PI VI
2
P2 V2 -
- + - + Z , =-+--+Z2
1
Fluid
r
CD
2"
Fluid - 7 (Math-ME Bd. Oct. 1995)

The now rate of water through a cast iron pipe is 5000 GPM. The pipe
is 1 ft and the coefficient of friction f = 0.0173. What is the pressure
W 2g w 2g
drop over a 100 ft length of pipe?
318 fluid Mechanics Fluid Mechanics 319

A. 3] 7 26 lblin z C. 337.261b/ft z . 8.02(62.4)(69.23)


B 21078 lb/in z D. 23.7801b/ft z Turbine power = -----'--'------'--
33,000
SOLUTION Turbine power = 1.05 hp

5000
o =
7481(60) Fluid - 9 (ME Bd. Oct. 1996)
o 11.139 ft3 Is
The fluid in a manometer tube is 60% water and 40% alcohol (SC
v =
~:.D1ft ) a=50~Ogpm 0.8). What is the manometer fluid height difference if a 6.2 psi
pressure is applied across the two ends of a manometer?
, L=100ft
A 15.5 in C. 36 in
v = B.186in D.215in
v = SOLUTION:
@
h, =
P J - w h P, v
T
2gD
PJ-P Z = wh
0.0173(100)(14.183)2 6.2(144) = (62.4 x 0.8)(h)
hr = _ SG=O.80
2(32.2)(1)
hi = 5.404 ft of water h = 17.88 ft = 214.6 in

Fluid - 10 (ME Bd. Apr. 1996)


Fluid - 8 (Math-ME Bd. Oct. 1995)
An air bubble rises from the bottom ofa well where the temperature is
Pressure drop across a turbine is 30 psi, the flow rate is 60 gpm. 25°C, to the surface where the temperature 27°C. Find the percent
Calculate the power output of the turbine. increase in volume of the bubble if the depth of the well is 5 m.
A 0.41 hp C. 6,30 hp Atmospheric pressure is 101.528 Kpa,
B. 105 hp D. 2.54 hp A. 49.3 C. 56.7
8413 D. 38.6
SOLUTION:
SOLUTION:
h = P/w t,=27"C
30 x 144 P2 T2 PlY] ·_·-~'V2
----
P,~~..... ~
~=60gpm
h
624 T2 T] , J:
h = 69.23 ft P,·P,=30psi
o = 60/7.481 PI wh+ 101.528
=
5m

o = 8.02 ft3 /min PI = 9.81(5) + 101.528


wOh PI = 150.378Kpa
.. _ _' V,
Turbine power = 33,000 150.378(Y 1 ) 101.528(Y2 )
t,=25°C
(25 + 273) (27 + 273)
320 fluid Mechanics. Fluid Mechanics 321
V2 ~ 1491V 1
Fluid - 12 (Math-ME Bd Oct. 1998)
V2 -V1
%lncrease in volume = ---
V1 '\ 24 inches long rod floats vertically in water. It has a I sq. in. cross
section and has a specific gravity of 0.6. What length L is submerged?
1.491Vj - VI
%lnerease in volume = A. 14.4 in C. 9.6 in
VI B. 24 in D. 18.0 in
%lnerease in volume = 49.10%

SOLUTION: ~w
R~
Fluid - II For floating object, A=1 in'---.

An empty, open can is 30 cm high with a Io-cm diameter. The can, W = BF 24"
with the open end down, is pushed under water with a density of 1000 ____ I_: J
W.~L t= ,I __
=!j=__ ..A
kg/m ', Find the water level in the can when the top of the can is 50 cm w., v, vc; v,
below the surface.
L

--. ·~U
(0.6 x 1000)( 1 x 24) = 1000 [I xL)
A. 17.20 em C. 4.20 em
B. 2.12 em D. 5.87 em L = 14.4 in BF
t
SOLUTION: Fluid - 13 (Power-ME Bd Oct. 1999)

Consider the water pressure:


t'w = wh + 101.325
P w = (0.8 - x)(9.81) + 101.325
P; = 109.173 - 9.81x

Consider the air pressure:


I
':]m
I,. ~-- -.-----,1
80
O

bo . ,- __=- x
.
P"P,
80-x
Pw
Flow of water taking over in a pipe having a velocity of 10 m/s.
Determine the velocity head of the water.
A 50.1 m
B. 5.1 ill
C. 8.2 m
D. 100 m

10 I

J.,
!
r'IV, = P ZV 2 SOLUTION:
-----=-------
10 1.325(A x 0.3) = Pz[A (0.3 - x)]
30.3795 h = vZ/2g
Pz =
Pw = Pz
(0.3 - x)

30.3795
h = (10)z 12(9.81)
h = 5.1 ill II . , . . ~10mJs
109.173 - 9.81x
(0.3- x) Fluid - 14 (Power-ME Bd Oct. 1999)
2
9.81x . 112.1 16x + 2.3705 = 0
The length of pipe is 168 meters. If the pressure drop is 50 Kpa for
By qua' Iratie formula: every 30 meters, what is the total oressure drop?
A = 0.02l18m A. 260 kpa C. 280 kpa
x = 2.12 em B. 300 kpa D. 100 kpa
,')')
.)-­
Fluid Mechanics Fluid Mechanics 323
"OU: IJON: , ?
PI-p" V7'-V t ­
=--~--+(z -z)
w 2g 2 I
Total Pressure Drop = SO kpa (168 m!30 m)
Total Pressure Drop = 280 kpa Z, - 7.1 15 m
Z2 - ZI -1.5 m
0.02
VI
(re / 4)(0.08)2
Fluid - 15 (I'O\~er-ME Bd Apr. 1999)
VI 3.98 m/s
0.02 z,
Hvd ra ulic clficicncv S5%, find Q in Ii/s, Power developed 10,500 kw \/2
under head of 320 Ill. (re I 4)(0.04)2
A 3935
B 3e.j.52 D 3623
C 327,1 V2
p,-p)
15.91 mJs

- - - ' =
(15.91)
2
-(3.98)
2
+ (-1.5)
1- Reference

SOU lION 9.81 2(9.81)


PI - P? = 104.016 kpa
Brake Power
11 = ~-

Water Power
10,500 Fluid - 17 (Power-ME Bd Apr. 1999)
0.85 =
Water Power
Water Power = 12,352.94 kw A fluid that has a velocity of 18 m/s will have an equivalent head of:
Water Power = w Q h A. 16.51 m C 18.34 m
B. 12.44 rn D. (0.34 m
12,35294 = 981 (Q) (320)
Q = 3935 m 3/s (1000)
Q = 3935 li/s SOLUTION:

v
h
2g
FIlJi,[ - 16 (l)o\\cr-i\lE Bd Apr. 1999)

A c vl iud rira l pipe with water flowing downward at 0.02 mJ/s having (18)2
h
lOp d ia me t cr "I' O.OS, bottom diameter of 0.04 m and height of 1.5 m. 2(9.81)
Find rhc prcxsur« between the pipe.
A 104 kpa C 120 Kpa
h 16.51 m
B 97 kpa 0 143 kpa

SUU.' IIO"-J.

Pi VI" P1 V," Fluid - 18


-+--+zl = -+--+Z)
\V ~g w 2g ­
Determine the size of pipe which will deliver 5 liters of medium oil (v =
6.10 X 10,6 m 2/s) assuming laminar now conditions.
324 Fluid Mechanics 325
Fluid Mechanics
A 522 111m C. 550 mm

B 454 !TIfT! D. 650 mm Fluid - 20

SOLUTION The type of flow occupying in a 30 em diameter pipe which water


6 2/s
flows at a velocity of 2.10 m/s. Use v = 1.13 X 10. m for water.
A turbulent C. laminar
Q B. constant D. none of these
v
A

0005

v SOLUTION:
2

4)d
(T[
V ~c o 006366 d)

dV dV

Re ~- Re

v
v
For laminar flow, Re = 2000
2 (0.30)(2.1 0)
d(0.006366 I d )
Re

2000 ~ _~
113 X 10-
0
610 x 10

d 0522 m

Re = 577,52212
d- 522 mm

Since it is greater than 2000, then it is turbulent flow

Fluid - 19

The power available in ajet having a cross-sectional area of II.OOS m Z Fluid - 21


with a velocity of 25.80 rn/s,
A. 34 kw
A man weighing 64 kg causes a flat area 30 em thick to be just fully
C 43 kw
B 49 k w submerged in a sea water (SG = 1.03). Neglecting weight the area
D. 23 kw

must be:

SOL!)TION A. 0.085 m ' C. 0.173 m 2


B. 0.756 m
2
D. 0.062 m 2
h = '/12g
~~' ~""..." ..
p

---+ 25.80m/s

S
h
(2580)2
----
tIJijiiiI1J SOLUTION:
2(98 J)
h = 33926 m

A"'O.005m
1

M
~;i,~·
Q .~ Av For floating object:

Q ~ (0.005)(2580)
W = BF

Q= 0129 mJ/s W = wV
·f••
P = wQh
P = (981)(0129)(33926)
P = 42.93 kw
64
A
=
=
(1.03 x 1000)(0.3 x A)
0062 m)
rSF
326 Fluid Mechanics Fluid Mechanics 327

V, o 778 rn/s
Fluid - 22 0.055
V2 ,
(rt . 4)(0.075)~
What force is exerted by water jet 50 mm diameter if it strikes a wall
at the rate of 15 m/s?
V,= 12456 mis
A. 342 N C. 764 N o )
B. 442 N D. 5113 N P, - P, (12456f - (0.778r
--- +-0
w 2(981)
P, -. P,
SOLUTION: 79 111

F = wQv

Q = A v

Q = [(11/4)(0.05)2](15) Flu id - 2-t

Q = 0.02945 m3/s

A jet of water 50 mm diameter with a velocity of 35 mls is being


F (1000)(0.02945)( I 5) eli,charged in a horizontal direction from a nozzle mounted on a fire
F 441.75 N truck. The force required to hold the nozzle stationary is:
A 1.34 KN C. 4.23 KN
B 240 KN D. 323 KN

Fluid - 23
SOLUTION:
A 300 mm x 75 mm venturi meter is inserted in a 300 mm diameter
pipeline where water flows at 55 liters/s. Neglecting friction loss,
compute the drop in pressure head from the inlet to the throat. F = wQ v
A8m C.6m Q = A x v
B. 10 m D. 12 m Q = [(11/4)(0.05/](35)
Q = 0.0687 m 3/s

\61 F (1000)(0.0687)(35)
SOLUTION:

"
F 2404.5 ~

V 75mm

v, , F 2.404 KN

2 Z, Z,
PI Vj P2 V/ Reference
-+--+Zl = -+--+Z2

w 2g w 2g
Ftuid-25
2 2
PI-PO V) -VI
---- - +(Z2- Z j ) An open storage vessel has 3 m of oil (SG = 0.82) and 6 m of water.
w 2g
The pressure at the bottom is:
0.055 A 45 kpa C. 83 kpa
VI
(11 /4)(0.3)2 B 65 kpa D. 92 kpa
328 Fluid Mechanics. Fluid Mechanics 329

SOLlfTiON
Fluid - 27
F=--C:=-Oil--­
P = pressure at the bottom 3m=h. A 300 0101 diameter pipe discharges water at the rate of 200 Ii/s. Point
p = ""0h, + Ww h., I on the pipe has a pressure of 280 kpa and 3.4 m below point 1 is
P = (082 x 9.81)(3) + 981(6) Water point 2 with a pressure of 300 kpa. Compute the head loss between
6m=h w
P = 83 kpa points 1 and 2.
p A 4.2 m C. 6.3 m
B. 2.5 m D. 1.4 m

Fluid - 26
SOLUTION:
A 200 0101 diameter pipe gradually reduces to a 100 0101 diameter.
The 200 0101 diameter pipe is connected to another pipe having a
pressure of 600 kpa horizontally with a flow of 0.04 m 3/s. Find the
pressure at the 100 0101 diameter. PI V j
2
P2 v/
A 588 kpa C 566 kpa
- + - + Z \ =-+--+Z2 +h L
W 2g w 2g
CD
B 642 kpa D. 598 kpa

PI - P2

I
hL = +(Z\-Z2)
p ,=280K Pa
w
;:,OLUT10N: 280- 300 3.4m
hL ----+3.40
9.81
hL 1.36 m P,=300Kpa

PI V\2 P2 V2
2
-+--+ZI =-+--+Z2

W 2g w 2g

Ffuid - 2~

0.04
VI An object weighs 90 N in air and when immersed in water it weighs 50
(n: /4)(0.2/ N. Compute the specific gravity of the object.
VI 1.2732 m/s Q=0.04m'/s A. 1.25 C. 2.25
-----+ 100mm
0.04 B. 3.25 D. 4.25
V2
(n: 14)(0.1/

SOLUTION:
V2 509 m/s

-W'$''--T~~~-c.:
600 (12732)2 P7 (5.09)2
BF = 90 - 40
-+ +0=-- + +0

9.81 2(9.81)

Po = 587.85 kpa
9.81 2(9.81) BF = 50 N

BF = W Va
50 = 9810 v;
11F
56N
V o = 0.00408 rrr'
Wo weight of object in air
330 Fluid Mechanics
Past Board Examination Elements (1994-1999) 331
Wo - w, v;
90 c. "0 (0.00408) PAST BOARD EXAMINATION

w., = 22,058.82 N/m J


SG = 22,058.82/9810 ELEMENTS

SG = 2249

Fluid - 29 Elements - 1 (ME Rd. October 1994)

A rectangular open box 7.6 m by 3 m in plan and 3.7 m deep, weighs When a substance is gaseous state is below the critical temperature it is
J5U KN and is launched in fresh water. If water is 3.7 m deep what called:
weight of stone placed in the box will cause it to rest at the bottom? A. Vapor
/\ 35034 KN C. 498.34 KN B cloud
B 65345 KN D. 477.57 KN C. moisture

Do steam

Sl)J.l iTlON:
Elements - 2 (ME Rd. October 1994)
w.s.
Is the condition of pressure and temperature at which a liquid and its vapor
are indistinguishable:
A. critical point
B. dew point
C. absolute humidity
Total weight = BF D. relative humidity

J."Oj W, = w V
Elements - 3 (ME Rd. October (994)
3"i() f W, = 9.111 [(7.6)(3)(3.7)]
If the temperature is held constant and the pressure is increased beyond the
W, ~ 477.57 KN saturation pressure, we have a:
A. saturated vapor C. saturated liquid
B. compressed liquid D. subcooled liquid

Elements - 4 (ME Rd. October (994)

A Francis turbine has what flow:


A. inwardflow reaction
B. outward flow impulse
C. outward flow reaction
D. inward flow impulse
Past Board Examination Elements (1994-1999) 333
332 Past Board Examination /:'/1'1111'/11\ (1994-1999)
A. velocity of now only
Elements - 5 (ME Rd. October 1994) B. pressure only
C. height above a chosen datum, density, internal energy,
The latent heat 01 vaporization in joules per kg is equal to: pressure and velocity ojflow
A 5.40 x 10
2 D. pressure, height above a chosen datum, velocity of f1ow,
3 density of fluid
B.4.13xl0
5
C. 22.6 X 10
5
D. 335 X 10
Elements - 10 (ME Rd. October 1994)

Elements - 6 (ME Rd. October 1994) A type of water turbine:


A. Parson
Form of energy associated with the kinetic energy of the random motion of B. Hero
large number of molecules: C. Pelton
A. internal energy D. Banki
B. kinetic energy
C. heat of fusion
o heat Elements - 11 (ME Rd. October 1994)

If the pressure of the confined gas is constant, the volume is directly


Elements - 7 (ME Rd. October 1994) proportional to the absolute temperature:
A. Boyle
In a P- T diagram of pure substance, the curve separating the solid phase B. Joule
from the liquid phase is: C. Charles
A. vaporization curve D. Kelvin
B. fusion curve
C. boiling point
D. sublimation point Elements - 12 (ME Rd. October 1994)

A theoretical body which when heated to incandescence would emit a


Elements - 8 (ME Rd. October 1994) continuous light-ray spectrum:
A black body radiation
The number of protons in the nucleus of an atom of the number of B. black body
electrons in the orbit of an atom: C. blue body
A. atomic. volume D. white body
B. atomic number
C. atomic weight
D. atomic mass
Elements - 13 (ME Rd. October 1994)

Elements - 9 (M E Bd. October 1994) Ignition of the air fuel mixture in the intake of the exhaust manifold:
A. backlash
The energy off1uid flowing at any section in a pipeline IS;I Iuncuon of B. backfire
334 Pas: Board Examination Elements (/994-1999) 335
Past Board Examination Elements (1994- / 999)
C. exhaust pressure
D. back pressure Elements - 18 (ME Rd. October 1994)

A simultaneous generation of-electricity and steam (or heat) in a single


Elemer..ts - 14 (ME Rd. October 1994) power plant:
A. gas turbine
Is the condition of pressure and temperature at which a liquid and its vapor B. steam turbine-gas turbine plant
are indistinguishable: C. waste heat recovery
A. relative humidity D. cogeneration
B. absolute humidity
C. critical point
D. dew point Elements - 19 (ME Rd. October 1994)

Is one whose temperature is below the saturation' temperature


Elements - 15 (ME Rd. October 1994) corresponding to its pressure:
A. compression
When a substance in gaseous state is below its critical temperature it is B. condensation
called: C. constant volume process
A. steam D. subcooled liquid
B. cloud
C. moisture
D. vapor Elements - 20 (ME Rd. October 1994)

Pump used to increase air pressure above normal, air is then used as a
Elements - 16 (ME Rd. October 1994) motive power:
. A. air cooled engine
B. air compressor
Which of the following a set of standard condition
A. 1 atm, 255k, 22.41 m 3/kg mole C. air condenser
B. 101.325,273 'k, 22.4 m'rkg mole D. air injection
C. 101.325, 273°k, 23.66 m3/kg mole
D. 1 arm, lOoC, 22.41 m 3/kg mole
Elements - 21 (ME Rd. October 1994)

If the temperature is held constant and the pressure is increased beyond the
Elements - 17 (ME Rd. October 1994) saturation pressure, we have a:
A. compressed liquid
B. subcooled liquid
Number of molecules in a mole of any substance is a constant called:
A. Rankine cycle C. saturated %por
B. Avogadro's number D. saturated liquid
C. Otto cycle
D. Thompson constant Elements - 22 (ME Rd. October 1994)

The locus of elevations:


336 Past Board Examination Elements (1994-/999) Past Board Examination Elements (1994-1999) 337

A. critical point
B. hydraulic gradient Elements - 27 (ME Bd. April 1995)
C. energy gradient
D. friction gradient What is the force which tends to draw a body toward the center about
which it is rotating?
A. centrifugal force
Elements - 23 (ME Ed. October 1994) B. centrifugal in motion
C. centrifugal advance
In sensible cooling process, the moisture content: D. centripetal force
A. does not change
B. decreases
C. indeterminate Elements - 28 (ME Bd. April 1995)
D. increases
A simultaneous generation of electricity and steam (or heat) in a single
power plant:
Elements - 24 (ME Bd. April 1995) A. steam turbine - gas turbine
B. cogeneration
What is the process that has no heat transfer? C. gas turbine plant
A. reversible D. waste heat boiler
B. isothermal
C. polytropic
D. adiabatic Elements - 29 (ME Bd. April 1995)

Percent excess air is the difference between air actually supplied and
Elements - 25 (ME Ed. April 1995) theoretically required divided by:
A. tlle theoretically air supplied
The internal combustion engines never work on_ _cycle: B. the deficiency air supplied
A. Rankine C. gas turbine plant
B. diesel D. waste heat boiler
C. dual combustion
D. Otto
Elements - 30 (ME Bd. April 1995)
Elements - 26 (ME Ed. April 1995)
What amount of air is required in a low bypass factor?
The dividing point between the high-pressure arid low pressure sides of the A. greater
refrigeration cycle occurs at the: B. lesser
A. expansion 'valve C. indeterminate
B. compressor D. does not change
C. condenser
D. cooling oil Elements - 31 (ME Bd, April 1995)

Work done per unit charge when charge is moved from one point to
another:
331< Past Board Examinution Elements (1994-11)99)
Past Board Examination Elements (1994-1999) 339
A. equipotential SlJrI~ICC
B. potential at II point
C. electrostatic unit Elements - 36 (ME Bd. April 1995)
D. potential difference
Vv hat is the clockwork-operated device which records continuously the
humidity of the atmosphere?
Elements - 32 (ME Bd. April 1995) A. hetograph
B. hygrometer
A pressure of lrnillibar is equivalent to: C. hydrodeik
A. 1000 dynes/em' D. hygrograph
B. 1000 em of Hg
C. 1000 psi

D 1000 kg/ern"
Elements - 37 (ME Bd. April 1995)

What is an apparatus used in the analysis of combustible gases? A.


Elements - 33 (ME Bd. April 1995) calorimeter differential
B. calorimeter gas
Heat transfer due to density differential: C. calorimetry
A. convection D. calorimeter
B. nuclear
C. conduction
D. radiation Elements - 38 (ME Bd. April 1995)

If the fluid travels parallel to the adjacent layers and the paths of individual
Elements - 34 (ME Bd. April 1995) particles do not cross, the flow is said to be:
A. turbulent
When a system deviates infinitesimally from equilibrium at every instant B. critical
of its state, it is undergoing: C. dynamic
A. isobaric process D. laminar
B. quasi-static process
C. isometric process
D. cyclic process Elements - 39 (ME Bd. April 1995)

What equation applies in the first law of thermodynamics {or an ideal gas
Elements - 35 (ME Bd. April 1995) in a reversible open steady-state system?
A. Q-W=Uz_U j
The ratio of the average load to the peak load over a designated period of B. Q+VdP=Hz_H 1
time is called: C. Q - VdP = Hz_HI
A. load factor D. Q-PdV=H2_H,
B. reactive factor
C. diversity factor Elements - 40 (ME Bd. April 1995)
D. plant use factor
Is one whose pressure is higher than the saturation pressure corresponding
to its temperature:
341
Past Board Examination Elements (1994-1999)
.' ·tIJ Past Board Examination Elements ( J fJfJ4-1999)
C. continuous flow
A. saturated Iiqu i d
B. compressed liquid D. turbulent flow
C. saturated vapor
D. compressed gas
Elements - 45 (ME Rd. April 1995)

Elements - 41 (ME Rd. April 1995) The hydraulic formula CA~2gh is used to find:
A. quantity ofdischarge through an orifice
The locus of elevation to which water will rise in the piezometer tube is B. velocity of flow in a closed conduit
termed: C. length of pipe in a closed network
A. energy gradient D. friction factor of a pipe
B. friction head
C. hydraulic gradient
D. critical path Elements - 46 (ME Rd. April 1995)

The sum of the energies of all molecules in a system, energies appear in


Elements - 42 (ME Rd. April 1995) several complex forms, is the:
A. kinetic energy
The total energy in a compressible or incompressible fluid flowing across B. potential energy
any section in a pipeline is a function of: C. internal energy
A. pressure and velocity D. thermal energy
B. pressure, density and velocity
C. pressure, density, velocity and viscosity
D. flow energy, kinetic energy, height above datum and Elements - 47 (ME Rd. April 1995)
internal energy
The temperature at which its vapor pressure is equal to the pressure exerted
on the liquid
Elements - 43 (ME Bd. April 1995) A. absolute humidity
B. calorimeter
The ratio of the density of a substance to the density of some standard C. boiling point
substance is called: D. thermal energy
A. relative density
B. specific gravity
C. specific density
D. relative gravity Elements - 48 (ME Rd. April 1995)

Type of turbine that has high pressure and low pressure is called:
Elements - 44 (ME Rd. April 1995) A. compound engine
B. gas turbine
At any instant, the number of particles passing every cross-section of the C. impulse turbine
stream is the same, the flow is said to be: D. compound turbine
A. steady flow
B. uniform flow
342 Past Board Examination Elements (1994-1999) 343
Past Board Examination Elements (1994-1999)
C. kilograms
Elements - 49 (ME Bd. April 1995) D. dyne

The design of an air supply duct of an air conditioning system:


A. adds moisture to the air Elements - 54 (ME Board Oct.l995)
B. lowers the temperature of the air
C. does no affect the distribution of air Heat exchanger used to provide heat transfer between the exhaust gaser
D. affects the distribution of air and the air prior to its entrance to the combustor:
A. evaporator
B. combustion chamber
Elements - 50 (ME Bd. April 1995) C. regenerator
D. heater
The changing of solid directly to vapor, without passing through the liquid
state is called:
A. evaporation
Elements - 55 (ME Board Oct.1995)
B. vaporization
C. sublimation
Heat normally flowing from a high temperature body to a low temperature
D. condensation
body wherein it is impossible to convert heat without other effects is called
the'
A. second law of thermodynamics
Elements - 51 (ME Bd. April 1995)
B. first law of thermodynamics
C. third law of thermodynamics
The volume of a fluid passing a cross-section stream in unit time is called: D. zeroth law of thermodynamics
A. steady flow
B. uniform flow
C. discharge
Elements - 56 (ME Board Oct.1995)
D. continuous flow
What are the immediate undesirable products from the petroleum-based
lubricating oil when subjected to high pressure and temperature?
Elements - 52 (ME Bd. April 1995)
A. gums, resins and acids
B. sulfur
Weight per unit volume is termed as
C. soots and ashes
A. specific gravity
D. carbon residue
B. density
C. weight density
Elements - 57 (ME Board Oct.l995)
D. specific gravity
The intake pipe to a hydraulic turbine from a dam is:
A. tailrace
Elements - 53 (ME Bd. April 1995)
B. spiral casing
C. surge tank
S.l. unit of force:
D. penstock
A. pounds
B. Newton
344 Past Board Examination Elements (1994-1999) Past Board Examination Elements (1994-1999) 345

Elements - 58 (ME Board Oct.1995) Elements - 63 (ME Board Oct.1995)

When 1 mol of carbon with 1 mol oxygen: Percent excess air is the difference between the air actually supplied and
A. 2 mols carbon dioxide the theoretically divided by: I
B. 1 mol carbon dioxide A. the deficiency air supplied I
C. 1 mol carbon and 1 mol CO 2 B the actually air supplied 1
D. 1 mol carbon monoxide C. none of these
D. the theoretically air supplied

Elements - 59 (ME Board Oct.1995)


Elements - 64 (ME Board Oct.1995)
A device for measuring the velocity of wind:
A. aneroid barometer Mechanism designed to lower the temperature of air passing through is:
B. anemometer A. air cooler
C. anemoscope B. air defense
D. anemograph C. air spillover
D. air cycle

Elements - 60 (ME Board Oct.1995)


Elements - 65 (ME Board Oct.1995)
Air standard efficiency ora diesel engine depends on:
A. speed The term "exposure" in radiological effects is used as a measure of a
B. compression ratio Gamma ray or an X-ray field in the surface of an exposed object. Since
C. fuel this radiation produces ionization of the air surrounding the object, the
D. torque exposure is obtained as:
A. x= no. of ions produced per mass ofair x coulombs per kg
Elements - 61 (ME BO?· _ O.l.l995) B. x= mass.of air x surface area of an exposed object
C. xc~ mass of air over surface area of an exposed object
Heavy water is: D. x= no.of ions produced per mass of air + coulombs per kg
A. B,O
B. H 20
C. W 20
D. D 20
Elements - 66 (ME Board Oct.l995)
Elements - 62 (ME Board Oct.1995)
The viscosity of most commercially available petroleum lubricating oil
The ratio of the sum of individual maximum demands of the system to the changes rapidly above:
overall maximum demand of the whole system: A. 120°F

A. demand factor B ISO°F

B. diversity factor C. I SO°F


C. power factor D. I :10°F
D. utilization factor
346 Past Board Examination Elements (1994-1999) 347
Past Board Examination Elements (1994-1999)

B, brew kelt le
Elements - 67 (ME Board April 1996) C. cooler
D. starting tubs
A type of water turbine where ajet of water is made to fall on the blades or
buckets and due to the impulse of water, the turbine starts to move:
A Pelton wheel Elements - 72 (ME Board April 1996)
B, Steam turbine
C. Francis turbine What keeps the moisture from passing through the system?
D. reaction turbine A. dehydrator
B. aerator
C. trap
Elements - 68 (ME Board April 1996) D. humidifier

What condition exists in an adiabatic throttling process?


A. enthalpy is variable Elements - 73 (ME Board April 1996)
B, enthalpy is constant
C. entropy is constant What are the main components in a combined cycle power plant?
D. specific volume is constant A. diesel engine and air compressor
B. gas engine and waste heat boiler
C. steam boiler and turbine
Elements -69 (ME Board April 1996) D. nuclear reactor and steam boiler

The specific gravity of a substance is the ratio of its density to the density
of: Elements - 74 (ME Board April 1996)
A. mercury
R. gas What do you call the changing of an atom of an element into an atcrn of a
C. air different element with a different atomic mass?
D. water A. atomization
B. atomic transmulation
C. atomic pile
Elements - 70 (ME Board April 1996) D. atomic energy

Which is used as a moderator in certain types of nuclear reactors?


A. vapor
B. heavy water Elements - 75 (ME Board April 1996)
C. hotwater
'''''-.,
D. cold water What do you call the weight of the column of air above the earth's surface'!
A. air pressure
B, aerostatic pressure
Elements - 71 (ME Board April 1996) C. wind pressure
D. atmospheric pressure
Yeast as raw material for beer making is added to the equipment called:
A. fermenters
348 Past Board Examinatian Elements (1994-11)1)1)) Past Board Examination Elements (1994-1999) 349

A. steam engine
Elements -76 (ME Board April 19(6) 8. steam turbine
C. gas turbine
C'-rnbined process of cooling and humidifying is also known as: D. diesel turbine
A. heating and humidifying
B. cooling tower
C. evaporative cooling process Elements - 81 (ME Board April 1996)
D. moisture removal process
What is the term as the ratio of the volume at the end of heat addition to
the volume at the start of heat addition?
Elements - 77 (ME Board April 1996) A. compression ratio
B. air-fuel ratio
What is the fore- required to accelerate a mass of I gram at a rate of I C. volumetric ratio
em/sec/sec? D. cut-off ratio
A dyne
B. poundal
C. slug Elements - 82 (ME Board April 1996)
D. kg force
What is the ideal cycle for gas turbine work?
A. Brayton cycle
Elements - 78 (ME Board April 1996) B. Stag combined cycle
C. Bottom cycle
What type of turbine has low head and high discharge? D. Ericson cycle
A Pelton Wheel
B. Francis turbine Elements - 83 (ME Board April 1996)
C. Jonval turbine
D. Kaplan turbine What do you call the passing of heat energy from molecuie to molecule
through a substance?
A conduction
Elements - 79 (ME Board April 1996) B. radiation
C. conservation
What is a Bull Head Tee? D. convection
A a pipe tee with head shaped like a bull
B. a welded built-up tee
C. a pipe tee with its run larger than its branch Elements - 84 (ME Board April 1996)
D. a pipe tee the branch of which is larger than tire run
What is the lowest temperature to which water could possibly be cooled in
a cooling tower?
Elements - 80 (ME Board April 1996) A. the effective temperature
B. the temperature of adiabatic saturation
What is the main power generating plant that produces the most electricity C. the wet bulb depression
per unit thermal energy in the fuel and has the greatest surplus of electricity D the dew point temperature of the air
for most cogeneration systems')
Past Board Examination Elements (1994-1999) 351
350 Past Board Examination Elements (1994-1999)
A isotropic
B. adiabatic
Elements - 85 (ME Board April 1996)
C. isometric
D. isobaric
The indicator used to determine the anti-knock characteristics of gasoline:
A. aniline point
B. Cetane No.
Elements - 90 (ME Board April 1996)
C. Octane No.
D. Diesel Index
A receiver in an air compression system is used to:
A. avoid cooling air before using
B. increase the air discharge pressure
Elements - 86 (ME Board April 1996) C. collect the water and grease suspended in the air
D. reduce the work needed during compression
Dew point is defines as:
A. the temperature to which the air must be cooled at
constant pressure to produce saturation
B. the point where the pressure and temperature lines meet
Elements - 91 (ME Board April 1996)
C. the temperature which dew is formed in the air
D. the pressure which dew is formed in the air
Foundations are preferably built of concrete in the proportion of what
measures of portland cement: sand: crushed stones?
A. 1:2:5
Elements - 87 (ME Board April 1996)
B. 2:4:6
C. 2:3:4
What type of lubricating oils are produced entirely from the crudes chosen
D. 1:2:4
through elimination of undesirable constituents by suitable refining
processes')
A. additives
B. inert
Elements - 92 (ME Board April 1996)
C. straight
D. premium
How does the values for work per unit mass flow of air in the compressor
and turbine influenced by the addition of a regenerator?
A. slightly increased
Elements - 88 (ME Board April 1996)
B. unchanged
C. greatly decreased
In a liquid-dominated geothermal plant, what process occurs when the
D. greatly increased
saturated steam passes through the turbine?
A. isobaric
B. polytropic
Elements - 93 (ME Board April 1996)
C. isometric
D. isentropic
The work done by a force of R newtons moving in a distance of L meters is
converted entirely into kinetic energy is expressed by the equation:
Elements - 89 (ME Board April 1996)
A. RL =' 2My2
Gas being heated at constant volume is undergoing the process of: B. RL =' RL N-m
Past Board Examination Elements (1994-1999) 353
352 Past Board Examination Elements (1994-1999)
C. RL = 1/2 MV1
Elements - 98 (ME Board April 1996)
D. RL= 1/2 MV
What is the suggested maximum permissible dose (MPD) of gamma ray
exposure for general individuals not working in a nuclear setting, by
choice, in rem/year?
Elements - 94 (ME Board April 1996)
A I
B. 5
In a steam generator with good combustion control, what occurs if the load
C. 1/2
is increased?
D. 3
A. air temperature leaving air heater decreases
B. air temperature entering heater increases
Elements - 99 (ME Board October 1996)
C. furnace pressure approximately constant There are two broad types in the classification of lubricating oils, they are:
D. economizer gas outlet temperature decreases
straight and
A. active
B. inactive
Elements - 95 (ME Board April 1996)
C. crocked
The color of lubricating oil indicates:
D. additives
A. does not indicated contamination
Elements - 100 (ME Board October 1996)
B. does not indicates qualities
C. qualities
Amount of air required in the low by-pass factor
D. viscosity
A. does not change
B. greater
C. lesser
D. indeterminate
Elements - 96 (ME Board April 1996)

For design stability, the center of gravity of the total combined engine,
driven equipment and foundation should be kept:
Elements - 101 (ME Board October 1996)
A. anywhere
What is the function of the compression joint of pipes or tubes?
B. above the foundation top
A. it is used to connect two pipes by welding
C. in line with the surface of the foundation
B. it is used to connect two pipes by pressing both ends
D. below thefoundation top
C. when tightened, compress tapered, sleeves so that they
form a tight joint on the periphery of the tubings they
Elements - 97 (ME Board April 1996) connect
D. it connects two pipes with the use of threaded couplings
Most commercially available petroleum lubricating oil deteriorates starting
from operating temperature of:
Elements - 102 (ME Board October 1996)
A. 150°F
B. 200°F
The components of a rotary pump;
C. 300°F
A. gears
D. 250°F
354 Past Board Examination Elements (1994-11)1)1)) Past Board Examination Elements (1994-1999) 355

R. piston
--------
C impeller Elements - 107 (ME Board October 1996)
D. screw
What is the most efficient thermodynamics cycle')
A. carnot
Elements - 103 (ME Board October 1996) B. diesel
C. rankine
An instrument commonly used in most Research and Engineering D. brayton
Laboratories because it is small and fast among the other thermometers:
A. mercury thermometer
B. liquid-in-gas thermometer Elements - 108 (ME Board October 1996)
C. gas thermometer
D. thermocouple How do you treat a statement that is considered a scientific law')
A. We postulate to be true
Elements - 104 (ME Board October 1996) B. Accept as a summary ofexperimental observation
C. We generally observed to be true
What is the term used in to express the ratio of specific humidities, actual D Believe to be derived from mathematical theorem
versus saturated')
A. relative hurrtidity
B. absolute humidity Elements - 109 (ME Board October 1996)
C. degree ofsaturation
D. percent saturation The transmission of heat from one place to another by fluid circulation
between spots of different temperature is called:
Elements - 105 (ME Board October 1996) A. convection
B. radiation
What is the process whereby a fissionable species utilized as a source of C. conservation
neutrons to produce more nuclei of its own kind than are used up') D. conduction
A. developing
B. culturing
C. multiplying Elements - 110 (ME Board October 1996)
D. breeding
What is referred by volume control?
A. an isolated system
Elements - 106 (ME Board October 1996) B. closed system
C. fixed region in space
A process of heat transfer due to motion of matter caused by a change in D. reversible process only
density:
.~]

A. absorption
B. radiation Elements - III (ME Board October 1996)
C. conduction
D. convection Which of the following types of flow meters is most accurate?
A. venturi tube
B. pitot tube
356 Past Board Examination Elements (1994-1999) 357
Past Board Examination Elements (1994-1999)
C. flow nozzle
D. foam type Elements - 116 (ME Board October 1996)

The specific measurement of moisture content in air:


Elements - 112 (ME Board October 1996) A. relative humidity
B. percent saturation
Pneumatic tools are powered by: C. degree of saturation
A. steam D. specific humidity
B. water
C. natural gas
D. air Elements - 117 (ME Board October 1996)

Elements - 113 (ME Board October 1996) Highest pressure drop in refrigeration cycle:
A. compressor
A graphical representation between discharge and time is known as: B. condenser
A. hectograph C. expansion valve
B. monograph D. evaporator
C. hydrograph
D. topograph
Elements -118(ME Board October 1996)

What is an expansion loop?


A. a double loop radius elbow to minimize friction losses
B. a pipe bent to a loop to change direction
Elements - 114 (ME Boa rd October 1996) C. a pipe expander fitting
D. a large radius bend in pipe line to absorb longitudinal
In a diesel engine, what elements in the fuel that make the work of the expansion in the pipe line due to heat
lubricant more difficult?
A. water and ash content
B high octane number
Elements - 119 (ME Boa rd October 1996)
C. high cetane number
D. sulphur and asphaltene content What is the color code of steam pipe lines?
A. silver gray
B. green
Elements - 115 (ME Boa rd October 1996) C. red
D. yellow
What is the function of a radiation pyrometer?
A. boiler water weight
B. boiler pressure
Elements - 120 (ME Board October 1996)
C. furnace temperature
D. boiler drum pressure What is absorbed by.sulphites in boiler water treatment?
A. oxygen
B. carbon dioxide
Past Board Examination Elements (1994-/999) 359
358 Past Board Fvaminntion Elements (1994-11)1)1))

C. impurities I "III,,£! ill mud drums C. sulfur dioxide


D. carbon diox idc and oxygen D methyl chloride

Elements - 121 (ME Board October 1996) Elements - 125 (ME Board October 1996)

What is meant by choking in pipe flow? Water turbine converts:


A. the specified mass flow rate cannot OCcur A. mechanical energy into electrical energy
B. shock waves always occur B. hydraulic energy into electrical energy
C. a valve is closed in a line C. mechanical energy into hydraulic energy
D. a restriction in flow area occurs D. hydraulic energy into mechanical energy

Elements - 122 (ME Board October 1996) Elements - 126 (ME Board October 1996)

What is a check valve? How do you differentiate surge from water hammer?
A. a valve design to allow a fluid to pass through in one A. time for a pressure to traverse the pipe
direction only B. the pressure of reservoir at the end of the pipe
B. a valve designed to release the excess pressure C. rate of deceleration of flow
C. a valve which allows flow of fluid in either direction D. relative compressibility of liquid to expansion
D. a valve used for checking the pressure of fluid
Elements - 127 (ME Board October 1996)

Throttling of the refrigerant throuzh the expansion valve III a vapor


refrigeration cycle is:
A. reversible adiabatic process
Elements - 123 (ME Board October 1996) 8. constant entropy process
C. irreversible adiabatic process
What is the purpose of providing the lubricating oil pre-heater in an D. isometric process
emergency stand-by diesel genset?
A. to keep the lube oil viscosity down under the cold
condition and enhance the starting of the cold engine Elements - 128 (ME Board October 1996)
B. to avoid moisture condensation in the engine
C. to avoid corrosion to engine parts Assuming real process, the net entropy change in the universe is:
D. to see to it that the lubrication system is functioning A. must be calculated
properly B. equal to zero
C. negative
D. positive
Elements - 124 (ME Board October 1996)

Which of the following refrigerants is most highly toxic? Elements - 129 (ME Board October 1996)
A. ammonia
B. freon 12 What characterizes a reaction turbine?
Past Board Examination Elements (1994-1999) 361
360 Past Board Examination Elements (1994-/999)
,.. steam losses velocity as it leaves the diaphragm D. nitrogen dioxide
B. steam strikes the blades at right angles
C. steam will react with a force in the diaphragm
D. steam is deflected Elements - 134 (ME Board October 1996)

What takes place in a uniflow scavenging?


A. turbo blower in exhaust header to create vacuum in
Element's - 130 (ME Board October 1996)
cylinders
The work done in an adiabatic process in a system: B. air reversing direction in cylinders
A. is equal to the change in total energy in a closed system C. uses two blowers to purge cylinders
B. is equal to the net heat transfer plus the entropy change D. air travelling in one direction
or
C. is equal to the change in total energy closed system
plus entropy change
D. is equal to the change in total energy of closed system Elements - 135 (ME Board October 1996)
plus net heat transfer
The diagonal lines in the Psychrqmetric Chart represent:
A. Effective temperature
Elements - 131 (ME Board October 1996) B. dry-bulb temperature
C. Wet-bulb temperature
Hr do you increase the output of a centrifugal pump? D. dew-point temperature
A. speed up rotation
B. install circulation line
C. increase the suction pipe area Elements - 136 (ME Ro~rrl Ol'tohpr 1996)
D. increase the discharge pipe area
An ideal gas is compressed isothermally. The enthalpv change is:
A. sometimes negative
Elements - 132 (ME Board October 1996) B. zero
C. sometimes positive
Based on the first law of thermodynamics, which of the following IS
D. indeterminate
wrong?
A. the heat transfer equals the work plus the energy change
B. the heat transfer cannot exceed the work done Elements - 137 (ME Board October 1996)
C. the net heat transfer equals the net work of the cycle
A system with paddle wheel work is irreversible, therefore, the change m
D. the net heat transfer equals the energy change if no work
is done its entropy:
A. is zero
B. greater than zero
Elements - 133 (ME Board October 1996) C. maybe negative
D. maybe positive, negative or zero
The main cause of air pollution as a result of burning fuel oil is:
A. sulfur dioxide Elements - 138 (ME Board October 1996)
B. silicon dioxide
C. hydrogen dioxide What is meant by brake horsepower?
362 Past Board Examination Elements (1994-1999)
Past Board Examination Elements (1994-1999) 363
A. power developed in the engine with cylinder
B. final horsepower delivered to the equipment
C. actual horsepower delivered to the engine drive shaft Elements - 143 (ME Board April 1997)
D. work required to raise a weight of 33,000 pounds at a
height of one foot in one minute time The products of complete combustion of gaseous hydro carbons.
A. Carbon dioxide and water
B. Carbon monoxide
Elements - 139 (ME Board October 1996) C. Carbon monoxide, water and ammonia
D. Water, carbon monoxide and carbon dioxide
Enthalpy of an ideal gas is a function only of:
A. entropy
B. internal energy Elements - 144 (ME Board April 1997) 1
~\

.f'~.
C. temperature
D. pressure The part that directs the flow of the refrigerant through the compressor:
...
A. wrist pin
B. valve
Elements - 140 (ME Board October 1996) C. piston ii
~.
D. connecting rod ·'··.·1.··

When droplets of water are carried by steam in the boiler: ~il


~ij
A. priming
B. foaming
C. carryover
D. embrittlement
Elements - 145 (ME Board April 1997)

An odorless refrigerant, its boiling point varies over a wide range of


I
'.1
.~
temperatures:
i~
A. Freon 22
Elements - 141 (ME Board October 1996) B. Freon 12
C. Freon refrigerant
11
Mechanical energy of pressure transformed into energy of heat: D.' Ammonia .' :
.1.:
!..
A. kinetic energy
B. enthalpy
Elements - 146 (ME Board April 1997)
i~
C. heat exchanger
D. heat of compression
The boiling point of Freon 22 is:
A. -41.04°F
Elements - 142 (ME Board April 1997) B. 40.60°F
C. -38AO°F
The ratio between the actual power and the apparent power in any circuit is D. -31AO°F
know as the of that circuit.
A. Measured Power
B. Capacity Elements - 147 (ME Board April 1997)
C. Power Factor
D. KVAR Medium pressure when applied to valves and fittings, implies they are
suitable for a working pressure of from:
A. 862 to 1200 kPa
364 Past Board Examination ElI.'IIIl'It(.\ (1994-1999) Past Board Examination Elements (1994-1999) 365

B. 758 to I ooo kPa


C. 500 to 1000 kPa Elements - 152 (ME Board April 1997)
D. 685 to 1050 kPa
A device whose function is to pass an information in an unchanged form or
in some modified form:
Elements - ] 48 (ME Board April 1997) A. relay
B. sensor
A general term for a device that receives information in the form of one of C. transmitter
more physical quantities, modifies the information and/or its form. if D. transducer
required, and produces a resultant output signal:
A. Converter
B. Transducer Elements - ] 53 (ME Board April] 997)
C. Sensor
D. Scanner A device whose primary function is to meter the flow of refrigerant to the
evaporator:
Elements - 149 (ME Board April 1997) A. sniffer valve
B. equalizers
In the process of pair formation, a pair cannot be formed unless the C. thermostatic expansion valve
quantum has an energy greater than: D. crossover valves
A 2m sub o C
B.ll2mY' Elements - 154 (ME Board April 1997)
C. 05MeY
D. hv/C The volume remaining when the piston reaches the end of the compression
stroke:
Elements - ]50 (ME Board April ]997) A. air cell
B. combustion chamber
The temperature of hot metals can be estimated by their color. For steel or C. turbulence chamber
iron, the color scale at 2200°F is roughly: D. pre-combustion chamber
A white
B orange
C. dark red Elements - 155 (ME Board April ]997)
J yellow
Specific heat capacity is anSI derived unit described as:
A. J/kg
Elements - 15] (ME Board April ]997) B. W/moK
3
C. J/m
Mathematically, a thermodynamic property is which of the following? D. JlkgOK
A. a pflint function
B. discontinuous
C. a path function Elements - ] 56 (ME Boa rd April] 997)
D. exact differential
The fundamental difference between pipe and tubing is:
A. The dimensional standard to which each is manufactured
jol
Past Board Examination Elements (1994-1 YYY)
366 Past Board Examination Elements (J 994-1999)

B. Compression joints Elenlents - 161' (ME Board April 1997)


C. The smoothness of the surface
D. Bell and spigot joint The temperature of the fluid flowing under pressure through a pipe is
usually measured by:
A. glass thermometer
Elements - 157 (ME Board April 1997) B. electric-resistance thermometer
C. thermocouple
One of the most popular types of compressor utilized for supercharging D. all of the above
engine is the:
A. Roots type hlower
B. Pulse turbocharger Elements - 162 (ME Board April 1997)
C. Constant pressure turbocharger
D. Turbo compressor An increase in the deposition of slag and ash on the surface for heating of
oil-fired boilers in both marine and stationary service has affected boiler
efficiency. The following are the causes except:
Elements - 158 (ME Board April 1997) A. Low temperature corrosion of the cold section of air
heaters and duct works
Crankshaft of reciprocating type compressor is basically made of: B. Slagging of high temperature superheater surfaces
A. semi-steel C. High temperature corrosion steel
B. aluminum alloy D. Increase of heat transfer in the hoiler
C. cast iron
D. steel forging

Elements - 163 (ME Board April 1997)


Elements - 159 (ME Board April 1997)
The type of filter where the filtering element is replaceable:
A chemical method of feedwater treatment which uses calcium hydroxide A. paper edge filter
and sodium carbonate as reagents: B. metal edge filter
A. thermal treatment C. pressure fi Iter
B. lime soda treatment D. filter with element
C. demineralization process
D. ion exchange treatment
Elements - 164 (ME Board April 1997)

Elements - 160 (ME Board April 1997) Which does not belong to the group?
A. air injection system
Engines using heavy fuels requires heating of the fuel so that the viscosity B. mechanical injection system
at the injector is: C. time injection system
A. around 200 SSU D. gas admission system
B. 100 SSU or less
C. 200 SSU±50
D. J50 SSU or slightly higher
· 368 Past Board Examination Elements (1994-1999) Past Board Examination Elements (1 YY4-1 YYY) .)o~

A. direct expansion system


Elements - 165 (ME Board April 1997) B. chilled water system
C. flooded system
Coaling water system consists of equipment to dissipate heat absorbed by D. multiple system
the, engine jacket water, lube oil and the heat to be removed from air
intercooler is measurable to keep the engine outlet water temperature
constant and the differential of the cooling water at a minimum preferably Elements - 170 (ME Board October 1997)
not to exceed:
A. IOt030°F When four events take place in one revolution of a crankshaft of an engine,
B. 10 to 50°F the engine is called:
C. 10 to zo-r A. rotary engine
B. steam engine
D. IOt040°F
C. two-stroke engine
D. four-stroke engine
Elements - 166 (ME Board October 1997)

In a water tube boiler, where is heat and gases of combustion passed? Elements - 171 (ME Board October 1997)
A. through the combustion chamber only
What occurs in a reversible polytropic process?
B. through the tubes
A. enthalpy remains constant
C. away from the tubes
B. internal energy does not change
D. around the tubes
C. some heat transfer occurs
D. Entropy remains constant
Elements - 167 (ME Board October 1997)

A pneumatic tool is generally powered by: Elements - 172 (ME Board October 1997)
A. water
B. electricity In a deepwell installation or operation, the difference between static water
C. steam level and operating water level is called:
D. air A. suction lift
B. drawdown
C. priming level
Elements - 168 (ME Board October 1997)
D. clogging
The instrument used to measure atmospheric pressure is:
A. rotameter
B. manometer Elements - 173 (ME Board October 1997)
C. venturi
What characteristics an impulse turbine?
D. barometer
A. steam striking blades on angle
B. no steam reaction to velocity
Elements - 169 (ME Board October 1997)
C. steam striking blades at zero angle
D. steam reversing direction
A refrigeration system in which only part of the refrigerant passes over the
heat transfer surface is evaporated and the balance is separated from the
vapor and recirculated:
-' 7() Past Board Evamination Elements (/1)1)4-/999)
Past Board Examination Elements (1994-1999) 371

Elements - 174 (ME Hoard October 1997) C. convection


D. conduction
Air receives in a compressed air plant must be:
A. without pressure gauges
B. vented to the atmosphere Elements -179 (ME Board October 1997)
C. rectangular in shape
D. installed with safety valve and drain valve Where is lithium bromide used in a refrigeration system')
A. condensate return lines
B absorbers
Elements - 175 (ME Board October 1997) C. centri fugal compressors
D. ion exchangers
A gas which will not be found in the flue gases produced by the complete
combustion offuel oil is: Elements - 180 (ME Board October 1997)
A. carbon dioxide
B. hydrogen Amount of heat liberated by the complete combustion of a unit weight or
C. oxygen volume of fuel is:
D. nitrogen A. heating value
B latent heat
C. sensible heat
Elements - 176 (ME Board October 1997) D work or compression

Scale in boiler can: Elements - 181 (ME Board October 1997)


A. create low steam quality
B. cause foaming A temperature above which a given gas cannot be liquified:
C. overheat blow-off line A. cryogenic temperature
D. inhibit circulation anti heat transfer B. vaporization temperature
C. absolute temperature
D. critical temperature
Elements - 177 (ME Board October 1997)

The effectiveness of a body as a thermal radiator at a given temperature. Elements - 182 (ME Board October 1997)
A. absorptivity
B. emissivity The ratio of the sum of individual maximum demands of the svstern to the
C. conductivity overall maximum demand of the whole system is:
D. reflectivity A. diversity factor
B. utilization factor
C. power factor
Elements - 178 (ME Board October 1997) D. demand factor

In a cooling tower, the water is cooled mainly by:


A. condensation Elements - 183 (ME Board October 1997)
B. evaporation
When fuel oil has a high viscosity, we mean that the fuel oil will:
372
Past Board Framination Elements (11)94-/999) Past Board Examination Elements (1994-1999) 373
A evaporate easily
B. have a low specific gravity Elements - 188 (ME Board October 1997)
C. bum without smoke
D. flow slowly through pipes In a hydro-electric plant using a Francis turbine with medium head, the
speed can be regulated by using the:
A. deflector gate
Elements - 184 (ME Board October 1997) B. nozzle
C. wicket gate
Percentage of excess air is the difference between the air actually supplied D. weir
and the theoretically required divided by:
A. actual air supplied
Elements - 189 (ME Board October 1997)
B. theoretical air supplied
C. theoretical less actual supplied To protect adequately the engine bearings, what type and better
D. deficiency air supplied arrangement of lubricating oil filter is most practical?
A. full-flow type filter installed between the lubricating oil
pump and the bearings
Elements - 185 (ME Board October 1997) B. duplex filter installed before the lubricating pump
C. by pass filter with cleanable and replaceable elements
In a refrigeration system, the heat absorbed in the evaporator per kg mass D. splash lubricating system in the crankcase
of refrigerant passing through is:
A equals the increase in enthalpy
B. does not depend on the refrigerant used Elements - 190 (ME Board October 1997)
C. is decreased if pre-cooler is used
D. equals the increase in volume In radiation, the heat transfer depends on:
A. temperature
Elements - 186 (ME Board October 1997) B. heat rays
C. heat flow from cold to hot
Air that controls the rate of combustion in the combustion chamber is D. humidity
known as:
A. secondary air
B. excess air
Elements - 191 (ME Board October 1997)
C. control air
D. primary air
The main purpose of a subcooler in a refrigerating system especially a 2-
stage system is to;
A. increase the heat rejection per ton and avoid system
Elements - 187 (ME Board October 1997) shutdown
B. improve the flow of evaporator gas pel ton and increase
An aftercooler on a reciprocating air compressor is used primarily to: the temperature
A. cool the lubricating oil C. reduce the total power requirements and return oil to the
B. condense the moisture in the compressed air compressor
C. improve the compressor efficiency D. reduce the total power requirements and heat rejection to
D. increase the compressor capacity the 2nd stage
Past Board Examinauon Elements (1994-1999) 375
j /4 Past Board Examination Elements (1994- 1999)
B. heat radiation
C condensate water level
Elements - 192 (ME Board October 1997) D. air volume

The performance of a reciprocating compressor can be expressed by'


A adiabatic work divided by adiabatic input Elements - 197 (ME Board April 1998)
8. adiabatic work divided by indicated work
C. isothermal work divided by indicated work Pres. F. V. Ramos approved on February 12, 1998 a Republic Act, which is
D. isothermal work divided by adiabatic work an act to regulate the practice of Mechanical Engineering in the
Philippines, otherwise known as the M.E. Law. What is this act?
Elements - 193 (ME Board October 1997) A. RA No. 9845
B. RA No. 8495
A reciprocating pump is considered positive displacement pump because: C. RA No. 8594
A. displacement ofthe liquid is affected by the displacement D RA No. 8945
of the piston -~_._----
----_._--~---

B positive pressure is given to the liquid Elements - 198 (ME Board April 1998)
C. liquid is discharge with positive pressure
D. liquid is lifted due to the vacuum created inside the The relationship of water vapor in the air at the dew point temperature to
cylinder the amount that would be in the air if the air were saturated at the dry bulb
temperature is:
Elements - 194 (ME Board October 1997) A. partial pressure actual at dew point
B. percentage humidity
A change in the efficiency of combustion in a boiler can usually be C reLative humidity
determine by comparing the previously recorded readings with the current D. partial pressure of water
readings of the:
A. stack temperature and CO Elements - 199 (ME Board April 1998)
8. over-the-fire-draft and CO
C. Ringelman chart and CO 2 What is the color code of air pipelines?
D. stack temperature and COl A. light-bLue
8. red
C brown
Elements - 195 (ME Boa rd October 1997) D. violet

A boiler steam gauge should have a range of at least:


A one-half the working steam pressure Elements - 200 (ME Board April 1998)
B. I 1/2 times the maximum allowable working pressure
C. the working steam pressure The CO (carbon dioxide) percentage in the flue gas of an efficiently tired
2
D twice the maximum allowable working pressure. boiler should be approximately:
A.I%
B.12%
Elements - 196 (ME Board April 1998) C 18%
D 20%
A manometer IS an instrument that is used to measure:
A air pressure
37(, Past BOl/rd Evamination Elements (1')')4-1 '1'1'1)
Past Board Examination 1:'1£'111£'111\ (I 994-1 999j 377
A. cleaning the cup on a rotary cup burner
Elements - 201 (M E Hoard April 1998) B cleaning a completely clogged oil strainer
C. replacing a leaking valve
An unloader is used on air compressor to: D. replacing a blown fuse
A. to relief air pressure
B. start easier
C. stop easier Elements - 206 (ME Board April 1
D. run faster
An air current in a confined space such as that in a cooling tower or
chimney is called:
Elements - 202 (ME Board April 1998) A. variable flow
B. velocity profile
How many pounds of air are theoretically needed to burn one pound of C. velocity gradient
diesel fuel oil? D. draft
A. 28
B 14
C. 18 Elements - 207 (ME Board April 1998)
D.22
What kind of a heat exchanger where water is heated to a point that
Elements - 203 (ME Board April 1998) dissolved gases are liberated?
A. evaporator
Which of the following is a great advantage of a fire-tube boiler? B. condenser
A. steam pressure is not steady C. intercooler
B. contains a large volume of water and requires long D. deaerator
interval oftime to raise steam and not so flexible as to
changes in steam demand
C. can not use impure water Elements - 208 (ME Board April 1998)
D. radiation losses are higher because fire is on the inside of
the boiler and is surrounded by water What is the function of steam separator?
A. trapping the steam and letting water through
Elements - 204 (ME Board April 1998) B. throttling
C. changing direction of the steam flow
What should be the temperature of both the water and steam whenever they D. steam metering
are present together?
A. saturation temperature for the existing pressure
B. boiling point of water at 101.325 kPa Elements - 209 (ME Board April
C. superheated temperature
D. one hundred degree centigrade Which of the following is not a main part ofa typical coal burner?
A. air registers
Elements - 205 (ME Board April 1998) B. a nozzle
C. an atomizer
One of the following tasks which is an example of preventive maintenance D. an ignitor
IS:
j/X Past Board Examination Elements (1994-1999) Past Board Examination Elements (1994-1999) 379

Elements - 210 (ME Board April 1998) Elements - 214 (ME Board April 1998)

Which of the following types of air dryers works by absorbing moisture on In the processing section, there is an instrument frequently used to measure
a solid dessicant or drying material such as activated alumina, silicon gel, the flow rate of fluids. What is the instrument consisting of a vertical
or molecular sieve? .passage with variable cross-sectional area, a float and calibrated scale"
A. Regenerative dryer A. rotameter
B. Deliquescent dryer B. pitot-tube
C. Spray dryer C. rota-aire
D. Refrigerated dryer D. manometer

Elements - 211 (ME Board April 1998) Elements - 215 (ME Board April 1998)

A heat-transfer device that reduces a thermodynamic fluid from its vapor How do you describe a non-flow process where in the volume remains
phase to its liquid phase such as in vapor-compression refrigeration plant constant?
or in a condensing steam power plant. A. isometric
A. flash vessel B. isentropic
B. cooling tower C. isobaric
C. condenser D. iscnthalpic
D. steam separator

Elements - 216 (ME Board April 1998)


Elements - 212 (ME Board April 1998)
A branched system of pipes to carry waste emissions away from the piston
A goose neck is installed in the line connecting a steam gauge to a boiler chambers of an internal combustion engine is called:
to: A. exhaust nozzle
A. maintain constant steam flow B. exhaust deflection pipe
B. protect the gauge element C. exhaust pipes
C. prevent steam knocking D. exhaust manifold
D. maintain steam pressure

Elements - 217 (ME Board April 1998)


Elements - 213 (ME Board April 1998)
Measure of ability of a boiler to transfer the heat given by the furnace TO
The law that states entropy of all perfect crystalline solids IS zero at the water and steam is:
absolute zero temperature. A. grate efficiency
A. Newton Law B. stocker efficiency
B. Third Law of Thermodynamics C. furnace efficiency
C. First Law of Thermodynamics D. boiler efficiency
D. Second Law of Thermodynamics
380 Past Board Examination Elements (/994-1999) Past Board Examination Elements (1994-1999) 381

A. reduce the speed 01 the 1T10tor when the maximum


Elements - 218 (ME Board April 1998) pressure is reached.
B. drain the condensate from the cylinder
A major cause of air pollution resulting from the burning of fuel oils is: C release the pressure in the cylinders in order to
A. nitrous reduce the starting load
B hydrogen D. prevent excess pressure in the receiver
C sulfur dioxide
D silicon
Elements - 223 (ME Board April 1998)

Elements - 219 (ME Board April 1998) Fluids thatare pumped in processing work are frequently more VISCOUS
than water Which of the following statements is corrects'!
An engine indicator is generally used to measure: A. Reynolds number varies directly as the viscosity
A. steam temperature B. Efficiency of a pump increases as the viscosity increases
B heat losses C Increased fluid friction between the pump parts and
C steam cylinder pressure the passing fluid increases useful work
D. errors in gauge reading D. Working head increases as the viscosity increases.

Elements - 220 (ME Board April 1998) Elements - 224 (ME Board April 1998)

The power required to deliver a given quantity of fluid against a given head The size of a steam reciprocating pump is generally designated by a three
with no losses in the pump is called: digit number size as 646. What would be the first number designate?
A. wheel power A. stroke of the pump in inches
B. brake power B inside diameter of the steam cylinder measured in
C hydraulic power inches
D. indicated power C percent clearance
D number of cylinders

Elements - 221 (ME Board April 1998)


Elements - 225 (ME Board April 1998)
A liquid whose temperature is lower than the saturation temperature
corresponding to the existing pressure. Peak load for a period of time divided by installed capacity is:
A. subcooled liquid A. capacity factor
B. saturated liquid B. demand factor
C pure liquid C utilization factor
D. compressed liquid D load factor

Elements - 226 (ME Board April 1999)


Elements - 222 (ME Board April 1998)
At what temperature where in an oil at any grade becomes cloudy and it
The function of an unJoader on an electric motor-driven compressor is to: freezes, thus in application is limited.
A. coLdpoint
382 Past Board Examination Elements (1994- J 999) 383
Past Board Examination Elements (1994-1999)
B. pour point A. critical temperature
C. flash point B. dry bulb temperature
D. freezing point C. dew point temperature
D. wet bulb temperature
Elements - 227 (ME Board April 1999)
-
A turbine pipe determined its "nominal" size refers to: Elements - 232 (ME Board April 1999)
A. inside diameter
B. outside diameter For reciprocating compressor slip at negative displacement:
C. approximate size
A. cd>}
D. pipe wall thickness
B. cd-cl
C. cd = 0
D. cd = [
Elements - 228 (ME Board April 1999)

If the exhaust lowered or the boiler is raised the moisture content of steam:
Elements - 233 (ME Board April 1999)
A. vaponzes
B. decreased
When the number of reheat stages in a reheat cycle is increased, the
C. liquifies
average temperature:
D. increase
A. increases
B. constant
C. decreases
Elements - 229 (ME Board April 1999)
D. zero

The relative humidity become [00% and where the water vapor starts to
condensate.
Elements - 234 (ME Board April 1999)
A. critical point
B. dew point
When the boiler pressure increases or when the exhaust pressure decreases,
C. saturated point
the amount of moisture:
D. cold point
A. increases
B. constant
C. decreases
Elements - 230 (ME Board April 1999)
D. zero
For reciprocating compression slip at positive displacement.
A. cd = ]
Elements - 235 (ME Board April 1999)
B. cd<1
C. cd» l The purpose of the nozzle in a combustor of gas turbine plant is to:
D. cd = 0
A. increase the velocity
B. increase the pressure
Elements - 231 (ME Board April 1999) C. increase the moisture
A temperature measurement in an ordinary thermometer wh ich has
D. increase the power
constant specific humidity.
384 Past Board Examination Elements (I <Jl)4-11)91)) Past Board Examination Elements (1994-1999) 385

B Throttle valve
-------

Elements - 236 (ME Board April 1999) C Thermostatic expansion valve


D. Control valve
During the sensible heating, the absolute humidity remains constant but the
relative humidity.
A. increases
B. remains constant Elements - 241 (ME Board Oct. 1999)
C decreases
D. zero A valve that sense the loss of ignition in a diesel engine.
A. combustion control
B. fire control
C flame detector
Elements - 237 (ME Board April 1999) D. fire extinguisher

For negative slip, the coefficient of discharge:


A. increases Elements - 242 (ME Board Oct. 1999)
B. zero
C decreases If evaporator oil clogs in the evaporator, it cause:
D. constant A. increase in heat transfer
B. vaporized oil
Elements - 238 (ME Board April 1999) C Low suction pressure
D. High pressure
For positive slip, the coefficient of discharge:
A. increases
B. zero Elements - 243 (ME Board Oct. 1999)
C decreases
D. constant In cooling, if humidity ratio remains unchanged,
A. sensible cooling
B sensible heating
Elements - 239 (ME Board Oct. 1999) C Pre-cooling
D. Latent cooling
Is an abrupt reduction in flow velocity due to sudden increase of water
depth in the down stream direction
A. Hydraulic energy Elements - 244 (ME Board Oct. 1999)
B Hydraulic jump
C Weirs Property of lubricating oil that responds at very low temperature, the oil is
D. Hydraulic gradient known as:
A. viscous
B. pour point
Elements - 240 (ME Board Oct. 1999) C cloud point
D. solid point
Is a valve that regulates the flow ofrcfrigerants.
A. Direct expansion valve
38!J Past Board Examination Elements (/994-/999) Past Board Examination Elements (1994-1999) 387

Elements - 245 (ME Board Oct. 1999) Elements- 249 (ME Board Oct. 1999)

A turbine in which all available energy of the flow is converted by a nozzle In a cooling tower the temperature of water is lower than the wet bulb
into kinetic energy before in contact to moving blades. temperature of entering air and it is found that air cannot cool. What
A. Kaplan turbine temperature of water in cooling water.
B. Francis turbine A. above
C. Propeller turbine B. lower
D. Impulse turbine C. constant
D. none of these

Elements - 246 (ME Board Oct. 1999) Elements - 250 (ME Board Oct. 1999)

A class of system of a refrigeration in which the wet bulb temperature is What is the critical temperature where water & vapor are in equilibrium to
not more than the temperature of air. the atmospheric pressure.
A. evaporator preload system A. ice point
B. direct system B. critical point
C. indirect system C. steam point
D. chilling system D. freezing point

Elements - 251 (ME Board Oct. ] 999)

Elements - 247 (ME Board Oct. 1999) In a pipe specification, schedule is used, when the pipe specified as "
schedule 80", the pipe corresponds to the
A type of throttle of air fuel ratio in constant charging. A. "extra standard" weight
A. quantitative B. extra strong
B. qualitative C. internal pressure
C. hit and miss D. "old standard" weight
D. none of these

Elements - 252 (ME Board Oct. 1999)


Elements - 248 (ME Board Oct. 1999)
The entropy of a substance at a temperature of absolute zer, is.
A company manager want to used comfort air, what is the most efficient A. unity
setting of the conditioning unit: B. infinity
A most attainable value C. zero
B. moderate value D. 100
C maximum value
D. minimum value
Elements - 253 (ME Board Oct. 1999)

The ratio of cross-sectional area of flow to the wetted perimeter.


A. hydraulic lead
388 Past Board Examination Elements (/994-1999) Past Board Examination Elements (1994-1999) 389
r,.
'.;,1·.·

,Ii;
B hydraulic radius
C hydraulic energy Elements - 259 (ME Board Oct. 1999)
D. hydraulic gradient
The ice making capacity is always:
A. directly proportional to the refrigerating effect
Elements - 255 (ME Board Oct. 1999) B. less than the refrigerating effect
C. greater than the refrigerating effect
The effect of superheating the refrigerant is: D. equal to the refrigerating effect
A. increased in COP
8. reduced in (lOP
C. high COP Elements - 260 (ME Board Oct. 1999)
D. constant COP
When the air is saturated, the wet bulb depression is:
A. zero
Elements - 256 (ME Board Oct. 1999) B. unity
C. constant
The absolute zero in Celcius scale: D. 100%
A. 100
B. -273
c.O Elements - 261 (ME Board Oct. 111l)Q)
D. 273
The process in which the heat energy is transferred to a thermal storage
device. It is known as:
Elements - 257 (ME Board Oct. 1999) A. adiabatic
B. intercooling
The water in the product of combustion is in vapor state. C. regenerator
A. Ultimate analysis D. isentropic
B. analysis
C. Lower heating value
D. Higher heating value Elements -262 (ME Board Oct. 1999)

The liquid pressure in the surface per area in the surface at the bottom is:
A. magnitude pressure
Elements - 258 (ME Board Oct. 1999) B. cohession pressure
C. intensity pressure
The thermal efficiency of gas-vapor cycle as compared to steam turbine or D. adhession pressure
gas turbine.
A. greater than
B. lower than
C. less than
D. equal to
390 Refrigeration Refrigeration 391
;Ii.

x = 0.1028 '.'
REFRIGERATION x = 10.28%

Refrigeration - 3
Refrigeration - 1
The refrigerating effect of 100 cons refrigeration is 117 KJ/kg.
The enthalpy at the entrance of the condenser is 1660 KJ/kg and exit is Determine the mass flow of refrigerant.
315 KJ/kg. The compressor has an enthalpy of 1450 KJ/kg at entrance. A. I kg/sec C. 3 kg/sec
Determine the COP. B. 2 kg/sec D. 4 kg/sec
A. 4.2 C. 5A
R 4.0 D. 6.0. SOLUTION:

SOLUTION: TR = ~(h I - h4 )
3.516
Re frjgerating Effect m (117 )
COP -s - -
Compressor Power 100
3.516
hi - h 4
COP
h, - hI ms 3 kg/sec
1450-315
COP
1660-1450
COP = 5AO Refrigeration - 4

The density of R-12 refrigerant at compressor suction is IlJ.81 kg/m'.


For mass flow rate of 2 kg/sec, determine the volume now at suction.
J '
A.O.IOm/sec C. 0.30 rrr'zsec
Refrigeration - 2 B. 0.25 mJ/sec D. OAO rrr'zsec

An evaporator has a temperature of 3°C with entrance enthalpy of SOLUTION:


352.75 KJ/kg. At 3°C, h r = 319.56 KJ/kg and h g = 642.45 KJ/kg. Find
the quality after expansion.
A. 16.27% C. 15.67% VI = mVI
B. 21A8% D. 10.27% VI = m (1/w))
VI = 2(1/19.81)
VI = 0.10 mJ/sec
SOLUTION:

h, = h, = h r + x(h g - hr) Refrigeration -

352.75 = 319.56 +x(642A5-319.56) A 100 tons refrigeration system has a COP of 5. Determine the
compressor horsepower.
392 Refrigeration Refrigeration 393

A. 94.26 lip C 8676 lip


B. 8676 Hp D. 65.65 Hp Refrigeration - 7

SOLUTION: A 90 tons refrigeration system has a compressor input of 0.97 KW per


ton refrigeration. If compressor efficiency is 75%. determine the heat
rejected from the condenser.
Re frigerating Effect A. 350 KW C. 500 KW
COP B. 250 KW D. 382 KW
Compressor Power
SOLUTION:
100(3.516)
5
Compressor Power RE = 90x3.516
RE 316.44 KW
Compressor Power = 70.32 KW x 1/0.746
Compressor Power = 097(90)(0.75)
Compressor Power = 94.26 Hp Compressor Power = 65.475 KW
Heat Rejected = Ref. Effect + Camp. Power
Heat Rejected = 316.44 + 65.475
Heat Rejected = 381.915 kw
Refrigeration - 6

A 90 tons refrigeration system has a compressor input of 0.97 KW per


ton refrigeration and COP of 5. What is the efficiency of the Refrigeration - 8
compressor?
A. 72.49% C. 56.34% The mass flow rate of refrigerant entering the compressor is 0.25 kg/~
B. 90.42% D. 83.33% and the change of enthalpy between the inlet and outlet is 320 KJ/kg.
If 134 Hp motor is used to drive the compressor, determine the heat
SOLUTION: loss from the compressor.
A. 15 KW C. 25 KW
B. 20 KW D. 30 KW
90(3.516)
Compressor Power =
5 SOLUTION:
Compressor Power 63.29 KW
Compressor Power mth, - hi)
Compressor input = 0.97(90) Compressor Power 0.25(320)
Compressor input = 87.3 KW Compressor Power 80KW

63.29 Power input 134 x 0.746


Compressor Efficiency
87.3 Power input 99.964 kw
Compressor Efficiency 72.497%
Heat Loss 99.964 - 80
Heat Loss 19.964 kw
394 Refrigeration Refrigeration 395

Refrigeration - 9 Refrigeration - 11

A 100 tons refrigeration system is used to produce chilled water from The difference in temperature between the water leaving the
22°C to 2°e. Determine the volume now of water in Ii/sec. evaporator and evaporator temperature is SoC. [f the temperature of
A. ).0 C. 4.2 water leaving is 3rF, what is the evaporator temperature?
B. 3.5 D. 5.5 A. O°C C. -4°C
B. _2°C D. -SoC

SOLUTION:
SOLUTION:

Ref Effect = m., cp (t 2 - t.)


°C = 5/9 (32 - 32)
"C = O°C
100x3.516 = m w(4.187)(22-2)
Evaporator Temp. 0- 5
m., = 4.187kg/sec
Evaporator Temp. -5°C
Q = 4.187 kg/sec (lIilkg)

Q = 4.1871ils
Refrigeration - 12

The heat rejected from condenser is 300 KW. The water required to
Refrigeration - 10 cool the refrigerant is S li/sec. Determine the temperature of water
leaving the condenser if water enters at 25°C.
An industrial plant requires to cool 120 gal/min of water from 20°C to A. 30°C C. 45.33°C
soc. Determine the tons of refrigeration required. B. 35°C D. 39.33°C
A. 100 TR C. 145 TR
B. 130 TR D. 135 TR SOLUTION:

QR = m., c p (t 2 - tt)
SOLUTION:
300 ~ (5 X 1)(4. I 87)(t 2 - 25)
Ref Effect = rn., c p (t 2 - t.)
rn., = 120 gal/min x 3.785 liIgaI x Imin/60sec t2 39.33°C
m., = 7.57 Ii/sec x Ikg/Ii
m., = 7.57 kg/sec

Ref. Effect = 7.57(4.187)(20 - 5) Refrigeration .: t::


Ref. Effect = 475.43 kw
The mass now of refrigerant entering the compressor is 0.1 kg/sec with
TR = 475.43/3.516 change of enthalpy of 400 KJ/kg. For compressor efficiency of 70%
TR = 135.22 ton of ref. and motor efficiency of 80%, find the electrical energy needed for one
day.
,(if, Refrigeration
R c] rr::';l'rl/tion 397

A 1524 KW-Ilt C 3455 KW-Iil SOLUTION:


B, 1685 KW-III D 1714 KW-Iil

SOLUTION Re frigerating Effect


TR ~
3S 16
Compressor Power m(h: - hi)
Compressor Power o 1(400) (12 / 60)( \ 000)
Compressor Power 40 kw TR ~

40 3.516
Input Power of motor TR= 56,88 tons of ref
0,7(0,8)
Input Power of motor ,"'" 71,428 kw
Energy = 71,428 x 24' hrs
Energy = 1714,28 Kwh Refrigeration - 16

The mass flow of water entering the condenser is 10 kg/sec. If the


temperature difference between inlet and outlet temperature is l8 F,
D

Refrigeration - 14 determine the heat rejected from the condenser.


A, 400 KW C 500 KW
An 80 tons refrigeration system requires lOS KW from VECO, If B. 419 KW D, 324 KW
motor compressor efficiency is 60%, find the COP,
A. 5,84 C 4,46
B, 3,56 D, 3,78 SOLUTION

SOLUTiON: .6. D C C~ 5/9 OF


.6."C = (5/9)( 18)
Compressor Power = 105(0,6)
.6. DC = lODC
Compressor Power = 63 kw
QR = 10(4,187)(10)
RE
COP = - - - - - - QR = 418,70 KW
Compressor Power
80(3,516)
COP Refrigeration - 17
63
COP 4,46
The change of enthalpy in the condenser is 1500 KJ/kg. The
temperature change of water is 8°C and the refrigerant flow is 0.13
kg/sec, Determine the gpm of water required for cooling.
Refrigeration - 15 A. 92 gpm C 45 gpm
B, 88 gpm D, 67 gpm
The change of enthalpy between the inlet and outlet of evaporator is
1000 KJ/kg and mass flow of refrigerant is 12 kg/min. What is the
SOLUTION:
capacity of plan!'?
A 15TR C 57 TR
B, 25 TR C, 60 TR QR m; cp (.6.t)
398 Refrigeration Refrigeration 399
0.13( 1500) rn., (4.187)(8) T] = -25 + 273
T, = 248°K
rn; = 5.8216 kg/sec x Iii/kg OR = T2 (s, - S4)
6000 = 345(5 I - S4)
Ow = 5.8216 Ii/sec x I gal/3.785 Ii x 60sec/min SI - 54 = 17.3913

Ow = 92.28 gpm Hp = (T 2 - Td(s) - S4)


Hp = (345 - 248)(17.3913)
Hp = 1686.96 KJ/min (Imin/60sec)
Hp = 28.116 KW/0.746
Refrigeration - 18 Hp = 37.69 Hp

The heat rejected from the condenser is 200 KW. The mass flow of
water entering is 5 kg/sec at 23°C. If the temperature between the Refrigeration - 20
condenser cooling water outlet and condenser temperature is 5°C,
what is the condenser temp? The temperature difference between the minimum and maximum
A. 30.56°C C. 35.78°C temperature of Carnot cycle is 50°C. What is the minimum
B. 39.94°C D. 37.67°C temperature if COP is 5.5.
A. 0.31°C C. 3°C
SOLUTION: B. 2°C D. 5°C

OR = rn., c p (t 2 -t.) SOLUTION:


200 = 5( 4.187)(t2 - 23)
t2 = 32.55°C T2 = 50 + 273
t.t ,= teon - t 2 T2 = 323°K
5 = t eond - 32.55
teond = 37.55°C T}
COP
T2 -T}
\
T,
5.5 = . --
Refrigeration - 19 232 - T)
323 - T 1 = 0.1818 T,
The heat rejected from the condenser of Carnot refrigeration is 6000 T) = 273.307°K
KJ/min. The minimum and maximum temperature is -25°C and noc t[ = 273.307 - 273
respectively. Determine the horsepower required to drive the t) = 0.307°C
compressor.
A. 30 Hp C. 38 Hp
B. 35 Hp D. 42 Hp
Refrigeration - 21
SOLUTION:
The power required to drive the compressor in a Carnot refrigeration
T 2=72+273
is 50 Hp. It operates between -SoC and 40°C. Determine the tons of
T 2 = 345°K refrigeration.
400 Refrigeration Refrigeration 401

A, 57TR C 55TR KJ/kg-OK and latent heat of fusion is 233 KJ/kg. If specific heat below
B. 34TR D, 63 TR freezing is 1.68 KJ/kg-OK, find the freezing temperature.
A. -565°C . C -1.0 I°C
SOLUTION: B. -219°C D. -10°C

1'1 = -5+273 SOLUTION


1'1 = 268°K
T2=40+27~
o m[cp,(tl-tr)f L+c p2(tr - t 2)]
1'2 = 313°K
268 01 6.4 37 = I 10[3 .:23 (20 - tr) -+- 233 + I. 68{tr - (- 18) }]
COP
313 - 268
COP 5,955 331.24 64.6 - 3.23tr + 233 + 1.68tr + 30.24
RE
COP 155t( = -3.4
Compressor Power
RE t( ,= -2.19°C
5,955
50xO,746
RE = 222,12 KW
TR = 222.12/3.516 . Refrigeration - 24
TR = 63,17 tons of ref
A refrigeration compressor has a specific volume of 0.0482 mJ/kg
entrance and 0.017 mJ/kg at exit. If volum'etric efficiency is 90%,
Refrigeration - 22 determine the percent clearance of the compressor.
A. 5.45% C 10%
Determine the heat to be remove from one ton of water at 26°C to an B 6.35% D. 12%
ice at -6°C.
A. 215,765 KJ C 345,654 KJ SOLUTION:
B. 4D 956 KJ D. 834,582 KJ
11v = I+ C - C(VI/V2)

SOLUTION:
0.90 = I +- c - c(0.0482/0.0 17)
Qremoved m[cpl(t,- tr) + L + Cp2(tr - t2 )1
0.90 = I + c - 2.835c
Qremoved 907[ 4.187(26-0) + 335 + 2.09( 0- (-6) )]
C = 5.45%
Qremoved = 413,956.514 KJ

Refrigeration - 23
Refrigeration - 25
The heat required to remove from beef 110 kg is 36,437 KJ which will
be cooled from 20°C to -18°C. The specific heat above freezing is 3.23 Milk must be received and cooled from 80°F to 38°F in 5 hrs. If 4000
gallons of fresh milk received having SG of 1.03 and = 0.935 Btu "p
per Ib-oF, find the refrigeration capacity.
402 Refrigeration: Refrigeration 403
A. 20 TR C.·22 ..:'i TR
h rg = 149.975
B. 30 TR D. 34.6 TR
235.503 = 202.78 + x(l49.975)
x = 0.218
SOLUTION: x = 21.80%
Ref Effect m cp (tz - t j )
w = rn/V
Refrigeration - 27 (ME Bd. Oct. 1991)
m
1.03 (62.4)
4000 I 7.481 An air-conditioning plant with a capacity of 400 KW of refrigeration
m = 34364.39 Ibs has an evaporating and condensing temperature of 3°e and 37°e
Ref. Effect = (3436439/5x60)(0.935)(80~38) respectively. If it uses Refrigerant 12, what is the volumetric rate of
Ref. Effect = 4498.298 Btu/min flow under suction condition?
Ref. Effect = 4498.298/42.42 A. 0.272 m 3/s C. 0.172 mJ/s
Ref. Effect = 106.04 Hp B. 0.453 m 3 Is D. 0.243 ml/s
TR = (106.04 x 0.746)/3.516
TR = 22.50
p
SOLUTION:
2
Refrigeration - 26 (ME Bd. Oct. 1991) From R-12 tables:
hi = h g at 3°e
An air-conditioning plant with a capacity of 400 KW of refrigeration hi = 352.755 KJ/kg
has an evaporating and condensing temperature of 3°e and 37°C h, at 3°e = 202.780 KJ/kg
respectively. If it uses Refrigerant 12, what will be the mass of flash "I = 0.05047 m
3/kg

gas per kg 'of refrigerant circulated? h, = h, = h rat37°e


A. 21.80% C. 14.56%
h, = h, = 235.503 KJ/kg v
B. 12.45% D. 1834%
Refrigerating Effect = mth, -14)
p
400 = m(352.755 - 235.503)
SOLUTION: m = 3.411 kg/sec
I,?",
VI = m VI
From R-12 tables: VI = 3.411(0.05047)
hi = h g at 3°e VI = 0.I72m
3/sec

hi = 352.755 KJ/kg
h r at 3°e = 202.780 KJ/kg
VI = 0.05047 m3/kg
h 3 = 14 = hr at 37°e
h, = h, = 235.503 KJ/kg v Refrigeration - 28 (ME Bd. Apr. 1986)

Let x = mass of flash gas or quality after expansion An air-conditioning system of a high rise building has a capacity of 350
h, = h, = h r + xh rg KW of refrigeration; uses R-12. The evaporator .and condenser
hrg = hg - hr temperature are ooe and 35°e, respectively. Determine work of
h rg = 352.755 - 202.780 compression in KW
404 Refrigeration 405
R efrigeration
'i Refrigerating capacity = rruh. - h 4 )
A, 34 kw C. 43 kw
Refrigerating capacity = 0,566(345 - 238,5)
B. 52 kw D,65kw
Refrigerating capacity = 60,279 kw
Tons of refrigeration = 60,279/3,516
Tons of refrigeration = 17.144 tons ref.
SOLUTION'
p
From R -12 tables and P-h chart:
hi = h g at O°C 'f ,.~ ~ \ 2, Refrigeration· 30 (ME Bd. June 1990)
hi '= 351.477 KJ/kg I ?"

VI = vgatO°C
3 A simple vapor compression cycle develops 15 tons of refrigeration
VI = 0,0553892 m /kg
using ammonia as refrigerant and operating lit condensing
At O°C:
temperature of 24°C and evaporating temperature of -18°C and
h f = 200 KJ/kg
assuming compression are isentropic and that the gas leaving the
h fg = 351.477KJ/kg
From R-12 chart:
v
• condenser is saturated, find the power per ton
A. 0.333 kw/ton C. 0.452 kw/ton
h z = 369 KJ/kg
B. 0.533 kw/ton D. 0.702 kw/ton
h, = h, = 233.498 KJ/kg
Refrigerating Effect = rruh, - h 4 ) p
350 = m(35 1.477 - 233.498)
SOLUTION:
m = 2,967 kg/sec .~
Work of compression m(hz-hl) From ammonia tables and chart:
Work of compression 2,967(369-351.477)
h, = 1439.94 KJ/kg
Work of compression 52 kw
h z = 1665 KJ/kg
h, = h, = 312.87 KJ/kg
3/kg
VI = 0.572875 m

Refrigerating Effect = m(h 1 - h 4 )


Refrigeration - 29 (ME Bd. Apr. 1990)
15(3.516) = m(l439.94 - 312.87)
v
m ~o 0.0467 kg/sec
A vapor compression refrigeration system has a 30 KW motor driving
the compressor. The compressor inlet pressure and temperature are
Power Requirement = m(h z - hi)
64.17 Kpa and -20°C respectively and discharge pressure of 960 Kpa. Power Requirement = 0.0467(1665 - 1439.94)
Saturated liquid enters the expansion valve. Using Freon 12 as Power Requirement = 10.531 lew
refrigerant, determine the capacity of the unit in tons of refrigeration. Power per ton = 10.531115
SOLUTION:
Power per ton = 0.702 K W Iton

From R-12 tables and chart:


hi = 345 KJ/kg
h z = 398 KJ/kg
Refrigeration - 31 (ME Bd. Apr. 1983)
h, = h, = 238,5 KJ/kg
Compressor work = m(h z - hi)
A vapor compression refrigeration system is designed to have a
30 = m(398 - 345)
capacity of 100 tons of refrigeration. It produces chilled water from
m = 0,566 kg/sec
- 406 Refrigeration Refrigeration 407

2rc to 2°C. Its actual coefficient of performance is 5.86 and 35% of By heat balance in the system.
the power supplied to the compressor is lost in the form of friction and OR = We + RE
cylinder cooling losses. Determine size of the electric motor required tc OR = 351.6 + 60
drive the compressor in kilowatts. OR = 411.6 KW
A. 92.31 kw C. 87.23 kw mwC4.187)(10) = 411.6
B. 76.34 kw D. 98.23 kw low = 9.83 kg/sec

SOLUTION: Refrigeration - 33 (ME Bd. Oct. 1984)

Ref. Effect = 100(3.516) A belt driven compressor is used in a refrigeration system that will
Ref. Effect = 351.6 kw cool 10 Ii/sec of water from BOC to l°e. The nelt efficiency is 98%,
COP = Ref. Effect/Compressor power motor efficiency is 85%, and the input of the compressor is 0.7
351.6 KW/ton of refrigeration. Find the actual coefficient of performance if
5.86 = - - - - - - EVAPORATOR
Compressor Power overall efficiency is 65%.
4
A. 2.34 C. 3.45
Compressor Power = 60 kw
B. 4.32 D. 6.44
Compressor efficiency = 1 - 0.35
Compressor efficiency = 0.65
SOLUTION:
Motor rating = 60/0.65
Motor rating = 92.31 kw
22°F+ +2°F
low Ref. Effect = 10 cp (t2 - t l )
Ref. Effect = (10 x 1)(4.187)(13-1)
Ref. Effect = 502.44 kw
TR = 502.4413.516
Refrigeration - 32 (ME Bd. Apr. 1983)
TR = 142.90
A vapor compression refrigeration system is designed to have a
Compressor Input 0.7(142.90) Expansion
Valve
capacity Of 100 tons of refrigeration. It produces chilled water from Compressor Input 100 kw
100(0.65)
2rC to 2°C. Its actual coefficient of performance is 5.86 and 35% of Compressor work e=98%
the power supplied to the compressor is lost in the form of friction and (0.98)( 0.85)
cylinder cooling losses. Determine the condenser cooling water Compressor work 78 kw
required in kg/sec for a temperature rise of 100e.
A. 9.83 C. 12.23 Actual COP 502.44178 13°C 11Oli/s 11 -c
B. 7.45 D. 4.34 Acw~COP 6.44 mw

SOLUTION:
EVAPORATOR
1
Refrigerating Effect = 100(3 .516) ~, Refrigeration - 34 (ME Bd. Oct. 1984)
Refrigerating Effect = 351.6 KW
COP = Ref. Effect/Compressor power A belt driven compressor is used in a refrigeration system that will
351.6 cool 10 Ii/sec of water from 13°C to 1°C. The belt efficiency is 98%,
5.86 = motor efficiency is 85%, and the input of the compressor is 0.7
Compressor Power 22°F 2°F
rn, KW/ton of refrigeration. Find the mass flow rate of condenser cooling
Compressor Power = 60 KW
water warmed from 21°C to 32°C if overall efficiency is 65%.
408 Refrigeration Refrigeration 409

A. 10.34 kg/s C. 23.23 kg/s VI = 5.937 m 3/min


B. 12.60kg/s D. 15.34 kg/s VD = 5.937/0.7
VD = 8A81 mvmin
2
VD = 11:/4 D L N c
For D= L (unity)
SOLUTION:
8A81 = 11:/4 (D)2 (D) (1200) (6)
Ref Effect = m c p (t 2 - tt) D=0.114m
Ref Effect = (10 x 1)(4.187)(13-1) D = 114mm
Ref Effect = 502A4 kw
TR = 502A4/3.516
TR = 142.90
Compressor Input = 0.7(142.90) Expansion Refrigeration - 36 (ME Bd. Apr. 1985)
Valve
Compressor Input = 100 kw
100(0.65) An am monia compressor operates at an evaporator pressure of 316
Compressor work = - - - - - ' - e=98"1o Kpa and a condenser pressure of 1514.2 Kpa. The refrigerant is
(0.98)(0.85)
subcooled 5 degrees and is superheated 8 degrees. A twin cylinder
Compressor work = 78 kw compressor with a bore to stroke ratio of 0.85 is to be used at 1200
QR = RE + We rpm. The mechanical efficiency is 78%. For a load of 87.5 kw,
QR = 502A4 + 78 13°C1101i/S 1°C determine the bore and stroke for 5% clearance.
QR = 580A4 KW rn, A. 123A4 mm C. 117.40 mm, 99.79 mm
QR =m., c p (t 2 - t 1) B. 109.23 mm D. 234.23 mm, 86.79 mm
580A4 = rn., (4.187)(32- 21)
m., = 12.6 kg/sec SOLUTION:

From ammonia charts and table: P


h, = 1472 KJ/kg
Refrigeration - 35 (ME Bd. Oct. 1988) h, = 1715 KJ/kg
h, = h, = hr at 34°C
Calculate the bore in mm of a single-acting, 6 cylinder ammonia h, = h, = 361.2 KJ/kg
compressor running at 1200 rpm to compress 700 kglhr of refrigerant VI = OAI mJ/kg
which vaporized at -15°C, given the following: V2 = 0.12 m 3/kg
a. Bore and stroke = Unity Refrigerating Effect = nuh, - h 4)
b. Volumetric Efficiencv = 70% 87.5 = m (1472 - 361.2)
c. Specific volume of NU3 = 8.15 felIb at 5°F m = 0.07877 kg/sec v
A. 110 mm C. 114 nun VI = rn VI
B. 106 rnm D. 124 rnm VI = 0.07877(OA1)(60)
VI = 1.9378 m'zmin
Since clearance is not given, then assume 5% clearance.
SOLUTION: T]v = 1 + c - C(VI / vz)
T]v = I + 0.05 - 0.05(OA I 10.12)
VI =8.l5fellb x 1m 3/35.31ft3 x 2.2051h/lkg
T]v = 0.879
vi = 0.5089 m 3/kg
VD = 1.9378/0.879
VI = m VI
VD = 2.204 mJ/min
V I = (700/60XO.5089)
410 Refrigeration 411
Refrigeration
VD~rr/4D2LNc Total Power required = 00863( 1590 - ~422.86)
D/L ~ 0.85 + 0.112([649 - 1462)
2.204 ~ (11:/4) D 2 (D/0.85)(1200)(2) Total Power required = 35.37 KW
D = 0.09979
D = 99.79 mm
L = 99.79/0.85
L = 117.40 mm
Refrigeration - 38 (ME Bd. Apr. 1981)

A refrigeration system operates on the reversed Carnot cycle. The


Refrigeration - 37 (ME Bd. Oct. 1984) minimum and maximum temperatures are -25°C and 72°C,
respectively. If the heat rejected to the condenser is 6000 KJ/min,
In a certain refrigeration system for a low temperature application, a draw the T-S diagram and find power input required
two stage operation is desirable which employs ammonia system that A. 20.34 kw C. 13.45 kw
serves a 30 ton evaporator at -30°C. The system uses direct contact B. 65.33 kw D. 28.12 kw
cascade condenser, and the condenser temperature is 40°C. Find the
total power required in kw, SOLUTION:
A. 23.43 kw C. 28.34 kw T
B. 25.37 kw D. 45.23 kw T[ = -25 + 273 2
0
.2

SOLUTION:

From Ammonia table:


PI = 119.9 Kpa
P 6 = 1557 Kpa
p

I 3i I 'A~/1:>"2
6
T 1 = 248°K
T 2 = 72 + 273
T 2 = 34SOK
QR = (SI - s4)(T2)
6000 = (s, - 54)(345)
(SI - S4)= 6000/345
Ref. Effect = (SI - s4)T I
4•
t:J _2S

oC
'""
1

r, ~PIP6
s
= Ref. Effect = (6000/345)(248)
Px = ";'---0-1-9.-9)-(1-55-7-) Ref. Effect = 4313 Kl/rnin
P x = 432.10 Kpa Power required = 6000 - 4313
From ammonia tables and chart: Power required = 1687 KJ/min
hi = 1422.86 KJ/kg h, = 1462 KJ/kg Power required = 1687/60
h 2 = 1590KJ/kgh6 = 1649KJ/kg Power required = 28.12 KW
h, = h, = 200.46 h 7 = h g = 390.587
Refrigerating Effect = ml(h 2 - h 3 )
30(3.516) = ml( 1422.6 - 200.46)
. ml = 0.0863 kg/sec Refrigeration - 39 (ME Rd. Oct. 1986)
By heat balance in cascade condenser:
m2(h s - hg ) = ml(h 2 - h 3 ) A refrigerating system operates on the reversed Carnot cycle. The
ml1462 - 390.587) = 0.0863( 1590 - 200.46) highest temperature of the refrigerant in the system is 120°F and the
m, = 0.112 kg/sec lower temperature is 10°F. The capacity is to be 20 tons. Determine
the heat rejected from the system in Btn/min
Total Power required = ml(h 2 - hi) + m2(h6 - h s) A. 4936 C. 3423
B. 5634 D. 7421
412 ReJrigeration .....1 _1
Refrigeration
j
SOLUTION: required plant refrigerating capacity in tons of' refrigeration if the
specific heat of fish is 0.7 above freezing and 0.3 below' freezing point
l
T1 = 10 + 460 which is -3°e. The latent heat of freezing is 55.5 Kcallkg.
T T, = 470 0 R A. 21.23 C. 28.34
B. 24.38 D. 32.12

~.
T2 = 120 + 460
T2 = 580 0 R _10°C
200 e
SOLUTION:
n~
~1

Ref. Effect = (s, - S4)Tl '11000k 9\ I


~
fish
20 x 200 = (s, - S4 )(470)
~, - (s, - S4) = 4000/470
Ref. capacity = m[ c, (t, - tr) + L + c2Ctr - t 2) ] t,::::3°e
Heat Rejected = (s, - s4)T2
11,000
Heat Rejected = (4000/470)(580) Ref. capacity = [0.7(20-(-3»+55.5+0.3{-3-(-10)}]
s Heat Rejected = 4,936.17 BtuJmin 11(3600) ,

Ref. capacity = 20.47 Kcal/s x 4.187


Refrigeration - 40{ME Bd, Oct. 1989) Ref. capacity = 85.72 KW

An Ice plant produces 20 tons of ice per day at -15°C from water at Tons of Refrigeration 85.72/3.516
25°e. If miscellaneous losses are 12% of the freezing and chilling load, Tons of Refrigeration 24.38 tons ref.
calculate the refrigeration capacity of the plant in tons of
refrigeration.
A. 21.35 C. 31.5 Refrigeration - 42 (ME Bd. Apr. 1986)
B.43.12 D. 36.3
25°e -15°e Compute tm; neat to be removed from 110 kg of lean beef if it were to

SOLUTION: ~ nn.. ''. r:::l


~lJL:J
2OtOM.
be cooled from 20°C to 4°C, after which it is to be frozen and cooled to
-18°e. Specific heat above freezing is given as 3.23 KJ/kgOC and below
freezing as 1.68 KJ/kg_°C. freezing point is -2.2°C and latent heat of
Product Load = mjc, (t. - t r) + L + C2 (t r- t 2 ) ] !,aooe fusion is 233 KJ/kg.
20(907) A. 32,455 KJ C. 36,437.5 KJ
Product load = [4.187(25 - 0) + 335 + 2.09{0-(-15)}]
24(3600) B. 23,455 KJ D. 54,223.2 KJ
200e
Product load = 98.904 kw
Considering the 12% freezing and chilling load:
Ref. capacity = 98.904(1.12)
SOLUTION: §-{][J------§
11... . 11_
_18°e

Q =m[cj(t,-tr}+L+c2(tr- t2)] ...... • • ,. . a.r • •


Ref. capacity = 110.773 kw
Tons of Refrigeration = 110.773/3.516 Q = 110[3.23(20 - 4) + 3.23(4 - (-2.2» + 233 + 1.68(-2.2 - (-18»]
Tons of Refrigeration = 31.5 tons ref. Q = 36,437.5 KJ

Refrigeration - 41 (ME Bd. Apr. 1989) Refrigeration - 43 (ME Bd. Apr. 1992)

Fish weighing 11,000 kg with a temperature of 20°C is brought to a Magnolia Dairy products plant must cool 4000 gallons of fresh milk
cold storage and which shall be cooled to -10°C in 11 hours. Find the received from the farm per day from an initial temperature of 80°F to
414 Refrigeration
Refrigeration 415
a temperature of 38°C in 5 hours. If the density of milk is 8.6
Ibs/gaIlon, specific gravity is 1.03, and specific heat is 0.935 how much Heat of fusion of ice ------------- 144 Btu/lb
brine must be circulated if the change in temperature is 15°F, specific A. 34.56 Ibs C. 43.23 Ibs
gravity is 1.182, and specific heat is 0.729? B. 74.23 Ibs D. 64.8 Ibs
A. 41.77 gpm C. 34.11 gpm

ITJ
B. 54.22 gpm D. 65.23 gpm 8S0F
SOLUTION: SOF
Vegetable
!
Ice
m 2S0lb !
SOLUTION. Cooling Load = m, cp ~t(U)
II

BO°F 3BOF . 250(0.80)(85 -- 45)(1.30)


i
,I
Brine Coolmg Load = --------
Ref. capacity = m c p ~t Cooler 24
Cooling Load 433.33 Btu/hr 4S0F
4000x8.6 =
m Ice
5 +
m = 6880 Ib/hr Heat gained by vegetable = Heat loss from ice Vegetable

Ref. Capacity = 6880(0.935)(80 - 38) (m/24)[ 0.463(32 - 25)+ 144 + 1.01(45 - 32)] = 433.33
Ref. Capacity = 270,177.6 Btu/hr m = 64.81bs

Q = m c p ~t
270,177.6 = m(0.729)(IS)
Refrigeration - 45 (ME Bd. Oct. 1990)
m = 24,707.6 Ib/hr
m = 411.793 Ib/min
A 10 tons ice plant using ammonia refrigerant operates between
411.793 evaporator and condenser temperature of -20°C and 35°C
v respectively. The ice plant is to produce ice at -12°C from water at
1.182 x 62.4 30°C in 24 hours. Assuming losses to be 18% of the heat absorbed
V = 5.583 fe/min from water, determine the power required by the compressor
V = 5.583 x 7.481 A. 21.23 kw C. 56.32 kw
V = 41.766 gal/min B. 16.79 kw D. 43.23 kw

300 e -12°e
Refrigeration - 44 (ME Bd. Apr. 1982) SOLUTION:
10 tons I
Water
I n .r:::l
LI1.:J
A mass of ice at 25°F is needed to cool 250 pounds of vegetables in a
bunker, for 24 hours. The initial temperature of the vegetables is t,=ooe
assumed to be 85°F. It is also assumed that the average temperature 10(907)
inside the bunker is 45°F, within the 24-hour period. If the heat gained RE = [4.187(30-0)+335+2.0935{0-(-12)}](1.l8)
24(3600)
per hour in the bunker is 30% of the heat removed to cool the
RE = 60.17kw
vegetables from 85°F to 45°F, what would be the required mass of ice? From Ammonia chart and table:
Specific heat of ice --------------- 0.463 hi = 1437.23 KJ/kg
Specific heat of vegetables ----- 0.80 h z = 1736 KJlkg
Specific heat of water ---------- 1.01 h, = !4 = 366.072 KJlkg
416 Refrigeration Refrigeration 417

Relllgnalmg Effect = rruh, - h.) SOLUTION


60.17 111(1437.23 - 366.072)
m 00562 kg/sec Ref. Effect = rruh, - h 4 )
Compressor Power m(h 2 - h) 5(3.516) = m(353.6 - 238.5)
Compressor Power 0.0562(1736 - 1437.23) m = 0.153 kg/sec
Compressor Power 16.79 kw
Heat gained by water Heat loss by refrigerant
=
m., cp (t) - t 2 ) m(h 2 - h 3 )
=
Refrigeration - 46 (ME Bd. Oct. 1994) ~w(4.187)(7) = 0.153(377 - 238.5)
m., = 0.723 kg/sec
An ideal vapor compression refrigeration cycle requires 2.5 KW to Vw = (0.723 kg/sec)(1 lilkg)(60sec/min)(1 gal/3 .785Ii)
power the compressor. You have found the (ollowing data for the Vw = 11.46 gpm
cycle: the enthalpy at condenser entrance = 203 KJ/kg, exit = 55;
evaporator entrance = 55 KJ/kg, exit = 178. If mass flow rate of the
refrigerant is 0.10 kg/sec, then the coefficient of performance of this Refrigeration - 48 (ME Bd. Apr. 1996)
refrigeration cycle is most nearly:
A. 592 C. 5.92 A refrigeration system operates on an ideal vapor-compression using
B. 59.2 D.4.92 Refrigerant-12 with an evaporator temperature of -30°C and a
condenser exit temperature of 49.3°C and requires a 74.6 KW motor
SOLUTION: to drive the compressor. What is the capacity of the refrigerator in
tons of refrigeration? Enthalpy of condenser entrance = 382 KJ/kg,
hi ~ h, exit = 248.15; at evaporator entrance = 248.15, exit = 338.14.
COP
A. 43.1 C. 21.3
h, - hi
B. 34.5 D. 18.2
178 - 55
COP SOLUTION:
203 -178
W = m(h 2 - hi)
COP = 4.92 74.6 = m(382 - 338.14)
m = 1.7 kg/sec

QA m/h, - h 4 )
Refrigeration - 47 (ME Bd. Oct. 1994) QA 1.7(338.14 - 248.15)
QA 153 kw
A Freon -12 waste water system operating at a 5°C suction
temperature and a 40°C condensing temperature has an evaporator Ref. Capacity 153/3.516
load of 5 tons. If the condenser is selected for a 7°c water temperature Ref. Capacity 43.5 tons ref.
rise, how many gpm must be circulated through the condenser?
The following enthalpies have been found: condenser entrance = 377
KJ/kg, exit = 238.5; evaporator entrance = 238.5 KJ/kg, exit = 353.6 Refrigeration - 49 (ME Bd. Apr. 1995)
A. 11.46gpm C. 13.45gpm
B. 15.23 gpm D. 23.22 gpm Liquid ammonia at a temperature of 26°C is available at the expansion
valve. The temperature of the vaporizing ammonia in the evaporator
418 Refrigeration 419
Refrigeration
is 2°C. Find the percentage of liquid vaporized while flowing through SOLUTION:
the expansion valve.
Temperature Pressure From 350 kpa to 1300 kpa:
(0C) (Kpa) hf hfg hg RE = m (h, - h 4 )
2°C 462.49 190.4 1255.2 1445.6 40 = m(189.023 - 87.796)
26°C 1033.97 303.6 1162.0 1465.6 m = 0.39515 kg/s
A. 9.02 C. 91.08
B. 90.98 D. 8.92 From 350 kpa to 1400 kpa
RE = m'(h, - h;')
SOLUTION: 40 = m'(189.023-91.355)
m' = 0.40955 kg/s
h3 = 14 = 303.6 KJlkg
Tlv Tlv
h, = hf + xh fg
mV
I
m'v I
303.6 = 190.4 + x(1255.2)
(n/4)(D)2 LN - (n/4)(D)2 LN,
x = 9.02% m m'
N N'

0.39515 0.40955
Refrigeration - 50 (ME Bd. Apr. 1998)
600 N'
A 140 mm x 140 mm single effect, twin-cylinder, single actmg Freon-
12 compressor with a refrigeration capacity of 40 kw operates between N' = 621.865 rpm
a discharge pressure of 1300 kpa and a suction pressure Of 350 kpa.
The speed of the compressor is 600 rpm. If the discharge pressure
shall be raised to 1400 kpa, at what speed (rpm) should the
compressor be run to produce the same refrigeration capacity and Refrigeration - 51 (ME Bd, Oct. 1997)
assuming the volumetric efficiency to remain the same?
Freon 12 Properties: A refrigeration system using R-22 has a capacity of 320 kw of
refrigeration. The evaporating temperature is minus 10 degrees C and
At 350 kpa p
h = 189.023 KJ/~ ~j -;r- \. lJ) the condensing temperature is 40"C. Calculate the fraction of vapor in
J
y = 0.04923 m /kg
the mixture before the evaporator.
At 1300 kpa Properties of R-22 are:
At -10 nC
h = 211.314 KJ/kg
h g = 401.60 KJ/kg
h, = 87.796 KJ/kg
At 1400 kpa h r = 188.426 KJ/kg
At 40°C
h = 213.692 KJ/kg
h r = 249.686 KJ/kg
h, = 91.355 KJ/kg I •
A. 0.287 C. 0.245
A. 610 C. 620 v B. 0.315 D. 0.227
B. 615 D. 630
420 Refrigeration Refrigeration 421

SOLUTION:
p SOLUTION: Condinser
h, = h, = h, + x (h g - he) " -- - .. \ ,,0
249.686 = 188.426 + x(401.60 - 188.426
Expansion
x = 0.287 Valve

v
Refrigeration - 52 (ME Bd. Oct. 1999)
COP = RE/W c
What is the coefficient of a vapor compression refrigeration system COP == 22/(7.8 x 0.746)
having the following data: Enthalpy entering the compression is COP = 3.78
181.79; enthalpy after compression work is 207.3 KJ/kg. After
condensation the enthalpy is 58.2 and the throttled from 0.19 Mpa to
0.18 Mpa.
A. 5.8 C. 4.75
B. 3.2 D. 5.6

SOLUTION:

COP = hI - h,
h, - h I

181.79-58.2
COP
207.3 -181.79 h,=181.79KJ
@ Kg
COP = 4.84

Refrigeration - 53 (ME Bd. Oct. 1999)

A refrigeration system having a 22 kw capacity needs 7.8 hp


compressor. Find the COP of the system.
A. 312 C. 3.78
B. 4.62 D. 6.34
422 Air-conditioning 42')
Air-conditioning

AIR-CONDITIONING Air Conditioning - 3

The change of enthalpy of air in a cooling tower is 35 Btu/lb and the


mass flow of air is 453.17 Ib/min. Water enters the tower at the rate of
50 gpm and 115°F. Determine the exit temperature.
Air Conditioning - 1 A. 45°F C. 65°F
B. 55aF D. 77°F
When 100 kg/min of outside air at 32°C dry bulb and 200 kg/min re-
circulated air at 22°C dry bulb are mixed with an air conditioning SOLUTION:
system, the resulting dry bulb temperature will be:
Heat carried by air = m(h l - h 2)
A 2533°C C. 4633°C
Heat carried by air = 453.17(35)
B. 35.44°C D. 26.88°C 15860.95 Btu/min
Heat carried by air
Heat carried by air Heat remove from water
SOLUTION:
Heat carried by air = m cp (t 2 - t.)
rna to + m, t, = m., t m m; = 50 gal/min x 62.4/7 .481
m., = rna + rn, m., = 417.056Ib/min
rn., = 100 + 200
m., = 300 kg/min
15860.95 = 417.056(1)(115 - t2 )
100(32) + 200(22) = 300 tm t 2 = 76.97°F
tm = 25.33°C

Air Conditioning - 2
Air Conditioning - 4
The amount water carried by air in a cooling tower is 15 Ib/min. The
The change of enthalpy in an air conditioning unit is 10 Btu/lb. The
change in humidity ratio in outlet and inlet is 0.025 Ib/lb. Determine
mass of supply air is 150,000 Ib/hr. What is the conditioner capacity?
the volume flow of air needed if specific volume is 13 fe/lb.
A. 125 TR C. 150 TR
A. 6000 ft3/min C. 7500 ft3/min
B. 100 TR D. 200 TR
B. 7800 ft 3imin D. 5000 ft3/tnin

SOLUTION:
SOLUTION:
m., = tn. (W2 - WI)
Conditioner capacity m, (h I - h4 )
15 = m, (0.025)
Conditioner capacity = 150,000 (10)
Conditioner capacity = 1,500,000 Btulhr x 1.055/3600
rna = 600 Ib/min
Conditioner capacity = 439.58 KW
Va = rna V
Cond itioner capacity 439.58/3.516
Va = 600(13)
Conditioner capacity = 125.02 ton ref
V, = 7800 ftl/min
424 Air-conditioning Air-conditioning 425

SOLUTION:
Air Conditioning' - 5

SHF
o
Re-circulated air of 8 kg/sec with 53 KJ/kg enthalpy and outside air of
2 kg/sec with 90 KJ/kg enthalpy enters the conditioning unit. OL + Os
Determine the air conditioning capacity if supply enthalpy to 120
SHF
conditioned space is 42 KJ/kg. 120+47
A 154 KW C. 174 KW
B 164 KW D. 184 KW

SOLUTION: Air Conditioning - 8

rna h, + m, h, = m, h, The total heat load and latent heat load of theater is 150 KW and 60
2(90) + 53(8) = (2 -t- 8)(h 4 ) KW, respectively. Supply air is at 15°C and has a mass of 9 kg/sec.
h, = 60A KJ/kg Determine the temperature to be maintained in the theater.
A ir conditioning capacity ills (h, - hi) A. 20 0 e c. 30 0 e
Air conditioning capacity (2 + 8)(60A - 42) B. 25°e D. 35°e
Air conditioning capacity 184 KW
SOLUTION:

Air Conditioning - 6 OT = Os + OL
150 = Os + 60
Outside air of an ail' conditioning system is 25% of re-circulated air. Os = 90 KW
Determine the mass of outside air if mass of supply air is 15 kg/sec.
A. 1 kg/sec C. 3 kg/sec Os = Ins c p (t2 - t j )
B. 2 kg/sec D. 4 kg/sec 90 = 9(1 )(tz- 15)
t2 = 25°e
SOLUTION-

rna + 01, = m,
rna 0.25 rn, Air Conditioning - 9
ill, 4 Ina
m, + -lm, = 15 A cooling tower has an efficiency of 65%. Water enters the tower at
rn, 3 kg/sec 55°C. The wet bulb temperature of surrounding air is 27°C. What is
the temperature of water leaving the tower?
A. 36.8°e c. 46.9°e
Air Conditioning - 7 B. 44.5°e D. 30Aoe
The sensible heat load and latent heat load in an air conditioning SOLUTION:
svstem is 120 KW and 47 KW, respectively. What is the sensible heat
.
factor? "'
ta - tb
A. 65.34% C. 76.54% Efficiency
R. 29A5% D 71.86% t a - t wb
426 A it-conditioning Air-conditioning 427

o. 65 __
_ ')') - I h S()UJ!]()N
5') - 27
Approach = t b - t wb
0.65(28) ~ 55 - t b 10 = 37 - t Wb
twb = 27°C
tb = 36.8°C . ta - t b
Efficiency = ---
t. - t wb
t a - 37
0.65
Air Conditioning - 10 t. - 27
t, = 52°C
The change of temperature entering the cooling tower and wet bulb
temperature of surrounding air is 25°C and efficiency is 65%. If mass
of water leaving the tower is 10 kg/sec, determine the heat carried by
air. Air Conditioning - 12
A. 720 KW C. 680 KW
B. 540 KW D. 700 KW A cooling tower is used to cool a jacket water loss from the engine. The
heat generated by fuel is 2500 KW and cooling loss is 30%. If
SOLUTION: temperature range of the tower is 15°C. Determine the mass flow of
water entering the tower.
A. 12 kg/sec C. 16 kg/sec
t. - t h B. 14 kg/sec D. 18 kg/sec
Efficiency -
t. - t Wb
SOLUTION:
t. - lb
0.65 = Cooling loss 0.3(2500)
25 Cooling loss 750KW
ta - tb = 16.25
Heat
Heat
carried
carried
by air
by air
= Heat loss by water
= me, (t a - tb)
Cooling loss = m., c p «, -
lb)
750 = mw (4 . 187)( 15)
Heat carried by air = 10(4.187)(16.25) m., = 1 1.94 kg/sec
Heat carried by air = 680.39 KW

Air Conditioning - 11 Air Conditioning - 13

The approach and efficiency of cooling tower is 10°C and 60%, A dryer is to deliver 1000 kg/hr of cassava with 2% moisture and 20%
respectively. If temperature of water leaving the tower is 37°C, what is moisture in the feed. Determine the mass of air required if change in
the temperature of water entering? humidity ratio is 0.0165.
A. 46°C C. 68°C A. 3.57 kg/sec C. 3.79 kg/sec
B S2°C D. 48° B. 4.67 kg/sec D. 5.36 kg/sec
428 Air-conditioning 429
Air-conditioning
SOLUTIUN:
SOLUTION:

Beat supplied = ma(h 2 - h.)


Let x = total amount of feed materials
Amount of solid in product = 1000(0.98) = 980 kg/hr 30,000
Amount of solid in the product = Amount of solid in the feed Heat supplied (--)(25 x 1.055)
980 = 0.8(x) 3600
Heat supplied 219.79kw
x = 1225 kglhr
Moisture removed = 1225 - 1000
Moisture removed = 225 kg/hr
'225 = rna (0.165)
rna = 1363636 kg/hr Air Conditioning - 16 (ME Bd. Apr. 1986)
rna = 3.7878 kg/sec
Water at 55°C is cooled in a cooling tower which nas an efficiency of
65%. The temperature of the surrounding air is 32°C dry bulb and
70% relative humidity. The heat dissipated from the condenser is
Air Conditioning - 2,300,000 KJ/hr. Find the capacity in liters per second of the pump
used in the cooling tower.
A. 8.55 C. 6.34
The moisture remove from a material is 250 lb/hr and change of
B. 7.34 D. 9.23
humidity ratio in dryer is 0.0175. Determine the fan capacity if specific
volume of air entering is 35 fellb.
A. 400,000 ft3/hrc. 500,000 ft3/hr
B. 450,000 fe /hr D. 550,000 ft3/hr SOLUTION:
®
ta - t 0

SOLUTION: Efficiency
ta - t Wb

Moisture removed = rna (W2 - WI)


From psychrometric chart:
250 = rna (0.0175)
At32°C and 70% RH:
m••~ 14285.71b/hr
twb = 27.35°C
55 - t b
Va = rnava 0.65
Va = 14285.7(35) 55 ~ 27.35
Va = 500,000 ft3lhr tb = 37.03°C

By heat balance in the condenser:


Air Conditioning - m cp (t a - tb ) = 2,300,00013600
m(4.187)(55 - 37.03) = 2,300,000/3600
The change of enthalpy in a heating chamber of dryer is 25 Btu and m = 8.49 kg/sec
the mass of air supplied is 30,000 lb/hr, What is the heat supplied by From steam table(Table 1), At 37.03°C ,
heater? Vf = 1.0067 li/kg
A. 560 KW C. 350 KW Pump capacity = 8.49( 1.0067)
B. 450 KW o 220 KW Pump capacity = 8.547 Ii/sec
Air-conditioning 4 .J
~ '
I
430 Air-conditioninp
SOIlJTION:
Air Conditioning - 17
From psychrometric chart:
hi ~ 21 Btullb
Fifty gallons per minute of water enters a cooling tower at U5°F. h 2 = 56 Btu/lb ~m
Atmospheric air at 60°F and 55% relative humidity enters the tower WI = 0.0061 lb/lb ~
at 6,009 cfm and leaves at 90°F saturated. Determine the volume of W2 = 0.03121b/lb
water that leaves the tower in gpm. VI = 13.24 ftl/lb
A. 32.34 C. 34.23 From steam table, at 115°F, Vf = 0.01618 ftllib
B. 48.62 D. 65.33 leal galIll:U, ,oy. air ~
Heat lost by water = heat
50
a
( )(1.0KI15-t) = (6000/13.24)(56-21)
SOLUTION: 7.481(0.01618) e

5''lPm t, = 76.61°F
~
From psychrometric chart:
hi = 21 Btullb Air Conditioning - 19 (ME Bd. Apr. 1987)
h2 = 56 Btulb
GO°F An atmospheric cooling tower is to provide cooling for the jacket
WI = 0.0061 lb/lb _____ t.
55%RH
W2 = 0.03I21b/lb ~ Q water of a four stroke, 800 KW diesel generator with useful output of
3/1b
Vi = 13.24 ft 34% and cooling loss of 30% . The cooling tower efficiency is 60% at a
m., C~ Amount of water carried by air temperature of approach of 10°e. If ambient air has a relative
mw = maC w 2 - WI) humidity of 70% and dry bulb temperature of 32°C, determine the
m., = (V, I V.)(W2 - WI) cooling water supplied to the diesel engine in IiIhr. Generator
m.; = (6000/13.24)(0.0312 - 0.0061) efficiency is 97%.
rn., = 11.37 Ib/min A. 41,713 C. 43,345
B. 45,232 D. 47,234
From steam table, at 115°F,
3/1b
\if = 001618ft SOLUTION:
Q = Volume of water that leaves the tower
Q = 50·· 1l.3'";'(001618)(7.48) From psychrometric chart, at 32°C and 70% RH,
Q = 48.624 gpm t wb = 27.50°C
Approach = t b - twb
o
10 = t b - 27.5
t b = 37.5°C
Air Conditioning - 18 ta - tb
Efficiency =
t a - t wh
Fifty gallons per minute of water enters a cooling tower at 115°F.
t a - 37.5
Atmospheric air at 60°F and 55% relative humidity enters the tower 0.60 = e=60% +---
at 6,000 cfm and leaves at 90°F saturated. Determine the exit ta - 27.5 air
temperature of water, of t, = 52.5°C
A. 56.23 C. 76.61 Brake power = 800/0.97
B. 65.33 D. 45.34 Brake Power = 824.742 KW
432 Air-conditioning Air-conditioning 433

824.742 rn., = 5,555 kg/hr


0.34 = - - -.._ - -
Heat sup plied Percentage Make-up water = 5,555/250,000
Heat supplied - 2425.71 KW Percentage Make-up water = 2.22%
Cooling loss ~ 0.30(2425.71)
Cooling loss = 727.713
Cooling loss = mCp(t 2 - t j ) Air Conditioning - 21 (ME Bd. Oct. 1995)
727.713 = m(4.187)(52.5 - 37.5)
m = 11.587 kg/sec x 3600 An auditorium is to be maintained at a temperature of 26°C drybulb
m = 41,713 kg/hr and 50% RH. Air is to be supplied at a temperature not lower than
m = 41,713 kh!hr(I li/kg) 15°C dry bulb. The sensible heat gain is 110 KW and the latent heat
m = 41,713li/hr gain is 37.5 KW. Take ventilating air as 25% by weight of the air from
the room, and is at 35°C dry bulb and 60% RH. Determine the
refrigeration capacity in tons.
A 43.45 C. 63.28
Air Conditioning - 20 (ME Bd. Oct 1991) B. 54.23 D. 76.34

250,000 kg/hr of water at 35°C enters a cooling tower where it is to be SOLUTION:


cooled to 17.5°C. The energy is to be exchanged with atmospheric air
entering the unit at 15°C and leaving the unit at 30°C. The air enters
Qs = m, cp (t, - t])
at 30% RH and leaves at 85% RH. If all process are assumed to occur
110 = ms(1.0)(26 -15)
at atmospheric pressure, determine the percentage of total water flow
that is make up water. m, = 10 kg/sec
A. 3.44% C. 3.94%
B. 8.34% D. 2,22% From psychrometric chart,
h, = 90.5 KJ/kg
h, = 53 KJ/kg
SOLUTION: ®S5%RH QT = Qs + QL
30 0 e QT = m s(h 4 - h])
From psychrometric chart 110+37.5 = 10(53-h])
At 15°C and 30% RH:
h] = 38.25 KJ/kg 01 _
m - O.25 m.
o

h, = 23.0 KJ/kg m,
By mass balance:
w, = 0.0032 kg/kg
rna + rn, = m,
At 30°C and 85% RH:
0.25m, + m, = 10
h 2 = 89.0 KJ/kg Air
rn, = 8 kg/sec ~ Q,
W2 = 0.0232 kg/kg
Heat loss by water = Heat gain by air • I I
By heat balance: 0 37.5 kW
m.h 1 + m.h, = m.h,
m w cp i1t = ma(h 2 - hI) (0.25 x 8)(90.5) + 8(53) = IOh2
250,000(4.187)(35-17.5) = m.(89-23) h 2 = 60.5 KJ/kg
rna = 277,746 kg/hr
Refrigerating capacity = m s(h 2 - h])
rn., = amount of make-up water Refrigerating capacity = 10(60.5 - 38.25)
rn., = rna (W2 - WI) Refrigerating capacity = 222.5/3.516
m., = 277,746(0.0232 - 0.0037)
Refrigerating capacity = 63.28 tons ref

'~'j?
..2.:
434 Air-conditio 11illg Air-conditioning 435

Air Conditioning - 22 (ME Bd. Apr. 1988) ---- ---

Air Conditioning - 23 (ME Bd. Oct. 1981)


An assembly hall was to have an air conditioning unit installed which
An air conditioned theater is to be maintained at 80°F dry bulb
would be maintained at 26°C dry bulb and at 50% RH. The unit temperature and 50% RH. The calculated total sensible heat load in
delivers air at 15°C dry bulb temperature and the calculated sensible the theater is 620,000 Btu/hr. and the latent heat load is 210,000
heat load is 150 kw and latent heat is 51.3 KW. Twenty percent by
Btu/hr. The air mixture at 84°F dry bulb and nOF wet bulb
weight of extracted air is made up of outside air at 34°C dry bulb and
temperature is cooled to 63°F dry bulb and 59°F wet bulb temperature
60% RH while 80% is extracted by the air conditioner from the
by chilled water cooling coils and delivered as supply air to the
assembly hall. Determine the air conditioner's refrigeration capacity
theater. Calculate the tons of refrigeration required.
in tons refrigeration and its ventilation load in, KW.
A. 123 C. 124
A. 83.22, 132 C. ~6.23, 45.32
B. 125 D. 128
B. 75.34, 412 D. 54.23, 83.23

SOLUTION:
SOLUTION: 01 m.
34°C db 11\ Q.=620000

Qs = ills c p (t 2 - t 1)
01 m60%RH Assembly Han From psychrometric chart: '1" 184°F db
~U/h
o r
j I
hi = 35.82 Btu/lb \..:.J m'72°F wb
ISO = fisC 1.0)(26 - 15) Q Q,=210000
h2 = 25.78 Btu/lb Btuth
rn, = 13.636 kg/sec 0 h) = 31.3 5 Btu/Ib
From psychrometric chart: 1 m, 150 kW

h 2 = 53 Kl/kg
Q, Total heat load = rruh, - h z)
h) = 86.5 KJ/kg 51.3 kW 620,000 + 210,000 = m(31.35 - 25.78)
m = 149,013 lb/hr
QT = Qs + QL
150+51.3 = 13.636(53-h l )
Conditioner capacity = fi(h, - h 2 )
hi = 38.24 Kl/kg
149,013(35.82 - 25.78)
fir =0.80(13.636) Conditioner Capacity =
fir = 10.909kg/sec 12,000
rn , = 0.20(13.636) Conditioner Capacity = 125 tons of ref
rn, = 2.727 kg/sec
By heat balance:
m, h) + fir h2 = m, h,
2.727(86.5) + 10.909(53) = 13.636 h, Air Conditioning - 24 (ME Bd. Oct. 1996)
h, = 59.7 Kl/kg
Capacity = m,(h 4 - hi) A room being air conditioned is being held at 25°C dry bulb and 50%
Capacity = 13.636(59.7 - 38.24) relative humidity. A flow rate of 5 ml/s of supply air at 15°C dry bulb
Capacity = 292.629 KW / 3.516 and 80% RH is being delivered to the room to maintain that steady
Capacity = 83.23 tons ref condition at 100 Kpa, What is the sensihte heat absorbed from the
Ventilation Load = maCh) - hi) room air in KW?
Ventilation Load = 2.727(86.5 - 38.24) A. 50.8 C. 40.5
Ventilation Load = 131.60 KW B. 60.8 D. 70.9

n
k
436 A it-conditio II illl{ Air-conditioning 437

S()[ UTION: SOLliTION

PY = m R T Let x = weight of original ")roduct per Ib of wet feed


100(5) = m(O.287)(15 +273)
III = 6.049 kg/sec Solid in wet feed = Solid in dried product
0.95 x = OAO( I)
Os = sensible heat x = OA21 Ib
Os = mCp(t z - t.)
Os = 6.049(1003)(25 - 15) Weight of water removed = 1 - OA21
Os = 60.80 KW Weight of water removed = 0.579 Ib/lb of orig. product
Weight of water removed per Ib of final product
Weight removed = 0.579/0A21
Weight removed = 1.3751b/lb
Air Conditioning - 25 (ME Bd. Oct. 1992)

Copra enters a dryer containing 60% water and 40% of solids and
leaves with 5% water and 95% solids. Find the weight of water Air Conditioning - 27 (ME Bd. Apr. 1983)
removed based on each pound of original product.
A. 0.34 Ib C. 0.86 lb The temperature of the air in a dryer is maintained constant by the
B. 0.63 D. 0.58 lb use of steam coils within the dryer. The product enters the dryer at the
rate of 1 metric ton per hour. The initial moisture content is 3
SOLUTION: kilograms moisture per kilograms dry solid and will be dried to a
moisture content of 0.10 kg moisture per kg solid. Air enters the dryer
Consider 1 Ib of wet feed: with a humidity ratio of 0.016 kg moisture per kg dry air and leaves
with a relative humidity of 100% while the temperature remains
Let x = weight of original product per Ib of wet feed constant at 60 a C. If the total pressure of air is 101.325 Kpa, determine
Solid in wet feed = Solid in dried product capacity of forced draft fan to handle this air in m 'zmin.
0.95 x = OAO(I) A. 80 C. 82
x = OA21 Ib B. 84 D. 86

Weight of water removed = I - OA21


Weight of water removed = O.5791b/lb oforig. product SOLUTION:

Let x= mass in kg of dry solid


3x + x = 1000
Air Conditioning - 26 (ME Bd. Oct. 1992) x = 250 kglhr
Since the given value is out ofrange, therefore:
Copra enters a dryer containing 60% water and 40% of solids and From steam table at 60 aC, P sat = 19.94 Kpa
leaves with 5% water and 95% solids. Find the weight of water 19.94
removed based on each pound of final product w = 0.622( )
J 01.3 - 19.94
A. 1.3751b C. 1.872 Ib w = 0.152 kg/kg
B. 119 Ib D. 2.345 lb Moisture removed = mtw, - WI)

Ii

j
4j'j
438 Air-conditioning Air-conditioning

3(~5())-0.1(250) = m(0152-0.016) W2 0.029 kg/kg


m 5,331 kglhr lct Ina amount of air
Using high temperature psychrometric chart: m a ( W 2 - WI) = amount of moisture removed
At 60°C(140°F) and 0.016 humidity ratio, m a(0.029 - 0.0087145) = 0.998747
v = 15.5 ttl/lb rna ~ 49.234 kg/sec
v .~ 0.968 rrr'zkg m, = 177,242.4 kg/hr
Fan capacity = (5331/60)(0.968)
Fan capacity = 86 m 3/min

Air Conditioning - 29 (ME Rd. Oct. 1985)

Air Conditioning - 28 (ME Rd. Oct. 1990) A Dryer is to deliver 1000 kg/hr of palay with a final moisture content
of 10%. The initial moisture content in the feed is 15% at atmospheric
Wet material, containing 215% moisture(dry basis) is to be dried at condition with 32°C dry bulb and 21 degrees centigrade 'wet bulb. The
the rate of 1.5 kg/sec in a continuous dryer to give a product dryer is' maintained 45°C while the relative humidity of the hot humid
containing 5% moisture(wet basis). The drying medium consist of air air from the dryer is 80%. If the steam pressure supplied to the heater
heated to 373°K and containing water vapor equivalent to a partial is 2 Mpa, determine the heat supplied by heater in kw. .
pressure of 1.40 Kpa. The air leaves the dryer at 310 0 K and 70% A. 323 C 4.23
saturated. calculate how much air will be required to remove the B. 5.46 D 6.23
moisture.
A. 213,233 kg/hr C. 177,242 kglhr SOLUTION:
B. 177,142kg/hrD. 198,234kg/hr

Let m amount ofpalay in wet feed


=

SOLUTION: Solid in wet feed = solid in product


0.85(m) = 0.90(1000)
m = 1,058.832 kg/hr
Let x = rate of flow of dried product From psychrometric chart:
Solid in wet feed = solid in dried product hi = 60.5 KJ/kg
I h2 = 74 KJ/kg
---(1.5) = 0.95x h, = 196 KJ/kg
1 + 2.15 WI = W2 = 0.0111 kg/kg
x = 0.501253 kg/sec
W3 = 0.0515 kg/kg
Amount of moisture removed 1.5 - 0.501253 3/kg
V2 = 0.915 m
Amount of moisture removed 0.998747 kg/sec
Amount of moisture removed = J058.823 - 1000
Solving for WJ :
Amount of moisture removed = 58.823 kg/hr
W
(
0.622l-
p-p v
Pv
j
"'
ma ( W 3 - W2) = 58.823
m a(0.05l5 - 001l!) = 58.823
W = 0.622 ( 1.4 ) rna = 1456.015 kglhr
101.325 + 1.4 Heat supplied by heater = m, (h 2 - hJ)
W = 0.0087145 kg/kg Heat supplied by heater = (1456.015/3600)(74 - 60.5)
From Psychrometric chart: Heat supplied by heater = 5.46 kw
440 Air-conditioning Air-conditioning 441

A. 0.123 C 0.054
Air Conditioning - 30 (ME Bd. Oct. 1985) B. 0.986 D. 0.019

A Dryer is to deliver 1000 kg/hr of palay with a final moisture content SOLUTION:
of 10%. The initial moisture content in the feed is 15% at atmospheric
condition with 32°C dry bulb and 21 degrees centigrade wet bulb. The
dryer is maintained 45°C while the relative humidity of the hot humid P,
RH
air from the dryer is 80%. If the steam pressure supplied to the heater Psat
is 2 Mpa, determine the air supplied to dryer in m 3/hr. Pv
A. 1332.25 C. 1234.23 0.7 =
B. 1532.34 D. 1982.34 4.29722
P, = 2.9722 kpa
Pv
SOLUTION w = 622 - -
P - P,

w = 0.622 (
2.9722 \j
Let m = amount of palay in wet feed 101.325 - 2.9722
Solid in wet feed = solid in product w ~, 001879 kg/kg
0.85(m) = 0.90(1000)
m = 1,058.832 kg/hr

From psychrometric chart: Air Conditioning - 32


hi = 60.5 Kl/kg
h 2 = 74 KJ/kg The humidity ratio of air is 0.045. If barometric pressure is 101 kpa,
h, = 196 KJ/kg find the partial pressure of water vapor.
WI = W2 = 0.0111 kg/kg A. 4.23 kpa C. 6.81 kpa
W3 = 0.0515 kg/kg B. 7.34 kpa D. 5.23 kpa
V2 = 0.915 m 3/kg

Amount of moisture removed 1058.823 - 1000 SOLUTION


Amount of moisture removed 58.823 kg/hr
m a(w3 - W2) = 58.823
m.(0.0515 - 00111) = 58.823
rna = 1456.015 kg/hr " P 'I
v, 1456.015(0.915)
w 622l-v
P-Pv
J
Va = 1332.25 m 3/hr '1
0.045 = 0.622I-
v
(

\.IOI-P v
P
--J
Air Conditioning - 31 101 - P, = 13.82P,
P, = 6.82 kpa
At 30°C, air-vapor mixture has a relative humidity of 70%. Find the
humidity ratio if barometric pressure is 100"C. At 30"C, P sn t = 4.246
kpa
442 .·1 ir-cotulitioning
A ir-conditioning 443

Air Conditioning - 33 p\
RH =
Psa t
Air at 36°C and pressure of 101.2 kpa has a density of 1.08 kg/m'.
P,
Find the humidity ratio of air. 060
A. 0.0352 C 00635 5.62~
B. 0.6350 D. 00173 P, = 3.3768 kpa

3.3768
w = 0.622 ( )
SOLUTION: 101.2- 3.3768
w = 0.0213
h = 1(35) + 0.0213(2565.3)
PY = mRT h = 89.63 KJ/kg
m P"
Y RT
108 = ---"--
0287(36 + 273) Air Conditioning - 35
P, = 95.777 kpa
P = P" + P" Air-vapor mixture has an enthalpy of 75 KJ/kg at 30°C. Find the
1012 = 95.777+P, partial pressure of water vapor. At 30°C: h g = 2556.3 KJ/kg
P, = 5.422 kpa A. 6.34 kpa C 4.231 kpa
p B 1.34 kpa D. 2.791 kpa
w = 0622 -----'-,-
P-P v
5.422 SOLUTION:
w 0622 ( - - - - - )
=
1012 - 5.422
w = 0.0352 kg/kg
h = c p t + W hg
75 = 1(30) + w(2556.3)
w = 0.0176
Air Conditioning - 34 P,
0.0176 = 0.622( )
101.2 - P,
What is the enthalpy of the air-vapor mixture at 60% RH and 35°C 101.325 - P, = 35.33 P,
when the barometric pressure is 102 kpa? At 35"C: P sat = 5.628, h g =
P" = 2.79 kpa
2565.3
A. 89.63 KJ/kg C 67.34 KJ/kg
Air Conditioning - 36
B. 7423 KJ/kg D. 53.34 KJ/kg
Air in an air-conditioner enters at 60% RH with w = 0.021 and leaves
SOLUTION:
at 25"C dry bulb and 16°C wet bulb. If mass of air is 10 kg/s, find the
refrigeration capacity in to ns of refrigeration.'
3/kg
At 60% RH, W = 0.021: It = 87 KJ/kg, v = 0.903 m
At 25°C db, 16°C wb: h = 45 KJ/kg;

You might also like